You are on page 1of 616

problem In their solutions, the authors have

attempted to convey a mode of ap-


book proach to these kinds of problems, re-

vealing procedures that can reduce the
In labor of calculations while avoiding
the pitfall of too much or too powerful
relativity formalism. Although well suited for in-

and dividual use, the volume may also be


used with one of the modern textbooks

gravitation in general relativity.

At Cornell University, Saul A. Teu-


kolsky is Associate Professor of Phys-
ics. Alan P. Lightman is Staff Scien-
tist at Harvard Smithsonian Center for
Astrophysics, and William H. Press
is Professor of Astronomy and of
Physics at Harvard University. Rich-
ard H. Price is Professor of Physics at

the University of Utah.


"This work is ful1 of interesting
problems, arranged by subject and
graded by difficulty. It is full of intel-
lectual content, and it is much more
than modern pedagogy. It is modern

physics, much of it at the frontiers,


done in modern ways."-John A.
Wheeler, Princeton University

Important and useful to every stu-


dent of relativity, this book is a unique
collection of some 475 problems-
with solutions-in the fields of special
and general relativity, gravitation, rela-
tivistic astrophysics, and cosmology.
The problems are expressed in broad
physical terms to enhance their per-
tinence to readers with diverse back-

grounds.

A complete catalog of Princeton


mathematics and science books, with
prices, is available upon request.

PRINCETON UNIVERSITY PRESS


PRINCETON, NEW JERSEY 08540
PROBLEM BOOK IN

RELATIVITY AND ORAVITATION

ALAN P. LIGHTMAN

WILLIAM H. PRESS

RICHARD H. PRICE

SAUL A. TEUKOLSKY

PRINCETON UNIVERSITY PRESS

PRINCETON, NEW JERSEY


Copyright @ 1975 by Princeton University Press
Published by Princeton University Press, Princeton and London

ALL RIGHTS RESERVED

Library of Congress Cataloging in Publication Data will


be found on the last printed page of this book
This book has been composed in VariTyper Bookman
Printed in the United States of America

by Princeton University Press, Princeton, New Jersey


Second printing, with corrections, 1979
CONTENTS

PRE F ACE . ..oo. ... ... . .. ..... . .. .. .... . .... . ... . ... ... . . .. . . .. .... . .oo . . .. ... . .. . ... . . ... vii

NOT AT ION ........ . . . . . .. . . .. . . . . . . .. . . .. . . .. . . .. . . . . .. . . . . .. . . . . . . . . .. . . .. . . ... .... . .. . . . xi

PROBLEMS SOLUTIONS

1. S peCla I R e I a t 1V1StlC K 1nemat1cs....................


. . . . . .

3 133

2. Special-Relativistic Dynamics ...................... 11 159

3. Special-Relativistic Coordinate Transforma-


tions, Invariants and Tensors ........................ 15 173

4. E lectroma gnetis m ............ .... .................... .... . ... 23 192

5. Matter and Radiation ...................................... 28 205

6. Me tr ics . . . .. . . . . . . . . . . .. . . . . . . . . .. . . .. . . . . . . .. . . .. . . . . . . .. . . . . .. .. . . . 37 233

7. Covariant Differentiation and Geodesic


C urv e s ................................................................ 40 243

8. Differential Geometry: Further Concepts .... 47 263

9. Curvature....... ....... ..... ........ ........ ..........oo .... ....... 55 284

10. Killing Vectors and Symmetries .................... 64 315

11. An gu la r Mom e n tu m ........ . .. . . .. . . .. . . . . . .... . .. . . .. . . .. . . . . . 67 327

12. Gravitation Generally................ ...................... 71 346

13. Gravitational Field Equations and


Linearized Theory ... ....................................... 76 364

14. Physics in Curved Spacetime ........................ 82 386

15. The Schwarzschild Geometry.................. ...... 87 404

16. Spherical Symmetry and Relativistic


Stellar Structure .............................................. 92 432

17. Black Holes ...................................................... 100 466

18. Gravitational Radiation .......... ........................ 106 490

19. C os mol og y . . . .. . . .. . . .. . . . . . . . . . . .. . . . . . . . . . . .. . . . . .. . . .. . . . . .. . . . 112 520

20. Exper imental Tests .u..................................... 122 560

21. Miscellaneous ................... ............................... 125 575

IND EX ..... ... . .. .. . ....... ........ . ...... ... . ... .. .... . . .. ... .... . .. . ...... . ...... . .... .... 593


PREFACE

This book contains almost 500 problems and solutions in the fields of

special relativity, general relativity, gravitation, relativistic astrophy-sics

and cosmology. The collection is motivated by a simple premise: that


the most important content of this field does not lie in its rigorous axio-
matic development, nor, necessarily, in its intrinsic aesthetic beauty, but
rather does lie in computable results, predictions, and models for pheno-
mena in the real universe. Accordingly, we have aimed for problems whose

statement is broadly understandable in physical terms and have tried to

make their statement independent of notational conventions. We hope to

awaken the reader's curiosity. ("Now how would one show that...?") We

have steered clear of purely technical problems, found in texts, of the form

"prove equation 17.4.38." In our solutions we also try to show the reader

"good" ways to compute things, methods and tricks which can vastly re-

duce the labor of a plug-in and grind-away approach, but we also try to

avoid the opposite pitfall of introducing too much confusing but powerful

formalism for an easy problem. There is often a lot of leeway in this

balance, and the reader should not be surprised if his solutions use a

rather smaller (or larger) set of calculational tools.

The first five chapters of this book deal only with special relativity,

and are designed for advanced undergraduates and graduate students in

any course in modern physics, classical mechanics or electromagnetism.

They are arranged roughly in order of increasing sophistication, beginning


at about the easy level of Spacetime Physics by E. F. Taylor and J. A.
Wheeler (Freeman, 1963); there are, however, both easy and difficult

problems in each chapter.

VII
viii PREFACE

The remainder of the book is aimed at the student in a course in

general relativity and/or cos


mology. The chapters cover as pects of metric
geometry, the equations of Einstein's gravitation theory (and some com-

peting theories), the effect of gravitation on other physical phenomena,


and applications to a variety of experimental and astrophysical situations.
A final chapter deals with some more formal topics whose applications are

less direct.

Each chapter begins with an


introductory note whose purpose is largely
to define the notation used. These by no means constitute a complete or

order ly presentation of the material covered in the chapter, but are intended

to aid the student familiar with a notation different from ours. We assume

that the reader has the benefit of one or more of the following texts (which

we have used heavily):

C. W. Misner, K. S. Thorne, and J. A. Wheeler, Gravitation (Freeman,


1973) [cited in this book as "MTW"] .

S. Weinberg, Gravitation and Cosmology (Wiley, 1972) [cited in this


book as "Weinberg"].
R. Adler, M. Bazin, and M. Schiffer, Introduction to General Relativity
2 nd ed. (McGraw-Hill, 1975).
We have also been influenced by the following texts or monographs:
Anderson, J. L., Principles of Relativity Physics (Academic Press,
1967).
Batygin, V. V., and Toptygin, I. N., Problems in Electrodynamics
(Academic Press Infosearch, 1964)

Hawking, S. W., and Ellis, G. F. R., The Large-Scale Structure of

Space-Time (Cambridge University Press, 1973) .

Landau, L. D., and Lifschitz, E. M., The Classical Theory of Fields,


3rd ed., (Addison-Wesley, 1971).

Peebles, P. J. E., Physical Cosmology (Princeton University Press,


1971) .

Robertson, H. P., and Noonan, T. W., Relativity and Cosmology


(Saunders, 1968).
Sexl, R. U., and Urbantke, H. K., Gravitation and Kosmologie,
(Wiener Berichte iiber Gravitationstheorie, 1973).
We have cited the primary literature where appropriate.
PREFACE ix

We are pleased to express our appreciation to colleagues who have


contributed original problems to this collection: Douglas Eardley,
Charles W. Misner, Don Page, Bernard F. Schutz, and our friend and

teacher, Kip S. Thorne.

We are also grateful to C. R. Alcock, B. C. Barrois, J. Conwell,

H. B. French, K. S. Jancaitis, C. Jayaprakash, S. J. Kovacs and

W. A. Russell for valuable help in improving the problems and solutions.


Our thanks go to Steve Wilson for preparing most of the illustrations in

this book. We acknowledge support from the Department of Physics at the

California Institute of Technology while we were there,. Of course, we

are responsible for the errors which inevitably must be present in a book

of this sort. We have tried particularly hard for problems and solutions

which are conceptually free from error, but we also apologize in advance

for the algebraic slips that the diligent reader will certainly find; we

invite his corrections.

A. P. LIGHTMAN
w. H. PRESS
R. H. PRICE
S. A. TEUKOLSKY

PASADENA, MAY 1974


NOTATION

It is intended that this book be compatible with several different text-

books, each with its own system of notational conventions. Thus, no

single notational system will be used exclusively in this book. In almost

all instances, meanings will be clear from the context. The fG ingoW
is

a list of the usual meanings of some frequently used symbols and


conventions.

a , {3 II
, r'
v... Greek indices range over0, 1, 2, 3 and represent space-
time coordinates, components, etc.
i , J , k... 1, 2, 3 and represent coordinates

Latin indices range over


etc. in 3-dimensional space

e e. ...
Basis vectors
a' J

A (Any boldface symbol) a spacetime vector, tens or, or form

A A 3-dimensional vector


All, B {3 ...
Tensor components

0 1 2
(A ,A ,A ,A 3) A vector represented by its components


(A ,
) A vector represented by its time component and spatial
part

(Caret) indicates unit vector, components in orthonormal
basis

d/dA Occasionally used to represent a vector (see Introduction


toChapter 7)
A(f) A vector operating on a function =
Aaf
,a

xi
xii NOTATION

(,,) A one-form

@ Outer product, tensor product e. g. A B has components


A#lB v

A Wedge product (see Introduction to Chapter 8)


V Covariant derivative operator (see Introduction to Chapter

.7). Also used as in ordinary physics V x curl, V = =

Laplacian, etc.

VA Directional derivative (see Introduction to Chapter 7)

D/dA Covariant derivative along a curve (see Introduction to

Chapter 7)
d Gradient operator as in e. g. the one-form df (see intro-
duction to Chapter 8)

Lie derivative (see Problem 8.13)


r Christoffel symbol (see Introduction to Chapter 7)
13y
d' Alembertian operator 2 2 2
o == V -
a / at in Special Relativity

Partial derivative

Covariant derivative (see Introduction to Chapter 7)

Riemann tensor (see Introduction to Chapter 9)


Ra 13yo
RY
Ral3 Ricci tensor ==
aY13

R Ricci scalar == R . Also scale factor in Robertson-Walker

metric.

G Einstein tensor (see Introduction to Chapter 9)


a l3
C Weyl (conformal) tensor (see Introduction to Chapter 9)

(3yo
K.
lJ
. Extrinsic curvature tensor (see Introduction to Chapter 9)

r Proper time
c Speed of light (usually taken as unity in the problems)
G Gravitational constant (usually taken as unity in the
problems)
u 4-velocity
a 4-acceleration == du/dr

p or P 4-momentum
NOTATION xiii

p or P Pressure

Tllv Stress-energy tensor (see Introduction to Chapter 5)

FIlV Electromagnetic field tensor (see Introduction to Chapter 4)

JIl Current density (see Introduction to Chapter 4)

JIlV Angular momentum tensor (see Problems 11.1, 11.2)

11 1lv Minkowski metric (see Introduction to Chapter 1)

h Metric perturbations (see Introduction to Chapter 13)


l1v

C.M. Center of momentum frame, center of mass

v,w Frequency in cycles per unit time, radians per unit time
2 2 1..
y Lorentz factor == (1 v / c )- 2,

or photon symbol


Lorentz transformation
f3

A matrix

det Determinant

Tr Trace

< > Average (as in < E> =
average energy)
<,> Scalar combination of vector and one-form, as in < w, A>

(see Introduction to Chapter 8)


[ ] Antisymmetrization (see Problem 3.17) or commutator (see
Introduction to Chapter 8) or discontinuity (as in Problem
21.9)
( ) Symmetrization (see Problem 3.17)


a f3yo The totally antisymmetric tensor (see Problem 3.20 J)

* Duality symbol (see Problem 3.25)


Re Real part
{} Solid angle (as in f dO), angular velocity

a f3 Projection tensor (see Problems 5.18, 6.6)
()

Expansion (see Problem 5.18)


G Shear (see Problem 5.18)
a f3
w Rotation (see Problem 5.18)
a f3
xiv NOTATION

k Reduced quadrupole tensor (see Introduction to Chapter 18)

HO Hubble constartt

Deceleration parameter
qo

Mass, radius,." of sun


Me' Re'
.oo

z Redshift factor (see Problem 8.28, Introduction to

Chapter 19)
(9 Order of magnitude

3 2
oc:
Proportional to (e.g., r oc: t
) or
parallel vector to (e. g., A oc:
B)
PROBLEMS
CHAPTER 1

SPECIAL-RELA TIVISTIC KINEMATICS

The path of an observer through spacetime is called the worldline of

that observer. The time measured by the observer's own clocks, called
his proper time r, is given by

2 2 2 2
_dr 2 == ds =
-dt + dx + dy2 + dz ,

where t, x, y, z are the observer's (Minkowski) coordinates along his

path. Here, and unless noted otherwise throughout this book, we use units

in which c, the speed of light, is unity.

The 4-velocity u, with components (dt/dr, dx/dr, dy/dr, dz/dr), and


2 2 2 2 2 2 2 2
4-acceleration a == du/dr, components (d t/dr d x/dr d y /dr d z/dr ),
, , ,

are defined on the worldline. The (contravariant) components of these or

other 4-vectors are denoted u



, a f3 AY,
, BO, etc., where a Greek index

indicates any of the 4 components t, x, y, z == 0, 1, 2, 3. Latin indices


i, j, k... are used to indicate only the spatial components x, y, z == 1, 2, 3.
The Einstein summation convention is used, that is, any repeated literal
index is assumed to be summed over its range. For example,

v =
Vile
11

expresses a vector as a sum of contravariant components multiplied by


basis vectors, eo ==
(1, 0, 0, 0), e

==
(0, 1, 0, 0), etc.

The invariant dot product of two 4-vectors is, in Minkowski coordinates,

A.B= _AoBo+AlB1+A2B2+A3B3.

This can be written as A. B


AIlBIl, where the numbers
All' called co-

All 1lV
variant components of A, are defined by
All
==
TJ IlVAv, or =
11 Av


4 PROBLEM BOOK IN RELATIVITY AND GRAVITATION

-1 0 o 0
o 1 o 0
11
11V

0 0 1 0
(als 0 ==
TJl1v) .

o 0 o 1

Vectors are called spacelike, timelike, or null, according to whether thei

square v. v is positive, negative, or zero. 4-velocities are always time

like.

Two Lorentz frames may differ by a relative 3-velocity y or by a

spatial rotation, or by a combination of relative velocity and rotation. If

t, x, y, z are the coordinates of one frame, then the coordinates in a

different frame are usually written t', x', y', z'. Similarly, vector com-

ponents in the primed frame are written All, Bv" etc., and its basis
vectors are The basis vectors and the components of vectors in
ell"
Lorentz frames are related by


11
, =

A e


' V,=Aa,Va
Il 11

, , ,

VIl = All

Va (All =

matrix inverse of A

,)
a 11

where the A's are Lorentz transformation matrices. Of special interest

are the "boost" transformations involving changes in velocity with no

rotation. For a primed frame with velocity f3 in the x-direction,

y -yf3 o 0
y o o


-yf3 o 0

All

0 1 0
Y _

(1_f3

)-2
o o 1

The velocity between two frames is sometimes parameterized by () ==


tanh- f3 ("the rapidity parameter").

A particle of rest mass m and 4-velocity u has 4-momentum p == ml

If m =
0 (photons), p is defined by its components in the frame of any

observer po ==
photon energy, pi ==


photon 3-momentum.
1. SPECIAL-RELATIVISTIC KINEMATICS 5

Problem 1.1. The 4-velocity u corresponds to 3-velocity v. Express:



(a) u in terms of
I I
(b) uj(j =
1,2,3) in terms of
Y-
° .


(c) u in terms of u

(d) d/dr in terms of dldt and v


j j
(e) v in terms of u

(f) Ivl-
in terms of uO.

Problem 1.2. Find the matrix for the Lorentz transformation consisting of

a boost v in the x-direction followed by a boost v in the y-direction.



Show that the boosts performed in the reverse order would give a different

transformation.

Problem 1.3. If two frames move with 3-velocities Yl and show


Y2'
that their relative velocity is given by


2 =
(Yl -2..2)2 -

(YlXY2)2
(1- Xl .

Y2)2
Problem 1.4. A cart rolls on a long table with velocity f3. A smaller cart

rolls on the first cart in the same direction with velocity f3 relative to

the first cart. A third cart rolls on the second cart in the same direction

with relative velocity f3, and so on up to n carts. What is the velocity



of the nth cart in the frame of the table? What does v tend to as

n oo?

Problem 1.5. A distant camera snaps a photograph of a speeding bullet


(velocity v) with length b in its rest frame. Behind the bullet and

parallel to its path is a meter stick, at rest with respect to the camera.

The direction to the camera is an angle a from the direction of the

bullet's velocity. What will be the apparent length of the bullet as seen

in the photo? (Le. How much of the meter stick is hidden?).

Problem 1.6. Tachyons are hypothetical particles whose velocity is faster

than light. Suppose that a tachyon transmitter emits particles of a constant


6 PROBLEM BOOK IN RELATIVITY AND GRAVITATION

velocity u > c in its rest frame. If a


tachyonic message is sent to an

observer at rest at a distance L, how much time will elapse before a

tachyonic reply can be received? How much time will elapse if the distant

observer is moving directly away at velocity v, and is at a distance L

at the instant he receives the message and replies? (Show that for u >


[1 + (1_v ) 2]/V the reply can be received before the signal is sent!)

Problem 1.7. Frame S' moves with velocity y relative to frame S. A

rod in frame S' makes an angle ()' with respect to the forward direction

of motion. What is this angle () as measured in S?

Problem 1.8. Frame S' moves with velocity (3 relative to frame S. A

bullet in frame S' is fired with velocity y' at an angle ()' with respect


to the forward direction of motion. What is this angle () as meas ured in

S? What if the bullet is a photon?

Problem 1.9. Suppose that an observer at rest with respect to the fixed

distant stars sees an isotropic distribution of stars. That is, in any solid

angle dO he sees dN =
N(dO/417) stars, where N is the total number

of stars he can see.

Suppose now that another observer (whose rest frame is S') is moving

at a relativistic velocity (3 in the e



direction. What is the distribution

of stars seen by this observer? Specifically, what is the distribution

function P(()', cp') such that the number of stars seen by this observer in

his solid angle dO' is P(()', cp') dO'? Check to see that fsphere P(()',cp')dO'


N, and check that P(()', cp') -+ as
{3 -+ O. Where will the observer see
417
the stars "bunch up"?

1 1
Problem 1.10. Show that A =
32et + 22 is a unit timelike vector in
ex

special relativity. Show that the angle between A and e



is not real.

Problem 1.11. Two rings rotate with equal and opposite angular velocity
w about a common center. Suppose Adam rides on one ring and Eve on

the other, and that at some moment they pass each other and their clocks

agree. At the moment they pass, Eve sees Adam's clock running more
1. SPECIAL-RELATIVISTIC KINEMATICS 7

slow ly, so she expects to be ahead the next time they meet. But Adam

expects just the reverse. What really happens? Can you reconcile this

with Adam's (or Eve's) observations?

Problem 1.12. Define an imaginary coordinate w =


i t. Show that a rota-

tion of angle () in the


xi' w plane (i= 1,2,3), where () is a pure imagi-

nary number, corresponds to a pure Lorentz boost in t, x, y, z coordinates.

How is the boost velocity v related to the angle ()?

Problem 1.13. Show that the curve

x =
r r c os () c os cp dA


r r cos () sin cp dA

z =
r r sin () dA

t =
r r dA ,

where r, () and cp are arbitrary functions of A, is a null curve in

special relativity. Under what conditions is it a null geodesic?


Problem 1.14. Show that an observer's 4-acceleration du /dT has only

3 independent components, and give the relation of these to the 3 com-

ponents of ordinary acceleration that he would measure with a Newtonian

accelerometer in his local frame.

Problem 1.15. Write the magnitude of the acceleration measured in the

observer's frame as an invariant.

Problem 1.16. A particle moves with 3-velocity u


--
and 3-acceleration a
.-

as seen by an inertial observer 0. Another inertial observer 0' has

3-velocity .y relative to 0. Show that the components of acceleration of

the particle parallel and perpendicular to y as measured by 0' are


3/2
(l-v 2
'II =

3 I
(l-y )


a' =
C1-v ) [a -
v x (axu)]

l \3
- .- .-.-

(1 -v .

JlJ
8 PROBLEM BOOK IN RELATIVITY AND GRAVITATION

Problem 1.17. An observer experiences a uniform acceleration in the x

direction, of magnitude g. Define a coordinate system (t, x, y , z) for him


in the following way: (i) Let the observer be at x =


z =
0 and let t

be his proper time. (ii) Let his hyperplanes of simultaneity agree with

the hyperplanes of simultaneity of an instantaneously comoving inertial


frame. (iii) Let the other "coordinate stationary observers" (for whom
x, y, z are constant) move in such a way that they are always at rest with

respect to the observer on the hyperplanes of simultaneity. At t =


0 label

all spatial points with the same labels as the momentarily comoving in-

ertial system t =
0, x, y, z.

Give the coordinate transformation between t, x, y, z and t, x , y, z .

Show that coordinate stationary clocks cannot remain synchronized.

Problem 1.18. A mirror moves perpendicular to its plane with a velocity v.

With what angle to the normal is a ray of light reflected, if it is incident

at an angle ()? What is the change in the frequency of the light?

Problem 1.19. A mirror is moving parallel to its plane. Show that the
angle of incidence of a photon equals the angle of reflection.

Problem 1.20. A particle of rest mass m and 4-momentum p is examined

by an observer with 4-velocity u. Show that:

(a) the energy he measures is E =


-p u;


(b) the rest mass he attributes to the particle is m = -

p;


(c) the momentum he measures has magnitude I pi =
[(p.u)2 + p .

p] ;

(d) the ordinary velocity v he measures has magnitude


[ ]
p,p
\yl =
1 +
(p .
u )2

(e) the 4-vector v, whose components in the observer's Lorentz

frame are
. .

o J J
v =
0, v =
(dx /dt)par t lC 1 e "

ordinar y velocit y ,



by

IS v = -
u -

given u



1. SPECIAL-RELATIVISTIC KINEMATICS 9

Problem 1.21. An iron nucleus emits a Mossbauer gamma ray with frequency
V as measured in its own rest frame. The nucleus is traveling with

velocity @, with respect to some inertial observer. What frequency does


the observer measure when the gamma ray reaches him? Express the

answer in terms of
, and the unit vector E. pointing towards the
vo'
nucleus at the time it emitted the y-ray, as meas ured by the observer.

Problem 1.22. An observer receives light from a source of light which is

moving with a velocity v;


---
the angle between v
---
and the line between

observer and source is () at the time the light is emitted. If the observer

sees no net redshift or blueshift, what is () in terms of Iy\?


Problem 1.23. Suppose in some inertial frame S a photon has 4-momentum
components
po =
pX =
E, pY =
pZ =
0 .

There is a special class of Lorentz transformations -


called the "little

group of p" which leave the components of p unchanged, e.g. a pure


rotation through an angle a in the y-z plane


1 o o o

o o
o 1 o o

o o

E E o o

E E o o

cosa -sin a

sin a cosa

is such a transformation. Find a sequence of pure boosts and pure rota-

tions whose product is not a pure rotation in the y-z plane, but is in the

little group of p.

Problem 1.24. Two giant frogs are captured, imprisoned in a large metal
cylinder, and placed on an airplane. While in flight, the storage doors
accidentally open and the cylinder containing the frogs falls out. Sensing
something amiss, the frogs decide to try to break out. Centering them-
selves in the cylinder, they push off from each other and slam simultane-

ously into the ends of the cylinder. They instantly push off from the ends
and shoot across the cylinder past each other into the opposite ends. This
10 PROBLEM BOOK IN RELATIVITY AND GRAVITATION

continues until the cy linder hits the ground. Consider how this looks from

some other inertial frame, falling at another speed. In this frame, the frogs

do not hit the 'ends of the cylinder simultaneously, so the cylinder jerks
back and forth about its mean speed {3. The cylinder, however, was at

rest in one inertial frame. Does this mean that one inertial frame can jerk
back and forth with respect to another?

Problem 1.25. J x ' J y Jz be infinitesimal rotation operators defined


Let '

that 1 + iJ /2 is a rotation by a small angle e around the j-axis.


so
je
Let Kx , K ,
Kz be infinitesimal boost O p erators defined so that

1 + i K.v /2

is a boost by a small velocity v in the j-direction. Show
that the following relationships, and all their cyclic permutations, are true:

[J x
' Jy ] =

2iJz
[ J x' Ky] =

2iKz
[K x ' Ky] =

-2iJz

Find a representation of the Lorentz group in terms of Pauli spin matrices

ax'
a , a

' and the unit matrix.

Problem 1.26. Two successive, arbitrary pure Lorentz boosts Yl and


Y2
are equivalent to a pure boost
Y3
followed by a pure rotation e ,where
n is a unit vector. Find the magnitude of e in terms of Yl and
.Y2
and

show that . 3

O.

Problem 1.27. Show that any proper (non time-reversing, non parity-

reversing) homogeneous Lorentz transformation leaves fixed at least one

null direction.

Problem 1.28. What is the least number of pure boosts which generate an

arbitrary Lorentz transformation? Note: This is a difficult problem!


CHAPTER 2

SPECIAL-RELA TIVISTIC DYNAMICS

In our laboratory frame, a particle with 4-momentum p has total ene'rgy



E =
p and 3-momentum p
.-

pl. If the particle has a nonzero rest mass m,

the 4-momentum, 4-velocity u and 3-velocity yare related by



2 -"2


mu =

m(y,yy), Y ==
(1 -

v) ,

so E =
ym, p
.-

ymv. The square of
.-
a particle's 4-momentum, an invariant

in all frames, is
222



-E + p =
-m .

The kinetic energy of a particle is T == E -


m.

The fundamental dynamical law for particle interactions is that in any

frame the vector sum of the 4-momenta of all particles is a conserved

constant in time.

o o o o o o o

Problem 2.1. (Compton scattering.) A photon of wavelength A hits a

stationary electron (mass m


e ) and comes off with wavelength A' at an

angle (). Derive the expression

A' -A =
(him) (l-cos ()) .

Problem 2.2.

(a) When a photon scatters off a charged particle which is moving with

a speed very nearly that of light, the photon is said to have undergone an

inverse Compton scattering. Consider an inverse Compton scattering in

which a charged particle of rest mass m and total mass-energy (as seen

11
12 PROBLEM BOOK IN RELATIVITY AND GRAVITATION

in the lab frame) E? m, collides head-on with a photon of frequency


v (hv? m). What is the maximum energy the particle can transfer to the

photon?

(b) If space is filled with black-body
radiation of temperature 3 K
20
and contains cosmic ray protons of energies up to 10 eV, how much
20 0
energy can a proton of energy 10 eV transfer to a 3 K photon?

Problem 2.3. Show that it is impossible for an isolated free electron to

absorb or emit a photon.

Problem 2.4. A particle of rest mass m



and velocity .Yl collides with

a stationary particle of rest mass m



and is absorbed by it. Find the

rest mass m and velocity .Y of the resultant compound system.

Problem 2.5. The beta-decay of a neutron is isotropic in the rest frame

of the neutron, with the velocity of the emitted electron v




0.77. If the

neutron is moving with velocity {3 through the laboratory, what values of

the electron's laboratory momentum vector .r are possible?

Problem 2.6. Evaluate the "available energy" of two different proton-

proton scattering experiments. The first is of the conventional type, where

a beam of protons is accelerated to 30 GeV and allowed to strike a target

(liquid hydrogen, for example). In the second., two separate beams of pro-

tons are accelerated to 15 GeV each, then directed toward each other and

allowed to collide. Evaluate the total energy of two colliding protons in

the center of momentum frame for each experiment. To what energy would

a beam in the first type of experiment have to be accelerated to match the

CM energy of the 15 GeV protons in the second experiment?

Problem 2.7. A particle of rest mass m collides elastically with a sta-

tionary particle of equal mass. The incident particle has kinetic energy

To' What is its kinetic energy after the collision, if the scattering angle

is ()?

Problem 2.8. Calculate the threshold energy of a nucleon N for it to

undergo the reaction


2. SPECIAL-RELATIVISTIC DYNAMICS 13

y+N N+17


where y represents a photon of temperature 3 K. Assume the collision

is head-on; take the photon energy to be ""


kT; m


940 MeV; m
17

140 Me V. (This effect probably produces a cut-off in the cosmic-ray

spectrum at this threshold energy.)

Problem 2.9. Consider the reaction 17+ + n K+ + A 0. The rest masses

of the particles are m =


140 MeV, m =
940 MeV, m =
494 MeV, =

17 n K mA
1115 MeV. What is the threshold kinetic energy of the 17 to create a K

at an angle of 90° in the lab in which the n is at rest?

Problem 2.10. Consider the reaction A B+ C (with particle masses


A ,

B ,
me) .

(a) If A is at rest in the lab frame, show that in the lab frame parti-

cle B has energy E



(mi m + -

m )/2m A'

(b) An atom of mass M at rest decays to a state of rest energy M-o

by emitting a photon of energy hv. Show that hv < o. In the Mossbauer

effect, why is hv =
a?

(c) If A decays while moving in the lab frame, find the relation
between the angle at which B comes off, and the energies of A and B.

Problem 2.11. Consider the reaction 1 + 2 3 + 4. The lab frame is de-

fined to be the one in which E2 =


O. The C.M. frame is defined to be the
C M c M
which 'p l 'f. 2 O. Show that:
. . . .

one in + =


c.M. 2 2 2
( a ) E tota I =
(m l + m

+ 2m


1 )
c M 2 M
(b ) E C.M.=
1 [ ( E total )
. .

+ m

2 _


2]/2E c
total
. .

C M
( c ) pC.M. P IE
. .
= m
1 2 1 total

(d) YC.M. =
(E l +m2)/Efo.t i(v C . M .
==
velocity of C.M. in lab frame,

2 -2
and Y C M. .
==
(1 -

c. M .
) .

(e) v = P I (E + m ).
C . M. 1 1 2
14 PROBLEM BOOK IN RELATIVITY AND GRAVITATION

Problem 2.12. Consider the elastic collision of a particle of mass m


with a stationary particle of mass m



< m

. Let ()
max
be the maximum

scattering angle of m]. In nonrelativistic calculations, sin (}max =




/m l "

Prove that this result also holds relativistically.

Problem 2.13.

(a) If a rocket has engines that give it a constant acceleration of 1 g

(relative to its instantaneous inertial frame, of course), and the rocket

starts from rest near the earth, how far from the earth (as measured in the

earth's frame) will the rocket be in 40 years as measured on the earth?

How far after 40 years as measured in the rocket?

(b) Compute the proper time for the occupants of a rocket ship to

travel the 30,000 light years from the Earth to the center of the galaxy.

Assume they maintain an acceleration of 1 g for half the trip and decelerate

at 1g for the remaining half.

(c) What fraction of the initial mass of the rocket can be payload in

part (b)? Assume an ideal rocket that converts rest mass into radiation

and ejects all of the radiation out of the back with 100% efficiency and
perfect collimation.

Problem 2.14. What is the maximum energy one could get out of a fixed

frequency electron cyclotron with accelerating potential V.


Problem 2.15. A new force field FIL(x ) is discovered which induces a


particle of

4-acceleration all == dull /dr =
m- FIL(x ) on a mass m, at


position xv. Notice that FIl does not depend on u . Show that this

force is not consistent with special relativity.


CHAPTER 3

SPECIAL-RELATIVISTIC COORDINATE TRANSFORMATIONS,


INVARIANTS AND TENSORS

Spacetime in special relativity can be descr ibed by more general


(curvilinear) coordinates than "inertial" or Minkowski coordinates, e.g.

coord inates xll, ,



xll =
f Il(X )

where XV are Minkows ki coordinates, and f Il four arbitrary functions.

One then shows that the basis vectors and components of vectors in the

new coordinates are related to the old by


axil ax
=-e,
e e
ell'

a' a

Il a
ax axil

axil, va'

ax
va' =
VIl , VIl =


axil ax

v ' =
axil v v =
ax
Va '


a Il' Il
ax axil

, ,

In other words, the transformation matrix A':z == axil / ax replaces the
less general Lorentz matrices (which are applicable only for transforma-
tions between two systems of Minkowski coordinates).
In general coordinates, the relation A. B =

AIlBIl still holds, but we

no longer have All


11IlVAIl. Rather, corresponding to every coordinate


system there is a metric tensor with components such that ds
{3'

dxadx{3,which A.B=gIl VAIlB


V v
g leads to A =g A and therefore .

UfJ Il IlV
Note also All gllV Av where gllV is the matrix inverse of
gllv.

Various formal definitions of a tensor are possible. Here, it suffices

to say that it is a geometrical object which, like a vector, has components

15
16 PROBLEM BOOK IN RELATIVITY AND GRAVITATION

whose numerica I values are different in different coordinate systems. A



tensor has 4 components, where n is its rank (number of "slots" or

indices for components). Slots may be contravariant or covariant; examples:



Tllv,

F R G Tensors transform with one matrix for each slot,

{3yo'

Ilv ll
e.g. , ,

G Y = Aa ,Av Q Ga fJ .

Il Il fJ

Tensors may be "contracted" (a covariant and a contravariant index

summed) or multiplied as a "direct product" with other tensors or with

themselves to form new tensors, e.g.

a V
R All GIlv B F
QIlV All Bv
= = =
' ' ・

/lClV IlV

A special case is contraction with the metric tensor, where the same

symbol is usually used for the result as in the relation of covariant and

contravariant vectors, FIl v =


gvaF/lCl. A tensor expression with no free

indices, e.g. F
/lVA/lAV or
Rf3 y Ff3y or
AaBf3 f3' is a scalar and is an

invariant number in all frames. The analog of the index free notation A

for a vector All is to write, e.g. T for a tensor Tllv. In both cases the

existence of covariant or contravariant slots must be deduced from the

context.

In index free notation, represents the direct product, e. g. FllvAP


is written F A; the contracted product is written with a dot, e.g. F. A

for Ff.lll Aa'



We denote partial derivatives by a comma, e.g. f a
== af/ax .

o o o o o o o

Problem 3.1.

(I) If 2 events are separated by a s pacelike interval, show that


(a) there exists a Lorentz frame in which they are simultaneous, and

(b) in no Lorentz frame do they occur at the same point.


(II) If 2 events are separated by a timelike interval, show that

(a) there exists a frame in which they happen at the same point, and

(b) in no Lorentz frame are they simultaneous.


3. SPECIAL-RELATIVISTIC COORDINATE TRANSFORMATIONS 17

Problem 3.2. Find 4 linearly independent null vectors in Minkowski space.

Can you find 4 which are orthogonal?

Problem 3.3. Show that the only non-spacelike vectors orthogonal to a

given nonzero null vector are multiples of it.

Problem 3.4. Show that the sum of two vectors can be spacelike, null, or

timelike, independently of whether the two vectors are spacelike, null, or

timelike.

Problem 3.5. Show that the cross-sectional area of a parallel beam of light
is invariant under Lorentz transformations.

Problem 3.6. Show that DIlIl and l under coordi-


Il

Il
DIlIl are not invariant

nate transformations, but that lD


Il
Illl is invariant. (Take D. to be a

tensor defined by its components Dllv .)

Problem 3.7. F

{3 is antisymmetric on its two indices. Show that





= _
F F

{3 .

Il,V a lla ,fJ

Problem 3.8. In a coordinate system with coordinates xll, the invariant

line element is ds

dxadx fJ If the coordinates are transformed
TlafJ

2 V
x/l /l, show that the line element is ds d x /ld x and express
g/lV

-> x ,

g- V

in terms of the partial derivatives axil / a .or two
F arbitrary
Il
4-vectors U and V, show that

lflv fJg
u V ・ =

uavfJTlafJ =

afJ

Problem 3.9. Show that the determinant of the metric tensor g == det
(gllv)
is not a scalar.

which transform the


Problem 3.10. If
A and
A are two matrices

components of a tensor from one coordinate basis to another, show that

the matrix
AA is also a coordinate transformation.
18 PROBLEM BOOK IN RELATIVITY AND GRA VITA TION

Problem 3.11. You are given a tensor K



{3. How can you test whether it

of two vectors K
a {3 A

B(3? Can you express the
is a direct product =

test in coordinate-free language?

Problem 3.12. Prove that the general second-rank tensor in n-dimensions

cannot be represented as a simple direct product of two vectors, but can

be expressed as a sum over many such products.


Problem 3.13. A two index "object" Xfl. is defined by the "direct sum"
V V V
of two vectors Xfl. =
Afl. + B . Is Xfl. a tensor? Is there a transforma-
""


tion law to take. X to a new coordinate system, Le. to obtain Xfl. from


Xfl. ?

Problem 3.14. Show that a second rank tensor F which is antisymmetric

coordinate frame is antisymmetric in all frames.


in (F -F
Vfl.)

one
fl.V

Show that the contravariant components are also antisymmetric (Ffl. =

FVfl.). Show that symmetry is also coordinate invariant.

be
Problem 3.15. Let
Afl.V an antisymmetric tensor so that
Afl.V AVfl.;
= -

and let Sfl.



be a symmetric tensor so that Sfl. V

SVfl.. Show that

Afl.V Sfl. O. Establish the following two identities for any arbitrary

tensor V
fl.v:
vfl.V A = 1. (Vfl.V -

VVfl.) Afl.V vfl.VS = 1. (Vfl. V + VVfl.) S .

fl.V

fl.V 2 fl.V 2

Problem 3.16.

(a) In an n-dimensional metric space, how many independent com-

ponents are there for an r-rank tensor rz{3'" with no symmetries?


(b) If the tens or is symmetric on s of its indices, how many inde-

pendent components are there?

(c) If the tensor is antisymmetric on a of its indices, how many

independent components are there?

Problem 3.17. We define the meaning of square and round brackets en-

closing a set of indices as follows:


3. SPECIAL-RELATIVISTIC COORDINATE TRANSFORMATIONS 19

== .1' l V

(a l'
...
, a)
p p.
, a

"'a

' Vr
La .a ]


.ll (-l)"V
p! a" 1
...

1 p 1 ,.. p a"

Here the sum is taken over all permutations " of the numbers 1, 2,'
. .

, p

and (_1)" is +1 or -1 depending on whether the permutation is even

or odd. The quantity V may have other indices, not shown here, besides
the set of p indices a
1 ,


,"', a ' but only this set of indices is

affected by the operations described here. The numbers "1'"2"" ,


"p
are the numbers 1, 2,." p , rearranged according to the permutation ".


Thus for example V
(a 1 a 2 )
==

( Va a +
Va 2 a 1 ) or equivalently V ( v ) =

1 2 Il
(V/Lv + V V/L)'
(a) If F is antisymmetric and T is symmetric, apply these defini-
tions to give explicit formulas for the following: F
V[ll v ]' F[ll v ]' (Il v )'

TijLv]' T(Il V)

V[a y]'T(a{3,y)' F[a ,y]-
(b) Establish the following formulae: V ((a '''a )) V
(a ."a );

1 pIp

V[[a 1 ...a ]]

V[a 1 ...a ]; V(a 1 ...[ana J... a )



0; V [a ."[ana J... a ]

p p Lm p 1 t.m p


[a 1 ...
an a'"
t. m


].
(c) Use these notations to show that F
Ilv
'=

V ,Il- AIl,v implies
O. (Half of Maxwell's equations!)
Fa ,vF v,aFva,
+ + =

Problem 3.18. Show for any two-index tensor X, that


Xa X(a )

[ ] denote symmetrization and antisymmetriza-


X[a ]where ( ) and

tion, respectively. Show that in general

1=
Ya yY(a y) Y[a y]
+ ・

Problem 3.19. Prove that the Kronecker delta, all is a tensor.


v'

Problem 3.20. Prove that, except for scaling by a constant, there is a

unique tensor which is totally antisymmetric on all its 4 indices.


Ea yo
The usual choice is to take E =
1 in Minkowski coordinates. What
0123
are the components of E in a general coordinate frame, with metric
gllv?
20 PROBLEM BOOK IN RELATIVITY AND GRAVITATION

Problem 3.21. In an orthonormal frame, show that

E = -

a (3yo
(3 yu

What is the analogous relation in a general coordinate frame with metric



gllv
P ro bI em 3 22 . . E va I ua t e E E Ilvpa .

/lvpa


Problem 3.23. Show that for any tensor A
(3

AUp.A{3vAYAAOu

Eu{3yo

EP.VAu
det II A {311
a a
where IIA (311 is the matrix of the components A
(3'
Problem 3.24. Show that four vectors u, v, W, x, are linearly independent
if and only if u A v A W AX O. Show that in this case U A v A W AX is pro-

portional to the totally antisymmetric tensor E. (The "wedge" product is


defined as the antis ymmetrized direct product, e. g. U A V =
U 0 v -
v 0 u.)

Problem 3.25. Let F be an antisymmetric second-rank tensor with com-



ponents F/l . From F construct another second-rank, anti-symmetric

tensor, *F, called the dual of F, as follows

*F = 1. E/lva (3 F e 0 e
2 a
(3 /l v.

Show that *(*F) = -


F.

Problem 3.26. Show that

V aVa = - 1 (*V) (*V)a(3y


(3 y

3! a

II'.A

Problem 3.27. The tensor u' is d efined b y


p...a

/l A
a ...

p p

/l...A == det
p" .a

a /l ...

a a
3. SPECIAL-RELATIVISTIC COORDINATE TRANSFORMATIONS 21

Show that if there are more than 4 upper (or lower) indices, the tensor

identically vanishes.

Problem 3.28. Show that o/lv = - ! E/lvpa E and g eneralize to


AK 2 AKpa'
o :: of other ranks.

Problem 3.29. Show that if the antisymmetric tensor pa(3 is a bivector

(i.e. pa(3 =
A [aB(3]) then

pa(3 pYa + pay p0(3 + pao p(3y =


(the Pliic ker relations).

Problem 3.30. In 4-space define the 3-dimensional volume element in a


hypersurface x

xa(a, b, c) by d

Epn.{3y(1/3!) da db dc [J(x
,x )/

III

,x{3 y

b,
J(a, c)], where the last factor is a 3 x 3 Jacobian determinant. Compute

for a space-like hypersurface xO
the components of d
III constant, =


parameterized by xl


a, x =
b, x =
c.

Problem 3.31. Show that the invariant proper volume element in

4-dimensional space is given by



dV =
(_g)2 d x


where d x dxdydzdt in the coordinate system of the metric
gl1 v

Problem 3.32. Show that the proper 3-volume element of an observer with

3 3
4-velocity u is d V =
(_g)2 uOd x, and show that this is a scalar

invariant.

Problem 3.33. What is the invariant volume element of contravariant


momentum d P for 4-dimensional momentum space? What is the invariant

3-volume "on the mass shell", i.e. when the constraint (_P.P)2 = m is

imposed?

Problem 3.34. A group of N particles is seen to occupy a volume



dxdydzdp dpY dPz in 6-dimensional phase space, so that the number
22 PROBLEM BOOK IN RELATIVITY AND GRAVITATION

density of particles in phase space n is given by

N =
n dx dydz dp x dpY dPz .

Show that n is a Lorentz invariant, Le. that all observers will compute

the same numerical value for n.

Problem 3.35. A vector field Ja(xll) satisfies


Jaa


0 and Ja falls off
2 3
faster than r- at large distances from the origin. (a) Show that fJOd x


is constant in time.

(b) Show that the integral is a scalar, Le. fJOd x =

rJO d 3x'.
CHAPTER 4

ELECTROMAGNETISM

The electromagnetic field is described relativistically by the anti-

symmetric electromagnetic field tensor (Maxwell tensor) FIlV. In any

Lorentz frame the components of Fllv are related to the electric and

magnetic field strengths, and in that frame


, by


0 EX EY E

_Ex 0 a -BY
Fllv =


-EY _B 0 aX
z x
_E BY _B 0

Here 11 is the row index and v the column index. Maxwell's equations

can be written
FIlV
,v

411 Jil

F + F + F =

a{3,y ya,{3 {3y,a

where Jil =
(p, J) is the 4-current density. The Lorentz force law is

d pll / dr =
eFllv Uv

for a particle of charge e, 4-momentum p and 4-velocity u.

2 2
The energy density 5; =
(E + B )/817, the Poynting energy flux S =

and the 3-dimensional stress-tensor


( x B )/417,

Tij =
[_ (EiEj + BiBj) + !

oij (E 2 + B 2 )]/417

are combined to form the electromagnetic stress-energy tensor

TIlV =
( F/lU- FVa _! TJllv Fa {3 F ) /417
4" a{3'

23
24 PROBLEM BOOK IN RELATIVITY AND GRAVITATION

Problem 4.1. Find the magnetic field from a current I in an infinitely


long straight wire, by appropriate Lorentz transformations and superposi-

tions of the electric field of an infinitely long straight charge distribution.


2 2
Problem 4.2. For electric and magnetic fields, show that B -

E and






are invariant under changes of coordinates and Lorentz transforma-

tions. Are there any invariants which are not merely algebraic combina-
tions of these two?

Problem 4.3. A particular electromagnetic field has its E field at an

angle eo to its field, and eo is invariant to all observers. What is

the value of eo?


Problem 4.4. Show that &2 -

1 12is a Lorentz invariant of the electro-

magnetic field, where & is the energy density and the Poynting flux.

Problem 4.5. Prove that except when ( '!:i (B +


2 -

E2)2 =
0, there is
a Lorentz transformation which will make E and.--..
B parallel (E'x B" 0). =

[Hint: Try v
--

aCE x B) for
--
some a.]

Problem 4.6. Suppose that B


. O.

Show that there is a Lorentz trans-
2 2
formation which makes =
0 if B -

E > 0, or one that makes B =



2 2 2 2
if B -
E < O. What if B -
E =
0 in addition to E. B -

O?

Problem 4.7. A collection of charged particles of charges e has


3-velocities v.
-1
and trajectories x


z.
-1 (t). The 4-current has components
JO =
L e.

o3[x -z.(t)]; Ji
-1
= L e
k vio3[x -z k (t)].
- -
Show that this can be

f e k o4[x 4-velocity of

written JI1 =
L -
za k(r)] u dr where u is the

particle k.

Problem 4.8. Show by explicit examination of components that the

equations
Fa{3,y

F{3y,a

Fya,/3


FU {3417 JU =

reduce to Maxwell's equations:


4. ELECTROMAGNETISM 25

V.B=O
- -

B + V xE
...... ...........-.-..

0, V. E .......... .......

417 P ,


x B = -
417 J .


Problem 4.9. If F/l is the electromagnetic tensor, show that Maxwell's
equations in vacuum can be written as F/l v =
0 and *F/l


O. [Here,
,v ,v
V V
*F/l is the dual of F/l ; see Problem 3.25.]

Problem 4.10. Write out the /l =


0 component of the Lorentz force equa-

tion dull Idr =


(elm) FILf3 u{3 expressing FILV in terms of Ei and
Bi'
to obtain

dp /dt =



Problem 4.11. From the spatial components of the Lorentz 4-force equa-

tion, find an equation for df/dt in terms of E and B. (Here P is the

spatial part of P).

Problem 4.12. A particle of charge q and mass m is coasting through


the lab with velocity v when
x it encounters a constant field in the

y-direction. Find y(x), the shape of the particle's subsequent motion.

Problem 4.13. A particle of charge q, mass m, moves in a circular

orbit of radius R in a uniform B field Be. (a) Find B in terms of


-z

R, q, m and w, the angular frequency. (b) The speed of the particle is

constant since the B field can do no work on the particle. An observer

moving at velocity x
, however, does not see the speed as constant.


fJ_
0' 0'
What is u measured by this observer? (c) Calculate du /dr and thus


dp /dr. Explain how the energy of the particle can change since the B

field does no work on it.

Problem 4.14. A small test particle (mass m, positive charge q) makes

circular orbits around a "fixed" (Le. very massive) body of positive

charge Q. A uniform magnetic field ]2 perpendicular to the orbital plane


serves to keep the particle in orbit. In the inertial frame in which the

central body is at rest, the test charge is seen to circle in the plane per-

pendicular to the B field with an angular frequency w. What is the

charge to mass ratio of the test particle in terms of w, R, B, Q?


26 PROBLEM BOOK IN RELATIVITY AND GRAVITATION

Problem 4.15. Show that the stress-energy tensor for the electromagnetic

field is divergenceless (Le. Tllv v =


0) in the absence of charge sources.

Problem 4.16. Show that the stress-energy tensor for the electromagnetic

fie ld has zero trace.

Problem 4.17. If TIlV is the stress-energy tensor of the electromagnetic

field, show that

Til





all
v [(E2_B2)2 + (2E.B)2]/(817)2
__

Problem 4.18. Write Ohm's law



= a E invariantly in terms of JIl, Fllv,
a and u ll (the 4-velocity of the conducting element).

Problem 4.19. Derive the Lorentz force law for a charged particle from the

action r Jil A d x -

m r d T, where Jil is the 4-current, A the vector


11 11
potential, and d T

== dJf dx{3
T/a{3

Problem 4.20.

(a) Show that E -+ -


Band B -+ E under the "duality transformation"
F -+ *F.

(b) Show that if F is a solution of the free-s pace Maxwell equations,

so is *F and also e*aF == F cosa + *F sina for arbitrary a. (F -+ e*aF


is called a "duality rotation".)

Problem 4.21. If one believes that esthetics should be an important con-

sideration in physical laws, then by symmetry Maxwell's laws should read

417 J 11

FIlV =

,v

*FIlV v =
417 Kil .

What would the significance of K be?

Problem 4.22. In Minkowski spacetime, there is an electromagnetic



current JIl(X ). Show that the solution to Maxwell's equation is
4. ELECTROMAGNETISM 27

[IL v] d 4 x -


r J
FILV (xa) =
4.
171 2
(r a ra )

where r/3 == x13 -


x 13 .
(Start by finding a Green's function of 0 AIL =


417 JIL.) How are the retarded boundary conditions specified?

Problem 4.23. Find the equation for the convective time rate of change

of a magnetic field which is "frozen in" a perfectly conducting fluid, in

terms of the expansion, shear, and rotation of the fluid. (See Problem 5.18
for definitions of these quantities.)
CHAPTER 5

MATTER AND RADIATION

A proper description of the energy, momentum and stress of a relati-

vistic fluid or field uses the symmetric tensor T, the stress-energy tensor

(also called the energy-momentum tensor). The components of this tensor

in the Lorentz frame of an observer are related to the measurements made

by that observer in the following way:

TOO ==
density of mass-energy (often denoted p).
jO
TOj =
T ==
j-component of momentum-density

j-component of energy-flux
XX
Tij ==
components of the ordinary stress tensor (e.g. T =
x-component
of pressure).

If TIlV describes all fields, fluids, particles etc. present in a system,


the interrelat'ion of momentum flow and energy change is summarized by the

equations of motion:
TIlV v =
0 .

The basic concepts of relativistic thermodynamics and hydrodynamics

which follow from this are developed in the problems.


With a view to developments later in the book, several problems in this

chapter use covariant differentiation, denoted by a semicolon. The reader

not yet familiar with this may replace all semicolons by commas (partial
differentiation in Minkowski coordinates). Also, the V notation is intro-

duced; e.g. VS for Sa. ,Vf for f


'fJ' a'
V. T for TIlV. v '

etc.

28
5. MATTER AND RADIATION
29

Problem 5.1. Calculate the nonzero components in an inertial frame S of

the stress-energy tensor for the following systems:


(a) A group of particles all moving with the same velocity =
{3 ex as

seen in S. Let the rest-mass density of these particles be as


Po
measured in their comoving frame. Assume a high density of particles and
treat them in the continuum approximation.
(b) A ring of N similar particles of mass m rotating counter-

clockwise in the x-y plane about some point fixed in S at a radius a

and angular velocity cu. (The width of the ring is much less than a.) Do
not include the stress-energy of whatever forces keep them in orbit. Assume
N is large enough that one can treat the particles as being continuously
distributed.
(c) Two such rings of particles, one rotating clockwise, the other
counter-clockwise, at the same radius a. The particles do not collide or

interact with each other in any way.

Problem 5.2. What is the stress energy of a gas with a proper number

density (i.e. number density as measured in the local rest frame of the gas)

N of noninteracting particles of mass m, if the particles all have the

same speed v but move isotropically? (Do not assume v << c.)

Problem 5.3. In the rest frame of a perfect fluid its stress energy tensor,

in terms of mass-energy density p and pressure p, is the diagonal tensor

p 0


TlLv =


o p

If a fluid element of proper density and pressure, p and p is moving

with 4-velocity u, what is its stress-energy?

Problem 5.4. Find the stress-energy tensor for a uniform magnetic field.
What is the average stress-energy if the B field is static but "chaotic"

Le. the direction of the B field varies, and is isotropic on the average?
30 PROBLEM BOOK IN RELATIVITY AND GRAVITATION

Problem 5.5. A rod has cross sectional area A and mass per unit length
/l. Write down the stress-energy tensor inside the rod when the rod is

under a tension F. (Assume that the tension is uniformly distributed


over the cross section.)

Problem 5.6. A rope of mass per unit length /l has a static breaking
strength F. What is the maximum F can be without violating the' 'weak"

energy condition that TOO should be positive to all observers? How close

is steel cable to this theoretical maximum strength?

Problem 5.7. An infinitesimally thin rod of length 2a has a point mass m

at each of its ends. The center of the rod is fixed in the laboratory and
the rod rotates about this point with a relativistic angular velocity cu.

(i.e. cue is comparable with c). Assume the rod is massless. What is

T/l for the rod and particle system?

Problem 5.8. A parallel plate capacitor consists of two large plates of

area A, perpendicular to the x-direction, separated by a small distance d.

The capacitor is charged so that a uniform electric field of magnitude E

is present between the plates; fringe effects at the edge of the plate can


be neglected. The "electrostatic mass" of this capacitor is E Ad/81T

in the rest frame of the capacitor. Show that the electrostatic energy is

smaller if the capacitor is moving in the x-direction! Consider now that

the plates must be held apart. Let the plates be held apart by an ideal

gas of proper density Show that the total energy (electrostatic + gas)
PO.
of the capacitor increases with velocity in the x-direction in precisely

the same manner that the energy of a point mass does.

Problem 5.9. Consider a system of discrete particles of charge qi and


mass m.

interacting through electromagnetic forces. From the explicit
V V
expression for T/l of the particles show that the total T/l (particles

plus field) is conserved, Le. that T/l v

O.

5. MATTER AND RADIATION 31

Problem 5.10. The specific intensity Iv of radiation measures the in-

tensity of radiation at a particular frequency v in a particular direction.


It is defined as the flux per unit frequency interval, per unit solid angle.

Show that Iv/v is a Lorentz invariant.

Problem 5.11. A star emits radiation isotropically in its own rest frame,
with luminosity L (energy per unit time). At a particular instant, as

measured from the earth, the star is at a distance R, and is moving with

a velocity v which makes an angle () with respect to the direction from

the earth to the star. What is the flux of radiation (energy per unit time

per unit area) seen by an observer on the earth in terms of R, v and ()

evaluated at the instant the radiation was emitted?

Problem 5.12. Consider a spherical particle of mass m which scatters

all electromagnetic radiation incident on it, isotropically in its rest frame.

Let A be the effective cross sectional area of the particle. Find the
equation of motion of the particle in a constant radiation field of intensity

S (energy per time per area), and solve it for the case of a particle initially
at rest. (Poynting-Robertson effect).

Problem 5.13. A thermally-conducting black sphere with a thermometer

attached moves with velocity v through a black body radiation field of

temperature To' What does the thermometer read?


Problem 5.14. In an electron gas of temperature T << c /k a photon
me
of energy E <<
mec2 undergoes collisions and is Compton scattered.

Show that, to lowest order in E and T the average energy lost by a

photon in a collision is

<dE> =
(E/m e c )(E-4kT) .

Problem 5.15. Show that in special relativity the stress-energy of an

isolated physical system of finite extent obeys the tensor virial theorem

f !.


Tij d3x =
TOOxixjd3x .


dt2
32 PROBLEM BOOK IN RELATIVITY AND GRAVITATION


Problem 5.16. Show that the stress-energy tensor T/l has a timelike

eigenvector if and only if there is a physical observer who sees no net

energy flux in any direction. What is the significance of the eigenvalue?

Problem 5.17.

(a) Consider a stressed medium which moves through a particular


inertial frame with velocity I I< < 1. Show that to first order in the

velocity, the spatial components of the momentum density are

jk
gj =
m vk ,

where m
jk the "inertial mass unit volume" is
, per

JOk 0'0' ok o'k'


m =
T oj + TJ

, ,

in terms of T/l v, the components of the stress-energy in the rest frame


jk
of the medium. What is m for a perfect fluid?
(b) Consider an isolated, stressed body at rest and in equilibrium

{30 laboratory frame. Show that its total inertial



(T ,

0) in the mass,

defined by


Mij ij dx
_
m dy dz

stressed
body

is isotropic and equals the rest mass of the body, Le. show that

Mij = Sij
J TOO dxdydz .

Problem 5.18. If u is the 4-velocity of a fluid show that Vu can be

decomposed as
U w + u +
()Paf3- a u

a ;f3 a f3 a f3 a f3'

where a is the "4-acceleration" of the fluid

:; u uf3
aa a
;f3 ,

() is the "expansion" of the fluid world lines


5. MATTER AND RADIATION
33


() == V. u =
u '

,a

W is the "rotation 2-form" of the fluid, and is the "shear



{3 aa{3

tensor'

f3
==
(ua;/lp/lf3 uf3;/lp/la)


a == !. ( u p/l + u p/l) - !. () p
af'J 2 a;/l f'J f'J;/l a 3 a,...,.

Here P is the projection tensor

P + u
u{3
==
a{3 ga{3 a

which projects a vector onto the 3-surface perpendicular to u.

Problem 5.19. Write the first law of thermodynamics for a relativistic

fluid. (i.e. Write the law of conservation of mass-energy for a fluid

element.)


Problem 5.20. Use the equations of motion (T/l .




0) to show that the

flow of a perfect fluid is isentropic.

Problem 5.21. For a perfect fluid with equation of state p



pen) (where
n =
baryon density) show that T/l /l' the trace of the stress-energy tensor

is negative if and only if


d log p/d log n < 4/3 .

Problem 5.22. Show that the velocity of sound v in a relativistic per-


fect fluid is given by 2





ap/apl s=constant

For a high temperature relativistic gas with an equation of state p 3P

(essentially that for a photon gas) show that v


s 1/V3.

Problem 5.23. The velocity of sound in a fluid is V


ap/ap\ s = cons tanto

Show that v; =
r P /(P + p) where 1
I 1
is the adiabatic index

11 =
a log pia log n\ .

s = constant
34 PROBLEM BOOK IN RELATIVITY AND GRAVITATION

Problem 5.24. What is the speed of sound in an ideal Fermi gas at zero

temperature?

Problem 5.25. A relativistic wind tunnel is to be fed from a tank of perfect

adiabatic compressed gas. Suppose the gas has an equation of state p oc n Y

with y constant, and the speed of sound in the tank is a. What is the

largest wind velocity v


rnax
which can be obtained? (No gravitational

forces; isentropic flow.)

Problem 5.26. An idealized description of heat flow in a fluid uses the

heat flux 4-vector q with components in the fluid rest frame qO =


0,
qj =
(energy per unit time crossing a unit surface perpendicular to e

, In

the positive j direction). What is the stress-energy tensor associated

with the heat flow?

Problem 5.27. Let s, nand q be respectively entropy per baryon,

number density of baryons, and heat flux, all measured in the proper

frame of the fluid. In this proper frame q is purely spatial. Let S be

the entropy density-flux 4-vector. Show that

S =
nsu + q/T ,

where u is the 4-velocity of the fluid rest frame.

Problem 5.28. A fluid is "perfect" except for admitting some heat con-

duction, described by a heat flow 4-vector q. Calculate the local rate of

entropy generation V. S.

Problem 5.29. In a uniformly accelerating system, show that the condition


for thermal equilibrium is not T =
constant =

To but rather is

T T 0 exp

(- )

where x is coordinate position in the accelerating frame.


Problem 5.30. The stress energy tensor of a viscous fluid is



{3 =
pu

u{3 + p pa{3 _

21J dl {3 _

,() pa{3 .
5. MA TTER AND RADIATION 35

Here 11 and , are res pective ly the coefficients of shear and bulk
viscosity. The definitions of cP{3, (), p

{3 are those of Problem 5.18.

The pressure and density are p and p. Show that the viscous terms

lead to the production of entropy at a rate

;a ?(, (j2
= + 2TJ aa{3 cP(3)/T

where T is the temperature of the fluid. (Hint: First show that


Sa'a,

[dp/dr + ()(p + p)]/T for a fluid without heat flow, then differentiate pu{3 =




{3u a
to get dp/dr.)

Problem 5.31. From the stress-energy


a {3 =
pu

u{3 + pp {3

_

211 dl {3 _

,()pa{3 ,

show that the equations of motion derived from


{3,{3 0 =
reduce to the

Navier-Stokes equations in the nonrelativistic limit.

Problem 5.32. As in non-relativistic thermodynamics, one defines the

specific heat of a gas at constant volume and constant pressure by

ds




In c



dT p I ・

For a perfect Maxwell-Boltzmann gas, show that c






+ k. (Here k =

Boltzmann's constant.) Show that the adiabatic index

a log p
r1 =

a log n

is equal to the ratio of specific heats,

Y ==
cp/c v

Problem 5.33. For a perfect Maxwell-Boltzmann gas, show that if y is

approximately constant in some regime of interest, then p



Kn Y and



mn + Kn Y /(y-1) under adiabatic conditions. (K =
constant, m =
mass

of partic les.)
36 PROBLEM BOOK OF RELATIVITY AND GRAVITATION

Problem 5.34. The invariant equilibrium distribution function of a relati-


vistic gas is 3
j((pU, xu) ==
dN =
(21_+ 1)/h
[ p.o ()

d 3x d 3p exp - - -

] -

kT

Here 1 =
spin of particles, h =
Planck's constant, 0 =
mean
4-velocity
of gas, E =
1, 0 or -1 for Bose-Einstein, Maxwell-Boltzmann or Fermi-

Dirac statistics respectively. The parameter () is independent of P.

The first two moments of n are

d 3P d3P
Jil ==

J npil
(-p.o)
, TIlV ==

J np/LpJ/
(-p.o)

Since u is the only free vector, these integrals must have the form

JI1 =
null, TIlV =
(p + p) ullU

+ pg
IlV .

(This is the kinetic-theory definition of n, p, p.)


(a) Obtain I-dimensional integrals for n, p and p.

(b) Show that dp =


(p + p)/T dT + nk Td ().

(c) Use the first law of thermodynamics to identify kT () as the chemi-

cal potential Il =
(p + p)1 n -

Ts.

(d) Show that for a Maxwell-Boltzmann gas, p =


nkT for all T.

(e) Show that for a Maxwell-Boltzmann gas, p =
n(m+ 2 kT) is an

approximation valid only for kT < < m. Find the exact expression


for pin. What is pin for kT? m? (Here m is the mass of a

gas partic Ie. )

Problem 5.35. For a perfect Maxwell-Boltzmann gas, find y(T), the

ratio of specific heats, as a function of temperature.


CHAPTER 6

METRICS

Metric geometry, geometry specified by 2


a distance formula ds =

d x f3 ,

gaf3
dx is the foundation for general relativity and for most of the

remaining chapters in this book. The most important metric is of course a

spacetime metric, formally a metric which can locally be transformed to

the Minkowski metric, Le. for every point P in the spacetime, there is
some coordinate transformation which makes at P.
ga{3 l1a{3

o o o o o o o

Problem 6.1.

(a) Prove that the 2-dimensional metric s pace described by

2 2 2 2
ds =
dv ...
v du (1)

is just the flat 2-dimensional Minkowski space usually described by

2 2 2
ds =
dx -

dt .
(2)

Do this by finding the coordinate transformations x(v, u) and t(v, u)


which take the metric (2) into the form (1).
(b) For an unaccelerated particle, show that the component of the

4-momentum P is constant, but that P is not.


u v

Problem 6.2. Show that the line element


ds =
R2[da 2 + sin 2 a (d &2 + sin 2 0 d <1>2)]

represents a hypersphere of radius R in Euclidean 4-space, Le. a locus

of points a distance R from a given point.

37
38 PROBLEM BOOK IN REl,A TIVITY AND GRAVITA TION

Problem 6.3. The metric for the surface of a globe of the Earth is

2 2 2 2
ds =
a (d'\ + cos ,\ dcp2)
where ,\ is the latitude and cp is the longitude. The metric of a flat
2 2
map of the world, with Cartesian coordinates x and y is ds =
dx + dy2;
however we are not interested in this geometry, but in that of the globe it

represents. What is the metric of the globe expressed in x and y coordi-

nates for (a) a cylindrical projection, and (b) a stereographic projection


map of the world?

Problem 6.4. Mercator's projection is defined as follows: The map

coordinates are rectangular Cartesian coordinates (x, y) such that a

straight line on the map is a line of constant compass bearing on the globe.

(a) Show that the map is defined by x =


cP, y

log cot 0, where

(0, cp) are the polar coordinates of a point on the globe.

(b) What is the metric of the globe in (x, y) coordinates?

(c) Show that the great circles are given by sinh y =


a sin (x + (3)
(except for the special cases y

0 or x =
constant).

Problem 6.5. A space purports to be 3-dimensional, with coordinates

x, y, z and the metric

\2

2 2 2 2 3 4 12
ds dx + dy + dz dx + dy + d
13 Z;
= -

13 13

Show that it is really a two-dimensional space, and find two new coordi-

nates ( and 11 for which the line element takes the form

2 2
ds =
d(2 +
d11 .

Problem 6.6. Show jhat a contraction of a vector V with the "projection

tensor" P ==
g + u u projects V into the 3-surface orthogonal to the

4-velocity vector u. If n is a unit spacelike vector show that

P==g-n n
6. METRICS 39

is the corresponding projection operator. Show that there is no unique


projection operator orthogonal to a null vector.

Problem 6.7. Show that a conformal transformation of a metric, Le.


ga(3-)
f(x/l) ga(3 for an arbitrary function f, preserves all angles. (Figure out

how to define angles!) Show that all null curves remain null curves.

Problem 6.8. One can put a metric on the velocity space of a particle by
defining the distance between two nearby velocities as their relative

velocity. Show that the metric can be written in the form

2 2 2 2
ds =
dX2 + sinh x (d0 + sin 0dcp2) .

where the magnitude of the velocity is v =


tanh X.

Problem 6.9. A manifold which has the topology of a 2-sphere has -


in a

neighborhood of 0 =
, X

0 -

the metric


ds =
d0

+ (0 _

(3)2 dX2 .

The manifold has precisely one point which is not locally flat, and that
point is a "conical" singularity. Show that there are two different maxi-

mal analytic extensions of the metric, Le. that there are two different ways

of extending the metric, and satisfying the condition that there is only one

conical singularity. Note that this shows that a metric in a local coordi-

nate patch does not always pin down the global nature of the manifold.

(Hint: consider the periodicity of the X coordinate.)

Problem 6.10. Find the most general form for a spacetime metric that is

(spatially) spherically symmetric.


CHAPTER 7

COVARIANT DIFFERENTIATION AND GEODESIC CURVES

The partial derivatives of a vector or tensor with respect to the coordi-

nates of a space (e.g. Afl,v or


QU{3"'yB... ,v) are not themselves com-

ponents of a tensor. Rather, the curvilinearity of the coordinates (optional


in flat space, but inevitable in a curved space) must be taken into account,

leading to the idea of covariant differentiation.

The tensor formed by differentiating a tensor Q with components

QU{3"'yB... is denoted VQ and has components denoted

Qaf3... yo Qaf3... yo. Q vf3... yo



== + r
;a a va
. ・ . . ・ . . ・

r f3 av. v
Qaf3.
・ . . .


va Q yo ・ ..
+ ...
_

ya va ・ . .

_r

af3...
oa Q yv...

where there is one "correction" term for every index of Q. The r's are

called Christoffel symbols or [affine] connection coefficients. In a coordi-

basis they are related to the partial derivatives of the metric by

a all r 1 all (
r +
g g
g(3y,1l )
- -

(3y ll(3y gll(3 ,y glly,(3


- - -

(the first equality defines r ll(3y). The r's are sets of numbers, but

they are not components of a tensor.


(They do not transform like a tensor.)
A covariant derivative which is dotted into a vector is called a direc-

tional derivative:
(V Q) Qa(3... yo''''v U


u == V Q == .

If the vector u is tangent to a curve parameterized by A, one sometimes

a a
writes u =
d/dA for u =
dx IdA and

40
7. COVARIANT DIFFERENTIATION AND GEODESIC CURVES 41

VuQ == ・

If the vector happens to be a basis vector, one writes

Ve Q ==
VaQ .

In terms of the basis vectors, the connection coefficients can be written

rfl.a
V{3e or
r/Kl{3 ell V{3e
= .

{3efl.
= .

a a

The covariant derivative operator V obeys all of the nice rules expected
of a derivative operator, except that in curved space VuV v
1= VvV u
(see
Chapter 9).
If u is the tangent vector to a curve, a tensor Q is said to be

parallel propagated along the curve if



Q = 0 .

If the tangent vector is itself parallel propagated,

Vu u =

(tangent vector "covariantly constant") the curve is a geodesic, the


generalization of a straight line in flat space. If xa(A) is the geodesic
a a
(with u =
dx /cL\) then the components of this geodesic equation are

dufl. a
u{3r ll
o =
( V u u)fl. =
+ u
{3

dA a

Here A must be an affine parameter along the curve; for non-null curves

this means A must be proportional to proper length.

If a curve is timelike, u is its tangent vector, and a ==


Vuu =
Du/dT,
then a vector V is said to be Fermi-Walker transported along u if

V V


(u a -
a u). V .
42 PROBLEM BOOK IN RELATIVITY AND GRAVITATION

Problem 7.1. Show that the connection coefficients r


a{3y do not obey
the tensor transformation law.

Problem 7.2. For a 2-dimensional flat, Euclidean s pace described by

polar coordinates r, 0, assume that the geodesics are the usual straight

lines.

Find the connection coefficients
{3y' using
(a) r your knowledge of

these geodesics, and the geodesic equation

2 IL
d x

dx

dx{3 rlL = 0
a{3

ds ds
ds2

(b) Next, in the coordinates x, y which are related to r, 0 in the


x x
usual wa y , take the covariant structure to be given by r = r ='.' = o.
xx xy

Using the transformation law for connection coefficients find the connec-

tion coefficients in the r, 0 coordinates.


2 2 2 2
(c) Finally, from the line element ds =
dr + r d0 find the

Christoffel symbols, in the usual way, as derivatives of the metric coeffi-

cients g
I1 V

(All three methods, of course, must give the same Christoffel

symbols.)

Problem 7.3. Consider the familiar metric space

2 2 2 2
ds =
dr + r d0 .

(a) Write the 2 equations that res ult from the geodesic equation, and

show that the following are first integrals of these equations:

2 dO

Ro constant
= =

ds

( / ( y:

+ r =
1 .

(b) Use the results in (a) to get a first-order differential equation for
reO). [That is: Eliminate s as a parameter and replace by 0].

(c) Using the fact that the metric space is just flat 2-dimensional
7. COVARIANT DIFFERENTIATION AND GEODESIC CURVES
43

Euclidean space, write down the general equation for a straight line in

r, () coordinates, and show that the straight line satisfies the equation
in (b).

2 2 2 2
Problem 7.4. For the 2-dimensional metric ds =
(dx -

dt )/t , find all

connection coefficients r and find all timelike geodesic curves.


af3y ,

Problem 7.5. Show that the metric tensor is covariantly constant.

Problem 7.6. For a diagonal metric, prove that (in a coordinate frame) the

Christoffel symbols are given by

rll \ = 0 r ll,\,\ = - -
1 ag,\X


Vl\
2gllll axil

glLlL I) t)
a ...L (log (I
rlL
pA

ax

(log (1
gIL1L1)t), rlL
ILIL

axil

Here 111= v 1=,\ and there is no summation over repeated indices.

Problem 7.7. Prove the following indentities:


(a) r +
rf3ay

f3,y af3y

(b)
lLgf{3,y glL{3 IL'Y = -

(c) ga{3,y = _


gll{3 _
r{3 gila.
IlY IlY

(d) g
,a
= -
gg
tJy,a
gf3Y =
gg{3y g(3 y,a .


(e) r f3(log IgI2),f3 in a coordinate frame.

(f) gllVr all,v = - l (gavi g\ t) in a coordinate frame.


,v
Iglt
(g) A

= l (Ig\ t A

) ,a in a coordinate frame.
,a

\ gI t
(h) A f3. = l (I gl t A (3) -
rA AXil in a coordinate frame.
a,fJ a,fJ all
1 g\ t

(I gl t A {3
{3;{3 (3),{3
a a
(i) A


in a coordinate frame, if A is

antisymmetric.
1 gl 2

(j) 0S =
S. ;a = -L (I gl t ga{3s )in a coordinate frame.
,a ,a
Iglt
,fJ
44 PROBLEM BOOK IN RELATIVITY AND GRAVITATION

Problem 7.8. Let A == det (A ) where A is a second rank tensor.


l1v l1v
Show that A is not a scalar. (Le. Show that its value changes under

coordinate transformations.) Since A is not a scalar one cannot define

A. =
A . How should A be defined (in terms of A and A)?
,a ,a, a ,a

Problem 7.9. If a geodesic is timelike at a given point P, show that it


is timelike everywhere along its length, and similarly for spacelike or

null, geodesics.

Problem 7.10. Derive the geodesic equation from the definition of a

geodesic as a curve of extremal length.

Problem 7.11. An affine parameter ,\ is one for which the equation of

geodesic motion has the form

dxa
+ r
a dx{3 d.x Y =

2 {3y

d,\
d,\ d,\

Show that all affine parameters are related by linear transformations with

constant coefficients.

Problem 7.12. Show that in flat spacetime, the conservation law for the

4-momentum of a freely moving particle can be written V


pP

O. Show

that particles with nonzero rest mass move along timelike geodesics.

Problem 7.13. Suppose the coordinate xl is a cyclic coordinate, Le. the

metric functions are independent of xl. If p is the momentum of


ru{3
an unaccelerated particle, show that the component is constant along
Pl
the particle world line.

Problem 7.14. Prove the general relativity version of Fermat's principle:

In any static metric


(gOj

(3,
0 0), consider all null

curves between

two points in space, x


j =

j and x
j =
bj . Each such null curve xj(t)
particular coordinate time tget from
to
j to bj Show
requires a a .

that the curves of extremal time t


are null geodesics of the spacetime.
7. COVARIANT DIFFERENTIATION AND GEODESIC CURVES 45

Problem 7.15.

(a) Show that the geodesics of the velocity space metric defined in

Problem 6.8 are paths of minimum fuel use for a rocket ship changing its

velocity.

(b) A rocket ship in interstellar space with velocity


.Yl (with respect
to the earth) changes its velocity to a new velocity Y2' in a manner that
uses up the least fuel. What is the ship's smalles t velocity relative to
earth during the change?

2 2 2
Problem 7.16. On the surface of a two-sphere, ds =
d0 + sin 0 dcp2,
the vector A is equal to
ee
at e =

eo' cp =
O. What is A after it is

parallel transported around the circle e=

eo? What is the magnitude of A?

Problem 7.17. Consider an observer with 4-velocity u who transports his

four basis vectors e


a along with him according to the trans port law

Vue a

Aa{3e{3' What is the most general form of Aa{3 if:

(i) the basis vectors are to be orthonormal?

(ii) in addition e" =
u? (Le. the frame is his rest-frame).

(iii) in addition the spatial vectors are to be nonrotating? (Le. He

sees a freely-falling particle move with no Coriolis forces.)

Problem 7.18. Show that the scalar product of two vectors is not altered

as they are both Fermi-Walker transported along a curve e.

Problem 7.19. Show that Fermi-Walker transport along a geodesic curve

is the same as parallel transport.

Problem 7.20. Write the following expressions in index-free notation:

(a) Ua ;{3 u{3UZ (b)


va;{3 u{3 UZ;{3 v{3

Ta{3;y vaw{3u
(c) Y
(d) wa;{3 V{3;y uy

uy u{3 Y u{3 UZ;{3 w{3;y uy
(e) W +
wa;y U

;y{3

;{3

Problem 7.21. Show that the paths of light rays in astatic, isotropic

spacetime can be described by taking the space to have a certain spatially


46 PROBLEM BOOK IN RELATIVITY AND GRAVITATION


varying "index of refraction" What is of Assume
n(x ). n in terms
f3?
g has the form 2 2
a(3 2 2 2
ds goodt f(dx l 3 ).
= -

+ dX + dx

Problem 7.22. An inebriated astronaut pulses his rocket, firing in a

random direction for each pulse. As measured in a momentarily comoving


frame, each pulse corresponds to a velocity boost of !1v < < c. Find the

probability distribution of his resultant velocity after n boosts, where n

is a very large number. Show that the drunk astronaut achieves highly
relativistic velocities less efficiently than a sober astronaut (who fires

his rocket always in the same direction), and takes on the average 3c/!1v

times as many pulses to achieve the same velocity.

Problem 7.23.
(a) Suppose a vector field k is orthogonal to a family of hypers urfaces
("hypersurface-orthogonal"). Show that this implies k[/l;V k,\]

O.

(b) What is the geometric interpretation if also vanishes?


k[/l;v]
Problem 7.24. Prove that any congruence of null curves that is hypersurface-

orthogonal consists of null geodesics.

Problem 7.25. Show that the variational principle


J (gaf3 x a x (3 ) ds =

gives the same geodesics as the defining property for geodesics



J (gaf3 x

x (3 ) ds = 0

when s is proper length (not



an arbitrary parameterization) and x ==

(gaf3 x x(3)"2,
a a
dx Ids. If y == show that


J F(y)ds =

gives the same geodesics for any monotonic function F(y).


CHAPTER 8

DIFFERENTIAL GEOMETRY: FURTHER CONCEPTS

A vector B is related to its contravariant components BI1 by

B =
Bile ,
11

where the basis vectors. The covariant components


ell
are
BI1 repre-

sent the same vector, but represent it as a different type of "vector",


called a one-form. (Loosely, one-forms are often called "covariant

vectors".) For one-forms the analog of the above equation is

B =
B w l1 ,
11

where ,...., indicates a one-form and (;,11 are basis one-forms with covariant

components (1, 0, 0, 0), (0, 1, 0, 0), etc. For an arbitrary tensor T, with
com ponents T
yo. . .

a(3.

. .

YO... -a (3 0...
_

T =
T CA) 0(,) 0e
e '.'
(3

a... y u

The scalar product of two vectors, or of two one-forms, involves the

metric tensor, and is denoted by a "dot":


A.B =
g AI1B
I1V


A.B =
gl1 A Bv
11


The scalar product of
(Here gl1 gl1v ')
is the matrix inverse of a vector

with a one-form does not involve the metric, only a summation over an

index. This is sometimes distinguished notationally as

- -

B.A ==
<B,A> ==
BI1AI1 .

47
48 PROBLEM BOOK IN REJ.JATIVITY AND GRAVITATION

Since <(;,11, e V > = all


v'
the basis w ll is said to be "dual" to the basis
- -

If B and A are one-forms corresponding to vectors B and A


ell' we

have, of course, A.B = A.1i =


< Ii, A> <A, B>. =

--

A one-form of particular usefulness is df, the gradient of any scalar

function f. When combined with a vector v, it gives the directional

derivative of f along v.

< df, v> = V f =
f va .

v ,a

The basis vectors e


a corresponding to a coordinate system are

tangent to the coordinate lines. This motivates the notation for a coordi-

nate basis vector



e =

a a
ax

Similarly, the coordinate basis one-forms are gradients of the coordinate


surfaces, -


--


w == dx .

Alternatively, since a spacetime metric can locally be transformed to

the Minkowski metric, it is always possible to find a set of orthonormal

basis vectors and one-forms at every point. These are not necessarily '"

tangents to or gradients of the coordinates; they are denoted bye"" (;,11,


11
'"

where indicates orthonormality. Note that the relations <w , e > =

{3
fP and c;,a. c;,f3 =
gaf3 always hold. When the basis is
ef3 f3'

f3' ea'
orthonormal, then in addition and gaf3 af3 If a local
f3TJa f3 TJ
= =

orthonormal frame in spacetime is freely falling (i.e. the basis is that of a

freely falling observer), then all the r


a{3y vanish at the center of the

frame.

The commutator of two basis-vector fields


e{3
V e =
[e a ,
e{3] == c
{3Y ey

e e a a
a {3

0, if and only if
vanishes identically, c
{3Y e and e are the

a a {3
tangent vectors to come coordinate system (a coordinate basis). In a
8. DIFFERENTIAL GEOMETRY: FURTHER CONCEPTS 49

general (not necessarily a coordinate) basis,


p.' (Vye (3)
+c
==

(gp.{3, y + gp.y,(3
+c C
(3yp.)

g{3y ,p.

p.{3y p.y{3
- -

p.{3y

A tensor of rank p with all of its indices covariant, and which is

totally antisymmetric on all indices, is called a p-form. A totally anti-

symmetrized direct product of forms is denoted by a wedge ",,".


The concepts of Lie differentiation, Lie trans port, and exterior differ-

entiation of forms are developed in problems.

o o o o o o o

Problem 8.1.
a a
(a) A spacetime has coordinates x with basis vectors a/ax and

basis one-forms a.
What are the values of:

< o, o
a/ax >, < , 2
a/ax

>, (a/ax ). (a/ax ), d;0
o l .
J;1 d;0 d;0? ,

--1
(b) To what vector does the one-form dx correspond?

Problem 8.2. The usual basis for polar coordinates, e..... e, e .....
r- e
()
= =



..... ()

is not a coordinate basis. Consider the one-form basis wi dual to this

bas is .....

i s:-i
<w
ej>
= U .

..... --

df and prove that there does not



Find the function f such that w =

exist a function g such that r::,O dg. =


Do this without imposing any

metric on the polar coordinates.

Problem 8.3. In 3-dimensional Euclidean space, what is a necessary and

sufficient condition on a fie ld of one-forms Go for there to exist a function


f such that a =
df?

Problem 8.4. If Ql IS a p-form and Q2 a q-form, show that

QI AQ 2 (-1)PQQ2AQl
= .
50 PROBLEM BOOK IN RELATIVITY AND GRAVITATION

Problem 8.5. The exterior derivative of a differential form g can be

defined axiomatically by the follow ing properties:

(i) If Q. is a p-form, dQ is a (p+1)-form;


(ii) d(Ql +Q2) =
dQl + dQ 2;
"'"'

(iii) For a zero form f (scalar), df is defined by < df, v> =

Vvf for

any v;

(iv) d(Ql AQ2) =

dQl AQ2 + (-l) PQ l AdQ2 where Ql is a p-form;


[Note: If p

0 (Le. f a scalar) this reads: d(fQ) =
df A g + f dQ.]

(v) ddQ =
0 for any Q,.

An alternative definition notes that a p-form is a completely antisymmetric


covariant tensor of rank p, and defines the exterior derivative as the com-

pletely antisymmetrized covariant derivative. Show that these definitions

are equivalent.

Problem 8.6. Consider a 2-form in n-dimensional space:

-...


-...

2 n 1 2
a =
f(x ,

,.", x ) dx A dx .

Suppose for some region of space including xl =


0, that

da =
0 .

Construct the I-form

If( xl
[XI.[ xn)d ]

/3 =
,x ,"', J;2

and show that a =


dp.

Problem 8.7. The components of the Maxwell tensor F can be regarded


a{3
as the components of a 2-form F. Show that Maxwell's equations in

vacuum can be written dF =


0, d*F =
O.

Problem 8.8. A 3-surface in spacetime is spacelike, timelike, or null, if


its normal vector is timelike, spacelike, or null respectively. It is desired

to find three orthogonal linearly independent vectors in a 3-surface. Show

that for a spacelike surface, these are all spacelike; for a timelike surface,
8. DIFFERENTIAL GEOMETRY: FURTHER CONCEPTS 51

two spacelike and one timelike; for a null surface, one null and two

spacelike.

Problem 8.9. Show that the integral


Is F/ld3IW where F/l is some

vector field and S is some oriented 3-dimensional hypersurface in space-

time, is independent of the parametrization x J1 =


xfJ.(a, b, c) used to

describe S. [See Problem 3.30 for the definition of d
1fJ..]
Problem 8.10. [Note: This problem assumes a familiarity with the Cartan
calculus, beyond the scope of most relativity texts. Succeeding problems
do not depend on it.] In the language of differential forms the generalized
Stokes' theorem is
r d9 = r 9
In

Jao
To what does this theorem reduce, in the following cases

(a) 0 IS 3-dimensional, 6 =
fkd 2S k
(b) 0 is 4-dimensional, 6 =
f J1d
31:fJ. .

FJ1Vd2fJ.V

(c) 0 IS 3-dimensional, 6 = where FfJ. is antisymmetric.
(d) Use the generalized Stokes theorem to derive the familiar relation

fl:.' i f ('
e =
'I) x 1:.) .

is ・

Problem 8.11. Show that there exists no tensor with components con-

structed from the 10 metric coefficients and their 40 first derivatives


ga{3
ga except g itself and products of it with itself, e.g. g g.
{3 ,J1

Problem 8.12.

(a) Show that, in a coordinate frame, r is symmetric on the last



{3y
two indices.

(b) Show that in an orthonormal frame, r is antisymmetric on the


a {3y
first two indices.

Problem 8.13. The Lie derivative of a scalar function is defined to be

the directional derivative: xfV xf.



For a vector field y, we define
52 PROBLEM BOOK IN RELATIVITY AND GRAVITATION

Lie differentiation as
xY[x,y]
== =
V y



yx.
The Lie derivative obeys

all the usual rules for a derivative operator and always gives a tensor of

the same rank as the tens or differentiated.

(a) What is the Lie derivative of a 1-form?

(b) What is the Lie derivative of a tensor whose components are


{3?
Problem 8.14. Suppose A =
d/dA is the tangent vector field to a con-

gruence (set of curves) XU =


xa(A) and B is a vector field. Show that

f. AB is that B
the geometrical interpretation
of} the trans port law 0

connects points of equal A on


4eighboring curves of the congruence.

Problem 8.15. Show that Lie differentiation commutes with the operation
of contraction.

Problem 8.16. Show that

uv -

v =


[u,v] .

Problem 8.17. Another definition of the Lie derivative of a geometric

object (l)A[xl1(p)] (A represents all the tensor indices, xl1(P) are the

coordinates of a point P) is as follows: Make an infinites imal point

transformation Po --) P
N by xl1(P 0) =
xll(P N ) + gl1(P N ). (Since 11is
infinitesimal, it can in fact be evaluated at either Poor P
N .) Also make
an infinitesimal coordinate transformation that makes the numerical values

of the coordinates of P

the same as those of Po in the original
coordinates:
x l1 (P
N)

xl1(Po) .

Then define

A A-A
rI>
(Po) =
LIM [rI> (Po) -<I> (P N )] .

e--)o

Show that this definition is equivalent to that of Problem 8.13 by examining

<l>A (Hi) <l>A


the cases (i) <l>A =
a scalar field (ii) =

Afl


/.
8. DIFFERENTIAL GEOMETRY: FURTHER CONCEPTS 53

Problem 8.18. It is desired to


transport a vector v along a curve with

tangent vector u. Which of the following are necessary for parallel

transport? for Fermi-Walker transport? -


for Lie transport?: metric;
affine connection; u(x) defined off the curve.

Problem 8.19. You are given a vector field vi =


(-y, x, za), a =
constant,
2 2 2
in a 3-space with metric ds =
dx + dy2 + dz . A certain vector
!!.
is Lie transported along from a point A to a point B and is then
parallel transported back to A by the reverse route. For what value of a

is there a
.! which is always left unchanged by this process?

Problem 8.20. Find the most general vector field which is everywhere

parallel-propagated along itself. Fermi-Walker transported along itself.

Lie transported along itself.

Problem 8.21. If Q is a p-form, show that fx(dQ) d(fxQ). =

Problem 8.22. Vector analysis in 3-dimensional orthogonal curvilinear

coordinates is a special case of tensor analysis where


gij

h a ij (not

summed). The h.'s



are functions of the coordinates called "scale

factors". Vector components are often referred to the ("physical") ortho-


'"

normal basis c:,i =

hid;i (not summed). Derive expressions for (i) V S,



(ii) V x V, (iii) V. V and (iv) V S, where S is
- -
a scalar field and V

a vector field.

Problem 8.23. Derive expressions for .


and x in spherical polar
coordinates.

Problem 8.24. If F prove that O.


AV;1l AIl;v' F[ll ;A]
= - =

IlV v

Problem 8.25. In an arbitrary spacetime manifold (not necessarily homoge-


neous or isotropic), pick an initial spacelike hypersurface SI' place an

2 3
arbitrary coordinate grid (xl ,x ,x ) on it, eject geodesic world lines
2 3
orthogonal to it, and give these world lines the coordinates (xl ,x ,x ) =

constant, xO == t =


+ T where T is proper time along the world line,
54 PROBLEM BOOK IN RELATIVITY AND GRAVITATION

beginning with T =
0 on
SID Show that in this coordinate system

("Gaussian normal coordinates") the metric takes on the synchronous

form
2 2 i
ds = -
dt + goo dx dx j .

lJ

Problem 8.26.

(a) If and are the components of two symmetric tensors,


g/lV g/lV
show that A -A A
S =
r -

/lV /lV /lV

are the components of a tensor. Here the r's and r's are Christoffel

symbols formed from the tensors g and g in the usual way.

(b) Suppose g/lV and g/lV have the same geodesics. Then show
that
S AII V
= oA 'II

+ oA v \II
r /l /l

where \II are the components of a vector.


/l

Problem 8.27. Compute the connection coefficients of the following metric


in an orthonormal frame

2 2
ds = -

2a
dt + e

/3dr 2 + e2Y(de2 + sin 2 e dcjJ2)
a, f3, y =
functions of rand t.

Problem 8.28. The redshift between two observers (with 4-velocities u


and be defined in two ways: (i) by the energies of


uB) can a photon
(4-momentum p) travelling along a null geodesic between them: 1 + z ==



p/U B ・

P, or (ii) by the proper time between two null geodesics,


emitted TA
apart and received apart, l+z==
TB TA/ TB'
Show that

these definitions are equivalent.


CHAPTER 9

CURVATURE

The study of curvature is based on the Riemann curvature tensor:

arll
{3 ar ll va
r ll Qr p

Ril =
+ r ll rp -

va{3 {3

pfJ va
ax{3 pa v

ax

in a coordinate frame. The covariant components of the Riemann tensor

are connected by several symmetries

Ra{3yo Ryoa{3' Ra{3yo R{3ayo


= = -


Ra{3yo Ra{3oy Ra[{3yo]
= - =

The (symmetric) Ricci tensor and the Ricci scalar are formed from the

Riemann tensor:
Rll
Ra{3
==
all {3


R==R .

The Weyl tensor,


gAK R IlV
C R R + R
(gAv RIlK gllv gllK Av )

AK
- -

AIlVK AllvK





(gAv gllK gAK gllv)

is also called the conformal tensor due to its invariance under conformal

transformations. It vanishes if and only if the metric is conformally flat

(i.e. reducible to Minkowski space by a conformal transformation).

The extrinsic curvature tensor of a hypersurface which has unit normal

o and which is s panned by basis vectors e.,



e.

...
is denoted K, with

components K.. = -
e. ・
V.o .

lJ J 1

55
56 PROBLEM BOOK IN RELATIVITY AND GRAVITATION

Problem 9.1. On a sphere of radius a


try to cons truct a local Cartes ian

system in two ways (a) from geodesics and (b) from the (orthogonal)
lines of longitud'e and latitude. Either way there will be deviations from

a good Cartesian system (e.g. the sum of "the angles in a coordinate box

will differ from 211, or the fractional difference in length between

"parallel" sides of coordinate box will not vanish).


a Show that such

deviations are of order [Area of coordinate patch/a ].

Problem 9.2. How many independent components does the Riemann tensor

have in n dimensions?

Problem 9.3. Mathematical manipulations with the Riemann tensor are



often done with computers. Rather than calculate and store the 4 256 =

components of the tensor as R(I,} ,K,L) with I,} ,K,L =


0,1,2,3, the sym-

metries of the Riemann tensor can be used to reduce the size of the stored

array. Design a subprogram which stores or recalls all components of

R(I,} ,K,L) in a linear array of dimensions ::; 21.

Problem 9.4. Compute all the nonvanishing components of the Riemann

tensor R
ijk1 (i,j,k,l

e,cp) for the 2-sphere metric

2 2 2 2
ds =
r (de + sin e dcp2) .

Problem 9.5. Find the Christoffel symbols and Riemann curvature com-

ponents for the two dimensional spacetime:

2 2 2 2
ds =
dv _
v du .

Problem 9.6. Set up a coordinate system on a torus (the 2-dimensional


surface of a doughnut in Euclidean 3-space). Calculate all components of

r/l and
g/lv' a{3 Ra{3yo'
Problem 9.7. In a space of fewer than 4 dimensions simple expressions
can be given for the Riemann tensor.

(a) What is the Riemann tensor in a I-dimensional space?


9. CURVATURE 57

(b) Express the Riemann tensor for a 2-dimensional space in terms

of the metric and the Ricci scalar.

(c) Express the Riemann tensor for a 3-dimensional space in terms

of the metric and the Ricci tensor.

Problem 9.8. Prove the relation

2V =
V -
V =
V Ra
a; [ VK ] a aVK

a;VK a;KV

and find its generalization to the commutator of second derivatives for a

tensor of arbitrary rank


Ta...(3....
Problem 9.9. Show that the second derivatives of a scalar field commute

compute
(Le. show that
S;a{3 S;(3a)' For third derivatives
S;a{3y'

and
S; (a{3)y S;a [f3y]-
Problem 9.10. Prove that for any second rank tensor

A fl

A f1
V -


;f1V ;Vf1

Problem 9.11. An infinitesimal circuit in the shape of a parallelogram can

be specified by the differential displacements u, v representing the sides


of the parallelogram. Let a vector A be parallel transported around this

circuit. (Le. Displace it successively by u, v, -u, -v.) Show that the

change in A due to the transport around this circuit is

oAa =

-Ra{3yoA{3uYvO .

Problem 9.12. Riemann curvature can also be computed with the Riemann

operator R
R(A,B)C =

(VAVB-VBVA-V[A,B])C
(a) Show that the value of R at a point P is linear in the arguments
their values at P and not the way in
A, B, C and depends only on on

which they vary around P.


58 PROBLEM BOOK IN RELATIVITY AND GRAVITATION

(b) Show that

,lAuC/l AA B

(R(A,B) C)a =


Problem 9.13. Two nearby geodesics have affine parameters such that

nearby points on the two geodesics have very close values of affine

parameter A. Let u == dxa/dA be the tangent to one of the geodesics,

arAu let n be the differential vector connecting points of equal affine

parameter on the two geodesics. Prove the equation of geodesic deviation

2 a
D n a
f3 nY u o
+ R u 0
f3yo


dA

Problem 9.14. In a suitable coordinate system the gravitational field of

the Earth is approximately (to lowest nontrivial order in M/r)

2 2
ds =
-(1-2M/r)dt + (1+2M/r)(dx2+dy2+dz2)


r ==
(x +y2+z2)2
M =
mass of Earth (c =
G =
1) .

Suppose a Skylab satellite orbits the Earth in a circular equatorial orbit.

What is the orbital period? An astronaut jettisons a bag of garbage into a

nearby orbit and watches it move relative to the satellite. At a given time
the separation of the Skylab and its garbage is described by the vector

gi == xi (garbage) _
xi (skylab) .

Using the equation of geodesic deviation, find the components of the rela-
tive motion gi as a function of time.

Problem 9.15. Prove the cyclic identity


Raf3yo + R
ao f3y +
Rayof3

and the Bianchi identities


Raof3y;v Raovf3;y + +
Raoyv;f3


9. CURVATURE
59

Problem 9.16. Show that Bianchi identities


imply that the Einstein tensor



G/l R/l
_

g/lV
= -

V V

has vanishing divergence (i.e. G/l V' =


0).
,/l

Problem 9.17. Show that the vanishing of the Riemann tensor is a suffi-

cient condition for a spacetime to be Minkowskian, i.e. a coordinate trans-

formation will bring into the form


g/lV 11
/lv'

Problem 9.18. A beam of light has a circular cross section at some point
along its path. Show that the beam experiences no shear (i.e. the cross-

section is not deformed into an ellipse) when the Weyl tensor is zero.

Problem 9.19. Compute the Riemann tensor, Ricci tensor, and scalar
curvature of the conformally-flat metric =
e 2CP where cP =
cp(x/l)
g/lV Tl/lV
is an arbitrary function.

Problem 9.20. Compute the Riemann tensor of the following metric in an

orthonormal frame:

2 2 2f3 dr2 e2Y(d02+sin20d<p2)


2a
ds =
_e dt + e +

a, f3, y =
functions of r, t .

What is the Ricci tensor for this metric? The scalar curvature? The

Einstein tensor?

Problem 9.21. Consider a Riemann tensor representing a plane gravita-


R where is "retarded time"
t onal
wave, i.e.
Raf3yo f3yo(u), u

(Vu. Vu=O). Find the number of independent components of such a Rie-

mann tensor. Do not assume that R satisfies the Einstein field



f3yo
equations.

Problem 9.22. At a given instant, the coordinate accelerations of n nearby


test particles are measured. What is the smallest n required to measure

all components of F/l ? of R/l
vpa?
60 PROBLEM BOOK IN RELATIVITY AND GRA VITA TION

Problem 9.23. Let A and B be two linearly independent vectors tangent

at a point to a two dimensional surface, in a space of dimension 2: 2. The

Riemannian curvature of the 2-s urface at that point is defined as


ay f38


Af3 BYB8
K =


A {3 B Y B


g(3y )

ga{3 gyo gao

Show that K is unchanged if A and B are replaced by linear combina-


tions of A and B.

Problem 9.24. Suppose K is the curvature at a point in a 2-dimensional

surface, as defined in Problem 9.23. If A and B are two vectors tangent


at a point to the two surface, and A is parallel transported around a small

circuit lying in the 2-surface, show that the change in the angle between

A and B is of magnitude
!1() =
IK!111

where !11 is the area enclosed by the circuit.

Problem 9.25. Suppose that the curvature K at a point P, as defined in

Problem 9.23, does not depend on the 2-surface which is chosen through

that point. Show then that

Ra
K{gay g(3o g(3y)

(3yo

Problem 9.26. If the Riemann curvature is isotropic, the Riemann curva-

ture tensor can be written as


K{gay g{3o gao g{3y)

(3yo

Use the Bianchi identities to show (Schur's theorem) that K must be a

constant.

Problem 9.27. Show that a space is conformally flat if the Riemann tensor

can be written as

RA/l VK

K(gAv g/lK -

gAK g/lv) ・
9. CURVATURE 61

Problem 9.28.' Suppose at a point q in a 3-surface I two curves are

tangent (i) CI a curve in the 3-surface and (ii) C a geodesic of the


4-dimensional space in which I is embedded. Let n be the unit

normal to I. The vector =


a u

u{3 (where u is the tangent
; {3
vector to
CI) measures the rate at which C and
CI separate. Show
that the rate of sep aration n' is

n .
= 1. K t:1. u

u f3
2 afJ

where is the extrinsic curvature tensor for


Ka{3 I.

Problem 9.29. What is the extrinsic curvature of the r = constant slice of


2 2 2 k i j
the metric, ds = -
dr + a ..(x ) dx
(r) [y lJ dx ]?

Problem 9.30. Prove that the extrinsic curvature of a time like hypersurface
with unit normal vector n is
nP where P n is
{3na {3
= -

a{3' a{3

the projection tensor into the hypersurface.

Problem 9.31. If we neglect gravity, the potential energy due to surface

tension of a soap film is proportional to its area. Thus in equilibrium a


62 PROBLEM BOOK IN RELATIVITY AND GRAVITA TION

soap film spanning a fixed closed wire loop will assume a shape of mini-
mum area. Show that this implies that the surface is one whose "mean

curvature" K == K .


is zero.

Problem 9.32. Let n be the unit normal to a hypersurface I, with


n.n == E =
+1 or -1 if I is time like or spacelike respectively. In
Gaussian normal coordinates (see Problem 8.25) based on I, the metric is

2 2
ds =
E dn + (3)g..lJ dx i dx j .

Derive the Gauss-Codazzi equations

(4) R m (3) R m m m

"k
lJ

lJ
"k

E(K lJ.,



1k



(4)R n

(Kikl j K
Ik)
= -

ijk

ij

Here 4 and 3 refer respectively to the spacetime geometry and to the

geometry of I; a s lash denotes covariant differentiation with res pect to

(3)g..;
lJ
the n index denotes the component on the n basis vector. Also

derive the equation for the remaining component of the Riemann tensor:

(4)R n

In k
= E (K'1 k , n + K.1m Km k )'

Problem 9.33. Using the results of Problem 9.32, derive expressions for

(4)G a
the components of the Einstein tensor, in Gaussian normal
{3'
coordinates.

Problem 9.34. The eigenvalues and eigenvectors of the extrinsic curva-

ture tensor are called the principal curvatures and principal directions.

Find the principal curvatures and directions for the following surfaces em-

bedded in a 3-dimensional Euclidean space.

(i) sphere: x

+ y2 + z2 =


2 2 2
(1 1 ) cy I Ind er:
' . .

x + y =
a .


(iii) quadratic surface (compute at origin only): z = (ax + 2bxy+ cl).
9. CURVATURE 63

Problem 9.35. Show that if I is a 2-dimensional surface in a flat

3-space, then the scalar curvature of I is


(2)R =

PI P 2

where Pl and are the principal radii of curvature of I. What is the


P2

analogous formula for a 3-surface embedded in a flat 4-space?


CHAPTER 10

KILLING VECTORS AND SYMMETRIES

Suppose that a geometry has a


symmetry such that a vector field

exists with the following property: If any set of points is displaced by

dA (dA a small number) then all distance relationships are unchanged.


The vector field is then called a Killing vector for the geometry, and it

satisfies Killing's equation

(a;{3) -

[ a;{3 (3;a] + =
0 ・

o o o o o o o

Problem 10.1. Solve Killing's equations to find the Killing vector fields

of the 2-s phere:



ds 2 =
d0 + sin

e dtp2 .

Problem 10.2. Show that Killing's equation


ga;f3 + gf3;a = 0 is equivalent

0, where g is the metric Interpret this result geometri-


to
fg tensor.

cally.

Problem 10.3.

(a) Show that the commutator of two Killing vector fields is a Killing
vector field.

(b) Show that a linear combination of Killing vectors with constant

coefficients is a Killing vector.

Problem 10.4. In Euclidean 3-space show that the 3 Killing vectors

describing rotations around the x, y, and z axes are linearly dependent


at any given point but that no constant coefficient combination of them is

zero. Show that the generators of the rotation group 0(3) are thus

2-surface forming, even though the group is 3-dimensional. Explain.

64
10. KILLING VECTORS AND SYMMETRIES 65

Problem 10.5. The metric for an axially symmetric rotating star admits

Killing vectors, (t)and there are no other indepen-


two
(<P)"Assume
dent Killing vectors. Prove that and commute.
(t) (cp)

Problem 10.6. Show that any Killing vector is a solution of the equation

V;A).. + R a
' =

0 .

Find a Lagrangian-type variational principle from which this equation can

be derived.

Problem 10.7. If is a Killing vector, prove that

gl1;a{3 Ry{3al1 gy
= ・

Problem 10.8. A metric is "stationary" if and only if it has a Killing


vector field g which is timelike at infinity (the "time" direction is

a/at). There are two ways to define a "static" metric:

(i) stationary and invariant under time reversal, a/at -

a/at, or

(ii) stationary and a/at is hypersurface orthogonal (see Problem


7.23). Show that the two definitions are equivalent.

Problem 10.9. In flat Minkowski spacetime find ten Killing vectors that

are linearly independent.

Problem 10.10. If (xl1) is a Killing vector field and u is the tangent

vector to a geodesic, show that u .is constant along the geodesic.

Problem 10.11. If is a Killing vector and T is the stress-energy

tensor, show that JI1 == Tl1v gv is a conserved quantity Le. JI1."




O.

Interpret J when is a time like Killing vector.

Problem 10.12. If T is the energy-momentum tensor, and is a time

Killing vector, show that the integral over a whole spacelike hypersurface

f F
-ra
f3 f3d3
Ia

is independent of the choice of s pacelike hypers urface F.


66 PROBLEM BOOK IN RELATIVITY AND GRA VITA TION

Problem 10.13. Given a divergenceless stress energy tensor in flat space-

time, Le.
T/lV ;v 0 0
Ral3yo
= =

show that one may construct ten global conservation laws and hence ten

conserved quantities.

Problem 10.14. If is a time like Killing vector and u =


/I \2" 'IS a

4-velocity, prove that a == V


UU

V log I 'I.

Problem 10.15. In a stationary metric with time Killing vector the

"energy at infinity" E = -


of a test particle with 4-momentum p IS

conserved. Find the minimum value of E//l (where /l



particle mass)
that the particle can have at a given point in the spacetime, in terms of

the norm of .

Problem 10.16. Show that a Killing vector is an admissible solution for

the vector potential of Maxwell's equations for a test field in a vacuum

spacetime. What elec tromagnetic field corresponds to the Killing vector

a/acp in Minkowski space?


CHAPTER 11

ANGULAR MOMENTUM

This chapter contains problems dealing with rotation, angular momen-

tum, spin, etc. in general relativity. Definitions are developed in the

problems.

o o o o o o o

II
Problem 11.1. In special

relativity, when a particle I

, \\
is at an event B and has

4-momentum p, its angular A

\\
momentum about event A

,I
IS
J =
f1x p-p f1x II
....

where f1x is the 4-vector I::. x


from event A to event B

(i) Show that for a

freely moving (i.e. /8


",,/
unaccelerated) /

particle J is con- /'"



served -
i.e. dJ/dr =
O. /

(ii J Suppose that x

several particles "

collide at an event B, thereby producing several other particles.


Show that the sum of the angular momenta of particles
he about

an event A is the same after the collision as before:

67
68 PROBLEM BOOK IN RELATIVITY AND GRAVITATION

I J=
(k) after IJ
(k) before

(k) (k)

Problem 11.2. Show

(a) that the total angular momentum of an isolated system in flat


space 3
Jaf3 == d x (x

Tf3 0 -

x f3 rzo)

is a conserved tensor (when rz f3 ,f3 =


0), but
a a
(b) that it is not invariant under the coordinate translation x x + aa.

Show also that

(c) the spin 4-vector defined by

1 a
S E s:- J f-I y U
_

- -

a 2 af-lYu

is both conserved and



(d) invariant under translations. Here u is the "center of mass

pa 1(- pf3 p
2:

pa is the total momentum pa
(3)
==
4-velocity" u == and


{d x TaD.

Problem 11.3. Show that a system's intrinsic spin 4-vector Sa is orthog-



onal to its 4-velocity u .

Problem 11.4. Show that a gyroscope with no applied torques Fermi-

Walker trans ports its s pin vector.

Problem 11.5.

(a) If angular momentum is computed about the center of mass of a

system show that Jaf3uf3 =


O.

(b) In this case show that the angular momentum ("intrinsic angular

momentum' ') can be found from the s pin vector as

Jafj(C.M.) safj == = -
Eafjyo S us:- .

Y U

Problem 11.6. Two bodies A and B have momenta PA and PB and

their centers of collision After


spin SA and SB; mass are on a course.
11. ANGULAR MOMENTUM 69

colliding they stick together to form a composite body C (spin Sc)'


Calculate
Sc in terms of P
A'

B ,
SA and
SB'
Problem 11.7. Thomas Precession: Consider a
("classical") spinning
electron which Fermi-Walker transports its spin angular
momentum, S, as

it moves in a circular orbit around an atomic nucleus. As seen in the

laboratory frame, the electron moves in a circular orbit of radius r in the

x-y plane with constant angular velocity, (u. Calculate Set), the s pin as

a function of laboratory time.

Problem 11.8. A nonspherical spinning body in an inhomogeneous gravita-


tional field experiences a torque which causes its intrinsic spin 4-vector
S to change with time. If u is the 4-velocity of the center of mass of

the object, freely moving along a geodesic, show that

DS K EK{3aJl u a A

u u t Rl1
{311 aaA

dr p.

Here is the "reduced t ij j


t{3113 quadrupole moment tensor" =
r p(xix -

o ij taf3 u


r )d x in the rest frame of the center of mass, =
0 and the

Riemann tensor is generated externally to the body in question and is

assumed to be approximately constant over the body.

Problem 11.9. Calculate the period of precession of the Earth's axis due

to the coupling of tidal forces from the sun and moon with the quadrupole

moment of the (s lightly nons pherical) Earth.

Problem 11.10. Consider a family of stationary observers in a stationary

spacetime, Le. their 4-velocity is proportional to the time Killing vector


.ach observer arranges his spatial basis vectors so that they connect

to the same neighboring observers for all time t, where =


(1) Show that .£ e",




0, where e

is a basis vector of the stationary

observer.

(2) Show that the rate of change of the components of any tensor quantity

Q as measured by the stationary observer in units of t, is


70 PROBLEM BOOK IN RELATIVITY AND GRAVITATION

・ ..

fja (() a ・ ・ ・

S rJsi" fj

(c:L Q) Wh at IS"

= ・

dt
dt

(3) The stationary observer carries a gyroscope with him applying no

torques to it. Show that the gyroscope's spin vector precesses (Lens-

Thirring effect) with an angular velocity relative to the stationary


observer, as meas ured in units of proper time,

avaA
E S
vs a;A
(lla =

2 Y

(4) Show that (ll = 0 if the spacetime is static, and not merely stationary.

Problem 11.11. A gyroscope is placed in a circular orbit about the Earth

and no torques are applied to it. What is the angular velocity of preces-

sion for its spin vector relative to a reference frame fixed with respect to

the distant stars? ("geodetic precession" and "Lens-Thirring effect".)


CHAPTER 12

GRAVITATION GENERALLY

This chapter contains problems dealing with the physical consequences

of gravitational interactions. Mos t of the problems use the Newtonian

limit, in which gravity is represented by a scalar potential U satisfying


v u = -
41TP

and generating a gravitational acceleration

g =
Vu .

(Alternatively, one sometimes uses a potential (I) == -

U.) Tidal forces


2 i k
depend on a <I>/ax ax in Newtonian theory; these forces appear as the

R terms of the Riemann tensor in the appropriate limit of the equation


jOkO
of geodesic deviation in general relativity. Some of the problems explore
consequences of gravity's spin-2 nature, and of its weakness in comparison

to other fields.

o o o o o o o

Problem 12.1. A small satellite has a circular frequency CtJ in an orbit of

radius r about a central object of mass m. From the known value of CtJ

show that it is possible to determine neither r nor m individually, but



only the effective "Kepler density" 3m/41T r of the object as averaged
over a sphere of the same radius as the orbit. Give the formula for CtJ2
in terms of this Kepler density.

Problem 12.2. Estimate the height of spring tides and neap tides.

71
72 PROBLEM BOOK IN RELATIVITY AND GRAVITATION

Problem 12.3. If the amplitude of solid-earth tides as a function of time

is fourier transformed, there are peaks at certain frequencies. What are

the frequencies (or periods) of the 10 strongest peaks?

Problem 12.4. The position of the sun in the sky can in principle be
measured by a sensitive tidal gravimeter. What is the angular difference
between this position and its position as measured optically? If the actual
pos ition of the s un were at its optical pos ition, there would be a force in

the direction -of the earth's motion. (Why?) If this the


'were case find the

radius of the earth's orbit as a function of time.

Problem 12.5. The "Eddington limit" for the luminosity of a star of mass

M is defined as the luminosity at which outward light pressure just

balances inward gravitational force everywhere within the star. Calculate

the luminos ity .


(You may ass ume that all matter is fully ionized hydrogen.)

Problem 12.6. Show that an electron does not fall down when released in

the center of an evacuated perfectly conducting closed container in a uni-

form gravitational field. ("Perfect conductor" ==


perfectly rigid positive
lattice with perfectly mobile conduction electrons.)

Problem 12.7. A tall, cylindrical, insulated bottle of height h is filled



with air at 300 K. It is then sealed and set on a scale at sea level. The

scale reads a weight W. For what range of h does the weight W


decrease as the contents of the bottle are slowly heated.

Problem 12.8. Define a stress tensor for the Newtonian gravitational

potential U as follows:

1 1
T O

Jk
== (U
4 7T,J,




a .



J,n
U,n).

Show that the Newtonian equations of motion for stressed matter with

proper density Po and velocity can be written in the form


12. GRAVITATION GENERALLY
73

dV
j a
Po dt
- = - -
(T'J k +t'J k )
ax k

(T jk 0
(Po Vj),t + + t
jk

Po V j vk),k

where t k is the ordinary 3-dimens ional s tress tens or.


Problem 12.9. Consider an extended body of mass M with several forces

f'i acting on it. Using the equivalence of gravitational mass and energy

show that the condition for equilibrium of the body is

l !\ (1- '.!/c2) = -

M!
where g is the acceleration of gravity and x.
-1
denotes the point of appli-

cation of each of the forces, meas ured in a local Lorentz frame.

Problem 12.10. From the result of Problem 12.9 show that the equation of

hydrostatic equilibrium in a star is

dp GM(r)
dr
= _


(p + p/c2)

where M(r) is defined as the "active" mass interior to the fluid shell at

radius r. This shows that in a fluid the "effective inertial mass" density

is P + p/c . Notice that this result does not depend on the field equations

of general relativity.

Problem 12.11. Show that the Newtonian equation of motion of a test parti-
cle in a Newtonian gravitational potential (I) can be written as a geodesic
equation in 4-dimensional spacetime. Compute the Christoffel symbols
and the Riemann tensor, and show that they are not derivable from a metric.

Problem 12.12. By examining the relative acceleration of a family of test-

particle trajectories in Newtonian gravity and comparing with the Newtonian

limit of the equation of geodesic deviation, derive the correspondence

a 2 (I)
R =

jOkO
Jxjax k
74 PROBLEM BOOK IN RELATIVITY AND GRAVITATION

between the Newtonian potential and the Riemann tensor. (A Newtonian


test particle is a on only by gravity;
ted a test particle in a relativistic

theory of gravity follows a geodesic.)

Problem 12.13. Write a Newtonian gravitational force law in covariant

4-dimensionallanguage, using a scalar field as the universal Newtonian

time function. Show that the resulting theory is consistent with special

relativity. Show that signals can be sent faster than the speed of light.

Is the theory acausal, Le. can an observer send signals into his own past?

Problem 12.14. Consider two particles of equal mass in a freely falling


elevator. One carries a charge q and the other is neutral. There is a

vertical electrical field E in the elevator. Write an equation for the

separation of the particles as a function of time, including tidal and

electrical effects. Reconcile the existence of both terms with the equiva-

lence principle.

Problem 12.15. New particles of mass are created which carry a new
mO
kind of charge, evidenced by a classical inverse-square force law. A con-

tainer holds a perfect monatomic gas of these particles in thermal equilibi-


rum. The total charge Q in the container is measured by the average

force on test particles outside, and it is observed to vary with temperature

What is the spin of the force field?


as Q ex
Qo(l + 6kT Im o )' new

Problem 12.16. Show (nonrigorously) that gravity is the only classical,

infinite range (massless quanta) pure spin-2 field; Le. that any other field

would couple identically to bulk matter and would thus be indistinguishable


from gravity.

Problem 12.17. In "geometrized units of length" (units in which the

gravitational constant G, the s peed of light c and the Boltzmann con-

stant k are all taken to be unity) give the values of the following,
expressed in terms of centimeters: t; the charge of an electron; elm for

an electron; the mass of the sun; the luminosity of the sun; 300 K; one

year; one volt.


12. GRAVITATION GENERALLY
75

Problem 12.18. Form "natural" units of mass, length, and time out of the

physical constants iI, G, and c.

Problem 12.19. Estimate the Bohr radius of a "gravitational atom", e.g.

two neutrons bound by their gravitational attraction in their lowest energy

state.
CHAPTER 13

GRAVITATIONAL FIELD EQUATIONS AND LINEARIZED THEORY

The gravitational field, described by the metric of spacetime is


g/lV'

generated by the stress-energy T/l of matter. Various field equations

relating to T/lv have been proposed. The most successful to date


g/lV
are the Einstein equations which are the foundation of general relativity:

R 817T
G/l R/l
;:
(1)
g/lV

V V
/lV

where and R the Ricci tensor and scalar curvature derived from
R/l
are

the metric and is the Einstein tensor. The equations


g/lV' G/l V
are non-

linear, since the left hand s ide is not a linear function of the metric.

Some other (less successful) field equations are discussed in the

problems, but unless specifically stated, the Einstein field equations are

to be assumed.

The equations of motion T/l .

0 are a consequence of Equation (1).



Other desirable properties of T/l , called "energy conditions", must be

independently postulated on phys ical grounds.


When the gravitational field is weak, the geometry of spacetime is

nearly flat and one writes


g/lV l1/lv

h/l

1. In this Equation (1) be solved


where all
\h/lvi are < < case can

approximately, by keeping only first-order terms in h . A number of


/lV
problems make use of this "linearized theory."

o o o o o o o

Problem 13.1. A somewhat generalized form of the Einstein field equa-

tions is

76
13. GRAVITATIONAL FIELD EQUATIONS 77

R 81TT
R/l ag/lV


/lV

where a is some dimensionless constant. Show that if a is not !.



the field equations disagree with experiment, even in the Newtonian limit.

Problem 13.2. A metric theory (devised by Nordstrom in 1913) relates

to T by the equations
g/lV /lV


C/lvpa

T /l


Kg/lV


where C is the Weyl tensor. Show that this theory, in the Newtonian

limit and with the proper choice of K, agrees with Newtonian gravitation

theory, but that this theory predicts no deflection of starlight passing near

the Sun. Does this theory agree with the Pound-Rebka experiments, Le.

are photons redshifted as they rise against the gravitational pull near the

s urfa ce of the Earth?

Problem 13.3. In the Brans-Dicke theory of gravity (see MTW p. 1070 or

Weinberg p. 160 for field equations), the locally measured Newtonian

gravitational constant G varies with position and time. Its value at

infinity is Goo' Show that G is a constant inside a self-gravitating



spherical shell of mass M and circumference 21TR. If R > > G oo M/c ,

express G inside the shell in terms of Goo' M, and R, to lowest order



in (G ooM/Rc ).

Problem 13.4. In relativistic quantum mechanics empty space contains

virtual particles. It is speculated that the vacuum therefore, has a non-

zero stress-energy.

(1) What form must the vacuum stress-energy tensor take, if there is to be
no preferred vacuum frame? Show that there is a resulting term in the

field equations which can be interpreted as an effective cosmological

constant.

(2) Suppose that the vacuum energy is due to the rest mass of virtual
78 PROBLEM BOOK IN RELATIVITY AND GRAVITATION

protons or electrons, produced with an average spacing of their Comp-


ton wavelength. Is such vacuum stress-energy ruled out by observa-
tions?

(3) Va. B. Zel'dovich has suggested that the mass-energy density should

be associated only with the gravitational interaction energy of nearby

virtual particles (separated by their Compton wavelength). What is

the predicted magnitude of the vacuum stress energy here? Is it ruled

out by observation?

Problem 13.5. In a local region of spacetime, an observer finds that the



Ricci curvature scalar is nearly constant, R + 1/a .
Why will the

sign to be "+"? If the region of spacetime is filled only with electromag-


netic energy what is R?

Problem 13.6. Normally it is assumed that a physically possible Tllv


must satisfy the (weak) energy condition, TOO 0 for all physical ob-

servers. Assume that Tllv has a timelike eigenvector; how can a given
satisfies the
single observer determine whether the Tllv he measures

condition?

Problem 13.7. The "dominant energy condition" on Tllv requires that

the weak energy be


cond tion satisfied (all observers see a nonnegative

energy density) and furthermore, that all observers see energy density

magnitude of the energy-flux 3-vector. Show that


greater or equal to the

the statement

(-T)n ・
u 0

for all nons pacelike vectors u reduces to the weak energy condition for

n =
1 and to the dominant energy condition for n = 2. What about n > 2?


[Here (T ) V == T


av
and so forth.]
Il 11

Problem 13.8. Is it possible to have a solution of the Einstein field

equations in which space is empty to the past of


some surface of constant

the future of
0, but in which there is a nonvanishing T v
to
time t =

Il
this surface?
13. GRAVITATIONAL FIELD EQUATIONS 79

Problem 13.9. A static metric is generated by a perfect fluid. Show that


the fluid 4-velocity is parallel to the time Killing vector.

Problem 13.10. At each point on an initial Cauchy hypersurface how many

numbers must be specified, to determine uniquely the evolution of the


metric field above that hypersurface. [Hint: First show that only spatial

components of the Einstein tensor contain second time derivatives of the

metric. ]

Problem 13.11. For the "nearly Newtonian" metric

2 2 k
ds = -
(1 + 2<1? dt + (1- 2<1?0 dx j dx
jk

calculate, to lowest nonvanishing order in <1>, the components of the


Landau-Lifschitz stress-energy pseudotensor tc:!!L (Landau and Lifschitz
p. 306). Assume that the field is changing so slowly in time that time

derivatives of <I> can be neglected compared to spatial derivatives.

Problem 13.12. A gauge transformation is an infinitesimal coordinate

transformation which relabels the coordinates of a point P according to

xl1
new
(P) =
xl1old (P) + l1(p) .

Such transformations induce changes, to first order in of the functional


forms of tensors. Find the gauge transformation laws for scalars, and com-

ponents of vectors and second rank tensors. For the linearized metric

perturbations g/lv

TJ
I1 V
+ h
l1V
show in particular that

h (x) =

hW d(x)

(p..v)

Problem 13.13. Show that in linearized theory the components of the

Riemann tensor are

(hav
Rap.l3v +
hp.13 ,va hp.v,al3 hal3,p.v)
= - -

,p.13

Also show explicitly that this Riemann tensor is invariant under a gauge

trans formati on.


80 PROBLEM BOOK IN RELATIVITY AND GRAVITATION

Problem 13.14. In linearized theory one often uses the "trace reversed"

form of the metric perturbations == h h


n h

Show that there - ! ・

afJ afJ 2 "afJ a


always exists a gauge transformation to the "Lorentz gauge", in which

the divergence of h vanishes. Is this gauge transformation unique?


a{3
Problem 13.15. Show that in the Lorentz gauge (see Problem 13.14) the

linearized field equations reduce to

,a
o h == h = -
167T T /I '
Il V Il v ,a rv

where is trace-reversed h
h/L v llV

Problem 13.16. In linearized theory a plane gravitational wave propagating


in empty spacetime can be represented as the real part of a complex ex-

press ion '1
1 x

_


h Re
[AIlVe

llV

where A is a constant tensor. Show that k must be a null vector,


IlV
and that A is orthogonal to k.

For a particular observer with 4-velocity u the "transverse-traceless"

gauge (a further specialization of the Lorentz gauge) is defined in the




observer's unperturbed rest frame (u =
1, u =
0) such that

h =
0 ' h ll =
0 .

Jl 0 ll

Find the gauge transformation which accomplishes this. Does A remain

orthogonal to k?

Problem 13.17. Show that in linearized theory there is no attractive

gravitational force between two thin parallel beams of light.

Problem 13.18. A rigid spherical shell of radius R and total mass M

(distributed uniformly on the shell), with negligible thickness, rotates

slowly with constant angular velocity n with respect to inertial frames

far away. Use the linearized equations of gravity to determine (il, the
13. GRAVITATIONAL FIELD EQUATIONS 81

angular velocity of dragging of inertial frames inside the shell, to first

order in OR < < 1. Show that

gorp 4 MO
(j) _
_ =


+ (')(02 R 2) '

gCPCP 3

which is constant everywhere inside the shell. [The constancy of w in

the cavity has been interpreted by some to mean that Einstein's equations

satisfy Mach's principle to some degree.]

Problem 13.19. In the linearized gravitational theory, show that the equa-

tions of motion for matter TIlV. v =


0 are inconsistent with the field equa-

tions for the metric perturbation. Show that this inconsistency is of

second order in the metric perturbation, hence negligible to first order.

Problem 13.20. A hypothetical particle of negative gravitational mass -


is released from rest at a distance e >> M from another fixed particle of

equal positive mass + M. As seen by a static observer, what is the magni-

tude and direction of the acceleration of each particle? Calculate the


motion of the particles after they are released, making any reasonable

approximations necessary.
CHAPTER 14

PHYSICS IN CURVED SPACETIME

This chapter deals with the generalization of the laws of special-

relativistic physics (e.g. hydrodynamics, electrodynamics) to curved

spacetime. Often this generalization involves only the replacement of

partial differentiation by covariant differentiation (' 'comma-goes-to-


semicolon rule"); for example the generalization of the equations of

motion is from Tllv v =


0 to TIlV. v =
0; this latter equation, with semi-
, ,

colons, includes the effects of gravity.

o o o o o o o

Problem 14.1. Write the stress-energy tensor for a single free particle,
and show that the equation of geodesic motion follows from TIlV. v =
O.

Problem 14.2. Show that the condition for therma I equilibrium of a static

system in general relativity is

T(-goo)2 =
constant

(where T is the temperature measured by a local static observer) rather

than the Newtonian relation T =


constant.

Problem 14.3. Derive the Euler equation


(p+p)Vuu =

-[Vp+(Vup)u]
from TlLv. v


0, and show that this equation has the correct Newtonian
limit.

Problem 14.4. Derive the general relativistic equation of hydrostatic

equilibrium, -ap a


(p+ p) ---


log (_goO)2
ax ax

and compare it to the Newtonian equation.

82
14. PHYSICS IN CURVED SPACETIME
83

Problem 14.5. Show that a highly relativistic fluid (p = 1. p) in hydro-



static equilibrium in a gravitational field can never have a free surface

(Le. a surface where p 0).

Problem 14.6. Two identical containers in a static uniform gravitational

field contain different substances. The containers and contents have

identical time-independent densities of mass-energy, but different (and

possibly anisotropic) stresses. Do the containers weigh the same if

weighed on the same scale?

Problem 14.7. If u is the 4-velocity of a perfect gas undergoing adiabatic


stationary flow in a stationary gravitational field, prove (the relativistic
Bernoulli equation) that along the flow lines




constant x n/(p+p) ,

where n =
baryon number density.

Problem 14.8. Show that the relativistic Bernoulli equation of Problem 14.7
reduces to the correct Newtonian limit for slow velocities and weak gravi-
tational fie Ids.

Problem 14.9. Consider a moving medium with four-velveity u(x). Choose


two arbitary neighboring particles A and B. At each event along the
world line of A define the four-vector separation of B from A, f,as
follows: (i) is an infinitesimal vector from a given event on A's

world line to B's world line; (ii) has vanishing time component,


0, relative to the orthonormal tetrad carried by A.

(a) Define the motion of the medium to be a rigid-body motion if and


only if the distance, ( I between


.)"2, any two neighboring particles -

e.g., A and B above -

is constant for all time. Show that a medium

undergoes rigid body motion if and only if u =


0 and () =
0, where
a {3
u and () are the quantities defined in Problem 5.18.

{3
(b) How many independent equations do these conditions constitute?

How many degrees of freedom are there in relativistic rigid-body motion?


84 PROBLEM BOOK IN RELATIVITY AND GRAVITATION

Problem 14.10. If 0 is the expansion of a fluid, derive the Raychaudhuri


equation
dO a
2 ill
2 2 1 2 a

- =
a. + -

2U - -

0 -
R Q U u fJ
dr ,a 3 a fJ

af3
aaf3,p.f3,
2 2
where w =
w w a

and the notation is the same
a f3 ,

as in Problem 5.18.

Problem 14.11. In a certain spacetime fluid flows along geodesics with


zero shear and expansion. (See Problem 5.18 for the definitions of shear

and expansion 0.) Show that the spacetime has a timelike Killing
ua{j
vector.

Problem 14.12. An observer in a closed box (not necessarily in free fall)

measures positions and times inside his box with rulers and a clock. Show

that the equation of motion for a particle, correct to first order in its

1 and position j is
measured velocity v < < x


dv /dt =

2<y'x )j [ -

x )x(!xd]j/dt)j

. .


aJ(l+!' ) RJ OkOx -

Here ill is the angular velocity of the box, its acceleration, and Rj OkO
the Riemann tensor evaluated at the origin.

Problem 14.13. The stress-energy tensor of a massless scalar field is

taken to be

p.v

(471)-1 (<I> ,/1<1> ,v -

gp.v
ell
,a
<I> ,a) ・

Derive the equation of motion of this scalar field from


Tllv;v =
O.

Problem 14.14. The equation for a scalar field, in flat spacetime, is

,v
(I) =
ps where ps is the density of "scalar charge." We are tempted
,v
to conclude that in curved spacetime the equation should be

m '.V
'¥
;v

Ps (1)

but another possible generalization is


14. PHYSICS IN CURVED SPACETIME
85

cI> ;v - 1. R (I) =
Ps (2)
;v 6

where R is the Ricci scalar. (Equation (2) is "conformally invariant"

while (1) is not.) Does Equation (2), in principle, violate the strong

equivalence principle? Find the influence of the R term on the force



(-., V(I)) between two "scalar charged" particles, assuming that R =
1/a
varies very slowly on a laboratory scale. In a practical laboratory experi-
ment what would be the magnitude of such anomalous R-term forces
compared to the ordinary scalar forces?

Problem 14.15. Show that Maxwell's equations Ff.l v .


4TTJf.l, imply
,v

Jf.l;/L O.

Problem 14.16. The generalization of Maxwell's equations to curved

spacetime by the "comma-goes-to-semicolon" rule (or principle of equiva-

lence) is not completely unambiguous. Show that the use of this rule with

the vector potential Af.l can lead to two different results for a relativistic

equation.

Problem 14.17. Estimate the fractional error introduced into Maxwell's

equations (applied to some earthbound process of characteristic frequency

v and size f) by our ignorance of what curvature coupling terms there

may be.

Problem 14.18. Show that except in the case =


0,.the sourceless

Maxwell equations F{3v.v ,



0 follow from the requirements V. T =
0,
where T is the electromagnetic stress-energy tensor, and from the fact

that Ff.l is derived from a vector potential.

Problem 14.19. Show that gp.v is a Hamilton-



(TTp.-eAp.)(TTv-eA)

ian giving the equations of motion of a test particle of charge e. Here

TT is the canonical momentum. (The canonical momentum TTf.l is not


f.l
equal to p/L, the particles 4-momentum unless the 4-potential Af.l is

zero. )
86 PROBLEM BOOK IN RELATIVITY AND GRAVITATION

Problem 14.20. Suppose is a Killing vector for a solution of the

Einstein-Maxwell equations. Write down an integral of the motion for


charged test particles. (Assume
fA 0 where A is the 4-potential.)

Problem 14.21. Show that Maxwell's equations are invariant under the

"conformal transformation"

ga{3 ga{3

fga {3

F F =

a{3 a{3 a{3
-2
J /1 ->
J /1 =
f J
/1

where f is an arbitrary function of position.


CHAPTER 15

THE SCHWARZSCHILD GEOMETRY


The vacuum (T/l =
0) solution to the Einstein field equations which
is spherically symmetric and static is called the Schwarzschild geometry.
In "curvature coordinates" (where 21Tr meas ures the proper circumference

of 2-spheres) the Schwarzschild metric has the form

-1

2) 2)

ds = _
(1_ dt 2 + (1_ dr + r2(d02+sin20dcP2) .

2 2 2 2
(One sometimes abbreviates d0 == d0 + sin 0 dcP .) The constant M is

the mass of the source of the field. If the metric is generated by a spheri-
cal star, the Schwarzschild metric holds outside the star and matches

smoothly to the star's Interior metric at its surface.

o o o o o o o

Problem 15.1. Prove that the total angular momentum squared

2 2 -2n 2

+ sin v p
PO

cP

is a constant of motion along any Schwarzschild geodesic.

Problem 15.2.
(a) Prove that all orbits in the Schwarzschild geometry are planar.
(b) Prove that all orbits are stably planar.

Problem 15.3. A particle falls radially into a Schwarzschild metric. As

measured by proper time at infinity, what is its inward coordinate velocity

(drjdt) at a (curvature-) radius r? What is the locally-measured velocity


relative to a stationary observer at the same radius?

87
88 PROBLEM BOOK IN RELATIVITY AND GRAVITATION

Problem 15.4. Derive the equations of motion (equations relating t, r

and r) for a particle falling radially in the Schwarzschild geometry. Con-

sider the three cases (i) particle released from rest at r =


R (ii) particle
released from rest at infinity (iii) particle projected inward from infinity
with velocity v
00

Problem 15.5. Derive a first-order differential equation for the trajectory

(r as a function of cp) for equatorial orbits in the Schwarzschild geometry.

Problem 15.6. Show that the trajectory of light rays in the Schwarzschild

metric obeys 2
d u 2
+ u =
3u
dcp2
where u == M/r, and r is the Schwarzschild radial coordinate. Denote

the minimum value of r along the trajectory by b, the" impact parameter."


In the case M/b << 1 what is the deflection of a photon as it passes a

spherical gravitating body? Give a formula for the deflection angle to

lowest nonvanishing order in (M/b).

Problem 15.7.

(a) For a nearly Newtonian planetary orbit (i.e. M/r? 1) calculate


to lowest order in M/r the advance of the periastron, per orbit, predicted

by general relativity.
(b) Suppose that the central star is somewhat oblate or prolate, so


that the form of the classical Newtonian potential is (I)(r) = -
M/r -

AM/r ,

where A is related to the magnitude of the oblateness or prolateness.


Calculate the advance (if oblate) of tl1e periastron, to lowest order in

A/r , per orbit. (This is a purely Newtonian calculation.)
(c) Assume that the oblateness of the sun is so large that the rate of

advance of the perihelion due to oblateness and the rate of advance due

to general relativity are equal for the orbit of Mercury. Compute the rate

of advance of the perihelion (in seconds of arc per century) for the four
planets closest to the sun, due to each of the effects. Note: to simplify
15. THE SCHWARZSCHILD GEOMETRY
89

the calculations assume,


throughout the problem, that the orbits are nearly
circular -

i.e. they all have negligible eccentricity.

Problem 15.8. A rocket ship in a circular orbit of circumference 21Tr

around a star of mass M fires a laser gun (rest frequency v



)' The gun

is aimed in the orbital plane, and at an angle a (in the ship's frame) out-

ward from the tangential direction of motion. What is the frequency of the
laser as seen by a
stationary observer at infinity?

Problem 15.9. A test particle of relativistic velocity v flies past a mass

M at an impact parameter b so great that the deflection 0 grav is small.

Calculate
Ograv' In flat space, a test charge e flies with velocity v

past a nucleus of charge Ze at an impact parameter b so great that the


deflection OEM is small. Calculate
OEM' Why is the formula for °
grav
different from that for
OEM?
Problem 15.10. A radio commentator is describing his radial fall into a

Schwarzschild black hole. Just before he crosses the Schwarzschild

radius his broadcast frequency starts becoming redshifted enormously with

a time dependence exp (-t/constant), where t measures proper time at

infinity. From the constant deduce the mass of the hole.

Problem 15.11. Calculate the cross section for capture of particles by a

Schwarzschild black hole of mass M, in the limits of very high velocity

particles (v c) and very low velocity particles (v << c).

Problem 15.12. Suppose that Paul is orbiting a neutron star in a circular

orbit at radial coordinate r =


4M. Peter, his colleague, has been fired

radially from a cannon on the neutron star with less than escape velocity;

he flies outward just meeting Paul as he crosses his orbit, reaches a

maximum radius and falls back down just happening to meet Paul again.
Between their two meetings Paul has completed 10 orbits of the neutron
star. Peter and Paul have an obsession about comparing their clocks
whenever they meet. They set their clocks to agree on their first meeting
90 PROBLEM BOOK IN RELATIVITY AND GRAVITATION

as Peter flies outward. When they again compare their clocks,


by how
much do they disagree?

Problem 15.13. Give the coordinate transformation from Schwarzschild



e <Pdt
2 2 2A 2 2 2
coordinates, in which ds + e dr + r d0
= -
to "isotropic

coordinates", in which ds = -


<Pdt 2 + e

/l(cff2 + r
2 2
d( ). Specialize to

the vacuum Schwarzschild solution and construct coordinate diagrams


showing the relation between (t, r) and (t, r) coordinates. Is the area A

of the surface r =
constant, t =
constant given by A =
4 1Tr ? Construct

an
embedding diagram (see MTW p. 613) for the spacelike hypersurface
t =
0, for 0 < r< 00 .

Problem 15.14.

(a) Show that in general a boost jn the spatial direction e,,:, leaves

invariant the P h y sical com p onents of the Riemann tensor ............... which
tj tj
are "parallel" to the boost. This is analogous to the invariance of E,,:,

and R.-. for a boost in the e- direction.
j j
(b) In the Schwarzschild geometry show that all the physical com-

ponents of the Riemann tensor are invariant for a boost in the r-direction,

but that all physical components are not invariant for a boost in the () or

<P direction.

Problem 15.15. Show that the spacelike slice v =


constant (Ivl > 1) of
the Schwarzschild geometry in Kruskal coordinates u, v cannot be em-

bedded in a Euclidean 3-space. What is the general condition on the

slope dv/du of a spacelike slice of the Schwarzschild geometry that

allows it to be embedded in a Euclidean 3-space?

Problem 15.16. Prove that the metric,

2/3 2/3 2

[ J
9M



2 9M
ds = -
dt +

9 2(r-t)
dr +

(r_t)2 d0

(which looks dynamical because the metric coefficients depend on t) IS

actually static. Show that it is in fact the Schwarzschild geometry.


15. THE SCHWARZSCHILD GEOMETRY 91

Problem 15.17. In the preceding problem (15.16) show that the set of

coordinate-stationary observers are all in free fall and have zero energy

(Le. they fell in from infinity with zero initial velocity).

Problem 15.18. A perfectly adiabatic gas with equation of state p =


Kn Y ,

4/3 Y :S 5/3, Y constant, accretes spherically onto a Schwarzschild

black hole of mass M. The speed of sound in the gas at radial infinity
is aoo' At what radius does the inward flow become supersonic? (Give
answer only to leading term in aoo/c.)
Problem 15.19. A scalar field satisfies 0(1) =
O. Show that in the Schwarz-

schild geometry (I) can be decomposed into spherical harmonic components

(Yfm =
spherical harmonic) as


(I) =
r-
tfr(r,t)Yfm({}'CP)
where tfr satisfies

tfr, tt (1- 2M/r) [(1- 2M/r)tfr ,r],r + V


fer) tfr = 0

fCf+ l)

2M
Vf(r) ==
(1- 2M/r)
[ r




Problem 15.20. Show that the Schwarzschild metric is also a solution of

the Brans-Dicke theory of gravity. (For the Brans-Dicke field equations,

see e.g. MTW p. 1070.)


CHAPTER 16

SPHERICAL SYMMETRY AND RELATIVISTIC STELLAR STRUCTURE

The geometry generated by a


nonrotating, perfect fluid star is spheri-
cally symmetric. Exterior to the star it is the Schwarzschild geometry,

even if the star is nonstatic (radially pulsating or collapsing). If the star

is static, the interior metric can be written

2 (1) 2 2 2
(1- 2m/r)-1 dr
2 2
ds = -
e dt + + r d0



where m =
r 417 r p dr. The pressure gradient inside the star is given by
.0
the OV (Oppenheimer-Volkoff) equation of hydrostatic equilibrium


dp =
(p + p) (m + 417r p)
dr r(r -

2m)

Here p and p are the pressure and mass-energy density of the fluid,

satisfying the equations of state

p =
p (n, T)



p (n, T)

If the entropy per baryon s is constant in the star, then p depends only
on p: p

pep). The metric function <P is determined by the Einstein

field equations:


del> _


m+417r p
and e
2(1) =
1- 2M/R, at r =

dr r(r -

2m)

where R is the radius of the star and M =


meR) is its total mass (mass
of the exterior Schwarzschild metric).

92
16. SPHERICAL SYMMETRY
93

Equilibrium stellar models may be unstable against gravitational

collapse. In this situation the relevant dynamical equations derive from


the Einstein field equations and/or from T/l v .

o.
v ,

A stationary rotating star is not spherically symmetric, but rather is

only axisymmetric. The structural equations for axisymmetric stars are

quite complicated. However, certain general properties can be deduced

(i) from the symmetries or (ii) when the star is assumed to be rigidly
rotating.

o o o o o o o

Problem 16.1. Find basis vectors (and a dual basis of I-forms) for ortho-

normal tetrads in a spherical geometry. Take the legs of the tetrad to be

in the t, r, e, and cp directions of the isotropic coordinate system


whose metric is
2 2 (1) 2 2 2 2 2
ds = _
e dt + e /l(dr +r d( ) .

Problem 16.2. Suppose that an observer, at rest at some point inside a

spherical relativistic star, measures the radial pressure-buoyant force,

Fb
uoy

on a small element of volume V, using the usual laboratory

techniques. What value will he find for F in terms of p, p, m, V,


buoy'
and dp/dr? If he equates this buoyant force to an equal and opposite

g ravitational force, F gra v ,what will F g r av be in terms of p, p, m, V,


and r? How do these results differ from the corresponding Newtonian

results?

Problem 16.3. Prove Birkhoff's Theorem: a spherically symmetric vacuum

gravitational field is always static, and is always the Schwarzschild


solution.

Problem 16.4. Show that test particles experience no gravitational forces

inside a self-gravitating hollow sphere.

Problem 16.5. In the Brans-Dicke theory of gravity (see MTW p. 1070 or

Weinberg p. 160 for field equations), show that the only static spherically
94 PROBLEM BOOK IN RELATIVITY AND GRAVITATION

vacuum solution which is regular at the origin is the flat-space metric '1

and constant scalar field (I).


Problem 16.6. How many algebraically independent components of T/l

are there in a spherically symmetric configuration?

{3;{3

Problem 16.7. Evaluate the 4 components of the equation T =
0 for

the stress energy tensor describing a static, spherically-symmetric perfect

fluid star.

Problem 16.8. Polytropic stars (stars with fluids described by p



KpY)
are unstable in Newtonian theory if Y < 4/3. Consider the influence of

small relativistic effects on this stability criterion. Show that the effect

is to increase the unstable range of Y to Y < 4/3 + E where E may

depend on the mass, radius, and structure of the star.

Problem 16.9. Express the Chandrasekhar and Oppenheimer-Volkoff upper

mass limits (for white dwarfs and neutron stars, respectively) as dimen-

sional combinations of fundamental constants and the mass of the nucleon

and electron. Similarly express the limiting radii corresponding to these

mass limits.

Problem 16.10. The mass mer) inside radius r for a spherical star

a ppea rs in the term of the line e leme nt


grr

2 2
ds
2 = _


cI>dt 2 + (1- 2m(r)/r)-l dr + r

d0 .

of the surface
Express mer) in a coordinate-independent manner in terms

area and radial separation of spherical surfaces.

Problem 16.11.

(a) What is the form of the Schwarzschild metric in "outgoing Eddington-

Finkelstein coordinates", obtained from curvature coordinates by the

transformation
dt =
du + (1- 2M/r)-1 dr .
16. SPHERICAL SYMMETRY 95

(b) Now let M be a function of the null coordinate u in part (a).


Show that the space-time is not vacuum, and find the corresponding T {3.

Give a physical interpretation. (This is the "Vaidya" metric.)

Problem 16.12. Solve the relativistic equations of stellar structure for a

static, spherically symmetric star of uniform density. Show that the mass

and radius of the star satisfy R/2M > 9/8. What is the smallest R/2M

can be if the dominant energy condition (see Problem 13.7) holds?

Problem 16.13. A static, spherically symmetric star is made out of a zero-

temperature Fermi gas with Fermi energies much larger than the particle
rest masses. Show that the equations of stellar structure have a solution

mer) =
3r/14. Find per), per), and n(r). Although n is infinite at r =
0,
show that the number of particles out to any radius is finite. Make an em-

bedding diagram of the 3-surface, t = constant. What kind of singularity


is at r =
O?

Problem 16.14. Calculate the surface stresses in a static self-gravitating


shell of mass M and circumference 21TR. What is the proper surface mass

density? Compare the stresses to the Newtonian limit when R >> M:

Problem 16.15. What is the smallest possible proper circumference of a

self-supporting spherical shell of mass M, if its matter satisfies the

dominant energy condition (as do all known forms of matter)?

Problem 16.16. What is the redshift to radial infinity from a thin spherical


shell in static equilibrium, in terms of its proper surface density A 0' and

()
A What is the largest possible redshift if it
proper surface stresses (j?
satisfies the dominant energy condition?

Problem 16.17. Show that for a rigidly rotating, self-gravitating, perfect


fluid star t
Vp =
(p+ p)V log u .


Here u is a component of the 4-velocity of the fluid in the canonical
coordinate system adapted to the Killing vectors (Le. (t)

a/at, (e/:?

a/ae/:?.
96 PROBLEM BOOK IN RELATIVITY AND GRAVITATION

Problem 16.18. Show that in a


rigidly rotating, self-gravitating, perfect
fluid star, the surfaces of cons tant p and p coincide.

Problem 16.19. Show that the surface of a rigidly rotating star, with
angular velocity of rotation (} as seen at 00, is given by


gtt + 2g trp 0 + 0 constant
grprp

Problem 16.20. Find the Doppler broadening for a spectral line from a

rigidly rotating star observed by an astronomer who is infinitely far away

along the axis of rotation. (The Doppler broadening is the variation across

the star in the Doppler shift:


vemitted

_

- -
1 )


obs rved

Problem 16.21. Derive the general relativistic criterion for convective

stability in a static equilibrium configuration of perfect fluid.

Problem 16.22. Prove the equivalence of isentropy and constant injection

energy [(p+ p)/(nuO)] for a rigidly rotating configuration.

Problem 16.23. Consider a stationary, axisymmetric star. There are two

Killing vectors, and Show that


(t) (<p r

M = -

J (2Tl1v-8I1v T) (t)d3II1

is the mass of the star as measured from infinity. Here d I is the
I1
volume element of the star at some instant of time t (the time coordinate

t is chosen such that


(t) a/at). Similarly, show that



TI1v
J =

(rp)
d I
I1

is the angular momentum as measured from infinity.

Prob Iem 16.24. Show that the integral for M given in Problem 16.23, in

the case of a static spherical star made of perfect fluid, is


16. SPHERICAL SYMMETRY 97

f R(p
cI>+A 2
M =
+ 3p) e 41Tr dr

2A
in curvature coordinates
(goo =


(1), grr

e ). Show that this is the

same as the expression derived from the equation of stellar structure,

=f Rp

M 4m dr .

Problem 16.25. For collapsing spherical stars we can't simultaneously


have the three nice properties: (i) radial coordinate is comoving with a

fluid shell; (ii) time coordinate is proper time for the fluid; (iii) the

metric is diagonal. Prove that we can have all three properties if and

only if the pressure vanishes.

Problem 16.26. If R is a comoving coordinate, the metric for a spherically


symmetric collapsing star (see Problem 16.25) can be written as

2 <P 2 2A 2 2 2
(t, R) d0

ds = -
e dt + e dR + r

where (I) and A are functions of R, t. If the star is made of a perfect


fluid it is often useful to define the following functions:



m '" 4m pr'dR

-<1>.r
U == e

-2A 2
r 2=


e (r ) .

Here primes denote partial differentiation with respect to R, and dots

with respect to t. The function m is interpreted as the mass interior to

the shell at R, and U is the rate at which a shell is moving with respect

to the proper time of a comoving observer.


Prove the following relations:


2 ・

(a) m = -
4 17
pr r .
98 PROBLEM BOOK IN RELATIVITY AND GRAVITA TION

[Hint: Use the first law of thermodynamics (Problem 5.19), baryon con-


servation, the equations of motion, and G R

0 (Problem 9.20).]
2 2
(b) r =
1 + U -

2m/r .

t t
[Hint: Use G t = -
81TP and G

= 0 (Problem 9.20).]

Problem 16.27. In a collapsing star made of perfect fluid show that once

a mass shell at comoving radius R has collapsed far enough so that

2m(R, t)jr(R, t) > 1, the mass shell will collapse to r =


0 in a finite

proper time.

Problem 16.28. For spherically symmetric pressure free collapse show


that the fall of a shell is governed by the mass interior to that shell in

the same manner as the radial fall of a particle in the Schwarzschild

geometry is governed by the Schwarzschild mass:

2 2 2
d r/dr = -
M/r .

Problem 16.29. For pressure free spherically symmetric collapse (see

Problem 16.26), show that both m and r are independent of time. Solve

the resulting dynamical equation



dr 2m(R) 2
( )
dr
_



r (R) _


in the three physically distinct cases r -
1 greater than, less than, and

equal to, zero.

Problem 16.30. Consider the gravitational collapse of a spherically sym-

metric, perfect fluid star of zero pressure and uniform density (i.e. uniform

throughout the star as seen by observers comoving with the fluid).


(i) Show that the metric inside the star is locally the Friedmann
solution with k =
+1 if the star collapses from rest at some

finite radius, k =
0 if the star is at rest at infinity, or k =
-1

if the star is projected with finite velocity from infinity.


16. SPHERICAL SYMMETRY
99

(ii) By Birkhoff's theorem (Problem 16.3), the exterior metric is the

Schwarzschild metric. Show that each point on the surface of

the star moves along a radial geodesic of the Schwarzschild

metric.

(iii) Show that the Friedmann and Schwarzschild metrics match

together smoothly at the surface of the star. (It is necessary

and sufficient to show that the intrinsic 3-geometry of the

surface and the extrinsic curvature of the surface are the same

whether meas ured in the exterior or the interior.)


CHAPTER 17

BLACK HOLES

The Kerr-Newman black hole is an exact solution of the Einstein field

equations possessing mass, angular momentum, and (in principle but not

in astrophys ic al cases) charge.

The metric describing this solution is (in "Boyer-Lindquist coordi-


nates "):

Q2
(1- 2Mr ) Q2);a
2 (2Mr- sin e
ds 2 = _
dt _

dt dcP


Q2) a 2


1 2 2 2 2 (2Mr- sin e 2
() d ",,2

dr e

+ -

+ a + r + a + SIn 'P

where


+ Q2 M2 ,

M -

mass, Q ==
charge
a =
angular momentum per unit mass
2 2
== r
_
2Mr + a + Q2
2 2 2
1 == r + a cos ().

The metric c oeff icients are independent of t and cP, s0 = a I at


(t)

and a / acP are Killing vectors. Among the properties of this



s olu-
(cP)
tion which follow from the metric are the orbital equations for test

particles: 1

1 r = :t (V r )2

1 () = :t (V e) 2

2 a

lcP = -
(aE -

Lz/sin ()) + P

2 2
2 r
li = -
a(aE sin e-
Lz) +

100
17. BLACK HOLES 101

Here "dot" indicates derivative with


respect to proper time or affine

parameter, and

2 2
P ==
E(r +a ) -

Lza -

eQr

Vr == p2 -

[/12r2+ (Lz aE)2 -


+ ]

(/12 E2) + L;/sin 2 ()]


()
== -

cos () [a -

E == conserved total energy

Lz == conserved z component of angular momentum


conserved quantity related to total angular momentum

/1 -
rest mass of particle

e ==
charge of particle.

A Schwarzschild black hole is the special case for which a =


Q =
O.

A Reissner-Nordstrom black hole is the special case a =


0, Q 1= 0; it is
spherically symmetric. A Kerr black hole is the case a 1= 0, Q =
O. The

defining property of a black hole is that it have a horizon, a surface

through which matter can fall, but from which no matter or information can

escape to infinity. For a Kerr hole, it is located at r+,


the larger root of

the equation =
O. The stationary limit of a rotating hole is the surface
within which all observers are dragged around the hole. For Kerr, the
stationary limit is at
ro'
the larger root of
gtt

O. The region between

the horizon and stationary limit is called the ergosphere.

o o o o o o o

Problem 17.1. Show that the constant M which occurs in the Kerr metric

is the mass of the system, and that the constant a is the angular momen-

tum per mass.

Problem 17.2. A suggested use of small ?< MEa) black holes is to crush

junked automobiles into neat spherical balls by allowing them to partially

collapse around a black hole. Estimate what mass hole, in orbit around
102 PROBLEM BOOK IN RELATIVITY AND GRA VITA TION

the Earth, would be appropriate for this application. How many wrecks

per hour could be processed?

Problem 17.3. Show that once a rocket ship crosses the gravitational

radius (horizon) of a Schwarzschild black hole, it will reach r = 0 in a

proper time T 17M, no matter how the engines are fired.

Problem 17.4. Show that Kepler's law

2 3
0 =
M/r

holds exactly for circular orbits around a Schwarzschild black hole, if r

is the curvature coordinate radius, and 0 is the angular frequency as

measured from infinity. Derive an analogous law for equatorial orbits


around a Kerr black hole of specific angular momentum a.

Problem 17.5. An observer in a circular orbit of circumference 217 r

around a charged, spherical, black hole (a Reissner-Nordstrom black


hole) of mass M and charge Q measures local electric and magnetic

fields. What are their strengths and orientations?

Problem 17.6. By considering the mass, charge, and angular momentum of

a "classical" electron, show that it cannot be a Kerr-Newman black hole.

Problem 17.7. For circular orbits in the equatorial plane of a Kerr black

hole, prove that the marginally stable orbit has minimum energy E and

minimum angular momentum L.

Problem 17.8. An observer (not necessarily freely-falling) orbits a Kerr

black hole in the equatorial (0 =


17/2) plane.
(a) Let his orbit be at constant r. Define 0 =
dcP/dt to be his

relative to In terms of
"angular velocity a distant stationary observer."

n, r, M, and a, find u

, ucf>, u
o'
ucf>'
(b) Suppose that the circular orbit lies in tne ergosphere (the orbital
radius is outside the horizon at r+
but inside the stationary limit at r
o )'
17. BLACK HOLES 103

Show that the observer cannot remain at rest with respect to a distant

observer. That is, show that {} for the observer must be nonzero.

(c) If the observer is in the region r_ < r <


r+, show that he cannot

remain at constant radius.

Problem 17.9. Show that there are negative energy particle trajectories
inside the ergosphere of a Kerr black hole (and outside the horizon!).

Show that it is possible for a rocket ship to increase its total energy by

firing a bullet into the hole during an orbital passage through the ergo-

sphere.

r+, the
Problem 17.10. Show that as a test particle approaches r =

horizon of a Kerr black hole, it has "angular velocity as seen from

infinity" equal to
11 =

dcp =

dt
2Mr+
Problem 17.11. Prove that there exist "quasi-circular, polar" orbits in

the Kerr geometry, i.e. orbits which pass alternately over the north and

south poles at a fixed radial coordinate distance. What is the smallest

pass ible polar radius of these orbits?

Problem 17.12. A Killing horizon is a null hypersurface generated by a

Killing vector. An ergosurface ("stationary limit") is an infinite red-shift

surface for static observers. Show that for a static black hole the ergo-

surface is a Killing horizon.

Problem 17.13. Show that the surface area of the horizon of a Kerr-

Newman black hole (area of surface r =

r+,
t = constant,

in Boyer-
2 2
Q2

Lindquist coordinates) is 417 [{M + (M _ _
a ) 2}2 + a ].

Problem 17.14. According to Hawking's theorem (that in a collision of

two black holes the total surface area must not decrease), what is the
minimum mass M of a Schwarzschild black hole that results from the

collision of two Kerr bl9ck holes of equal mass M

and opposite angular
104 PROBLEM BOOK IN RELATIVITY AND GRAVITATION

momentum parameter, a? Suppose I al M ;



what fraction of the original
mass can be radiated away? Are there any other uncharged black-hole

collisions that can get this much energy out?

Problem 17.15. Use the theorem that the area of a black hole is non-

decreasing (cf. Problem 17.14)to prove that a Kerr black hole amplifies
(rather than absorbs) certain modes of an incident radiation field.

Problem 17.16.

(a) Write down the scalar wave equation o<P =


0 in the Kerr geometry

in Boyer-Lindquist coordinates.

(b) Show that the equation can be reduced to ordinary differential

equations by separation of variables.

(c) Find the asymptotic form of (I) for r 00.

(d) Find the asymptotic form of (I) for r


r+.
(e) What boundary condition on (I) corresponds to ingoing waves as

seen by a physical observer on the horizon?

(f) Show that, for a wave of the form (I) =


exp (-iwt + im<p) f(r, ()),
0 < wlm <
energy flows out of the hole if a/(2Mr+). Compare to Problem

17.15.

Problem 17.17. Charged particles are dropped radially into a Reissner-



Nordstrom black hole with Q2 < M . Show that it is never possible to


drop in enough charge to make Q2 > M (a solution which would be a

naked singularity, not a black hole).

Problem 17.18. The "Zero Angular Momentum Observers" (ZAMO's) in

the Kerr geometry have basis I-forms



1-
t 2
g<P<P \2

I gt t -
w dt

if> t
(gcf>cf? (.14> (i.Jllt)
= -


1 -.I

r(l/

dr
)2

()=
l2 dO
where w ==
gt<p/g<P<P.

17. BLACK HOLES
105

(a) Show that these basis I-forms are orthonormal.

(b) Find the dual basis vectors.

The 4-velocity of the ZAMO is


(c) u
et'; show that u has zero

rotation.

(d) The ZAMO is not an inertial observer; show that the acceleration

g.p.p I
is a =
V log I gtt -
w .

Problem 17.19. Calculate the Gaussian curvature of the horizon of a Kerr



black hole and show that it becomes negative for a > 3 2 M/2. (This shows
that the horizon cannot be globally embedded in a Euclidean 3-space if

a > 32 M/2.) Use the Gauss-Bonnet theorem to check that the horizon is

topologically a 2-s phere.

10
Problem 17.20. Show that a primordial, rotating, black hole (,...., 10 years
15
old), of mass 10 gm. will have already lost most of its angular momen-

tum to spontaneous quantum emission of photons or gravitons. What frac-


33
tion of the angular (,...., 10 gm.) rotating hole
momentum of a
1MG)
would be lost in the same time?

Problem 17.21. Consider the vacuum metric

ds2 (1- t m/p)2 2 (1- t m/p)2 2


(dp +p2 d 02 +p
2 2
= _
dt + sin 0d.p2)
(1+ t m/p)2 (1- m/p)2
(which is a particular solution to the static, spherically symmetric problem
in the Lightman-Lee theory of gravity). Does the above metric describe a

black hole; and, if so, how do the hole's properties differ from those of

the corres ponding hole in general relativity?


CHAPTER 18

GRAVITATIONAL RADIATION

Weak gravitational waves are described by linearized theory (see

Chapter 13). The basic equations for waves propagating in vacuum are

gllv TJ + h
(Ihllvl ? 1)

llv llv



h/l h/l
==
TJ/lV

V V a

D h == h ;a=O
Il V IlV a


lLU;a = 0 ("Lorentz gauge")



llO

0, h a =
0 ("TT" or "Transverse-Traceless" gauge) .

The effective stress-energy tensor for gravitational waves is


(GW) = ..l. <h. h jk >
Il V 3217 Jk,1l ,v

where < > denotes an average over several wavelengths and h is in


jk
the TT gauge (see e.g. MTW Section 36.7).
The gravitational wave power L
GW '
emitted by a nearly-Newtonian,
slow-motion (v? c) gravitating source is

1 G'" ..'jk
L i.
5 5 <'1jk
GW

>,

where 't is the "reduced quadrupole moment tensor" VI. the source,


jk
given by




P(XjXk- 8jk r2 )d
==
x,
jk

and < > denotes averaging over several characteristics periods of the

source. 106
18. GRAVITATIONAL RADIATION
107

Problem 18.1. A Massachusetts motorist shakes his fist


angrily at another

motorist. What fraction of his expended energy goes into gravitational

rad ia tion?

Problem 18.2. A gravitationally bound dynamical system (e.g. a binary


star) has, in order of magnitude, mass M and size R. Estimate the time

for radiation reaction forces to affect the system substantially, and com-

pare this time-scale with the dynamical time-scale for the system.

Problem 18.3. For an electric dipole, and its radiation pattern, there are

three independent orientations, corres ponding to the three directions in

which the dipole may point. How many independent orientations are there

for a traceless quadrupole tensor?

Problem 18.4. Calculate the gravitational radiation luminosity of a spin-


ning thin metal rod of mass M and length e, spinning at frequency (U

around a symmetrical perpendicular axis. Estimate the electromagnetic


luminosity which would arise from the slight excess of electrons pushed

toward the ends by centrifugal force. If the rod has a reasonable density

(10 gm/cm ) and is rotating at a reasonable frequency (1 kHz) will
electromagnetic or gravitational radiation be more important in slowing
the rotation?

Problem 18.5. The radiation reaction forces on a slow-motion, weak field


source can be derived from an addition to the Newtonian potential

<l>react. =
xjx k .


(Cf. W. Burke, J. Math. Phys. 12,402 (1971); MTW pp. 993.) Here
.ljk is


the reduced quadrupole moment of the source i.
jk
==
J p(xjx k -

8jk r

)d x

at a given time. The superscript (5) indicates the fifth time derivative.
From this potential derive expressions for the time-averaged rates at

which the source loses energy and angular-momentum, in terms of d riva-

tives of
.ljk.
108 PROBLEM BOOK IN RELATIVITY AND GRAVITATION

Problem 18.6. Two stars of mass Ml and M separated by a distance R



revolve about each other in a nonrelativistic circular orbit. Due to gravita-

tional radiation reaction, R changes with time. Find R(t).

Problem 18.7. Two point masses m and m are in a Newtonian ellipti-


l 2

cal orbit with semimajor axis a and eccentricity e. Compute da/dt and

de/dt due to gravitational radiation reaction. Show that the elliptical


orbit tends to be circularized.

Problem 18.8. A plane gravitational wave propagates through nearly flat

empty spacetime, in the xl direction (i.e. the metric perturbations


ha{3
are functions only of u =
t-x). Give an explicit coordinate transformation
which makes all the ha zero except h 23 =
h and h = -

h33' Show
{3 32 22
that the same resulting components could have been obtained directly by
projection into the transverse traceless gauge.

Problem 18.9. Show that gravitational radiation generated by an axisym-


metric system carries no net angular momentum. (Do not assume that the

sources have weak internal gravitational fields.)

Problem 18.10. Define Stokes parameters for a plane gravitational wave

and show how to calculate the fraction of circular polarization, linear

polarization and the orientation of maximum linear polarization from the

three Stokes parameters.

Problem 18.11. An initially static source undergoes violent motion which

generates gravitational radiation and then, after a finite time, becomes

again static. A distant observer detects the gravitational waves by


watching the motion of two free particles which are initially at rest with

respect to each other. Show that after the passage of the waves the ob-

server sees the particles back in their original positions and at rest with

respect to each other, to linear order in the wave amplitude.

Problem 18.12. An elastic rod can be used to detect gravitational waves

not only at its lowest normal mode frequency w ' but also at harmonics

18. GRAVITATIONAL RADIATION
109



== nw

. What is the sensitivity of the nth mode relative to the zeroth,

Le. how does the ratio of maximum squared amplitude of the displacement

to energy flux of wave vary with n? (Assume the rod has the same

mechanical damping time for all modes.)

Problem 18.13. A (weak) plane gravitational wave travelling in the

x-direction is normally incident on a slab of cement. The cement absorbs



energy E from the plane wave. Show (e.g. as a result of T/l .




0) that
the slab must absorb x-momentum E also, and find the relationship be-

tween the rate at which energy and x-momentum are absorbed.

Problem 18.14. In the previous problem it was shown that materials must

be able to absorb momentum in the direction of wave propagation. This


seems incompatible with the description of gravitational waves as trans-

verse! To investigate this point idealize the cement molecules of the

previous problem as
being harmonically bound to their equilibrium positions,

and having a damping force due to internal friction of the cement. Assume

the gravitational wave is monochromatic and linearly polarized. Using the

equation of geodesic deviation find the time average force, and hence the

rate of momentum absorption, in the direction of wave propagation. Show


that this rate of momentum absorption is equal to the rate at which the

molecule absorbs energy.

Problem 18.15. A weak, plane gravitational wave of frequency wand

dimensionless amplitude h passes through a "hard-sphere" gas of

temperature T. The mean free path of atoms in the gas is E, and the gas

is dilute enough so that E? clw. Show that in a finite amount of time

the particles in the gas will be heated to relativistic temperatures. Esti-

mate this time. Estimate the distance over which the wave is damped by
a factor of e in amplitude.

Problem 18.16. Estimate the number of gravitons emitted in an asymmetric

explosion of energy E.
110 PROBLEM BOOK IN RELATIVITY AND GRAVITATION

Problem 18.17. Roughly how many thermal gravitons does a 100 watt

lightbulb emit in its rated lifetime of 1000 hours? What is the approximate

wavelength and number of gravitons emitted when the lightbulb is dropped


and broken on a cement floor?

Problem 18.18. Calculate in detail the lifetime of a hydrogen atom in the

3d state against decaying to the Is state by gravitational radiation.

Problem 18.19. By symmetry, the thermal graviton flux from a spherically


symmetric star is evidently isotropic; reconcile this fact with Birkhoff's
theorem. Approximately what is the multipolarity 2£ of the flux (f =
1 ==

dipole, etc.) for a typical star, such as our sun.

Problem 18.20. Consider the following "gravitational wave modes" repre-

senting a gravitational wave travelling in the z direction


qt2 = -



zozo 'P4 =

yOYO


xoxo
+ 2i R
xOyO

qt3 =

(-RxOzo+iRyOzo) 'P4 =

yOyO


xOxO

2i R
xOyO

qt3 =

(-RxoZO-iRyOzo) (1)22= -

(R xOxo + RyOYO) ・

Which of these waves are transverse?

The spin of a wave indicates (among other things) the relation of the

orientation of polarization states. For a spin 0 (scalar) wave the mani-

festations of the wave are symmetric about the direction of propagation.


For a spin 1 (vector) wave (e.g. an electromagnetic wave) the independent
states are oriented at 90° to each other; rotation of 180°
polarization a

returns to the original polarization state, with only a sign change. In

general for a spin s wave a rotation of 17/ S brings bac k the original
polarization state. Which of the waves above are spin O? Spin I?

Spin 2? Which are possible in general relativity?

Problem 18.21. Draw the force field of each of the modes in the preceding

prob lem .
18. GRA VITA TIONAL RADIATION 111

2 2 2
Problem 18.22. Consider the metric ds =
dx + dy2 -
dudv + 2H(x,y,u)du .

What form must the function H have for this to represent a (strong) plane
gravitational wave propagating in vacuum?
CHAPTER 19

COSMOLOGY

If the universe is homogeneous and everywhere isotropic, its geometry

is that of the Robertson-Walker metric

[ ]
2 2 2 dr 2 2
ds = -
dt + R (t) + r d0 ,

1-kr

where k =
+ 1, 0, -1 for a closed, marginally open, or open universe.

When the Einstein equations are used to determine the time development
of R(t) and the value of k, the resulting spacetime is called a Fried-
......

mann model (sometimes called a Lemaitre model, especially for nonzero

cosmological constant). The first two derivatives of R(t) at the present

epoch (denoted by subscript 0) are parameterized by the Hubble constant

HO ==
(dR/dt)/R at R =

Ro

and the "deceleration parameter"


2 2
qo == -

[(d R/dt )RJ/(dR/dt)2 at R =

Ro .

The matter in a cosmology is generally in a state of expansion or con-

traction, so that light received by an observer is generally red- or blue-

shifted relative to its source by an amount z,

1 + z ==
vemitted =
Aobserved .

vobserved Aemitted

Often the magnitude of z varies monotonically with distance from an

observer so one speaks of "an object at redshift z."

112
19. COSMOLOGY 113

If p and p are the density and pressure of the smoothed-out mass-

energy content of the universe, the universe is said to be "matter domi-

nated" when p? p, and "radiation dominated" when p


j p.
The 3. oK black-body "cosmic microwave background," when extra-

polated back in time, implies high temperatures at early times in the


Friedmann model, a "hot big bang." However it is also possible that the

large "entropy per baryon" implieq. by this radiation was generated by


some dissipative process during the evolution of our universe.

o o o o o o o

Problem 19.1. Show that the equations of Newtonian gravity and hydro-

dynamics do not admit a cosmology which is isotropic, homogeneous, and


static (i.e. an unchanging universe filled with a uniform perfect fluid).

Problem 19.2. A spacetime contains no matter and is everywhere isotropic.

Prove that it is flat Minkowski space.

Problem 19.3. An object emits black body radiation of temperature T in

its own rest frame; we see it at a redshift z and subtending a solid

angle O. What flux do we measure? What if the redshift is due to doppler

motion of a local object instead of a cosmological redshift?

Problem 19.4. Homogeneous, isotropic spatial hypersurfaces must (by


spherical symmetry) have a line element of the form

2 2
[f2(r)dr 2 112]

da =
a + r d ,


constant.

(1- kr ) -1, where k



Show that f2(r) must have the form =
0, :t 1.

Problem 19.5. Show that the Robertson-Walker metric

[ ]
2 2 2 dr2 2 2
ds = _
dt + R (t) 2
+ r

(d0 +sin 0d4>2)
1 -
kr

can also be written as

2 2 2 2 2
ds = -
dt + R

(t) [dX + 2(X)
(d0 + sin 0 d4>2)]
114 PROBLEM BOOK IN RELATIVITY AND GRAVITATION

or as

2 2 2 2 2 2
ds =
R (11) [-d11 +dX + 2(X)(d02
+sin 0d<p )]

2 2
where 2(X)sin

x or X2 or sinh x (k =
1,0, -1).

Problem 19.6. Show that the spacelike 3-surfaces of a closed, isotropic,


homogeneous universe possess a translation symmetry which leaves no

points fixed. (Notice that this is not true in 2 dimensions; a 2 sphere

cannot be combed smooth!)

Problem 19.7. A bullet is shot out into an expanding Robertson-Walker


universe with a velocity V 1 (relative to cosmological observers). Later,
when the universe has expanded by a scale factor (l+z)-l, it has a

different velocity V 2 with respect to cosmological observers. Find V 2


in terms of z and V Show that in the limit V c, the formula for the
l' 1

redshift of photons is obtained.

Problem 19.8. Show by an explicit coordinate transformation that the


Robertson-Walker metric is conformally flat. Write in terms of
R/l va {3
and p and the 4-velocity u IL of the matter.
p,
g/lv
Problem 19.9. In a Robertson-Walker metric show that angular diameter

distance (d A ), luminosity distance (d L ) and proper motion distance

(d M ) are related by
(1+z)2 dA =
(l+z)d M =


Problem 19.10. Suppose astronomers are able to find a family of objects


whose absolute luminosities L are known. Suppose their apparent lumi-

nosities e (or equivalently their luminosity distance d L) and their red-

shift z are measured. Using the Robertson-Walker line element, find an

expression for e (or d L ) as a function of L, z, Ho and


qo
for small z.

Problem 19.11. Let n(t o ) be the number density at the present epoch of

a (mythical) family of identical light or radio sources distributed uniformly

throughout the universe.


19. COSMOLOGY 115

(a) Show that the number of such sources with redshifts less than z

as observed from the Earth today is

411 n(t o )


3 3
N(z) = -






(1--z(1+q 2 0 )+...

Ignore evolutionary effects, Le. the number of sources in a unit comoving


volume remains constant.

(b) If the sources all have intrinsic luminos ity L, show that the
1 2
number with fluxes (ergs sec- cm- ) greater than S as observed from

the earth today is

1/2
L\3/2
( 417S )
411 L
N(S) "3 n(t o ) (417S) [ 1- 3Ho +

...
= ・

Problem 19.12. A ray of light travels along a radial line in the Robertson-

Walker metric

[ J
ds
2 = _
dt
2 2
+ R (t)
dr2 + r

d 02 .


1 -
kr

How is the coordinate r related to the affine parameter A. along the ray,

namely what is dr/dA.?

Problem 19.13. By requiring that the Robertson-Walker metric satisfy the

Einstein field equations, derive the dynamical equations for a perfect


fluid Friedmann cosmology:
..

3R + 411G (p+3p)R = 0 (1)


RR + 2R 2 + 2k -

411G(p -

p) R =
0 .
(2)

Problem 19.14. Show that the two second-order equations of Problem 19.13
are equivalent to the first order equations

811G 2
R2 + k =


pR (1)

d 3 2
(pR ) = -
3pR ・
(2)
dR
116 PROBLEM BOOK IN RELATIVITY AND GRA VITA TION

Problem 19.15. For a Friedmann cosmology derive the relations

81TGp .l
( R2 + H2
) (1)


87TGp =


+ H2(1_ 2q) .
(2)

If the cosmology is matter-dominated (p? p) show that

..k (2q -1)H



(3)


81TGp -

2 qH 2 (4)

If it is radiation dominated (p 1/3 p) show that

k 2


( q-1)H (5)

81TGp 2

qH ・
(6)

Problem 19.16. For a Friedmann cosmology what equations relating p, p,


and result from the equation of energy conservation,
R(t)
TV/l;v O?

Problem 19.17. For a k =


-1 Friedmann cosmology with p =
p = 0 show

that the line element becomes

2 2 2 2 2 2 2 2
ds = -
dt + t [dX + sinh x (d0 + sin 0 d<p )] .

Exhibit an explicit coordinate transformation to show that this metric

describes Minkowski space.

Problem 19.18. Solve the first-order Friedmann equation

R 2
87TG
( )



_

R2
for R(t) when the density p is dominated by a) matter and b) radiation.

Express any parameters of the present epoch in terms of Ho and


qo'
19. COSMOLOGY 117

Problem 19.19. A bullet is shot out into an


expanding Friedmann universe.
When k =
-1, show that it approaches the same velocity as some cos-

mological observer, but a position which is a constant


proper distance

away from him. When k =


0, show that the bullet again approaches the
velocity of some cosmological observer but that the proper distance be-
tween the bullet and that observer becomes
arbitrarily large as t 00.

Problem 19.20. For a closed (k= 1) Friedmann universe in which radia-


tion dominates for only a negligibly short fraction of the life of the uni-

verse, how many times can a photon encircle the universe from the moment

of the creation of the universe to the moment of its death?

Problem 19.21. If an idealized k=O matter dominated Friedmann cos-

mology with Hubble constant


Ho contains homogeneously distributed
sources of constant luminosity L, and if the local number density of

such sources in space is now n, what is the brightness B of the night

sky (energy per steradian of sky per collection area per time)? (If the
universe were static and infinitely old the brightness would be infinite;

this is called Olbers' Paradox.)

Problem 19.22. Suppose that at the time of hydrogen recombination (which,


say, occurred at a redshift of z =
1500) the deceleration parameter was



0.5002. What would qo be today? Repeat for q =
0.4998 at z =

1500. (Assume a matter-dominated universe.)

Problem 19.23. A closed (k= 1) Friedmann universe has Hubble constant

and deceleration parameter Assume that the universe has always


Ho qo'
been matter dominated.

(a) What is the total proper volume of the universe at the present

epoch?
(b) What is the total proper volume that we see, looking out into the

sky?
(c) What is the total proper volume now occupied by the matter which

we see, looking out into the sky?


118 PROBLEM BOOK IN RELATIVITY AND GRA VITA TION

Problem 19.24. What is the apparent angular size of an object of proper

diameter e seen at redshift z in a matter dominated Friedmann cos-

mology with pre'sent parameters Ho and qo? (Analogous results for


apparent proper motion and apparent luminosity follow from Problem 19.9.)

Problem 19.25. In a "hot" Friedmann cosmology, two unrelated important

epochs are when matter first begins to dominate radiation (p ma tt er R:::

Pradia tion)' and when protons and electrons recombine to form hydrogen.
Given that these epochs happen to be nearly the same in our universe,

deduce a numerical value for the conserved entropy per baryon a== 4aT /3n

(where T =
temperature, a =
radiation constant, n =
number density of

baryons ).

Problem 19.26. In terms of the conserved entropy per baryon a of a hot

big bang model, find the temperature at which hydrogen characteristically

recombined, Le. had an equilibrium ionization fraction of 0.5. Evaluate


8 9
your answer for a = 10 ,
10 .

Problem 19.27. In a radiation dominated Friedmann cosmology at times

near the big bang singularity, calculate the temperature T as a function

of (proper cosmological) time t. Assume that only photons, electrons and

positrons contribute to the energy density p. How is the answer changed


if neutrinos and antineutrinos are also allowed?

Problem 19.28. A Friedmann cosmology has temperature T 1 at expansion


scale R 1 ,
and is dominated by relativistic electrons, positrons, muons,

photons, and neutrinos in thermal equilibrium. Later, at expansion scale


2 ,
the muon pairs have annihilated, but the other particles are still

relativistic and in equilibrium. Find the temperature T2 in terms of T


1 '

R and R
1 2

Problem 19.29. Under which of the following suppositions would the hot

big bang have produced less He than predicted by the "standard"

model? Less H (deuterium)?
19. COSMOLOGY
119

(i) Suppose the baryon dens ity in the universe today is larger than

we now think.

(ii) Suppose the weak interaction constant is smaller than we now

think.

(iii) Suppose there are many more neutrinos than antineutrinos or

photons in the cosmic background today.


(iv) Suppose there are many more antineutrinos than neutrinos or

photons in the cosmic background today.


(v) Suppose that the gravitational constant G varies with cosmologi-
cal time and was slightly larger in the past.

Problem 19.30. Suppose that a universe is isotropic, homogeneous, and


empty except for a "vacuum polarization" stress energy of the form 817 T V
Il
(In old language: there cosmological l\..)
Agp.v. is a non-zero constant

Find a k =
0 cosmological solution. Find a coordinate system in which

it is static. (This is the "de Sitter universe".)

Problem 19 31.A universe is isotropic, homogeneous, and contains only

pressureless dust and "vacuum polarization" stress energy, so that T


Il V

Po up.


t;, gp.v where up. is the 4-velocity field of the matter. Show

that there is a static solution for the metric, but that it is unstable. (This

cosmology is called the "Einstein universe".)

Problem 19.32. What is the proper volume of the "Einstein universe" of

Problem 19.31, in terms of the density of its dust.


Po'

Problem 19.33. At one time observations seemed to suggest that there

was an unusual clustering of quasar redshifts around z =


2.. One proposal

to explain this is that our universe is a k = + 1 dust cosmology, with


nonzero cosmological constant A only slightly greater than the value for
a static "Einstein Universe" (Problem 19.31). Show that for this model

at
the universe will expand at a decreasing rate to a certain radius Rm'
which radius it will remain for a long time while expanding very slowly,
before expanding again at a rate which asymptotically approaches H =

(A/3) 2".
120 PROBLEM BOOK IN RELATIVITY AND GRAVITATION

Suppose quasar formation occurred at the time of near ly constant

radius. What does this model predict for Pma tter today? [Use Ho =

28 1
10- cm- ].

Problem 19.34. What is the order of magnitude of the influence of the

cosmological constant on the celestial mechanics of the solar system if


57 2
A 10-
,....,
cm- ?

Problem 19.35. Prove that for a physically possible perfect fluid no solu-

tion of the Einstein equations is homogeneous, everywhere isotropic, and

static. (Before Hubble's discovery, Einstein considered this a failing of


the theory and introduced the "cosmological constant" term as a remedy.)

Problem 19.36. Prove that no solution of the Einstein equations for a

pressureless fluid is static. Do not assume isotropy or homogeneity. (The


difficulty of this problem depends on the definition of "static." Easy

case: Take static to mean time invariant and time reversible, the first

definition of "static" in Problem 10.8. Harder case: Use the second

definition in Problem 10.8.)

Problem 19.37. Prove that no solution of the Einstein equations for a

perfect fluid is static and homogeneous. Do not assume isotropy. (Again


as in Problem 19.36 there are easy and harder cases depending on the

definition of "static" that is used.)

Problem 19.38. In cosmology one usually uses coordinates comoving with

the galaxies. Let (r, xi) be such a system, and let the metric take the

general form 2 2 i i
ds = -
dr + 2g o 1 dr dx

+ g dx dx j
..

1]

where
go 1
. and g..
1]
can be functions of r and xi. Show

(a) that r is the proper time measured by a galaxy;


(b) that gij governs proper distances in the hypersurface of constant r;

(c) that if gOi and gij


are independent of all xi, then the universe

is homogeneous but that the converse is false;


19. COSMOLOGY 121

(d) that if gOi and gij are independent of r, then 0, e 0


aa{3
= =

(but w 1= 0 in general);
a {3

(e) that if g Ol .

0 and g..
IJ

f(r) g .

IJ
.(x ), then a..
lJ

0;
(f) that gOi,O 0 if =
and only if galaxies fall on geodes ics;
(g) that if w a {3 1= 0, no choice of r and xi can make
gOi

0 every-

where and that this means that w 1= 0 implies nongeodesic motion of



{3,o
the galaxies.


Problem 19.39. The distance between two neighboring galaxies is ox =


Rn where n is a unit, purely spatial vector in one galaxy's rest frame.

Show that
a Q I

- =
a n n fJ + -

R a {3 3

(where a is the shear tensor and e is the scalar expansion) and that

averaging over all directions n gives



> = ! e
R 3

Problem 19.40. For the congruence of galaxy world lines in a Robertson-

aa{3' (See
Walker cosmology find (), w and Problem 5.18 for

{3
definitions.) Do the same for the anisotropic metric

2 2 2
dy2
2a 2 2h 2c
ds = _
dt + e dx + e + e dz

where a, b, and c are functions of t and where x, y and z are

coordinates comoving with the galaxies.

Problem 19.41. Consider the homogeneous, anisotropic cosmological

model with metric


2 2 i j
ds = -
dt + g .

IJ
.(t) dx dx .

where the space slices t =


constant have a flat geometry. Find the

evolution of the g..


IJ
when the universe is "gravitation dominated," Le.


set Tf.l to zero in the field equations. Show that the volume of the uni-
3 /2
verse goes to zero linearly with t as t 0 (in contrast to the t or


t behavior for radiation- or matter-dominated Friedmann models).
CHAPTER 20

EXPERIMENTAL TESTS

This chapter explores concepts relevant to experimental tests of

gravitation theory (light deflection, perihelion shift, etc.). A number of

problems elsewhere in this book are


similarly relevant: 11.9, 11.11,

12.2 -12.4, 12.6, 12.7, 13.2 -13.4, 14.12, 15.6, 15.7.

o o o o o o o

Problem 20.1. Gravitational bending of light: An evacuated tube of

length e is set up horizontally in a uniform gravitational field -

e.g., in

the field of the earth at sea level with e? r


ear th
so that gravitational
inhomogeneities (tidal forces) are negligible. A laser beam passing

through the tube is deflected from the horizontal by the uniform gravita-

tional field. Calculate the angle of light deflection measured relative to

the axis of the tube. Express the answer in terms of the length of the

tube, e, and the acceleration of gravity, g. Discuss the feasibility of


performing such an experiment in an earth-based laboratory.

Problem 20.2. Calculate the gravitational light deflection of a ray passing


near the sun using Newtonian gravity and the fact that light moves along
straight lines in the local frame of a freely falling observer. Since the

light is always in a weak gravitational field, the Newtonian approximation

seems justifiable. Why doesn't this answer agree with the general relati-

vistic answer?

Problem 20.3. Derive the general expression for the angular deflection of

light by the sun's gravitational field, if the light comes from a star which

is at an angle a from the sun as seen from earth. Take the earth-sun

distance to be R. Do not assume that a is small, but show that in the

122
20. EXPERIMENTAL TESTS 123

limit of small a the answer reduces to the conventional result (Problem


15.6) oa =
4M/b.

Problem 20.4. Show that the sun's angular momentum J modifies the

light deflection formula (Problem 20.3) from ocjJ =


4M/b to

J.n
ocjJ =
4M
b ( 1 _

Mb )
where n ==

unit vector in direction of angular momentum of the photons

about the center of sun.

Problem 20.5. In addition to the general relativistic deflection of electro-

magnetic waves by the sun, there is a frequency-dependent deflection


caused by the solar corona, which must be taken into account in the inter-

pretation of measurements. Estimate the impact parameter at which the

general relativistic and coronal deflections of an electromagnetic wave of

frequency v make approximately equal contributions. Take an approxi-


mate coronal electron density of

( n3) ( )

log10

8.4-6.5 log10 R
1cm G

for r
4RQ' Evaluate your answer, in solar radii, for v =
1000 MHz.

Problem 20.6. The deflection angle of light passing near the sun is given

by a =
1.75"/b, where b is the impact parameter in solar radii. Design

a thin lens (Le. give thickness as a function of radius) which models this

focal behavior. Take the solar disc to be a black mask of 8mm diameter

in the center of the lens, so that you can simulate the light deflection

experiment by holding the lens at arm's length. Assume an index of re-

fraction appropriate to ordinary crown glass, n =


1.52.

Problem 20.7. Calculate the expected perihelion shift of the planet

Mercury in terms of the semimajor axis of its orbit a, the eccentricity e,

and the mass M of the sun.


124 PROBLEM BOOK IN RELATIVITY AND GRAVITATION

Problem 20.8. Newtonian gravitation theory can be modified and made

covariant if the force equation on a point particle is written as

d plL ILV (I) dx13 pa (I) ,a dxlL


,v P {3
= -
11 +

where (I) is a scalar potential which is related to stress energy by

cI>;1L =
477 T IL ・

;IL IL

Investigate whether this theory agrees with experiment and observation:

(a) Is this theory in agreement with the experiments of Eotvos and

Dicke showing the equivalence of inertial and passive gravitational mass?


(b) Is this theory in agreement with the Pound-Rebka experiment on

the gravitational redshift of a photon on the earth's surface?

(c) Does this theory predict the bending of starlight near the sun?

Problem 20.9. A physicist wishes to take advantage of the tremendous

precision of current atomic clocks by using them to test both special and

general relativity. He places various clocks at different locations on the

earth (assumed to be rigidly rotating) and measures their ticking rates with

respect to some standard clock. Both the doppler shift, due to the earth's

rotation, and the redshift effect, due to the earth's gravitational field, make
contributions to deviations in ticking rates. Calculate the measured tick-

ing rate of a clock located at (r, e) relative to a standard clock of your

choice. Take into account the rotational deformation of the earth's surface,
assuming the earth is a rigidly rotating perfect fluid.
CHAPTER 21

MISCELLANEOUS

ProbJems in this chapter deal mostly with variational techniques, thin

shells of matter, and spinors.

o o o o o o o

Problem 21.1. Show that



11 2"
(i) o(-g) 2" (-g) gllV
ogllv

(ii) ogllV = -
gPIl gCW a gpa

Problem 21.2. Let L =


L(cI>A, gl1 v ) be the Lagrangian density for some


field or matter distribution. The field is described by the variables (I) ,

where A represents any tensor indices. The action is

s =

J L(_g)t d 4 x ・

The functional derivative oLio (l)A is defined by making a variation (l)A


(l)A + a (l)A and taking the change in S to be

J (_g)t d 4
oL a (I)

aS = x .

a (l)A

Show that oLio (l)A =


0 is the usual Euler-Lagrange equation when L

depends on (l)A and its partial derivatives


(l)A,a'
Problem 21.3. If L is a Lagrangian density as in Problem 21.2, the

stress-energy tensor can be defined by a variation of in S:


gllv

O(L(-g)2)
J J (_g)t d 4
4 1
aS =
og IlV d x == TIlVOg x .

o g 2 Il V

Il

125
126 PROBLEM BOOK IN RELATIVITY AND GRAVITATION


Show that T/l .

0 follows from the equation of motion of the field and

the fact that S is a scalar.

Prob Iem 21 .4. Cons ider the action

J (-g)t f Lmatter(-g)t
4 4
S =
(1617)-1 Rd x + d x ,

where R is the Ricci scalar and


Lmatter contains g's but no r's (so
that r"s are present only in R).
(a) Treat the g's and the r's as independent field variables
("Palatini method "), and show that aS =
0 leads to the Einstein field

equations and the usual formula for the f"s in terms of the g's. (Assume


rav .)

13v
(b) Now assume r"s are Christoffel symbols used to define covariant

derivatives in the usual way. Show that aS 0 (where now


ora v
IS

not independent of oga )leads to the Einstein field equations.

Problem 21.5. The Lagrangian density for a scalar field is

(817)-1(<<1>.,a (I);a + m ?1>2).



L = -
Find the equations of motion and the stress-

energy tensor. Verify explicitly that the stress-energy tensor has vanish-

ing divergence.

Problem 21.6. The electromagnetic Lagrangian density is

L =
(1617)-1: F/Lv F /LV where F /LV AV;/L A/L;v' Show that the Maxwell
- = -

equations Fa . 0 follow from setting to



the variation of zero
,fJ


f L(-g) "2 d x with respect to A/l' Find the stress-energy from the pre-

scription
oL
T "V 2 +
g"v L
= - -

og/lV
r r

Show that an equivalent Lagrangian density is

1 1
L = -
F F llv -
F llv A
1617 IlV 417 Il;V

Fllv is antisymmetric and FIlV and must be varied inde-


where
All
pendently.
21. MISCELLANEOUS 127

Problem 21.7. A Lagrangian for the Brans-Dicke theory is

L ((I)R w(l),a (I),a (1)-1 + 1617


Lma tter)
= -

where (I) =
scalar field, R =
curvature scalar, w =
coupling constant.


Derive the field equations from {) fL(-g)2d x = 0 by varying
f3
and <II.

Problem 21.8. A surface layer is a timelike 3-surface separating two

regions of spacetime. In general relativity the intrinsic geometry of such

a 3-surface is well defined, but the extrinsic curvature may be discon-

tinuous. That is, we may get a different extrinsic curvature tensor K if

we evaluate it with respect to the 4-geometry on one side or the other. The

surface stress-energy Sa contained in such a surface layer is defined as


{3
+E


f3;;


f3
dn
-E

where n is proper distance perpendicular to the 3-surface. Use the

initial value equations to find the discontinuity in K in terms of Sa


{3.

Problem 21.9. For a surface layer described with Gaussian normal coordi-

nates n and xi (i =
1,2, 3) (see solution to Problem 21.8) derive the
equation of motion of the surface layer

Si, + [Tn,] =

\1

j J

where the square brackets denote a discontinuity across the surface and

the slash denotes covariant differentiation with respect to the intrinsic

geometry of the 3-surface.

Problem 21.10. A thin shell of dust in vacuum has surface density of

mass a as measured by an observer comoving with the dust. If u is the

4-velocity of the dust, show that

[K..]
lJ

817a
( u.u.+
1 j


(3)g..lJ )
da i

au
Ii

dr

+ -

a -
a =
4 17an
+ -

a + a =
0 .
128 PROBLEM BOOK IN RELATIVITY AND GRAVITATION

Here a+ and a- are the 4-accelerations measured on the outside and

inside of the shell, respectively.

Problem 21.11. The vacuum geometries exterior to and interior to a

collapsing spherical shell of dust are the Schwarzschild geometry

_(1_ 2)dt2 (1_ )-ldr


2 2M 2 2 2
ds =
+ + r d0

for the exterior, and the flat geometry

2 2 2 2 2
ds = -
dT + dr + r d0

for the interior.[The radial coordinates in these metrics both, clearly,



have the property that 47Tr is the proper surface area of the spherical

surfaces r =
constant, and t or T =
constant.]
Showthat for the collapsing spherical shell of dust the "rest mass of

the shell" /l == 47TR (r) a is constant. Here a is the surface mass density

of the shell and the area of the shell as a function of proper shell time is

47TR (r). Derive the equation of motion of the shel1

2 !
/12
[ ( )J
dR
M =
/l 1 + -

dr 2R

and integrate the equation to find (in implicit form) R(r) in the case

dR/dr = 0 at R =
00.

Problem 21.12. Find an instantaneous spatial metric which represents

N point masses at arbitrary positions at an instant of time symmetry.

Problem 21.13. Suppose one identifies four-vectors if with 2-index



AA
spinors U by
O 1 2 3

[ ]
1 U + U u + iU
O 1 2 3
(U ,U ,U ,U ) 2-2 .

2 3 O 1
U _iU U _

What is the analog of the Minkowski metric in spinor language? Le. Find
AA' BB'
U V {3
a ..

LAA'BB' such that U.V U V (Hint. LAA'BB'


l1a{3
= =
an .

(_ ).)What is the analog of


AB
use the spinor (E AB ) =
(E ) = a Lorentz

trans formation?
21. MISCELLANEOUS 129

[Note: The spinor notation used here and in the following problems is

that of e.g. F. A. E. Pirani in A. Trautman, F. A. E. Pirani, and H.


Bondi,
Lectures on General Relativity, Brandeis 1964 Summer Institute on Theo-
retical Physics (Prentice-Hall, 1965).]

Problem 21.14. Show that

(a) 0
EA[se CD ]

'AB 'CAB) + t cAB 'c



(b) ==

where eAB is an arbitrary 2-spinor. [Note: This problem and the two

following were suggested by T. Sejnowski.]

Problem 21.15. Let Tab Show that if Tab is antisymmetric


TAA'BB'.
then its dual in a spinor representation is i
*T
ab

(TABB'A' -

TBAA'B')'
Ptob,lem 21.16. Let Tab ==

AA'BS'
in a spinor representation.. What
tensor corresponds to
TBA'AB'?
SOLUTIONS
CHAPTER 1: SOLUTIONS


2 -2 °
Solution 1.1. u =
(y, yv), where y=(l-y) =u =dt/dr. Thus:

(1_ 2)-2

(a) u =



(b) u =

(1_ 2)-2vj
(c) u.u =
-1 since it is a 4-velocity, so

1 1
1 2 22 3 22 j



[ 1+ (u ) + (u ) + (u ) ] ==
(1+u u.)2

(d) d/dr =
(dt/dr) d/dt =
(1- y?)-2 d/dt


. . . .

l 2
(1 + u u.)-
J J J
(e) v = u /u = u



(f) II =
[ 1- (u

) -2] ,from (a).

Solution 1.2. The product of the transformations is

0 0 0 0 0
yx yxvx yy yyV y YxYy Yxvx Yxyyv y
0 0 0 1 0 0 Yx 0
Yxvx Yx _ Yxyyv x YxYyVXv y
0 0 1 0 0 Yy 0 0 Yy 0
YyV y YyV y
0 0 0 1 0 0 0 1 0 0 0 1

but

0 0 0 0 0
Yy YyV y Yx Yxv x YxY y YyYxv x YyV y
0 1 0 0 0 0 Yx 0 0
Yxv x Yx Yxv x

0 0 0 0 1 0 0
YyV y Yy Yxyyv y YxYyvxv y Y

0 0 0 1 0 0 0 1 0 0 0 1

which is different.

be the 4-velocities of In
Solution 1.3. Let u

and u

2 -2
tre two frames.

frame 1, (l,Q), u (y, yy) where Y



(1- v) Since y= ul. u 2'

u = = .

1 2

133
134 PROBLEM BOOK IN RELATIVITY AND GRAVITATION

we just have to evaluate this scalar in general frame where


a u (y l' Y 1-1
v )

1 '

(Y2'Y2 v
Q ):
u =



2 2"
Y 'u
YI Y 2 YI Y 2 ..Y.l.X2 (1- v )-


u = -

1 2


1-v =

(YI Y 2)-2/(1-.Yl'.Y2)2


( 1- '.Y2)2

(1 -

1 2) ( 1 -



) -:i.2)2
1 -

<Y.l x:i.2 )2
v =

(1- ' 12)2 (1- ..Y.l'Y2)2

(The solution could also be verified by a


painfully tedious application of
Lorentz transformations.)

Solution 1.4. We use the fact that in dimegsion the "rapidity parameter"
one

1 1 1
() == tanh- v adds linearly. Thus tanh- v () n(} n tanh- =
(3, or = =

n n





tanh (n tanh- (3)
1+
n /2
(3

tanh log
( ) 1-{3
(easily verified!)


1 -

[(1- {3)/(1 + (3)]n


1 + [(1- (3)/(1 + (3)]n

As n 00, [(1- (3)/(1 + (3)]n 0 and v



1, the speed of light.

Solution 1.5. The usual b/y answer refers to a measurement made simul-

taneously in the lab frame, but the photons are not emitted simultaneously

in the lab frame. They are received simultaneously so that photon 2 (see
diagram) has to travel for an extra time b' cos a, where b' is the

apparent length. From the Lorentz transformation

b = =
y(
x x'(3 t") y(b' (3b'

= -

cas a) ,

so that

b' =
b/y(l- (3 cos a) .
CHAPTER 1: SOLUTIONS 135

. b' .

.b/y+ .b¥
II11I1IIIIII111111111111
- - -,
..
. ) I
'---t ,

\\

,a ,"
u_
,""
\ "
-'" y' \q,
\ \/
\ \





\q, \
\ \


\ e(C)
\ (jo
\0
'-

Solution 1.6. When the two observers are at rest in the same Lorentz

frame, we have (distance) (velocity) x (time) and tround trip 2L/u.


= =

Things are more complicated when one is moving. The moving observer's
coordinates are

t =
yet -

vx)
x' =
y(x-vt) ,

and the tachyon emitted from the moving frame back towards the stationary

observer has velocity


dx'/dt' = -
u .

Now dx' =
y(dx -

vdt), dt' =
y(dt -

vdx), so substituting and solving for


the apparent velocity in the stationary frame gives

dx/dt =
(u-v)/(l-uv) .

Finally we add up times in the stationary frame ttotal


tout + t
back

L/u + L(l-uv)/(u-v) =
L[l/u+(l-uv)/(u-v)]. One can easily calculate


that 2
ttotal < 0 if u>[1+(1-v )2]/v.
A spacetime diagram is helpful in understanding this effect.
136 PROBLEM BOOK IN RELATIVITY AND GRAVITATION


II

...

.,

0"
".







\\ //
'\,/
\'


\\
-0
c:
6 \
;\ \
ci)
0" \
i\ \

o -c

,I c;
. ,


\0


, \.,c::,








y-


..

)(


/'


..
..
CHAPTER 1: SOLUTIONS 137

Solution 1.7. The rotation comes from the contraction in the x direction

but not in the y direction.



That is, cot () =
I1x/ y.Because of the
2"
motion, I1x = I1x' (1- f32) while l1y =
l1y'. Thus,

2"
cot () =
(1 _/32) cot ()' .

Solution 1.8. Not only are lengths contracted, but also times are dilated.
In S the bullet's velocity is

v =
(l1x/l1t,l1y/ t) .

From the transformation formulas for I1x and 11 t




I1x/l1t =
(v x '+f3)/(l+f3 v x ') .

Also, since l1y =


l1y', we have

1 1




l1y/l1t =
l1y'(1-f3 )2"/(l1t'+f3l1x') =


,(1-f32)2"/(1+
fJ x v
,) .

The direction of motion then is given by

1 1

tan () =

vy/v x
= v

'(1-/32)2" /(v '+f3) x

tan (}'(1_f32)2" /(l+f3/ v
x ') .

Except for the factor of (1- f32) 2", this change in direction is identical
to the Galilean result. It represents a "funneling" of the motion toward
2 2
the x axis. For a photon, v

+ v


1, v


cos (), so that

2"
()
cos ()' + f3 tan (}'(1- f32)
cos =
, ta n () =

1 + f3 c os ()' 1+ f3 sec ()'

Solution 1.9. If () is the direction of the motion of a photon from star to

observer then the star is observed at angle () = 7T -


(). The transformation

of photon directions was derived in the previous problem. From that re-

sult we have here


cos ()' -

f3
cos () =

1- f3 cos ()'

The number of stars observed in solid angle dO is then


138 PROBLEM BOOK IN RELATIVITY AND GRAVITATION

dN =
NdO/417 (N/417) 217 d(cos (})

= 1. N d(cos (})
d(cos (} )

2 d(cos (}')


N (1-{32) ,

2 d(cos (})
{3 ()')2

(1- cos

Since dN =
217 P({}', cp') d(cos (}') the distribution in S' IS

l! (1- {32)
P?()', cp') =

417
(1- {3 cos {}')2
Checks: {3 0 P({}', cp') N/417 =
P((), cp)

2 +1



P?()', cp')dO =

N(1 {3 J -1 (1-
217d

{3
cos

cos
()'

{}')2
=N.

sphere

You can easily verify that half the stars are contained between ()' =

-1
and {}'1..


cos {3 < 17/2. Thus the stars "bunch up" in the forward direc-

tion. For {3 1, {}'1.. [2(1- (3)]2 and the bunching is pronounced.


Solution 1.10. The length of A is IA.A\2 and

1 1 1 1
A.A =
(3 2 e t +22e x ) ・

(3 2 e t +2 2 e x )

3e.e
t t
+ 2e

'e

+ 2 V/6

e 'e
t x


-3 + 2 + 0 =
-1 .

If () is the angle between A and e then


A.e 1

cos () =


= -3 2

\A.A\2 \e t .e t \2

which is satisfied by no real ().

Solution 1.11. By symmetry, the clocks must read the same the next time

Adam and Eve meet. An easy way to see this is by considering proper

time intervals in a coordinate system fixed to an inertial observer at rest.


CHAPTER 1: SOLUTIONS 139

In polar coordinates
2 2 2 2
dcp2


dr = _
dt + dr + r + dz .
(1)

The coordinates of an instantaneously comoving inertial frame attached to

Adam are cp A =
wt, with rand z constant, and the coordinates of Eve
are
CPE = -
cut, with rand z constant. Thus


dr 2A 2 2 2

dr


dt (1 -

r cu ) (2)

and proper time intervals for Adam and Eve are identical.

A second method is to consider a noninertial coordinate system

attached to Adam. He may define surfaces of simultaneity by extending

hypersurfaces orthogonal to his world line, the distance between hyper-


surfaces being at equal intervals of his proper time. At points where his
hypersurfaces intersect Eve's world line, her proper time r

is read off

and Adam can compute r



as a function of r
E'
One proceeds by deter-

mining the 4-vector W which connects Adam's and Eve's world lines and

which is orthogonal to Adam's 4-velocity:

t =
yr A
x =
sin cut =
sin cuyr

Adam's world line: (3a)
y =
c os cu t =
cos cu y r

z = 0

t =
yr E
x =
-sin cut = -sin cu y r

Eve's world line: (3b)


cos cut = cos w r
y E
z =

where we have taken the ring to have unit radius. From these relations

W == x

-x

[y(rA-r E ), sincuyr A + sincuyr E , coscuyr A


coscuyrE,O]



(y, cuy cos cuyr
A'

cuy sincuyr A' 0)

so that requiring W.U =


0 gives







cuy-l sin [cuy(r A +r E)] ・
(4)
140 PROBLEM BOOK IN RELATIVITY AND GRAVITATION

Equation (4) is transcendental for r



(r E)' One may easily see, however,

that r =
r when
A E
sin 2wyr =
sin 2wt 0 (5)


Le. whenever Adam's and Eve's \vorld lines cross.

Solution 1.12. The Lorentz transformation is

t =
y( t vx) --

x =
y(x vt ) -

if w == it, y/' = it', this becomes

w =
yew ivx) -

x' =
y(x +ivw) .

To make this look like a rotation

w' =
cos Ow -
sin Ox

x =
c os 0x + s in Ow ,

we
obviously want

sin () =
ivy
c os 0 =
y .

2 2
y2(1_

sin 0 + cos 0 =
v ) =
1 .


So 0 =
arcsin (ivy) =
arctan (iv) =
+ i tanh- v .

Solution 1.13. Along the curve we have

2 2 2 2 2 2 2
dy2 + dz 2 2 2 2 2 2
dx + _
dt =
(r cos () cos cp + r cos 0 sin cp + r sin 0 _
r ) dA
2 2
dA2(cos


r 0+ sin 0 -1) 0 ,

so the curve is null.

To be a geodesic (Le. a straight line) we must have dz/dt =


constant,

but dz/dt =
sin (), so () must be a constant. From the expression for

dy /dt or dx/dt we see that cp must be constant. These are the require-
ments for a geodesic; theAe is no restriction on the function reA). If, how-
ever, A is to be an affine parameter (e.g. proper time) we must have

dt/dA a constant, and hence r a constant.


CHAPTER 1: SOLUTIONS 141

Solution 1.14. 4-velocities are normalized so that uaua =


-1, so

E.. (uau ) d ua
o = =
2 U =
2a au .

dr a dr a a

This is one constraint on the 4-components of acceleration aa. In the

momentarily comoving frame u




(-1,Q), and the constraint requires

aO =
0, with aj(j =
1, 2, 3) arbitrary. A Newtonian accelerometer can be

modeled as follows: Have the observer release a particle in the comoving

frame and see how much velocity dy- the observer picks up relative to it

in a short time dr, then calculate a N ew t onlan dv / dr. -




Of course the

particle is really stationary in the momentarily comoving inertial frame,


and we accelerate relative to it by an amount du j =
ajdr. Now u
j =

1 1 1

2 2
du j j



(1_v )-2, so =
(1_v )-2dvj + v d(1-v )-2. But v =
0 since
j dv j
the frames are momentarily comoving, and thus du =
, so finally


j =
dv j /dr =
duj /dr =

j .

Newtonian

The 3 independent components of 4-acceleration in a comoving frame are

just 3 Newtonian accelerations.

Solution 1.15. Since a.u =


0, in the observer's local rest frame


a =
(O,a ),

where aT is the jth component of locally measured acceleration. Then,


the squared magnitude of his locally measured acceleration is

" " "


aja--=- j j
a _



(0, a ). (0, a ) = a.a .

Solution 1.16. The transformation of 3-velocity, from Problem 1.8, is

-1
u'
y u =
U\\-V
u\\

1 l'
(l-vu\\) 1-vu\\
142 PROBLEM BOOK IN RELATIVITY AND GRA VITA TION

and the time transformation is

t' =
y( t -

v. X) .

--

Now take differentials:

y-l du 1 vy- 1u l y-l[(1- vu


ll)du 1 +vu
1 dUll]
dui =

(1- vu
ll)

dUll

(1- vu
ll)2 (1- vu
lI)2
, dUll v(v-U\\)dull y- 2du ll
du -

II

1- vu
ll (1-
vUII)2 (1- vU11 )2

dt' =
y(dt -

y.d =
ydt (1- .}!) ydt (1- vUII)

d!!'11 , Y
-3
y-3
dt'
==
\I


-11

!I

(Answer)
(1-
VUII)3 (1-v.u)3
--

and

-2
d 'l, Y
==
l [(1-VUII) +V.!!l
1 an]

dt'"" 3
(1
\I )

vu

-2

(l- vu ll)
3 [ -V(UII
l l all)]
-.l!l

-2

[a1-vx(axu)] (Answer)
(1-v.u)3
- - - -

--

[In the last line we have used the fact that

(x+l II) (]!l + II) x


x + x

E.II

II E.l
and the rule for expanding vector triple products.]

Solution 1.17. If the acceleration is in the x direction we can clearly


choose

y =
y, z= z. Note that the world lines

t =
A sinh gt + B
x =
A cos h gt + C
CHAPTER 1: SOLUTIONS 143

represent world lines with constant acceleration. [Easily verified. See

also Problem 2.13.] If A, B, C are functions only of x then the above

equations represent a
4-velocity

u =
cosh gt e

+ sinh gt ex '

independent of x, and they therefore give us a set of world lines which

are parallel on a t =
constant surface. If, furthermore Band Care
constant, u is perpendicular to the hyperplane dt =
0 (Le. the hyper-
planes t =
constant are hyperplanes of simultaneity for the momentarily
comoving inertial frame). It remains only to choose A( x ), B, and C
correctly: -1
t =
(g +x ) sinh gt

x =
(g-l+ x )cosh gt _

g-l .

Notice that the element of proper time for a coordinate stationary ob-

server IS 1
2 2 2"
dr =
(d t -
dx ) =
(1 + gx ) dt .

Since it does not depend only on t, clocks will not remain synchronized.

Le. They can only agree on one hyperplane.

Solution 1.18. Without loss of generality take the mirror to be in the xy

plane, and take the photon to be in the yz plane. If "before" and


"after" mean before and after reflection, we have for a photon in the lab

frame:
Pbefore

(E, 0, E sin e, E cos e) ,

Lorentz transforming to mirror frame,

Pbefore

[yE(l+v cos e), 0, E sine, yE(v+cos e)] .

Reflecting in mirror:

Pafter

[yE(l+v cos e), 0, E sine, -yE(v+cos e)] ,

Lorentz transforming back to lab frame:


144 PROBLEM BOOK IN RELATIVITY AND GRAVITATION


P after

Iy E [(1+v cos O)+v( v+cos 0 ) ] ,
0 ,

E sin 0, y2E[_ v(l + v cos 8) -

(v + cos 8)]} .

So that, finally:
p3 2
(1+v )cosO+2v
cos
81after =


= +

p l + 2v cos 0 + v

1+2V coS8+v2


E =
p after =

after 2 before
1 -v

Solution 1.19. Let E and £ denote the energy and momentum of the

photon. In the frame of the mirror (primed frame) E'.In =


E'au t ' P'x .

In

P'x the lab frame, 81 82


P'y P'y In wish to show


we =

out' in

out'

tan 8

(P x /P y)in P.
x In
P'.
y In
P .

x In
= = =

tan ()
2 (Px/-Py)out P
x out
-P'
y out

x out

where have used


we
P'y P Then, Lorentz transforming P

y' x'

tan 8
1 =
y(P'x + {3E')in =
1 .

tan 8
2 y(P + {3E')out

Hence the angle of incidence equals the angle of reflection in the lab

frame as well as the mirror frame.

in I

t y


82
: x




CHAPTER 1: SOLUTIONS 145

Solution 1.20. In the rest frame of the observer,

uo =
1 u
j =

°

= -

Po

pj =
p.



_

so, we verify the invariant formulas in this easiest frame:

(a) E=-pouo=-p,u

_lp12

(b) m =

--

-p o po_p.pj


-p,p
1 -

2 2- 2
(c) Ipi-

(E -
m )2 =
[(p.u) +p.p]2
Ipl i 1-


2 2

J [ J
=- E -
m p'p 2
(d) Ivl =


= =
1+

(p.u)2

(e) In the rest frame,



= _



pO = -
1+ 1 =

(- E)


j = -


- -
pj =
0 +
d J
.-i
(-E) dt

Solution 1.21. There are three 4-velocities to consider. The laboratory

observer has ulab


(1,.2); the nucleus has u


F e

(y, y(3),-
where y =

the photon has


(1- . )-2; a 4-momentum constant x (1, -n)

Py ,

since it is traveling toward the observer. Also notice that the photon is

-1
null,
Py.pyex

(- (-
). =

)O. From Problem 1.20, now


Vobserved =
lab =
-ulab.Py =

(Answer).
Vemitted E
Fe
-u
Fe 'Py y(l + (3.n)
--
--

Solution 1.22. The doppler-shift formula (Problem 1.21) gives




1 =

obs _


C1_v )2
v l+v.n--


v.n 2 2
cos (} _----

- -
C1 -v ) -

v V

() _

cos
-1

[ C1-v2)2_1

] .
146 PROBLEM BOOK IN RELATIVITY AND GRAVITATION

Solution 1.23. Make an arbitrary boost in a direction which lies in the

y-z plane. The photon now may have nonvanishing pY and pZ as well
as pX. A pure rotation lines up the coordinate frame
again so that only

pX is nonvanishing, but pX does not now have its original magnitude.


So, make a final boost along pX either to redshift it or to blueshift it to

the original value. Since E -

p2 =
0, E also has its original value.
You can easily convince yourself that the product of these transformations

is not a pure rotation; there is in general a net boost left over. An exam-

p Ie is:

x -

boost x-y rotation y



boost

y y'v' 0 0 0 0 o

y yv
, , , 2"
yv y 0 0 (1_v2) V

0 0 1 0 0 E

2"
0 0 1 0 -v (1-v2) 0 yv o y 0 o

0 0 0 1 0 0 1 0 o 0 1 o

where v' is chosen to satisfy the equality

y'(l + v') =
l/y ,

which gives
2 2
v' = -
v / (2 _

v ) .

Solution 1.24. It is not possible for one inertial frame to jerk back and

forth with respect to another. The "paradox" comes from the tacit and

erroneous assumption that the cylinder remains rigid when the frogs hit
the ends. This cannot be true since the elastic waves that inform one

end of the tube that the other has been hit must propagate along the walls
of the tube at less than the speed of light. In the freely-falling frame of
the cylinder, the two ends are driven outward by the impact of the frogs.
Tension waves fly from each end of the cylinder toward the other. Not

until the waves meet at the center of the tube do they discover the exist-

ence of each other. Then they pass each other and counteract each

other's effects, pulling the tube back to its original shape. The tube
then pulsates back and forth in its fundamental mode of vibration. During
CHAPTER 1: SOLUTIONS 147

Q. -cu
4&) o u

.." u c
--

I ....
...

CD cu._
C U >-0

"1:J 0
-.::"
o c
. Q)


q-

eQ)
as
....
C+.I

rn

......
Q)
"tj

.....
....-4

>a

(J

.....


Q) Q)
rn

rn

CD

+=
CI)
> E
0 .e
. ..

c .a
.2 I
en
c CD
CD
..

:2

..
148 PROBLEM BOOK IN RELATIVITY AND GRAVITATION

'- --
OU au
uc
I .5 -a
..
CD>' fit
CD._
c u


i:
o c

Q)

as

.....
as
'.-4

""
Q)
c=
'.-4


as

c=
..-4

c=
Q)
Q)
rJJ

rJJ

-E

;:
..
CD

>a

CD

..

:I .. c

0=
co

.CiS I



.-


:2.
..


CHAPTER 1: SOLUTIONS 149

this pulsation the frogs hit the ends of the tube time and again, each time

changing the amplitude and phase of the pulsation.


From another freely-falling frame (perhaps instantaneously comoving
with onlooking birds), the cylinder rushes by at speed f3, and the frogs
do not hit the ends of the tube simultaneously, the pulsations of the two

ends will be out of phase with each other, but the overall picture will not

be much different. In particular, the center of the tube (which is at rest in

the inertial frame of the cylinder) will not jerk back and forth in any other

inertial frame. The accompanying spacetime diagrams illustrate the

phenomena.

Solution 1.25. Let us first derive the forms of the infinitesimal operators

by considering their action on a spacetime function f(x, y, z, t):


(i) Rotations about z axis:

x' x -

yO
y' y + xO

(1)
z = z

t =
t ,

f(x', y', z') -

f(x, y, z) =
0 [- ya x +
xay]f , (2)
thus

J z

-2i(xa y -ya x ) , (3)

since

f(x',y',z') ==
(1+iJ z O/2)f(x,y,z) .

Jy , Jx are obtained by cyclical permutations of Equation (3).

(ii) Boosts in z direction:

z' z -

vt

t t -
vx

y =

z =

150 PROBLEM BOOK IN RELATIVITY AND GRAVITA TION

f(x', y', Z', t') -

f(x, y, z, t) =
[- ta z -
Za t ] V f(x, y f z, t)
thus

Kz

2i
(t fu.-), + z etc. (4)

(iii) Now, commutation relations may be worked out from Equations

(3) and (4) by the usual procedure, for example:

[Jx,J y ] =
-4[ya z
-za ,za X -xa ]
y z

4(xa y -ya ) x

2iJ z ' (5)

and similarly for all other commutators.

(iv) A representation of the Lorentz group is specified by giving ex-

plicit matrices for the generators. We can associate the operators with the

Pauli spin matrices

[ ]

a =
etc.

according to

J -+ a

K -+ ia .

The commutation relations for J and K


__
are satisfied since [a x
, a


] =

__

2iaz etc. To find the matrices corresponding to finite transformations L,

note that for example J z is defined by

dL iJ z
dO 0 I =


so that

L(O) =
exp (iO J z/2) ・

An arbitrary finite transformation L will have six parameters; in the

general case the rotation parameters !!..* (here treated as a 3-vector) will
be different from the physical rotation angle !!.. and the parameters repre-

senting boosts, v*,


--
will differ from the physical relative velocity v.
--

These are details; the Lorentz group is defined completely by its genera-
tors J, K;
--
--
its parametrization is largely arbitrary. The infinitesimal
version of an arbitrary transformation L is
CHAPTER 1: SOLUTIONS 151

* * * * * * * *
(O X J x+ O Y J y+ 0zJz+v x K x+ v yKy+vz Kz)/2 ( 10 -y)' ,!!12.
. .

oL =
1 =

Let us define


--
==
(iO*---


---
*)/2
1. (_0*2 -2iO*. V*

q2 _

q.q
........

4 ,..", .......
+V )

(Note that pure boosts and pure rotations correspond respectively to pure

real and pure imaginary q.) We have then, for the general finite

trans-

formation: 00

L( ,y) exp<1'£)
= =

2 (.?!'£f In!
n=O

00 00

2 qnIn! +
2 (q.a)qn-l In!
--

n=O,2,4". n=1,3,S".


cosh q + (q. al q) sinh q .

Here we have used



..

lJ
(q ) ( q a) (a., a.) q.1 q.J a q


a . =
q.1 q.J
= .

_
_
_
_ 1 J

Solution 1.26. The hard way to solve the problem is by multiplying 4 x 4

matrices together and then decomposing into rotations and boosts. An

eas ier way is to use the 2 x 2 complex unimodular matrix representation

of the Lorentz group developed in Problem 1.25. Represent a pure boost

of velocity vl by

L(Y1)

exp (- v;,!!.1'QI2) =
cosh (v /2)(.!!.1',q) sinh (v:12)
+ (la)

where .!!.1 is a unit vector in the direction of the boost and v


1 parame-

trizes the magnitude of the boost. Represent a pure rotation !l. by

L( 1)

exp (iO*E.'£/2) =
cos (0* /2) -
i (,!!.'£) sin (0* /2) (lb)

where n is the rotation axis and 0* parametrizes the magnitude of the


rotation.
152 PROBLEM BOOK IN RELATIVITY AND GRAVITATION


For pure boosts in the direction, is in fact the
same v
rapidity
* 1 *
parameter, v =
tanh- Ivl, since v adds linearily, e.g.
* * * *
-v



/2 -v a /2 -(v 1 +v2)ax /2
2 x
L(v 1 )L(v 2 ) =
e e =

Likewise one can see that for pure rotations in the same direction, 0* is

just the magnitude of the rotation angle, 0* =


1 0 1.
Using Equations (la) and (lb) one now solves the relation

LC! 1) L(y 2) =

L(O.!!) LCy 3) (2)

for 0, , in terms of and


1 A useful identity in multiplying
Y3 2.
the right hand sides of Equation (2) together is

(A, a) (B. a) .,.....,....--..,



A. B + i (A x B) ・
a .

.--... ....--.., "....,

Since the a and identity 2 x 2 matrix are all independent, one may

equate, in Equation (2), the real and imaginary parts respectively of terms

multiplying the identity matrix and of terms multiplying the a matrices,


yielding two scalar and two vector equations:

cosh
O v0 cosh
O v;) + sinh
O V:)SinhO v;) 1' 2)
(3a)

cosh
G V;) G 0*) cos

sin
G 0*) sinh
G v;) <' .E.3) =
0 (3b)

cosh
O v:)SinhO V; 2sinh( v:)cosh ( ;)n +
O O*)sinhC} V: 3 v


cos

G O*}inh G v;)( X!!3)


+ sin (3c)

Sinh
O vnSinhG V;)( xl 2)-sin

G O*)coshG v;) .(3d)


In the general case, the solution to Equation (3b) is
CHAPTER 1: SOLUTIONS 153

n.n =
0 (4)
--
--3

Le., the resultant rotation is orthogonal to the resultant boost.

Let y angle between


be the and
1 ..!!2' Then, dotting into
Equation (3d) and combining with Equation (3a), one finds

tan(l e"\) 2

cosh


Note that () (= () by above discussion) may take on all values 0 () 2"

except () =
", and that for n



2 (y =
0), () = 0 and there is no net

rotation as expected.

Solution 1.27. An astronaut looking at the sky (by null photons!) makes
an arbitrary Lorentz transformation, and looks at the sky again. There
results a continuous map of the sky onto itself. Any continuous map of a

2-sphere onto itself has at least one fixed point.

Solution 1.28. From Solution 1.25, any homogeneous proper Lorentz trans-

formation corresponds to a 2x2 complex unimodular matrix L, or a

complex 3-vector f, say. We will write the 2x2 matrix as

L(P)


(1+p2)2I + p.a
- -

where p2 =
p.p, and I is the identity matrix which
--
we will often not

write in explicitly. (In Solution 1.25, we used P




q(sinh q)/q.)

For a

pure boost with velocity v and direction n, P is real: P =


n sinh (tfr/2),
.,....., .........

where tfr =
tanh v is the rapidity. For a pure rotation of magnitude ()

about an axis , is imaginary: f = i sin ?()/2) and p2 2: -1.


The composition of two Lorentz transformations represented by P

and Q IS 1 1

poQ =
(1+p2)2Q+(1+Q2)2P+ipxQ. ...................

This follows from multiplying the matrices corresponding to the Lorentz

transformations:
154 PROBLEM BOOK IN RELATIVITY
AN:Q GRAVITATION

1 1
2 2
L(P)L(Q).........

[(1 + p2) + p.a] [(1 + Q2)
........
+ Q.a]
....................

1 1 1 1

(1+p2)2(1+Q2)2 +[(1+p2)2Q+(1+Q2)2p]. a+P'Q+iPxQ.a. --

..-' ..-'
-----
........

The idea of the solution is first to find a characterization of the product

of two pure boosts (Lemma 1), then to find a criterion for the case that an

arbitrary Lorentz transformation is the product of three boosts (Lemma 2),


and finally to show that this criterion is always satisfied except for the
case of a "180° screw", which requires four boosts.

LEMMA 1. P is the product of two pure boosts C and D if and only if


p2 is real and positive.

Proof. If P is the product of £ and Q (real, non-null vectors), then


1 1
2 2

..........

CoD
.........,.......

(1+c )2D .........
+ (1+D )2C + i CxD ,..,....

(1)

1 1
2 2 2 2 2 2 2
p2 =
(1+c )D + (1+D )C + 2(1+C )2(1+D2)2C. D ...........
_
C D + (C'D)2
...........

1 1


[(1+c )2(1+D2)2+£'Q]2 -
1 (2)

2 2 2 2

[(1+2CD+C D )2 +£'Q]2 _
1 since C + D > 2CD

2: (1+ IC.DI +C.D)2


..--,.............. ........

1 since CD 2: IC.DI
..........

>0.

Conversely, if p2 is real and positive, then one can write =



i ,
2 2
where B .0 =
and A -
B > O. To construct boosts C and D, choose

a vector E

orthogonal to both A
--
and B with E to be determined. As

an ansatz, take
C =
aA + D
, =
aA -
E , (3)

where a is a normalization constant to be determined. From Equation (3)


2 2 2 2 2
C =
D =
a A + E and by Equation (1)
CHAPTER 1: SOLUTIONS
155

1 1
2 2 2 2 2
A =
(1+C )2D +
...................
(1+D )2C =
(1+a A +E )22aA ...........

and


.-......

C x D
........,.......

2aE x A .

Thus the construction will succeed as long as


2 2 2
1 =
2a(1+a A +E )2 , (4a)
2 2 2 2
B =
4a E A .
(4b)

Square Equation (4a) and solve for a :




{[(1+E2)2 +A2]2 -(1+E 2 )1/(2A 2 ) .
(Sa)

Equation (4b) now implies




B =
2E2{[(1+E2)2+A2]2 -(1+E 2 )1 .

After squaring and simplifying, this reduces to

2 2 4 2 2 4
4(A _
B )E _
4B E -
B =
0 , (5b)

2 2 2
which always has a positive solution for E since A > B . The

parameter a is given by (Sa) and the proof is now complete.

Now try to express any Q


---
as a product of three boosts. By Lemma 1,

this is possible if and only if there exists a boost £ such that p2 is

real and positive, where P


--
==

--

(-C). By algebra similar

to that used

to reach Equation (2), we find

1 1 1

(1+p2)2 =
(1+Q2)2 (1+c )2 -

Q' C
--
--

(6)


So we must find C
--
such that (1 + p2) evaluated from

Equation (6) is

real (Q is in general complex) and (1 + p2) > 1.

Let Q
,.."",

A+ iB
........
(A and
.................

........
real), and let the boost D be some

linear combination of A and B.


156 PROBLEM BOOK IN RELATIVITY AND GRAVITATION

LEMMA 2. There exists a boost £ satisfying the required properties if


and only if there exists a boost .Q which is a linear combination of A

and B such that 1 1

d ==
(1+Q2)2(1+D2)2 -Q.Q (7)

with d real and strictly positive.


Proof. Let F be a vector orthogonal to both A and B with F < 1 to

be determined shortly. Define




C =
Q2+f)/(1-F )2
2 2 2
so that 1+C =
(1+D )/(1-F ). From Equation (6),

1 1 1 1 1

(1+p2)2 =
[(1+Q2)2(1+D2)2 _Q'0](1_F2)-2 ---

d(1_F )-2 .


Thus if and only if d is real and positive, we can choose F close


enough to 1 that (1 + p2) is real and has magnitude greater than 1.
Lemma 2 is proven.

Now consider two cases:

2 2
Case 1: not parallel to A
, 1= 0, and B 1= 0 .

In this case Equation (7) gives

Q '.Q)2 (1 + Q2) (1 + 0 )

(d + =

Substituting 2.

+ i gives two real equations:


(d+A'D)2
..........

(B'0)2
.........

(1+A -

B2)(1+02) , (8a)


(d+AoO)B.D ................. ....-...........

A.B(1+D ).
.................
(8b)

Solve Equations (8) simultaneously for d + A.D and B.D to get:


--

(d+ 'Q)2 =

(1+D )1[(1+A

_B2)2 +4( '! +(1+A
i]t 2
_B2)} , (9a)

(B.D)2
...................
= 1. (1+D 2 )1[(1+A 2

B2)2 +4(A'B)2]t_ (1+ A 2 B2)}.


..................

(9b)
CHAPTER 1: SOLUTIONS 157

equations do 2
.Q and/or
These not determine d explicitly, because D
occurs on the right-hand side. One possible solution of these equations

is to take .Q =
bJ2, b to be determined. For this choice Equation (9b)

becomes



B2)2 +4(A.B)2]2
2 4 2
(2b B )/(1+ b B2) =
[(1+A _

--
_

(1+A
2 _

B2) .
(10)

To show that there is always a real solution b of Equation (10), note

that the right-hand side is 2:: 0 (obvious) and also < 2B2; this follows

since
2 2
4(A.B)2
--
< 4A B


(1+A2+B2)2 _

(1+A2_B2)2 _
4B

(1+A +B2)2 (1+A _B2)2


< 2 _


Because the left-hand side of Equation (10) varies from 0 to 2B as b

varies from 0 to 00, there is always a real constant b such that Equa-

tion (10) holds. Thus Equation (9b) is satisfied by our ansatz, and then

Equation (9a) (with Q =


b and with the sign of b chosen appropriately)

defines a positive value of d. Thus Q is the product of 3 boosts. --


2 2
Case 2: -
A parallel to B (including A =
0 or B =
0) .

Now Equation (7) becomes a scalar equation since all the vectors are

collinear: 1 1

d =
(1+Q2)2(1+D2)2 -QD. (11)
Let
-1
tfr =
sinh Q =
a + i {3 ,

and make tfr single-valued by the restriction -TT/2 < (3 :S TT/2. Also let


cp =
sinh- D (real) .

Equation (11) then becomes

d =
cosh(tfr-cp) =
cosh(a-cp)cos{3 -

isinh(a-cp)sin{3 .
158 PROBLEM BOOK IN RELATIVITY AND GRAVITA TION

If {3 0, done is boost). 0,
we are (Q a pure If {3 for d to be real

we must choose c:p =


a (this fixes D); Le. d =
cos {3. If {3 11/2, d IS

positive and 2. product of three boosts. If {3


is the =
11/2, d =
0 and Q
is not the product of three boosts.
The case {3=11/2 corresponds to Q=sinh(a+i11/2)=icosha. So Q

is pure imaginary and Q2::; -1. This corresponds to a rotation through


180 0
composed with a boost of rapidity 2a, a "180

screw." (In particu-

through 180

lar, a pure rotation cannot be attained by three boos ts..)

A 180 screw can be achieved by four boosts; for, if C is a boost

and Q =

i with B 1, then
--

1 1


.....
==
QoC
........

i(B -1)2C+i(1+C 2 )2B_BxC
"""'"" ......... ..........


is not a 180 screw provided £. is not parallel to B . Therefore R is

the product of three boosts and so Q = 0


(-C) is the product of four
boos ts .

This solution is due to D. M. Eardley.


CHAPTER 2: SOLUTIONS

Solution 2.1. From 4-momentum conservation we have

P + P =
P' + P' .

eye y

(Here y indicates the photon; prime denotes values after scattering.)


Since we are not interested in the electron's final momentum we use a

generally useful trick to eliminate it:

(P e +P _p')2 =
p2,= _m2
y y e e

or, since
p =
0,

2 2

m + 2P .
P -
2P ・
P' -

2P ・
P' = -
m .


ey ey yy e

In the lab frame, we have

Pe =
(me'.Q)



(h/A, h/A -1
e.), e.
--1

unit 3-vector in incident direction,



(h/A', h/A' o), o =
unit 3-vector in outgoing direction,

so that
h h 2 2
me me h h

+ + -
cos 0 =

T T XX' XX'

Multiplying through by AA', we find

A'-A =
(him)

(1- cos 0) .

Solution 2.2. Use y to denote the photon, and prime to denote values

after scattering. We proceed by eliminating P' from the equation of

4-momentum conservation:

159
160 PROBLEM BOOK IN RELA TI\7ITY AND GRAVITA TION

(P'

+ p' _


)2 =
p2 = _

p. (P

p P )

= -

y y

obviously

Maximum energy transfer occurs for a 180 scattering angle. With
this condition and
Ifyl Ey' the above equation becomes


E E' + P P' ・
= -
E(E -

E') + P .

(P -
P' )
y y -y -y y y - -
y -y

-2E E' =
-E(E y -E') + P(-E -E')
Y y Y Y Y

so that

E' =
Ey(E+P) E
y 2
2Ey+E-P l+m /4E
Ey
The second expression for is result of using the approximation
E a

P=(E2_m )2

E - 1. m

/E

appropriate to our assumed condition E? m .

0 4
For 3 K, the photon energy is of order kT 3 x 10- eVe Using
9 20
this, m =
.938 x 10 eV, and E =
10 eV in our equation we find
pro t on
E' 101geV.

Solution 2.3. By conservation of 4-momentum, we would have to have

(1)
Py+Pe Pe'

where is the photon momentum, Pe and P are the electron's momen-


Py
tum when the photon is/is not present.

Squaring both sides of Equation (1)

, ,

Py Py
+ 2p e

Py
+ P
e' Pe =
Pe' Pe

2 2
o + 2p P m ・ -
= -

eye e



Pe Py


CHAPTER 2: SOLUTIONS 161

But in the frame where (m, Q) and this


Pe
= =
(E, p ), says that the
Py
photon energy must be zero, Le. there is no photon. Thus the process
cannot occur.

Solution 2 4 . If P is the 4-momentum of the resultant system, and


'1 y ==


(1-v I )-2, we have

P =

(mly, m 1 YYI) + (m 2 ,Q)


and
1 1
22-
m =
(_p'p)2 =
(m 1 +m
2 +2ym 1 m 2 )2

p mlYYI Yl


= = =


E m
1 y+m 2 1+ m

/m 1 y

Solution 2.5. Let the neutron travel in the x direction. In its rest frame

the electron has momentum 4-vector



p' =
(E', P'cos 0, P'sinO, 0), E' =


(1_v e )-2

(where we have, without loss of generality, oriented our coordinate system

so
f' is in the x-y plane), and P' is isotropic. Using the Lorentz

transformation to get the lab momentum , we have:

Px =
Y (P'cosO+{3E')

P =
P'sinO

or
2 2


P Y {3E

( ) ( )



+ =

P'

yP'

The last equation says that in the lab, the momentum vector in momentum

space lies on an ellipse of semi major axis yP', semiminor axis P', and
origin (y{3E',O). The plot of such an ellipse falls into three cases:
162 PROBLEM BOOK IN RELATIVITY AND GRAVITATION

--2yP --+


2P'

yfJE

{3E' > p' {3E' p' =
{3E' < p'
Solution 2.6. Let P and Q be the 4-momenta of the incident protons,

and W the total energy in the C.M. frame. In this frame,

w2 =
(p

+QO)2 = _
(p+Q)2 ,


which is an invariant expression for W and can be evaluated in any

frame. In the conventional experiment, in the lab frame

P =
(E, )

Q =
(m, 0) ,

so


w =
(E+m)2 _

p2


2Em + 2m

2Em for E ? m .

For E =
30 GeV, m =
.94 GeV, the available energy is W 7.5 GeV.

For the colliding beam experiment, in the lab frame

P =

(E, )

Q =

(E,- )

2 2
So W =
4E ,
i.e. W =
2E. For E =
15 GeV, the available energy is

W =
30 GeV. To achieve W =
30 GeV in a conventional experiment would

require E =
W /2m 480 GeV.
CHAPTER 2: SOLUTIONS 163

Solution 2.7. Let subscripts 0 and 1 denote respectively the initially

moving, and the initially stationary particle. Balancing 4-momentum before

and after the collision, we have

Po +
PI =

Po' +
PI'

(PO+Pl _PO,)2 =

pi,
2 2

3m + 2P 0 ・
P1 -
2P 0' .

(P 0 + P 1) = -
m .

Now we put in

Po =
(E, r), E =
m +
To
PI =;
(m,Q), E'= m+T'

Po' =

(E', '), p.p' =


pp' cos 0

to find

_m -
Em + E'(E+m) -
PP'cosO =

I.e.
1 1

(E2_m )2 (E'2_m 2 )2 cos() =
(E'-m)(E+m) .

Squaring and factoring the difference of squares, then gives the kinetic
energy of the scattered particle:


(E-m)(E'+m)cos () =
(E'-m)(E+m)


TO(T'+2m)cos (} =
T'(T o +2m)


()
T'(-Tocos (}+To+2m) =

2mTocos

2m (}



To cos .


2m sin (}

To

Solution 2.8. Conservation of 4-momentum gives us

Py+ PN =
P N '+ P1T (N' =
nucleon coming out of reaction)

(P y+ P N )2 =
(P N'+P1i ・
164 PROBLEM BOOK IN RELATIVITY AND GRAVITATION

In the lab frame (where 0


Ey

3 K -
2.5 x
lO-lOMeV),
P P
(Ey,fy), (EN' P N )

y N .

In the C. M. frame at threshold,



, + P =
(m N +m 11 ,Q)
17
so that
2P




m = -
(m N + m 1l


2E E + 2P .
R = -
2m m m

N N


-y

11 11

Now and, for head-on collision


Py=Ey since
my=O a
Py ' =-PyPN'
so that

(E N
2 2-

)2
2m


11

m;
EN + -m

2Ey

2" 940" 140 + 140 14

MeV =
6x 10 MeV

2 .

(2.5 x 10- °)

2 2 2
S ince E

? m

, we ca n re p lace (E N -



) by EN" Thus
14
3x 10 MeV.
EN

Solution 2.9. We proceed by conserving 4-momentum and removing informa-

tion about the A particle:

P 11 +P n =
PK+PA

P -m = =
(P 1T +P n -PK )2 =

-m;-m -mi+2P1T'Pn-2Pn'PK-2P1T'PK'
In the lab


17

(E 11 , 11)
Pn =
0',
(m n ::.J , P


(E K r K) ,

so,

m; m mi- 2m
- -


E 1T + 2m nE K + 2E E
1T K

2£1T"£K
= -

m ・

scattering) and find



We now set P 'P =
0 (90
-11_ K
CHAPTER 2: SOLUTIONS 165

2 2 2 2
m -m -m -m +2m E
A 11 n K n K
E =

11
2(m n -


K)

Thus, to make E
11
a minimum we must make E as small as possible

(as you might have guessed!), so we try E K =
m . Thus

2 2 2 2
mA


11

fin



+ 2m n m K
E =
11 (threshold) 2(m n -

K )

2 2
mA m11

m m 2 2
940-494


n K

1115 -
140
2(m n -m K ) 2 2(940 -

494) 2


1149 MeV ,

hence threshold kinetic energy =


1149-140 =
1009 MeV.

Solution 2.10.

(a) From conservation of 4-momentum

P = -

m =
(P A -

B)2 = -

m m - -
2P A ・



(1)

In the lab frame P




(mA,.Q), B

(EB' B)'so
2 2 2
-m 2 mAE B
-mB
= -

-me A

and hence

2 2 2
EB =


+m

me /2 m A .

(b) Here
mA=M, mB=O, mc=M-o, so


E =
hv =
M2 -

(M_o)2 = u _

< a

2M 2M

Physically hv 1= a because some energy goes into the recoil of M which

is necessary to conserve momentum. In the Mossbauer effect, the recoil


23
momentum is shared among ,....,
10 atoms, so the recoil energy is

negligib le.
166 PROBLEM BOOK IN RELATIVITY AND GRAVITATION

(c) In this case


PA=(EA,fA)' PB=(EB' B)'so (1) becomes

1 1
2 2 2 2"22 2"2 2
-m
A -mB + 2E



2(E A -m
A) (EB-m B ) cosO =

-me'

Solution 2.11.

(a) The total 4-momentum is P total =


P1 + P2 =
P3 + P 4' In the C.M.

frame P total =

(E o't Q)i' so that

P otal = -

(E o't i)2(P 1 + P 2)2 =



(1)

We can evaluate (P 1 +P 2 )2 in the lab frame, where P 1 =

(El' l)'P2 =

(m 2 Q):

-(E o.t i)2pi+p = + 2P 1 ・


P2 =

-mi -m -
2E 1 m 2 ・

(b) In the equation


P1
Pl' (P 1 + P 2 )
..

t 0 ta I

evaluate the left side in the C.M. frame and the right side in the lab frame:

_EC.M'E

C M
tota I
. .


_m2_E
1 1

2 1 C. M. 2 2 2
= -



[(E total ) -





] (from (a))

Hence, the result:

C M C M 2 2 2 c M
E [( E total ) ]/2E total
. . . . . .
_

+ m _
m .

1 1 2

(c) By using the result of part (b) we have

2 2

( )
m + E1m
(EC.M.)2
1 2
(pC.M.)2
1 1
= _

mi =

C M
. .

mi

tot


2 2 2
(m 1
2 2
+2E m )

(E 1

) m2p2
2)
-m
(m 1 +E

m -m

+m
2 1 2 _

2 1

2 1
= '
2 2
C.M. 2
(E tot ) ( Ec.M.
tot ) (EC.Mo)
tot

and therefore pf.M. =

m2Pl/E oOt i'
CHAPTER 2: SOLUTIONS 167

(d) If u is the 4-velocity of any observer, then it is easy to verify


in that observer's frame (hence in all frames) that

P(3) ==
P+(P'u)u

is the 3-momentum measured by that observer. Therefore u


C . M .
is de-

fined by

Ptotal +
(Ptotal' uC.M.)u C . M .

Now if two observers have 4-velocities u and u then y = -

ul


1 2 ' 2

is the Lorentz transformation factor between the two frames. (Proof: In


rest-frame of 1, ul' u 2 =
(1, 0) (y ・

, y:v )
= -
y). So y C .M.
= -
u.u
C . M .

gives the transformation to the C.M. frame. Thus

(P total 0
P U + U
.) u U

・ ・ =

tot C . M C . M .

Etotal +
E o.YC.M.
i =

C.M.

E total IE
YC.M.

tot

In E1 + hence
our case,
Etotal m

2 ,

YC.M.

(E 1 +
m2)/E o't i ・

(e) From (d), we have

1 -

v .M. [E =
)]2
t i/(El + m

thus
2 2 t

[ ]
E -m P
1 1 1
v =

C M

. .

(E I +m
2 )2 EI +m

Solution 2.12. In the C.M. frame since the collision is elastic, Ef.M. =

C M .

E l'
. .


1 ..,

P 1"
Pl' u
C . M .


C . M .

(The prime denotes values after the collision.) Evaluating this equation
in the lab frame yields
168 PROBLEM BOOK IN RELATIVITY AND GRAVITATION

-E 1 -E ,+
+ P
l' .M. fl"YC.M.

1 .

But
C.M.

.Pli(E1 + m
2 ), so we have

p21 Pl,Pl cos e


+ -E
1 ,+
-E =

E + m E1 + m
1 2 2

or

()

1 ,(E 1 +m
2)




2 -mi
c os
(1)


22-
P (E '-m )2
1 1 1

(This equation could also have been derived without using the center of

mass frame.) We can find the minimum value of cos () (maximum value of
sin ()) by setting d cos ()/dE 1 , =
0, solving for E 1 " and then substitut-

ing into Equation (1). A quicker way is to consider the intersection of

the graph of cos () (E 1 ,) with the straight line cos () =


K. The points of

intersection are given by


2 2 2 2 2 2
,-m ) (E 1
1 (E 1 2)
K P =
E , +m
1 1
(2)
222

2(E 1 + m
2) (E 1 m 2 + m 1 ) E 1 ,+ (E 1 m 2 + m
1 ) .

At the minimum value of cos () , the straight line is tangent to the curve

and the discriminant of Equation (2), a quadratic in E " is zero. This


gives
2 2 2
o =
-(E 1 +m
2)
K 222
P1 m1 + K



1 (E 1
m +m
1)
+ K
442
P m
2 1 1 '

or

2 2 2 2 2 2
2 (E 1 + m
2)


(E 1 m 2 +
fPl) m




K = =

m2p2
1 1


and, finally 1

cos () min =
(mi m -

)2

sin
()max =


/m 1 .
CHAPTER 2: SOLUTIONS 169

Solution 2.13.

(a) Take the rocket's motion to be along the x-axIS. We have the

following constraints on its 4-velocity u and 4-acceleration a:

t 2 x 2
u.u = -
I = -
(u ) + (u ) (normalization of u) (1)
t t x x
a.u =
O = -
a u + a u (a orthogonal to u) (2)
2 t 2 2
(Proper acceleration

a.a =
g = -
(a) + (a ) is g) .
(3)

From these equations, we get

2 2

( :: ) (aX) [1- (u jut) ] g2


at = aX ,

which imply




gu (4)

at =
gu .

(5)


By differentiating Equation (4) we can get a differential equation for U

2 x x t
d u

da du t 2 X


- =

ga

g u ,
2 dr dr
dr


with solutions U =
A sinh gr + B cosh gr. Since the initial conditions on

the motion are U =
0, dux /dr =
g, at T =
0, u must be


U =
dx/dr = sinh gr .
(6)


u =
dt/dr =
cosh gr (from Equation (1))

The integrals of these equations (with x =


t =
0 at r =
0) give us

x =
g-l (cosh gr

1) t =
g-l (sinh gr) .
(7)

In units with c =
1, it turns out by numerical coincidence that

g( =
980 cm/sec ) is almost exactly equal to one inverse year (as a time)
or one inverse light-year (as a distance). Thus for 40 years as measured

on Earth (t =
40 yr), (7) gives
170 PROBLEM BOOK IN RELATIVITY AND GRAVITATION


T [sinh- (40)] yr 4.38 yr

and

x [cosh [sinh- (40)] -1] L.Y. 39.01 L.Y. (8)

For 40 years in the rocket T =


40 yr and from (7)

17
x [cosh(40)-1]L.Y. 10 L.Y. (9)

(b) To travel halfway requires x =


15,000 L. Y., or


T cosh- (15,000 + 1) yr 10.3 yr .
(10)

The deceleration half of the trip is identical, so the total time is 20.6 years.

(c) Denote the rest mass of the rocket (which changes) by M. The

change in energy of the rocket equals the energy radiated

°
d(Mu ) = -
dE (11)

ra .

Since the energy is radiated as photons

dE rad .

dP
rad .
(12)

and by conservation of momentum,

dP =
dP (13)
rad .

where dP is the change in momentum of the rocket. Combining Equations

(11)

(13) gives
d(MuO) = -
dP = -
d(Mu )

(dM) uO + MduO = -
(dM) U

Mdu

dM/M =
-d(uO+uX)/(uO+u X

so that

e- gT
Mo/(UO+u

M =
) =

Mo (14)
CHAPTER 2: SOLUTIONS 171

where we have used Equations (6) to give us uO(r) and uX(r).


gr
From part (b), e =
30,000 for half the trip. Thus, M =

o /30,000

and

Mfinal =

o /(30,000)2 10-
Mo .

Solution 2.14. A cyclotron applies an accelerating potential with a fixed

frequency, to the gap between two "dees." The frequency is chosen so

that the potential is always in the right direction when the electron crosses

the gap, Le. it is the cyclotron frequency CU




eB/mc. The reason that

there is a maximum energy is that the electron actually circles at the

"synchrotron frequency" cu =
eB/ymc =

cuom/E (where E =
electron

energy), and as the electron becomes relativistic this gets out of phase
with the fixed cyclotron frequency. Eventually the electron arrives at the

gap when the potential is 90° out of phase and the accelerating process

breaks down. Quantitatively if the phase of the accelerating potential is

a when the electron passes through the gap, the electron receives energy

V cos a, where V is the maximum accelerating potential expressed in

electron volts. In dN cycles of the electron motion then

d(electron energy) dE rt..


= =
2V cos ( 'P _

CU t)
dN

cudt/21T o

[Note that the electron passes through the gap twice per cycle, hence the
factor of 2.] Here rp is the angular distance traveled by the electron

1>
f wdt W

J,t(m/E)dt
= =

and

; 2VCOS[Wo it dt-wot]

" [w l ( dt]
wom cos
= -

d t2 [w l t( 1) dt]
a cos


172 PROBLEM BOOK IN RELATIVITY AND GRAVITATION

where a == 2V cu

m/7T. Differentiating we have


r r


d ( E2 )
L J ( ) ( )

m m
= -
a SIn CiJ -

1 dt CiJ -

2 o
E o

dt

.1

[a ( d;2)2J CiJo( 1)

= -

The negative square root has been used because the sine function above

is clearly negative.

We now find a first integral of this equation. Let q == dE /dt so that
2 2 2 2
d E /dt =
qdq/dE , then our differential equation becomes

dq !

[1 ]
2 2 m
-q

[a -q ] CU

l'

dE 2
(E2)
This can be integrated with the initial condition that q =
a at E =

(Le. at t =
0): 1
2 2 2 2
(a _

q ) =cu
o (E -m ) .

The energy keeps increasing until q =


0, at which time the energy must be

Emax

m +
a/cuo =
m + (2Vm/7T)2 .

Solution 2.15. If F is not to be identically zero then for some 4-velocity


u, the product a.u must be nonzero. This contradicts the requirement

that a.u =
d(u .u)/dT =
O.
CHAPTER 3: SOLUTIONS

Solution 3.1. The neatest way to do this problem is by using spacetime


diagrams.
First rotate the coordinate system until both events lie on the x-axIs

and the first event occurs at x = t =


O.

The two events A and B at spacelike separation ( x,


t) are depicted
in Figure (i), with the world line of a light ray indicated as the dotted

45° line. A Lorentz transformation, shown as the t', x' axes in Figure


(ii), can obviously be made to make A and B simultaneous. (The

magnitude of the Lorentz boost is (3 =
t/ x tanh-
and tan () in

Figure (ii) is the rapidity parameter.) Since the t' axis cannot be rotated

below the light cone line in a Lorentz transformation, the two events can-

not be made to occur at the same location.

t t
t'

// B

x'
T /

t /
x x
A/ x--+ A
/ /
/ /
/ /
/ /

(i) (ii)

173
174 PROBLEM BOOK IN RELATIVITY AND GRAVITATION

t t

B.


t /

j x x
Ax" A/
/ /
/ /
/ /

( iii) (iv)

Figure (iii) depicts events A and B at timelike separation. As

shown in Figure (iv), a Lorentz transformation can obviously be made to

bring A and B to the same location in the primed frame. The magnitude
of the required boost is a = /1x/ /1t. Since the x' axis cannot be rotated

above the light cone line, the two events cannot be made simultaneous.

Solution 3.2. Four linearly independent null vectors are e.g.

ez+e t ,
ez-e t ,
ex+e t ,
ey+e t

Suppose there existed 4 null vectors A, B, C, D which were mutually


orthogonal. Linear independence implies that any vector can be written

v =
aA + bB + cC + dD

but the length of this vector is clearly zero if A, B, C, D are null and

mutually orthogonal. Hence 4 such vectors cannot exist.

Solution 3.3. Without loss of generality, choose coordinates to make the

null vector V + e A general vector is S Ae + Be Ce + De z


ex
= = +

. '

t x y
The dot product is S.V A(et'e t ) ) A must equal

B(ex.e B-A so B
= =

if S is orthogonal to V. Now the vector S is spacelike unless


CHAPTER 3: SOLUTIONS
175

2 2 2 2 2 2
A B + C +D which means 0 and hence
C +D =
C =
D =
0, there-
fore S =
A(e x + e
t ),
which is a multiple of V.

Solution 3.4. Take flat Minkowski space, so that X, y, z are spacelike


unit vectors; t is a timelike unit vector; t::tz, t:!:y are null, etc. The

following table gives examples in the order: spacelike sum, null sum,

timelike sum.

spacelike null timelike



'" '"

x+y x + (x 1) -

+ (E t)
(X+Et)
'" '"

spacelike (-X+EZ)
+ (x+ 1) + (-2x) x + t

(x+ Et) + (-x) (-x) + (x- 1) ( EX) + t

(X- t) + (x+t) (x-t)+t


'" '" '"

null (x+ 1) + (x+ t) (x -

t) + 2t
(t-x) + (t+x) (x+t) + t

spacelike sum (t+EX) + (-t)


time like null sum (t+EX) + (-t+Et)- '"

timelike sum t + t

Here E represents any small constant, e.g. 0.1. Not all possibilities

would be allowed if one required the timelike vectors to be "future

(The reader

directed. Le. u.l < 0 for timelike vectors u. can work

this out.)

Solution 3.5. Let k be the null vector along which the light is traveling.

A given observer takes a little square element of the beam whose sides

are the vectors A and B, which are purely spacelike vectors in his

frame, and orthogonal to the light beam, so A.k =


B.k =
O. Also A.B=O

since the element is square. The area of the element is IAI IB\.
A different observer can unambiguously identify the same element of

the beam, say by the rays making up its corners, but he slices it differ-

ently in time: the old vectors A and B are not orthogonal to his

4-velocity u, so are not a purely spacelike cross section in his frame.


176 PROBLEM BOOK IN RELATIVITY AND GRAVITATION

N ow for any constants a and {3, the vectors

A' =
A + ak

B' =
B + {3k

span the same beam element as A and B (the "tip" of each vector is

just moved to a different point along the ray by adding a multiple of the
wave vector k). The new observer requires A =
uB'.u =
0, which is
always possible, choosing a = -
A.u/k.u and {3 = -
B.u/k.u. (k.u 1= 0,
since k is null and u is time like.) Notice that A'.k =
B'.k = A'.B' =

Since k.k =
0, so A' and B' are orthogonal vectors which are the sides

of the new observer's cross section.



The new area is IA'I IB'I =

(A'.A')2(B'.B')2 =
IAI IBI, the same area as seen by the original observer.

Solution 3.6. It is easy to show by example that l DIlIl is not invariant.


Ox p.
Let D =
1 and all other components be zero. For a boost in the
, , , ,
0' 0

X x
x-direction D == -
{3 y, D -{3y so that l DIlIl =
0 but l DIlIl =

Il 11'
-2{3y. The proof for l
DIl/l is similar.

For a Lorentz tran:formation A(, we know


,D
D :A A =
so that

11' A ll' Df3


a a

Dp.'
A aa D'l
{3 D{3
;= =

11'

{3 a
/1'
11 11 11

and hence this summation is invariant.

Solution 3.7. Since ",a{3 is constant we have

Fa F{3 =
(F l1ya ) (3 (F{3al1aa )
11 , {3 a
p.y ,



/lY, (3
F{3a(l1yal1 aa
) =

/lY, {3
F{3o aaa = F
p.y,fJ
F{3y

=-F F

{3
/la, {3

where the last equality follows from relabelling dummy indices, and by
antisymmetry.
CHAPTER 3: SOLUTIONS 177

Solution 3.8. We express the differentials in terms of the new coordinates:

ds

dx

dx{3
Tfa{3

/(JXfl) dXIL ( ax{3 /(JXv) dX


a v

Tfa{3 (ax

ax(3
( )
axa d-Il
x dx
-v
1Ja
_

..----

{3 axil a x v

2 v'
If we write the line element as ds =
g dx fl dx it follows that
fl v

==
ax

ax{3

11 v l1a{3 aXIl ax

A vector transforms by the following rule


U {3
T,.a ax

_

ax t3

therefore

U.V =

UaVAl1aA
(if (y{3 ai' -va y{3
ax U ai,
)\ ) Tf aA Tf a
= =

\ iJY!l ax13 iJY!l ax 13


-va y{3 g
a{3


Solution 3.9. Under the coordinate transformation xll x ll(X ) the trans-

formation of is
ga13 ax

ax 13
g ga (3

Il v
axil aY!"

and therefore the transformation of g is

g =
det(g--)
Il v

det(ga (3 )det
aXa
( ) ( )

det
axv

g rd taXa
L ( axil )1 .

Since g 1= g, g is not a scalar.


178 PROBLEM BOOK IN RELATIVITY AND GRAVITATION

Solution 3.10. Suppose one coordinate transformation is

X f1
A = /ax{3


X f1 (X ), so
(1)
and the other is

ax / ax {3
v u
x/l =
;{/l(x ), so A
(3

-u

(2)

The product matrix is then

AU AY.
a aiY
Y J3

(3)
ax Y ax{3
This looks suspiciously like the chain rule for partial derivatives, so one

is led to consider the coordinate transformation

x ll (x{3) =
xll [x Y (x{3)] , (4)

whose transformation matrix is obviously

A ll axil axil axY


13
= =

(5)
ax13 ax Y ax{3

This differs from (3) only by a twiddle. A little thought shows that the

twiddle is not meaningful. Partial derivatives are taken with respect to

argument variables; it makes no difference what symbols are used to repre-

sent these variables. Thus (4) is in fact the coordinate transformation of

the product matrix.

Solution 3.11. In a basis K is represented by a matrix K



{3. An obviously
necessary and sufficient condition for K

{3 to be expressible as A

B{3
is that all columns of the matrix are proportional to each other.

In basis-free language, K can be written as

K =
AB

if and only if the vector W K.v (i.e. wa K



{3v is in the same direc-
(3)
= =

tion for any choice of v. Proof: In some frame choose 4 basis vectors

By linearity the direction of W

e , e ,... to satisfy ell. e v = all
v'
CHAPTER 3: SOLUTIONS 179

is independent of any v if and only if it is independent of the 4 basis

vectors ell. Our condition then is

o 1
K.e -

A W K.e = A W ...

0 ' ,

or

aO a a1 a
A W
K =

AoW , K =


...

Our basis-free criterion then is equivalent to the columns being proportional

Solution 3.12. A direct product of two vectors (see Problem 3.11) has all

of its columns proportional to each other (similarly for all of its rows).
This is not true for a general second rank tensor. If e

i =
1,n are the
th
basis vectors (0,0,...1,...0,0) where the 1 is in the i slot, then the

n products e


j obviously span the space of second rank tensors,

since there is one with a 1 uniquely in any row and column. Sums over

these times constants obviously can give the general tensor. (Anyone
specific tensor can be represented as a sum of n outer products, as the

reader is invited to verify.)

nd
Solution 3.13. The object cannot be a tensor. A 2 rank covariant

tensor is a functional of 2 covariant tensors, Le. Xf.l V W would have
f.l v

to be a scalar, but
Xf.lV V W v =
(A. V) W v + (B. W) V
f.l f.l

which is coordinate dependent.


V V
If one defines the two index object by the relation Xf.l =
Af.l + B in

II'V'
any coordinate system, then the coordinate transformation law is X"'" =

A B{3.

Af A + It is not possible to express this as

X f.l'V' =
T f.l'v' Xa{3
a{3

where T is some transformation. This follows from the fact that there

are many choices of Af.l and B which give, in one coordinate system,
v V
the same Xf.l In particular, in a given coordinate system Af.l + C, B C


180 PROBLEM BOOK IN RELATIVITY AND GRAVITATION


give the same Xfl. as Afl. and B V , but the two choices give very

different Xfl.'v'.

Solution 3.14. We need only transform and check the results for antisym-

metry:
{3

A-A-F

{3
-A-A-F {3 a
F--
-AJi A F -F

= = =

{3 a

{3 a{3

fl.V vfla v fl v
Vfl.

FI1V gfLU gv{3 Fa{3 gfLU gv{3 F {3a gl1{3 gva Fa{3 VI1
= = - = - = -
F .

For variety, we treat the symmetric case by a different argument: Consider


V v v
the tensor (!fl =
Sfl. -

SVfl., where Sfl is a symmetric tensor. Since

(!fl. v
vanishes (by hypothesis) and since (ffl. V
is a tensor, it vanishes in

all coordinate systems, hence Sfl. is symmetric in all coordinate systems.

Solution 3.15. By the given symmetries A Sfl.V = -


A SVfl., but /1, v
fl. V Vfl.
are dummy indices so we can interchange fl. and v to get


A Sfl.V = -
A Sfl. = -
A S fl. v.
fl.V Vfl. fl.V


Thus
Afl.V Sfl. O.

Any tensor V can be written as a of its symmetric part V ==


fl.V s -
m fl.V
1 1
and its antisymmetric part thus
(V +V
Vfl.) V ==
(V V
Vfl.)'

2 fl.V J1v 2 fl.V

Vfl

A =
Vfl.V A + Vfl. V A =
Vfl. v A = 1. (Vfl. V -

VVfl.) A fl.V
fl.V fl.V fl.v fl.V 2

Vfl.

S =
Vfl. V S + Vfl. V
S =
VIlV S = 1 (vfl. V + VVfl) S .

fl.V fl.V flV fl.V 2 fl.V

Solution 3.16.

(a) If there are no symmetries there are clearly n components.

(b) For s symmetric indices and r-s asymmetric ones, consider:

How many inequivalent ways are there of choosing the values of the s

indices from the n possibilities? It equals the number of ways of

choosing s things from n possibilities, including repetitions:

(n+s-1)!/(n-1)!s! .
CHAPTER 3: SOLUTIONS
181

[See, e.g. J. Mathews and R. Walker, Mathematical Methods of Physics


(W. A. Benjamin, 1965) Section 14.3.] The other r-s indices can be

chosen in nr-s ways so the number of independent components is

nr-S(n+ s -

1)!f(n -1)! s! .

(c) For a antisymmetric indices and r-a other indices, consider

first: How many ways are there of choosing the a indices? This is

equal to the number of ways of choosing a things from n, with no

repetitions:
n!f(n -

a)! a! .

The number of independent components is then

nr-a n! f(n -

a)! a! .

Note that if a = n there is only one possibility for the a indices. If

a > n, there are no possibilities; all components must be zero!

Solution 3.17.

(a) V 0
Vij.tv] (V V
vp) T[p.V]
= - =

2 Il
F F T T
ij.tv ]
= =

IlV (Il v ) Il V

F =

(Ilv )



[a,By] [Va,By Vay,B V,Bya V,Bay + +V V
y,8a
= -

ya,B
- -

[Ta

(a,8, y) ,8 ,y

Tay ,,8 T,8y ,a]

F[a,8 ,y]

[F
a,8 ,y
+F
ya,,8
+ F
,8y,a] ・

(b) If A

is a totally antisymmetric tens or, i. e.
Il V
' . .


( _1 ) A a'. .a =

a...a
1 P
"l"p
then

A_I
I ( _1)

A =

[a ]

--. ...a " '..a a



'..a
1 p p. 1" p

But A == Vr is totall y antis y mmetric, so




..'a La

'.'a ]
p p
182 PROBLEM BOOK IN REL,ATIVITY AND GRAVITATION


] V[[a 1 V[a 1 ]

]]

[a 1

...a ".a "'a


p p p

The theorem for V follows analogous ly.


((a 1 "'a ))

V V but by antisym-
By symmetry [anLm ]...a p ) [a mL
ao]...a p )

(a 1 ."
a (a 1 ...

metry V (a

V Hence V must vanish.
[a mL
ao]...a p )"


...
[aoa
Lm
]...a 1 ) (a 1 '"

Antisymmetry on two indices is easily expressed as


[Va 1 ]
Va 1 ...[aoa ]...a p -Va 1
= .

"'a a ao...a
2 "'aoa ...

Lm Lm p mL p

Thus

V[a 1 ".[aoa l l(-l)17 ( V V"' a




]...a p ]
= ...

p!

17
..'a a ."a
17 17 a17
£ m
L m 1 17m 17£ P

But V


V so that
"'a a ."a a "'a a ..'a
a 17 17 17
17
1 17£ 17m
17
p 1 m 17£ P


V ] [a 1 ]

].

[a 1 [a Lm a

a 0 a'.
・ ・ ・

・ ・
.. ・
0a
p Lm p

(c) From part (a),

F + F + F
3F[a{3,v]

va,{3

a{3,v {3v,a

and by part (b)


But
-A[a,fj] F[afj,v] -A[[a,{j],v]'

so
Fafj

A A
[[a, {3] v] [a {3 ]


, , ,

which must vanish because Aa {3 , ,



Aa ,
v , {3

Solution 3.18. The first part is straightforward:

(Xafj Xfja) XCafj) X[afj]



(Xafj Xfja)
+ + = ・

Xafj

If a similar relation,

Ya Y + Y (1)
[a{3y]

{3y (a{3y)

CHAPTER 3: SOLUTIONS 183

rd
were true for a 3 rank tensor, we would have:

Y Y + Y Y Y
[a{3y ]

{3ay [{3ay]

({3ay)

(a{3y)


Y(y{3a) Y[y{3a] Y(a{3y) Y[a{3y] Y (2)


(3ya

= =

{3y

Since Y does not in general have the symmetry properties implied



{3y
by Equation (2), the simple decomposition in Equation (1) cannot be
correct.

Solution 3.19. One way is to show that


o can be combined with the

com p onents of two vectors to form a scalar. But AVa of.l =


A.B ' hence
f.l v

S is a tensor.

Alternate ly one can check the transformation properties

a x f1' ax 13 axil' axa


aa = =
all: ,

{3 v

ax axv'
a v a
ax ax

, ,
a a
where the last equality follow from the fact that axf.l / ax and ax / axf.l ,

are matrix inverses of each other. Hence


of.lv transforms as a tensor.

Solution 3.20. If
Ea{3yo
is totally antisymmetric, then given anyone

component e.g. E
0123 any other component (with nonrepeated indices)
can be found by permuting the indices on that given component. Com-

ponents with repeated indices, of course, vanish. The tensor is thus

fixed once E
0123
is given, Le. the tensor is unique up to scaling by a

constant. With the usual, position independent choice of normalization,


we have
E =
-E =
E = ... =

0123 1023 1032

in Minkowski coordinates. If we now transform to other coordinates

xf.l '(xa) we get



ax

ax{3 ax Y ax ax

Ef.lv'A'a' ' E det
[ ] E
= =

a{3yo IlVAU


axA-' ax

axf.l' ax v a
ax

184 PROBLEM BOOK IN RELATIVITY AND GRAVITATION

Now

ax

ax{3
g/l v'

axil' ax v l1a{3

det
(gl11/) \det ( :,)
I det
(Tfafj)

so that

a a'
[ 2
det (ax lax ) det
(g/lt/)]
= -

and finally

[- det (gu'v')] 2
E/lV'A'a' E/l VAa

Solution 3.21. In an orthonormal frame

E a{3yo = .,.,a/l.,.,{3v
.,
.,.,ya
.,
.,.,OA E
., .,
/lVaA.

For nonzero components of E



{3yo, precisely one of the indices a, {3, y, a
00 jj
must equal zero. Since 11 =
-1 and l1

1 this introduces precisely

one minus sign in what is otherwise an identity transformation. Thus

Eafjy8
= -
Eafjy8 .

In some other frame, this equation becomes

1. 1.
[- det (g )]- 2 E = -
[- det (g )] 2 E

{3 yo
/lV a
{3 yu /lV

according to the transformation derived in Solution 3.20, and its obvious

analog for E

{3yo. Thus in general

de t ( a{3y8
gllv )

Ea{3yo

Solution 3.22. Evaluate this scalar in a local orthonormal frame. Accord-

ing to Solution 3.21, E/lvpa so


E/l vpa
= -


E E /lvpa =
I EI1vpa I =

IIE012312
- -

I1 v pa "'-
/lvpa
CHAPTER 3: SOLUTIONS 185

where the sum is over permutations of 0123. Since there are 4! = 24

permu t a t Ions, E p.vpa


24
Ep.vpa


Solution 3.23. In orthonormal frame (with


(3l1a (3)
an =
it should be

evident that the antisymmetric nature of E is precisely what is needed to

perform all of the operations in finding the determinant. That is,

Ea{3y8 A A A A IIA II

det ・

Generalizing from the (0,1,2,3) to arbitrary (p.,v,A ,a) requires placing


on the right-hand side to account for the minus signs introduced by
Ep.vAa
interchange of rows.

To show that the result is true in all (not just orthonormal) frames, we

need to show that it is a tensor equation. This is equivalent to showing


that det
IIA II is a scalar:

ax{31

ax
det IIA<II
{3

det A

{3 ax

ax{3'

a ax

ax{3 a

det \I A {3 \I
det det =
det II A{3 II .

axa ax{3'
, ,

a a a a
[The last step follows from the fact that ax / ax and ax / ax are

matrix inverses.]

Solution 3.24. Suppose au +


{3v + yw + ox = 0 for a, {3, y, a not all zero.

If a 1= 0, take the wedge product of this equation with v A w A x and get

u A V A W A X =
O. If a =
0, then form analogous product using the {3, y, or

a term. Conversely, if u, W, x and v are linearly independent, then any


vector can be written as a linear combination of them; in particular, any

4 orthonormal basis vectors can be so written. Thus

01= e

Ae

Ae

Ae

(alu+{31v+Ylw+olX) A (a 2 u+.")

A (a 3 u + ...) A (a 4 u + ...) ex
(u A V A W A x) ,
186 PROBLEM BOOK IN RELATIVITY AND GRAVITATION

since cross terms with a duplicated vector vanish. Hence u A V A W A X 1= O.


Since U A V A W" x is totally antisymmetric on all "slots" and is a multiple
of eo A e 1 " e

A e

, it must uniquely be proportional to the E tensor. See

Problems 3.20, 3.21.

Solution 3.25.

*FIlV = 1. Ellva(3 F (1)


2 a (3

*F -! 11 l1vT E
aTa(3 F = 1. E a(3 F
a(3 a(3

IlV 2 1la 2 IlV

(E/lVAY ."aA .,f3Y)



a{3

E/lVA/."aA .,,{3y F
a(3)
_

E F
a (3 (2)
(3

2 Il va


E/l
va
{3 (*F E/l
va
{3 E FAa (3)
*(*F/l ) a(3) {3Aa
= = .

Now, E/l {3 Ea{3Aa


va
= -

o (this defines the symbol o


2 ).
One should

convince oneself that


o has the following properties:

o =
+ 1 if A =
11, a=v

= -1 if A =
v, a=/l

= 0 otherwise .

Thus
ollV = allA oVa _
all oV
a A
Aa

and therefore

o FAa

(F/l

FV/l) F/l
*( *F/lv)
_ = _
_ = _ .

Aa
Solution 3.26. From the definition *V

{3y =

VA E {3y, and

*V *Va(3y =
VIl V\1\ E
Ila (3 y
EAa(3y .

a (3 y

and A must also


Since 11 must be a different index than a, (3, or y,

be, it follows that


CHAPTER 3: SOLUTIONS 187

E E
Aa (3y C oA
p.a(3y

fl

for some constant C. If we sum over fl and A, use


o =
4, and use

the result of Problem 3.22 we see that C =


-6 so

f3y
-60 VflVA
a a
W w -6V V
f3y
= =

a a

Solution 3.27.

lower indices.
Notice that
o::::
This follows from the fact that the sign of
is antisymmetric in both its upper and

a determinant

changes for an odd permutation of its rows or columns. Since, for example,

the upper indices are antisymmetric there can be no repeated index. If


there are more than four upper indices there must be a repeated index so

fl'
uA
a must be identically zero.
p...a

Solution 3.28. The indices fl, v, A, K must take on different values from

p, a. For given p, a, we can have fl equal to K (with v then equal to

A) or fl equal to A (with v then equal to K). All other possibilities

vanish because of the antisymmetry of Eflvpa. Furthermore, because of


that antisymmetry, the fl =
K, V = A case has a minus sign. Thus

Ellvpa E \
I\Kpa

C(o 1\ K
-oflo ) Co
ov K 1\
==
I\K

for some C. Summing on fl,


A and v, k and using the results of

Problem 3.22 gives C =


-2. Thus

Ilv 1 IIv a
uAK
= -
Er p E
AKpa

In general
Efl vAr E = _
oflvAr
lKpa lKpa
vAr E ofl VA
Efl = _

lKpr lKp

Solution 3.29. If pa(3 =


A [aB(3], then
188 PROBLEM BOOK IN RELATIVITY AND GRAVITATION

pa(3 pay pao


det A(3 A



= 0

B(3 BY B

since the first row is a linear combination of the other two rows (first row

a a
= -
A x third row + B x second row). Multiplying out the determinant

gives the desired res ult.

Solution 3.30. For each value of fl, there are 3! arrangements of a, (3,
y 1= fl which make the E nonzero. The Jacobian determinant has the
same property of changing sign that the E does, so with the 3! in the

denominator, we can just take one representative element of each permuta-


tion and get the right answer:

d3 O [a(a,b,c)/a(a,b,c)]dadbdc
= =
dadbdc

d3 1
= -
[a(const., b,c)/a(a,b,c)]dadbdc =

d3 2 -[a(a,const.,c)/a(a,b,c)]dadbdc
= =

d3 3 -[a(a,b,const.)/a(a,b,c)]dadbdc
= =
0 .

Solution 3.31. Let barred coordinates represent an orthonormal frame


4 4
where the metric is 'YJ
.,
. Then dV == d x det(ax/ax)d x because the =

flV
Jacobian determinant relates volumes. Now

( )
ax fl
-g =

-det(ga(3)

_det

1J fl V =
[det(ilX/ax)]2(- det l1) ,

ax ax fJ

so that


(_ g)2 =
det(ax/ ax)

and hence


dV=(-g)2d x.


Solution 3.32. In a comoving local orthonormal frame we have d V =

dx dy dz; we want a scalar invariant quantity which reduces to this.


CHAPTER 3: SOLUTIONS 189

4 .1
Start with d V =
(- g) 2 dx dy dx dt, a scalar in any frame. Multiply by
uO luo =
uO I(dt/ds) and get


d V =
[(- g)t uO dx dy dz] ds .


Since d V and ds are invariants, the term in parentheses must also be,
and it reduces to dxdydz in a comoving orthonormal frame, so



d v =
(-g)2uodxdydz .


Solution 3.33. Since the contravariant momentum vector p transforms

just like the contravariant displacement vector x , the 4-dimensional

momentum element must be of the same form as the invariant 4-volume

element (Problem 3.32), so


4 x t
d P =
(_ g) 2 dp dpY dPz dp .

For the 3-volume element we multiply by a a-function of the constraint

and integrate over pt:

J o[(-ga{3p1Ip{3)t (_g)t dpxdpYdpzdpt



d P =
-m]

1 _.1 -l
dp Y dpz

(3 ) 2.
2g ta pII


(_g)2dp (- {3p1Ip


x -

(Here we have used the identity



O(f(X)) dx =

If (xl) I

where xl
is a zero of f.) Rewriting the term in brackets, we get

(_g)t dpxdpY dPz


( J

d P =


Notice that in a local orthonormal frame comoving with the particles in

momentum space, this reduces to


d P =
dpx dpY dPz

as it should.
190 PROBLEM BOOK IN RELATIVITY AND GRAVITATION


Sometimes d P is renormalized by dividing by m. This gives an

invariant volume element which is valid for massless particles as well.

Solution 3.34. Clearly the number of particles N is invariant. We need



to show that dx dy dz dp dpY dPz is also invariant. If the particles are

moving with 4-velocity u relative to the observer's xyz coordinate

system, he measures an invariant 3-volume element occupied by them (see

Problem 3.32), 1
3 O
d v =
(-g)"2 u dxdydz

and an invariant 3-momentum element occupied by them (see Problem 3.33),


(_g)t dpxdpY dP
( u ).

d P =


Since he measures with an orthonormal frame, we have -

g =
1 and u =


u so
o'
x 3 3
dx dy dz dp dpY dPz =
d Vd P

which is an invariant.

Solution 3.35.
0 0 0
d by
" "
> Sl d e
(a) Choose

a 4-volume b oun d ed b y x

' by x



' an

hypersurfaces at infinite dis tance from the origin. By Gauss' theorem

0=
fj ad40=fj adtdXdYdZ=fjad3Ia.
The contributions on the sides can be ignored as they are moved to

infinity, so

0=
f jad3Ia f fId3Ia f jOdxdydz i jOdxdydz
+ = -

000 0
x x
x A
x A

o 0' .

(b) The planes x =


constant, and x =
constant intersect f orm1ng

two 4-dimensional regions: I and II. Close off these regions with

spatial hypersurfaces at infinity forming two bounded 4-volumes. By

Gauss' theorem, for region I


CHAPTER 3: SOLUTIONS
191

0=



,a
d4Q =
0 =


Ix

JOd

IO -


Ix
0....
JO'd 3 I O

or

J f

O O
J dxdydz =
J dx'dy'dz' ,

0 0

Ix Ix

and similarly for region II. The proof is completed by adding the equations

for the two regions.


CHAPTER 4: SOLUTIONS

Solution 4.1. Suppose the wire lies on the z-axis of cylindrical polar

coordinates and has proper charge density Po' In the rest frame of the

wire JO =

Po and 1

0, and from Gauss' law the only nonvanishing
FOr,

component of the field tensor is E =
P o A/21Tr =
where A is the

cross sectional area of the wire. If the wire is moving in the +z direction


with velocity (3 relative to the lab, the Lorentz transformation A 0

y,
0'



(3y gives ,
0'

YPo JZ =
(3ypo

" , ,

B cp oFOr
Z r Z r


F = =

(3YPoA/2TT r E =
YP o A/21Tr .

(B?> is the "physical component," i.e. the component related to the unit

vector If the field of similar wire of charge


ecp') we now superpose a

density at rest in the lab, the total charge density and field
-yp,o' ,

b thcancel. The current I =


f JZ dx dy =
(3 YP oA produces only a B field

BCP 1/ 21Tr

Solution 4.2. The invariance of these quantities follows from the fact that

they are equal to scalars:



_E2 =
lF (3Fa

2 a
(3

!. (3 ! Ffl v F (3
a V
E. B * F F

= E =
det (Ffl )
flva(3
= .

- --

4 a(3 8

invariant for rotations in 3-space. Thus the


Any invariant must be an

invariants can only be constructed from the scalars . , and .


. .
{f there were three independent invariants, then it would follow that e.g.

B.B would be invariant. This is obviously untrue, since B changes in
-- -

general under a Lorentz transformation.

192
CHAPTER 4: SOLUTIONS 193

Solution 4.3. In any frame the angle is given by

80 B /IEIIBI
cos


Now is invariant (see Problem 4.2), but


. IEIIBI is not. Thus, ° 0
can be invariant if and only if . 0, i. e. (J 0
= = 1T /2.

Solution 4.4. The expression is shown to be invariant by relating it to


2 2
the invariants E -
B and E.B:
-- --

641T(&2-lsI --

) =
(E2+B2)2 _

4(ExB)2
-- --

4 2 2
(E. B)2]
4 2 2

[E + 2E B +B ] _

4[E B -

-- --

4 2 2 4

E _2E B + B + 4(E.B)2-- --


(E
2 _

B2)2 4(E.B)2 +
--

Solution 4.5. Let v


--

aCE x B), then
----


E'y-l
""""""

E + vxB
....................

(1-aB )E + a(E.B)B
...................--...


B'y-l =
B -

vxE
...................

(1-aE )B + a(E.B)E
,.....,,..............

Case i) If =
0 and lEI
. =
IB\, the transformation is equivalent to

the redshift of a plane wave, and E and cannot be made parallel.

2 2 2
Case ii) If E B . =
0 but E 1= B choose a =
1/max(E , B2) to make

E' or B' vanish. Then E'x B' = O.

Case iii) If E.B


........
1= 0, then E' and
........
B' can be made parallel by choosing
2 2 2 2
)2

[ (E B E B ] 1 a(E + B ), that
d to satisfy a
. - = -
so

ExB

--


a( x ) =
a( x ) =
- -

2 2 2 2 2 2
1+v 1+a2[( x
)]2 1+a [E B _( B )2]
' E +B

2 2
Solution 4.6. If E -
B > 0 perform a Lorentz transformation with
2 The transformation is analo-
v =
Ex B/E to get B' =
y(B -
v x E)
,..."

O.
...--...
,...... .....-..

2 2 2 2 2
E B < 0 and B /B If E B =
0 then clearly
gous for
x E

= -


- .

IBI after any transformation. A transformation of the form (see


lEI =
194 PROBLEM BOOK IN RELATIVITY AND GRAVITATION

Problem 4.5) v =
aCE x B) reduces the magnitude of E or B by a factor


y(1-aE ) =
y(l-v) =
[(1-v)/(1+v)]2. In the limit v 1 the magnitude

of E and B can be made arbitrarily small.

Solution 4.7. The integration in the expression for JI1 can be performed

by using

J F(r)o[t-zO(r)]dr J F(r)o[t-zO(r)](dr!dt)dt = =
F(r[t])!uO ・

The res ult is


u f.1.
k 3
JI1 =

0 eko [ -!k(t)].
k u

The time and spatial components obviously reduce to the correct

express Ions.

Solution 4.8. For this problem it is useful to introduce the totally anti-

symmetric tensor in three dimensions, E


ijk
(with normalization E
123

1).
Note that
F ij = E
ijk
Bk Bi=!.EijkF'J k

(Ux V)i
ijk j k
= E U V .

--- --


Now, for the "electric" equations Ff.1 =
417 JI1 :
,v



(3 =
Fl.
O'

V .
E =
417 J


417p
, (3 ,1..- --

i (3 ij iO i

,(3

F .

,J
+ F
,0


ijk

k,J,O

_
E =
417 Ji .

th
This last equation is the i component of


.......
x B
....-...-

E =
417 J .

........

In the "magnetic" equations first assume a, (3, yare all spatial.


Take a= 1, (3=2, y=3.

3 2 1
F + F + F =
B + B + B = . =

12,3 31,2 23,1 ,3 ,2 ,1
CHAPTER 4: SOLUTIONS 195

Now take a =
0, and {3, y spatial:

F O' .k

1, J

F.o
J, 1
. + F"
IJ , 0

E.
1,j

+ E.
J ,


+ E. .

IJ k


E"
lJ k
(V--
xE)k--
+ E"
IJ k
Bk =
0 .

Multiplying this equation by E


ijm
gives us V
--
x E + B=
..-
O.

Solution 4.9. It has already been shown (Problem 4.8) that Fllv =
0 is
,v
equivalent to the "electric" Maxwell equations (in vacuum). As for the

magnetic equations:

*FIlV =!. (F E a (3flV) = 1. F E



(3/lv
,v 2 a(3 ,v 2 a(3,v

1. Fr Ea(3/lv
(3 ,v ]


2 La

*F l1
so that
v,v =
0 is equivalent to F
a(3 ,v
+ F
va ,(3
+ F
(3v ,a

O.

Solution 4.10. With fl =


0 the Lorentz force equation is

o Oi i
dP / dr =
eF u =
eE yv i .

Since dr =
dt/y this equation can be written

o i
dP /dt =
eE vi .

Note that this differs from the nonrelativistic equation only in that pO IS

the re lativis tic mass-energy.

Solution 4.11. The spatial components of the Lorentz equation are

dPi dPi i i0 1j u
. .

I 1J
..


- =

- =
qF Ilu =
qF U

+ qF .

qyE +
qy E Bk V J '

dr dt 11

so that

dP/dt
........

q(E+vx B)
....... ........

196 PROBLEM BOOK IN RELATIVITY AND GRAVITATION

Solution 4.12. t
The only nonzero components of FIlv are F =
FYt =
E.


The initial 4-velocity u of the particle has components (y, yv, 0, 0),

2;


Y ==
(1- v ) the Lorentz force equations

dpll/dr =

qFllvuV
dp / dr

then give us = 0 for 11
= 1 and hence


U =
yv r =
x/ yv .

The equations for 11 =


0, 2,

Y Y t
dut/dr =
(q/m)Eu du /dr =
(q/m)Eu

can be combined to give the equation

2 2
d u

/dr =
(qE/m)2 u

with the solution

[(qE/m)r]

u y sinh

t Y
where the initial conditions on u and u have been used. Integrating
this result with res pect to r, substituting r =
x/yv, and using the initial

condition _


0 when x =
0, gives

y =

(q )[ Y cosh
(q; ) :Y ] -
1 .

Solution 4.13.

(a) From the Lorentz force equation (see Problem 4.11)

wp =
\dp/dtl =
qvB ,

therefore

wp mw mw
B = =



qv 2 2
q(1_V )2 q(1_w R2)2

(b) The 4-velocity components seen in the lab are

1 1 1
2 2
)-2
-2 2 2-2
R2)
O 2 X
(1- w R)

(1- =
wy(1_w R = wx

u =
w U u -

CHAPTER 4: SOLUTIONS 197

The components measured by the moving observer are found with a Lorentz

transformation, e.g.
1-
0' 0 2 2
)-2

u =
y(u -(3u ) =
y(1-{3wy)(1-w R

where y ==
(1_(32)-z.
0'
(c) Since u is not constant, energy is seen to increase:



0 0 Y
dp =

m =
m {3 wyu
dr dr 1


(1_w2R2)

In the moving observer's frame he observes an field EY = -


y{3B.
(This is easily computed by Lorentz transforming FIlV.) The observer
should then expect to see work being done on the particle at a rate

, ,

0 Y Y
dp mwyu
, ,

Y mwyu

dr

qEY u =




(1_w2R2)

(1-w2R2)
This resolves the' 'paradox."

Solution 4.14. From the Lorentz force equation (Problem 4.11),

dp qQ
( )

dt -qwRB +
rm dt (yy)

= ・



2 2-z
(1- w R ) is constant.
No work is being done on the particle so y
= a


Since dv/dt = -
w Re we have
_ -r


2 2
qwRB -

qQ/R

= mw

R/(1_w R )Z

and hence






(1_w

R2) Z (wB _

Q/R )

Solution 4.15. The form of the electromagnetic stress-energy is given


in

the introduction. Taking its divergence we have:


198 PROBLEM BOOK IN RELATIVITY AND GRAVITA TION

417T JLlI =
FJLa FV + FfLaF v _ 1. F F

{3,11
,11 ,v a a, v 2 a{3


FJLaF V + F (Ff.1U ,11 _ 1. F
va
,JL)
a,v va 2


Ff.1U F v + 1. F (Fall ,fl + FJLa,lI + FlIJL ,a)
a, v 2 va

lI
According equations, both F a,v and the terms
to the free field Maxwell
ll
in parentheses vanish, hence Tfl O. Note: If a charge current J is =

11 ,

present this result is modified to TJLlI 11 ,


= -
Fila J


as is easily verified.

."IlVFaf3Fa(3)
a v
Solution 4.16. From the stress-energy Tllv =
(I/411)(Fll F -

the trace is easily computed to be

TIl
Il

(1/411)(FIU2 F
Ila


4 ・

af3 Fa(3) =
0 ・

Solution 4.17. It is sufficient to show that this relation is true in one

frame. 2 2
Case 1: E =
B , E. B
-- --
= 0 .

Choose E = E e ,
B = E e ; then the only nonvanishing components of
__ -x --
--Y

TO

TOO 1417 from which it follows that Tf.L a T v
ll z ZZ 2
Tfl are = = T = E

O.


Case 2: (E -

B2)2 + (E. B)2 1= 0


-- --

From Problem 4.5 we can make and parallel. Choose =

E x'
B =
Be. The nonvanishing components of the stress-energy are then
- -x

TOO = _

XX
= TYY =

ZZ

(1/817) (E + B2) and

2 2
TJL T


11
= ofl11 [E

+ B2]2 1(817)2 = ofl [(E 2
11
_

B2)2 + 4E B ]/(817)2

8 [(E

_B2)2 +(2 .!!)2]j(811)2 .

Solution 4.18. If u JL is the 4-velocity of the conducting element, then


the field carriers in the conducting element is Efl
by charge

seen

ll
FJL11 u . A vector which reduces to J in the rest frame of the charge
_


carriers is Jfl + ufl J 11 u . Ohm's equation can then be written as
CHAPTER 4: SOLUTIONS 199

JIl +
ullJvuV =

aFllvuV. Since this is a tensor equation and is correct in

the rest frame of the conducting element, it must be correct in all frames.

Solution 4.19. From Problem 4.7 we know that, for a particle of charge q



J/l(x) =
q 8 (x -

z[r]) u/l dr ,

so that the action can be written as

J Ldr =

J u/lA/ldr

J dr

and the Lagrangian is:



7.

f3u uf3)

L =

qu/lA/l m(- T/ a

The coordinates zA specify the position of the particle parametrized by


rand
aL aL


qAA + mU

a(dz / dr) au

The Euler-Lagrange equations then give us

dU A
dzll
( )
aL
-!x

dr auA

qAA,/l T+

(fr""

az
L =

qU/lA/l,A

so that the equations of motion are

u ll

duA/dr q(AJL,A AX,Il)
= -

or

ll
dPA/ dr =

qFAllu
Solution 4.20.

(a) From the form of Fllv in terms of E



and B

(see the introduc-
tion to this chapter) and from the definition for *F (see Problem 3.25) we

have
200 PROBLEM BOOK IN RELATIVITY AND GRA VITA TION

*F

{3 = 1 E a{3ll v F
2 Il V


0 _B -BY _B

X Z
B 0 E -EY


BY _E 0 EX

B EY _Ex 0

therefore F *F corresponds to E -

Band B E.

(b) [Note: Since *(*F) = -


F, if we treat * as an operator it has
the same algebraic properties as i, hence we write e *aF = F cos a +

*F sin a.] According to Problem 4.9 Maxwell's equations, in the absence


of sources, are

*FIlV =
0 FIlV =

,v


Clearly the transformation F *F leaves these equations invariant i.e.

*F is a solution if F is. Since the equations are linear, a linear com-

bination such as e *aF must also satisfy the equations.

Solution 4.21. Since we know that F *F under the transformation

B; B '

then K would be interpreted as (minus) a charge-current of "magnetic



charge." Note that by the F *F duality V. B = -
417K implies the

existence of magnetic monopoles.

Solution 4.22. In 4-dimensional Euclidean space described with cartesian

coordinates xi(i =
1,'''' 4), consider the function G =
1/ (x

)2. It is
2 i
2 2 i .

straightforward to show that V G == a G/ax =


0 except possibly

at the point xi =
O.

If we introduce 4-dimensional spherical coordinates


x =
U cos V, r =
U sin V (O:S U < 00, 0 ::; V 17)

(J=(J cp=cp.
CHAPTER 4: SOLUTIONS 201

2 2 4
then G =
U- . The integral fV G d x can easily be written in

4-spherical coordinates. If the 4-volume is taken to be the pseudo-

spherical region U:S U


o ' the volume integral can be converted into a

surface integral on the 3-spherical surface U =


Uo

TT TT 2TT

J I I f
2 4 x=
dCP( )v3sin2vsinO


V Gd

dV

dO

g =
_417 ・

u<u

2 2 1 2 3 4
It follows therefore that V G = -
4TT 0(X )0(X ) 0(x ) 0(x ) and that we

can therefore use G as a Green's function to solve the 4-dimensional



problem V (I)

Poisson = -
4TT f(x ) as

f?(i)d4(
(Xi) 1 =

TT
J I

(,i_xi)2

Now suppose that f is somehow defined for imaginary ,4. Introduce



new variables ,4 =
ix , and ,i =
xi, i =
1,2,3. Define S such that


fe' i) =
S(x ) .

2'l 1 2 3
Notice now that V (I) 4TT f(x )
= -
becomes 0 (I) -= -
4TT Sex ,

, X
, t). The
solution to this problem is the above integral. If we transform variables

we have "


Sex, t) d x dt




(I)(x, t) =

1T
xll) (xV xV)

TJ V (xll-

Il

where it is understood that the t integration extends from ioo to -


ioo.

Assume that S (x , t), the source term in the d' Alembertian equation, is

defined and has no poles on the real t axis for t < t. Take the defini-

tion of S elsewhere in the complex t plane to be defined by analytic


continuation. The contour of integration is shown as C in Figure 1. The

boundary conditions are specified by how the contour C is deformed to


run along the real axis where the source


S(!., t) is defined. The poles of
202 PROBLEM BOOK IN RELATIVITY AND GRAVITATION

III
,.

Imt 1m t

c'
..
..

Ref Ret

-18'-31 t t+li-.!I t

(0) (b)

1/ (xp. xp.) (xp. t t:t Ix .!I. Retarded boundary conditions


Xp.) are at
- - = -

correspond to deforming C into C' in Figure 2. This can be verified by

using the above result to solve

o A = -
4 TT J
f1

11

We get:
a 4
J 11 (x )d x



A (x ) =


11 TT1 r r
c'" a

We now show that this reduces to the familiar retarded integral solution
by performing the integral over dt:

Jp.(x ,t)d3xdt


Af1(x, t)
= -
-: " "

TT1
(t-t-Ix-xl)(t-t+lx-xl)
........,....,. .......

= _

111
f [!p.(R,t)d3xl
t-t-I.!-.!IJt=t_I _!.1 f
2TTi Jp.(x,t-I!.!-.!i)d3x

= .

)-l/ax )-2,
a V V a
Since a(r a r = -
2x (r a r a formal expression for the

fie ld tens or is
"a 4"
)d
X[vJp.](X x


F 2A = .i
[v,ll]
= .

Il v TTi a

(r a r )
CHAPTER 4: SOLUTIONS 203

Solution 4.23. Since the conductivity is infinite, the Lorentz force on a

charge carrier must be zero, so E + v x B =


O. Maxwell's equation

B = -
V x gives


.......,

Vx(vxB)
...........--...........

-B(V.v)+(B.V)v-(v.V)B. (1)
""""...-.. ........ .....-,.......,.......

If we now specialize to the instantaneous rest frame of the fluid, =


0,
t = r
(proper time), then y

1 and has vanishing first derivatives, and

a/at becomes alar (Le. the convective derivative and the partial deriva-

tive are equal), so

i j j i
dBi/dr = -

B v. + B v. = _




. + Bju i .

, J , J ,J ,j


Since u .
is the projection perpendicular to u, of uf.L it follows that
,J ,v


u..
1,J

w..
lJ
+ a..
lJ


g..O
lJ


The contraction of this 3-tensor is of course u .

() and the proper time
,1

derivative of B is then

i 2
dBi/dr =
_B ()+ B.(w ij +


ij +!. gij()) = _
Bi() + (a
ij +w ij B.
) J

(2)
3 3

To Equation (2) may be added any term proportional to the spatial com-

ponent of the 4-velocity (since that vanishes in the comoving frame). Thus
write Equation (2) as

dBi/dr 2
BiO ij i
) B.J
ij
= _
+ (a +w + fu (3)

where f is as yet arbitrary. Now transform Equation (2) into a tensor

equation by
(a) Letting indices run from 0-3 instead of just 1-3;

by Baua

(b) Defining a 4-vector B =
0 and the spatial part of B

equal in the comoving frame to ;


(c) Changing ordinary derivatives to covariant derivatives;

(d) Defining f so that d(B'u)/dr =


0, Le. so that the normalization

of B is preserved.
204 PROBLEM BOOK IN RELATIVITY AND GRAVITATION

The unique equation satisfying (a), (b), (c), (d) is

(w {3 + aa(3)
B{3
{3B{3
a a
(jpa (4)
DBa /dr u
a{3

B{3
= -

where a is the 4-acceleration, and pa is the projection operator de-


{3 I

fined in Problem 5.18. Equation (4) is obviously a tensor equation and


reduces to Equations (3) and (1) in a comoving orthonormal frame, thus it
is the correct equation in all frames.
CHAPTER 5: SOLUTIONS

Solution 5.1.

(a) In the rest frame of the particles the only nonzero component is
/. ,

TO =

PO' By transforming to a frame moving in the x-direction with

velocity -{3, we find the nonzero components

Ox xO 2
TOO =

poy2 T -


T -

Po Y {3 ' T
XX

P o{32y2 ,

where y ==
(1- (32)-"2. In general, if u is the 4-velocity of the particles

T=pou u.
(b) Let the circle of motion be in the x-y plane. Consider a con-

tinuum of rest mass density Po at the point x =


0, y= a moving with
velocity (3 =
wa. From part (a), at x =
0, y

a we have TOO =

poy2 etc.

Now if we consider a ring of matter, all points on the ring are equivalent
so that in polar coordinates the nonzero components of TIlV are

TOO =

Poy2, T
o 4> =

Poy2 f3 T4>O, =
T4>4> Poy2 f32

We now must relate to the collection of particles. If we assume that


Po
N is large enough so that we can view the particles as continuously dis-

tributed around the ring, then


Po
ex
oCr -

a) o(z). The total energy of the

particles must be yNm so

TOO 217 r dr dz =
yNm

and Po

Nmo(r- a) a (z)/217ay .
The results can now be expressed in car-

tesian coordinates by making the usual transformation x =


r cos cp ,

y =
r sin cp :

205
206 PROBLEM BOOK IN RELATIVITY AND GRAVITATION

1 -{3 sin cP {3

cos cP o

TI1V =

yNmo([x +y2]2 -a)o(z) {32 sin2cP _{32 cp cos cp sin o
217a
{32 cos 2cp o

(symmetric) o

(c) If we have an additional ring rotating in the opposite direction

then that ring's stress-energy is the same as that in (b), with the sign of

{3 =
(Va reversed. We can then add the two cases. In the sum, terms

linear in {3 disappear; terms quadratic in {3 are doubled. In polar

coordinates, the nonzero components are

"" ""

TOO =
TcPcp /(32 =
y Nmo(r- a)8(z)/17a .

Solution 5.2. In Problem 5.1 we saw that for particles with 4-velocity u,
the s tress-energy is proportiona I to u u. F or an isotropic collection
with u =
y(l, v )
we have

T = K < U U > , K =
some constant,

where the average is taken over the orientations of n. By symmetry, off


diagonal components average to zero:

Oi 2 i
T =
Ky V < n > =

2 2
Tij =
Ky v < ninj > =
0, i 1= j .


To evaluate the diagonal, spatial, components note that < nXn > =


< n



> = < nZn > and that

x z 2
< nXn > + < n



> + < nZn > = < n > = 1

so that

1 ii 2 2
< nini > =
T =KYV /3 .

We know that the moving particles each have energy my so the density

of mass energy must be


CHAPTER 5: SOLUTIONS 207

00 2 2
myN =
T =
KY < 1> =
KY .

Thus K =
mN/y and the nonvanishing components are

00 2
ij
T =
myN Tij =
(myNv /3)a .

For photons v c, my hv. For cold dust v =


0 so TOO =
mN is the

only nonvanishing component.

O i
Solution 5.3. In the rest frame of the fluid element u =
1, u =
0 so that

TIlV =
pgllV + (p + p) ulluV .

Since this is a tensor


equation which is true in the rest frame of the fluid

element, it must be true in general.

Solution 5.4. We can always choose the spatial coordinates to diagonalize


the 3-dimensional ij
stress tensor T . It is obvious by symmetry that one

of the spatial directions which is singled out is the direction of B,



and

that the choice of the other two orthogonal directions is arbitrary. If we

XX XZ
choose B


Be
-z
it follows that T = TYY and that TXY =
T = TYz =
o.
Oi
Since there is clearly no energy transport in this static problem T =
O.

The energy density of a magnetic field is TOO =
B /817.
Xx Zz
We must now calculate the "pressures" T , TYY, T . We do this

by considering a box x, y, z of the field and imagining it expanded

adiabatically. (An adiabatic expansion keeps constant the magnetic flux

Bx y

i. e. the number of "lines" of B in the volume.) The energy

in the box is

& =
(B2/817) x =
y(B z x Z/817
y)2 X .

The pressures can now easily be found:

T XX =
1 d& =

B /817 =
TYY
y z d( x)


ZZ

1 d& = -

B /817
x d(y z)
208 PROBLEM BOOK IN RELATIVITY AND GRAVITATION

so that

2 1 0

Tllv =

817 1
0 -1

To find the stress energy of a chaotic field start by choosing a set of

orthogonal axes and average the fields in the x, y, and z directions.

From our previous result, the averaging gives


13

Tllv>
1 3
< =

817

1 3

where B is the average strength of the field. If we picked any other

orthogonal directions we would get the same result because the 3 x 3

identity matrix block is invariant under rotations; hence the above result

also represents the average over all directions.

Solution 5.5. We are given the linear mass density as Il, so TOO III A.

Oi
In the rest frame of the rod there is no energy flow so T =
O. Choose

the z direction to be along the rod axis. By symmetry [see the argument
Xz
in Problem 5.4] the off diagonal stresses TXY, T , TYz, must be zero.

FlA. Since there is


ZZ
Since the tension is in the z-direction, T = -
no

applied stress in the x or y directions (therefore no transport of x or

XX
y momentum) T = TYY = O.

Solution 5.6. Suppose a force infinitesimally smaller than F is applied.

If the cross section is A and the axis of the rope is in the z direction,

the components of T in the rest frame of the rod are given in Problem
5.5. An observer moving with velocity u =
(y, yy) sees energy density

0'0' 2 2
T =

TafJuaufj

(y IA)[Il- Fv z ]'
CHAPTER 5: SOLUTIONS 209

0'0'
Clearly the observer with the greatest chance of seeing a negative T
is the one moving in the z-direction with v c. To guarantee that he
, ,

sees a positive TO 0, the weak energy condition requires F < f.l, which

is the required upper limit. In physical units, and for good steel rope

this would be
2 2 13 2
< c =
pc

7x10 Kg-wt!mm .


as compared to its actual breaking strength of 200 kg -

wt/mm . The
11 5
for this difference by factor of order 10 strength

reason a is that the

of steel comes essentially from the energy of the molecular bonds

( leV per atom), not from the total mass density, which is virtually all
10
in the nucleus (,,- 5x 10 eV).

Solution 5.7. Let S' be the instantaneous rest frame of an element of

the rod. In this rest frame the

y only nonzero components are


, , , ,


TO =
p, T
X x

p. If we

Lorentz transform to the lab

frame we find the nonzero com-



=y2{32p,
XX
ponents: T =
p, TYY

TOO y2p, TOy =y2{3p,


here

2 -z
{3 =
wr and y ==
(1- (3) . If

spherical polar coordinates are

used the nonzero components are


rr

p, Tcpcp =
y2{32p/r2, TOcp =
y2{3p/r, TOO =
y2p .

To find per) we can use the r component of the equation of motion

o =

rv
jV


rr

,r
+ Tcpcp rr,./,.,./,. + T
rr
[lo g ( _

g) !] ,r


(T
rr


) ,r /r
2 _


sin 0 Tcpcp ,

so that
210 PROBLEM BOOK IN RELATIVITY AND GRAVITATION

2 2
(pr ) ,

= r sin e y2{32p .

Since the only mass density is in the point masses we have


P =
mo(r-a)0(cose)[0(</>-cut)+0(</>-cut-17)]/r .

We can now integrate the equation of motion with the boundary condition
p =
0 for r > a, to find p. When this is done the components of the

stress-energy tensor (in the lab, described by coordinates t, r, e, </?


are seen to be


TI1V =

2 2
o(cos e) [o(</> -

cut) + o(</> -
cut -

17)]
l_cu a

oCr -

a) 0 0 cu o Cr -
a )
2 2
a 2 a
cu a
--
0 0

x r

0 0


cu 0 (r -
a )
symmetric

Solution 5.8. The electrostatic field for the moving capacitor remains E,

since electric fields parallel to the direction of motion do not change.

The moving capacitor, due to Lorentz contraction, has thickness



d/y,
) -"2. The moving capacitor then has electrostatic

where y ==
(1- v
2 2
energy E Ad/8ry < E Ad/817.
The pressure of the gas must be equal in magnitude to the (negative)

pressure of the E field, so p

E /817. The nonvanishing components
of the stress-energy of the gas in the capacitor rest frame are

00 XX ZZ 2
T =

Po' T = TYY =
T = E /817 .

The total rest mass of the capacitor is


M =
E Ad/817 + Ad
Po

CHAPTER 5: SOLUTIONS 211

If the capacitor is moving calculate the energy density of gas


we can
by
using a Lorentz transformation:

, ,

TOO =
y2(TOO+ 2TXX) y2(pO+ 2E2/81T) =

If the capacitor is moving, then its total energy is


& =

y2(pO+ 2E2/81T)Ad/y
Ad/8rry +


(YPO+yE2/81T[1/y2+ 2])Ad Y(PO+E =


/81T)Ad =
yM .

Solution 5.9. For simplicity assume Minkowski coordinates. The stress

energy for the particles is

Tf.LpV(x) m.

f ulf(r.)u (r.)04[x-x.(r
.)]dr
= '


1111 1 l l

Thus

f ulfu
f.L V
Tp
,v

m.
1 1 1 (a/axV)04[x-x.(r
1 .)]dr l



The delta function depends only on x -

xi so we can replace a/ax by


a/ax .Since u a/ax =


d/dr.1 we have then
1 1 1

mi f u

Tf.LP,v = -

(d/dTi)a4[x-xi(T)]dTi




f (du xi(Ti)]dT
/dT)a4[x- i '

where the second result follows from integration by parts.


th
N ow for the i particle

miduf /dr i qi Ff.Lv(x i ) ur



so , with the result of Problem 4.7,

T ,v

qi
f FlLvura4[x-xi(Ti)]dTi

Ff.Lv Jf.L .
212 PROBLEM BOOK IN RELATIVITY AND GRAVITATION

But, from Problem 4.15,

TlLv
EM,v

(41T)-l F lLa F V

a,v
= -
FflD. J a

so that ( TlLv

+ TILV)
EM ,v

O.

Solution 5.10. Consider a small number, dN, of photons with a small

spread in frequency and position, moving in a narrow cone, and impinging

on a small area in a short time. By definition

d (energy)
Iv

(frequency) d (time) d solid angle d cross-sectional




[ of c one ] [ area

With no loss of generality take the narrow cone of the motion, whose size

is d (solid angle), to be along the z-direction, then

d (energy) =
hv dN
2 2
dpx dpY Ih

d (solid angle) =
dv dv Y =
d(pX IE) d(pY IE) =

d(time) =
dz/c =
dz

d (cross sectional area) =


dx dy

d (frequency) =
d(hv)/h =
dpz Ih

so that

4 3
dN h v
I =


dx dy dz dpx dpY dpz
3 3
Number density in phase space, dN/d x d p, is a Lorentz invariant (see

Problem 3.34) so
Ivlv3 is invariant.

Solution 5.11. If spherical polar coordinates are used to describe the

3-d imensional s pace in the frame of the star, then in that frame the radia-

tion has stress energy


CHAPTER 5: SOLUTIONS 213

t r () cp
1 1 0 0

2 1 1 0 0
Tllv (r , () , cp) =
(L/417 r ) 0 0 0 0
0 0 0 0

Let t be the vector (null, of course) from the emission of a photon to the

reception of that photon. In the star's frame, if a photon is received at a

distance r from the star, eO =


r, e


r, e(} =
ecp =
0, and the stress-

energy tensor at the event of reception can be written in the frame inde-

pendent form

T =
[LI 417( us' f ) ] f t

where Us is the 4-velocity of the star so that, evaluated in the star's

frame, us. f = -
r. It is now simple to use the frame independent expres-

sion for T to evaluate the flux the observer sees. Let n =

(O,!V be the

spatial vector which, in the observer's frame, points to the observed posi-

tion of the star, so that f =


(R, -RE)' The flux measured by the observer
will be, in the observer's coordinates,

Oi
F = -
T n. =


T .
n .

o b s 1 obs

In the observer's coordinates

Us. f (y, y,y)' (R, -R!V -yR(l+v 0)


= = cos ・

Furthermore t. U = -
Rand t. n = -
R so that
ob s

Fobs =
L/[417y4(1 + v cos (})4R2] .

Another way of arriving at this result is to use the fact that Ivlv3 is a

Lorentz invariant. (See Problem 5.10.)

Solution 5.12. Let the 4-velocity of the particle be u. Let the null

4-vector along which the radiation propagates be t. The particle absorbs


a 4-momentum flux -

SAC U. f) f, i. e. the stress-energy of the radiation


214 PROBLEM BOOK IN RELATIVITY AND GRAVITATION

T =
Sf f multiplied by the effective area of the particle and dotted into

its 4-velocity (to get the flux in its frame). The minus sign is due to the

+++ signature. The time component of this in its rest frame is

SA(u. f) (u. f); this is the absorbed energy which the particle reradiates
in its own res t frame, so the net change in 4-momentum is

dp mdu
dr

dr
= _
SA [(u. f)t + (U' f)2u] .
(1)

This is the equation of motion for u.

To solve, let W == u. f, and dot the equation of motion with f to get


dW/dr =
-(SA/m)W
and hence

W = -
[(2SAr/m)+ K]-2 (2)

where K is some constant. We now treat W as the independent variable



and substitute dr = -
dW m/SAW in Equation (1) to get

du u
l f
_




(3)
dW
W2

This has the integrating factor l/W and can easily be integrated:

J!.
dW ( W1 )U = .l. f
w3
(4)

u = _.1.. f + Wq (5)
2W

where q is a constant of integration. The conditions u.f =


Wand

u.u =
-1 imply that q.f =
1 and q'q

O. Substitute for W from Equa-

tion (2) and integrate Equation (5) to find

x =
+ (2ar + K) i e -

(2ar + K) t q +
Xo

where a =
SA/me If we choose the constant of integration so that the
CHAPTER 5: SOLUTIONS 215

particle starts from rest at the origin in a radiation field parallel to the

x-axis, this is
t = l
6a
(2aT+1)t + l
2a
(2aT+1)t_1-
3a

x = ..!. (2aT + 1) t - 1.. (2aT + 1) t + 1...


6a 2a 3a

y =

z =

(The solution here follows that of Robertson and Noonan, pp. 116-118.)

Solution 5.13. An observer on the sphere sees the black body spectrum

doppler shifted by different amounts in different directions. Since the


hV /KT
black body spectrum has
Iv lv3

constant x (e -

1)-1, and since

Ivlv3 is a Lorentz invariant, the effect of a doppler shift V



v is just

to change the effective temperature T 0 T =


T o(vlv o ).
What is the doppler shift seen by the comoving observer when he looks
at an angle () to the forward direction of motion? This standard result is

easy to rederive with invariants: Let u be the 4-velocity of the sphere,




be the rest 4-velocity of the radiation, and p be the 4-momentum of

a photon in units of Planck's constant. Then v = -


u'p, V

= -

uo'p and

2 -z
(1- v) ==

= -
u'u

' In the sphere's frame,

u =

(1,.Q), u


(y, -

y,V, p =
(v, v

where .!! is a unit 3-vector. The unit spatial vector in the direction of
.Y
IS

U +
(uo'u)u
(0, I)

yv

(This is verified by substituting in components in the sphere's frame.)


Similarly, the vector (0, is invariantly written as

p + (u'p)u
(0, n)."...


216 PROBLEM BOOK IN RELATIVITY AND GRAVITATION

so the angle of the photon in the sphere's frame is

( )

Vo yv

cos (J =
(0, . 0, =- =

Ivl yvv

The las t line is obtained by dotting together the invariant 4-vector ex-

pressions and using v =


-u.p etc. Solving for vivo' we have

T = = 1


To o y(l+v cos(J)

Finally, the equilibrium



temperature is given by the average over all

directions < T((J)4 > 4". [This is because the radiation rate, which equals

the absorption rate in equilibrium, goes as T (Stefan-Boltzman law).]
Thus cos(J=l

< ) J
T 1 d(cos (J) 1 1 1 1

[(1-v)3 ]
_ -

- -

2 6v
To y4(I+v cos(J)4 y4 (l+v)3
cos(J=-l

and
1.. 1..
4 4
(y2(I+v /3))4 TO

T =
< T > =

equilibrium

Solution 5.14. Let the average transfer of energy be < We


E>. are

2 2
given that E/mc ? 1 and T/mc ? 1 (units with k= 1), so we want

only the first nonvanishfng terms in the double series expansion


< E>
mc

[a 1 + a
2 (E/m) + a
3 (T/m)
(1)
2 2 2 2 2
+ a
4 (E
/m ) + a
s (ET/m )
+ a
6 (T
/m ) +...] .

If T=E=O, nothing is happening, so a




O. If T=O, E O,
we have

standard Compton scattering with a cross section da/dO ex
(1+ cos (J)

and an energy transfer E =
(E /m) (1- cos (J). Since the cross section

is forward-backward symmetric, the cos () term cancels out when we

average over angles, and


< E>(E /m),

T=O.
CHAPTER 5: SOLUTIONS
217

This implies that a



=0 and a

=1. If E=O, T O,
the photon has
zero enei'gy and is vacuous, so a =
a =
O.
3 6

Finally, we need Take a thought experiment where there is


as: a

dilute black-body flux of photons with the same temperature as the gas so

that
d (number of photons)/dE 2 E/T

constant x E e- .

Plugging in Equation (1) with a



=a

=a

=a


0, a


1, and requiring

thermal equilibrium, namely foo < E>
E e- E/T
dE =
0, gives

0= (3T/m)(4T+a s T) or
as

-4. Expression (1) now takes the final form

2 2
<E> =
(E -4TE)/mc +... .

Solution 5.15. The technique here is to use the equations of motion



(T/l , v

0) to convert first derivative terms to divergences and then, by
Gauss's theorem, to vanishing surface integrals outside the system. By

repeated use of the equations of motion we have

00 Ok kO km
T = -
T = -
T T
00

, , kO ,
Ok , mk'


Now multiply by xix :

OO km
(xixjT ) ,00 =
xixj(T ) , km


(Tkmxix j ) , km

2(x



) "

Tkm m

Tkm(xix j ) , km

j i
(Tkmxix j ) 2(x Tim xi Tj 2T j


km
_




, m
) _

The first derivative terms can be simplified, e. g.

j Tim j oj m Tim

,m

(x Tim) ,m -

If we make this simplification, integrate over all space, and ignore all

divergences which can be converted to vanishing surface integrals we see


218 PROBLEM BOOK IN RELATIVITY AND GRAVITATION

J XiX j TOO d 3 x

d i


[O _2(_Tij_Tij) -
2T j] d 3 x
dt



J Tij d 3 x ・

Solution 5.16. Suppose v is a timelike eigenvector

T .
v =
av .

Consider the stress-energy measured by an observer with 4-velocity

u = :t v / I vi. (Choose the sign to make u future-pointing.) In that ob-

server's rest frame u =


(1, Q) and the energy flux vector S is

S T.u
I I T.v
,:\
v au (a, 0 )
- = = = = =

Thus the observer sees no energy flux, and a is the negative of the

energy density he sees.

Conversely if an observer with 4-velocity u sees no spatial flux,


then -
S =
T.u =au for some a, and u is thus a timelike eigenvector of T.

Solution 5.17.

(a) If secoqd order terms in velocity are ignored, the general Lorentz
transformation is . .'
° i
AOo '=
1, A .'= v , A 1. , = 0

., .

1 J J

, ,


If T/l are the stress-energy components in the rest frame of the fluid,

then TO'i' =
0, and

0'0' ° k'
, , . . .

° °

T J
0' v l

gJ == = A
/l,AJv,T/l
= A
o,AJo,T + A k,A\,T

= v
j TO'o' + vkTj'k' vk(To'O'ojk+Tj'k') = .

For a perfect fluid TO'o' =


p and Tj'k' =
p ojk so that the "inertial mass

per unit volume" is


jk
m =
ojk(p + p) .
CHAPTER 5: SOLUTIONS 219

(b) Since
Mij == oij
f TOO dx dy dz +
f Tij dx dydz ,

we need only show that the second term on the right vanishes. Note that
the 3-divergence of Tjk vanishes

Tjk k =
Tj _

TjO 0 =
0-0 ,

and that


(x Tji) ,1 .

ok.1 Tji + xk Tji.,1 =
Tjk .

It follows by Gauss' theorem that

f Tjkdxdydz f (xkTji),i =
dxdydz =

I S
xkTjini dS

where S is any surface outside the stressed body and is the outward

normal to S. Since the surface integral obvious ly vanishes, the proof is

complete.

Solution 5.18. When the definitions of and w are used, we can


aa(3 a
(3
see we need only verify


ua; PP.f3 aa uf3

;f3

It is s trai ghtforward to use the definition of pp. to verify this. Ins tead
f3
we follow a more instructive approach of projecting the equation with u

and with P. Note that

u. P =
0, p. P = P .

The projections with P and with u are:

u P f3 P P (3 pf3 -

pp. 0
aa uf3 ua;p.
_

ua;p. {3
_

;f3
- - -

a y y y y

and
f3

u u = u p P. u(3 a U u(3 =
o +
aa
f3 f3

a;
(3 a;p. a
220 PROBLEM BOOK IN RELATIVITY AND GRAVITATION

Both results check. Since the projection of the equation onto u and

perpendicular to u are true, the equation is verified.

Solution 5.19. If the fluid has mass energy density p, pressure P,

temperature T and entropy S, then conservation of mass-energy for a

fluid element of volume V is

d(pV) = -
PdV + TdS .

Here S is the entropy in the volume V. This differs from the nonrela-

tivistic law only in that we have replaced energy density by mass-energy

density, since mass is not conserved in special relativity. Baryon number


is conserved so the first law is often written in terms of baryon number

density n, and entropy per baryon s, by eliminating V ==


(number of
baryons / n):
d(p/n) = -
Pd(l/n) + Tds

or

dp =
(p+P)dn/n + nTds .

v v
Solution 5.20. For a perfect fluid T/l =
(P + p) u/lu + Pg/lV so that in

the fluid rest frame the 0 component of the equation of motion is

Ov o
T/lV _

P gOV
TOv;v,V =
+ Pg + (P+p) ,v uVu
u/l

;v ,v

+ (P+p) [u


uO + uO, v UV ] =
0 .

, ,

IIV ° o a_I a
0 th a t
Now 0' 1 an d u .1J ( )


U - so
gr 2 uau .V



U''U

, ,
v,v,a
;v

the equation becomes

o = -

gt P + t (P+p) (P+p)uv;v + ・

If n is the density of baryons in the fluid rest frame then the number-

flux vector of baryons nu is conserved. In the rest frame then

v v v dn v
(nu).,v =
0 =
n u + nu =
+ nu
'V


v 'V
, dt ,
CHAPTER 5: SOLUTIONS 221

When this equation is used to eliminate V


u .
the equation of motion

becomes
dp =
(P + p) dn

dt n dt

If we compare this with the first law of thermodynamics (see Problem 5.19)

we see that ds/dt =


0; the fluid flow is isentropic.

Solution 5.21. From the first law of thermodynamics for a perfect fluid
(see Problem 5.20) having equation of state p

pen)

dp _


p+P dn
p p n

so that

d log p/d In n =
(p+P)/p .

Thus p is greater than 3P if and only if d log p / d log n < 4/3. Since

T/l
/l

(p + P) u/lu /l + pg/l/l = -
(p + P) + 4P =
3P -

for a perfect fluid, the proof is complete.

Solution 5.22. Assume that the acoustical wave is a perturbation


(isentropic) in a uniform static fluid with parameters Po' PO' and no'
Let the perturbations be Pl' P
1 , and n

and the fluid velocity be


(1' 1)in the rest frame of the unperturbed fluid. The first order

perturbation terms in T/l =
0 give us
,v


apl 1
(i) From p.

V'Yl = -

dt (Po+Po)-

aYl Pl
(ii) From p. =
1,2,3
at
_

(p 0 + P 0)
which can be combined into


a2pl/at2 -
V P


0 .

Since P1 and Pl are related for isentropic flow by


222 PROBLEM BOOK IN RELATIVITY AND GRA VITA TION



ap/apls Pl '

the above equation becomes a wave equation

a2 p 1
2 1
V Pl -- = 0
2 2
V at

with characteristic velocity v


s'

For an equation of state p 3P,

v; =
ap/ap =
1/3

so that v

3-"2 in a highly relativistic gas.

Solution 5.23. From the first law of thermodynamics (see Problem 5.19)
with ds = 0 we have
dn/dp =
n/(p+P) .

Now we use the definition of r 1 and the expression for v; from

Problem 5.21:

v; =
dP/dp _

P dn =




(ndP)/(Pdn) dp p+P

Solution 5.24. At zero temperature all energy states in the Fermi gas are

filled up to the Fermi level E =




. Since there are two spin states for

fermions the density of fermions in phase space is 2/h so that

3 3
dn/V =
(2/h )d p



t 3
(2/h ) 41Tp2dp



(p2 + m )


p2(p2+m2)-t (2/h


P =
) 41Tp2dp

where P is the momentum corresponding to the Fermi energy. In


Problem 5.22 we found v; =


dP/dp so
CHAPTER 5: SObUTIONS
223

dP/dPF P
\2
( )

dP F 1
v2 1 m

\1-
= = = =

dp /dPF

dp

3 E 2
F } 3


Note that for a highly relativistic Fermi gas v



3-"2.

Solution 5.25. The relativistic Bernoulli equation, which expresses the

conservation of energy along flow lines of a perfect fluid, is (Problem 14.7)

(1_v

)-t =

( p:p ) x constant.

(This equation follows from TOv,v =


0, from (nu/l),/l

0 and from the

first law of thermodynamics dp/(P+p) =


dn/n.) Since the gas starts in

the tank at zero velocity, we have: constant =

(Po + po)/n o ' An upper

bound for n/(P + p) which will give an U Pp er bound v


max
for v, occurs

when P O
implying p =
mn, so

.! P +P

( O)


! o
(1 -

max
) =

m n

Now we have to try and write the right hand side only in terms of a, the

velocity of the We have P KnY where K


sound gas with Po' Po' no.

is a constant telling what adiabat the gas is on. The first law reads

dp _
Kn Y + p


dn

which is integrable and (with p mn when n 0) gives



mn + ...!L nY .

y-l
The speed of sound a is given by

yKny




dP =
dP/dn _

(1)
dp dp / dn m+yKny

/(y-1)

and the first law of thermodynamics gives us


224 PROBLEM BOOK IN RELATIVITY AND GRAVITATION

P+p dp dP dp
( )( )
1 2
(yKnY )(1/a )

(2)
= = =
n dn dn dP


But Equation (1) contains just this y

same combination yKn . So we

can solve Equation (1) for

y 1 2 2
/[1- a /(y_1)]

yKn =
ma

and get

Po+Po
( )
1 =


no 1-a 2 /(y-1)
and therefore

v ax 1 /(y_1)]2

= -

[1- a .

(Notice that v
max
1 in the limit of highly relativistic gas, for which

y 4/3 and a 1/3.)

Solution 5.26. In the fluid rest frame TOj =


TjO =
qj are the only stress-

energy components associated with q. Since u for the fluid is (1,.Q) in

its res t frame, then


Ta f3 =


qf3 uf3 qa

is true in that frame and hence true in general.

Solution 5.27. In the fluid rest frame

SO =
entropy density =
ns

sj =
entropy flux =
heat flux/T =
qj /T

qO

Since =
0, u =
1 in the fluid rest frame S =
nsu + q/T is true in

the fluid rest frame, and hence in general.

Solution 5.28. For the stress energy of the system we have

T =

Tfluid + T
heat

[(P+p)u u+Pg]
+ [q u+u .
q]

We compute the equation expressing energy conservation along the fluid

flow:
CHAPTER 5: SOLUTIONS
225

O=(V.T).u =

(V.Tfluid)oU +
(V.Theat).u ・
(1)

By a short calculation (see Solution 5.20) the first term is seen to be


st
-dp/dr + (dn/dr)(P+p)/n which by the 1 law of thermodynamics equals

nT ds/ dr (see Problem 5.19). The second term is

(V.Theat).u =

(q/luv+u/lqV);vu/l .

Since qa ua =
0 (heat flux is spacelike in comoving frame) and u


a =-l,
which imply

(q/lu/l);v q/l;vu/l q/lu/l;V

= =

o =

(u/lu/l);v =

2u/l;vu/l '

the second term reduces to -q.a


V.q. Thus Equation (1) becomes

o =
-nT ds/dr -

q.a

V.q .

Finally using the definition S =


nsu + q/T, we compute

t7
S 1 VT
V' =
sV.(nu)+n(Vs.u)+-V.q--.q

T2
ds
+ 1.
VT'q
drT V.q

n _

T2

= -
q.a VT
---'q

T2

The second term says entropy is generated by heat flow thru a tempera-
ture gradient, as in nonrelativistic thermodynamics. The first term says

that it is generated by flow along an acceleration, this is a "redshift"

effect arising from the fact that in a accelerated system, constant tempera-

ture is not an equilibrium state: photons from the "forward" side are

blueshifted in going backward, thus carrying a net heat flux.

Solution 5.29. An equilibrium state obtains when there is no entropy


generation, so according to the results of Problem 5.28
226 PROBLEM BOOK IN RELATIVITY AND GRAVITA TION

VT
q.a q

= -.

must hold for all heat flows q. Thus we must have a = -


V en T. The
solution to this equation is en T = -
a.x +
--
constant, or

T To exp(- . )

Solution 5.30. Since T



{3u a
= -
pu{3 there are no energy flow terms,

hence q =
0 and s =
nsu. The rate of entropy production is thus

a a ds ds
S =

(n u;a s + n = n
;a dr dr

where the second equality follows from baryon number conservation

V.(nu)=O.

With the first law of thermodynamics

dp p+p dn ds

+ nT
n dr
dr dr

and the law of baryon conservation

(nua);a =
+ nO

the rate of entropy production can be written as

dP

[ ]
a 1
S = + () (P + p) .

;a T dr

To express the right hand side in terms of the viscosity coefficients

take the divergence of the vector T



{3u a

_(puf\fJ = -

: -

pO =

(-rafJua);fJ -rafJua;fJ = ・

The last term on the right can be calculated using the stress-energy for

the viscous fluid and the expansion for u t:l given in Problem 5.18. The
a,fJ

-rafJwafJ term vanishes by symmetry; the -rafJaaufJ term vanishes since

-rafJufJ and left with



oc
u we are
CHAPTER 5: SOLUTIONS 227

-ra(3u [pu u{3 + (p-'£1) pa{3



27Wa{3] [U + £1p
a{3]

;{3

a a
{3

{3 £1(p-'£1)
2rWa{3 u +
= -

[Here we used the easily verified relations



U =
0, p u
a {3 =
0, P pa {3 =
3.]

{3 a{3 a{3

The res ult then is

: 2rWa{3u {3
a 2
+ £1(p+p) =
+ '£1 .

When this is substituted in our previous formula for Sa'a ,


the desired

equation results.

Solution 5.31. We know that projecting


-ra{3,{3 along u
a gives an equa-

tion of local energy conservation; to get an equation of motion for the


fluid, we project perpendicular to u

o = pYa TJ-{3 t:l =


pY (P t:lUa u{3 + pua t:l u {3 + pua u{3 t:l + P t:l pa {3
,fJ a,fJ ,fJ ,fJ,fJ

ppa{3,{3 2." ,{3u {3 {3,{3 ({3£1pa{3 '£1 ,(3pa{3 '£1pa{3,(3)


a a
+ 2rw
- -
- - -

We now use the following identities: pYaua =


0,

p Yau

uY uY
a,{3 + uY u u
,{3

,{3

,{3 a

pYa pa{3 =
py{3

pYa pa{3 t:l


= pYa (uau{j) t:l
= uY t:l u{3
,fJ ,fJ ,fJ

pYa aa{3 = a y {3

pY
a,fJ


{3t:l =
uy{3t:l
,fJ
+ uYu u
a,fJ

{3t:l =
uy {3t:l-
,fJ
uYu
a,fJ
t:lUa{3 =
uy {3t:l
,fJ

uYuat:lUa{3.
fJ

With these identities the projected equations of motion become:


228 PROBLEM BOOK IN RELA TIVITY AND GRAVITA TION

o =
(p + p) UY ,{3u{3 + p
,{3pa{3 -

2(TfUy{3 +
(,()Py{3),{3
211 UY Ua {3U {3 (,()2UY

+ + .
(1)

In the nonrelativistic limit,

j j 2


1, u v , p

(9(v ), P =
(9(1) .


Taking the j-component of Equation (1), and working to (9(v ), we get

. .

J J k
o =
p( v "

+ v

v ) + p

[l1(V ,m)].k



Vk,j
v +
?(,vm,m),j

j ,k jk

which is the Navier-Stokes equation.

Solution 5.32. For a perfect Maxwell-Boltzmann gas,



nkT (1)

pin =
U(T) (2)

Equation (2) says that the energy per particle is a function of temperature

alone. From the first law of relativistic thermodynamics,

Tds =
d(p/n) + pd(l/n)
(3)

¥dT+pd a )
we see that

dU


_



(4)
dT

Equation (1) implies that

pd a) + dp =
kdT (5)

so Equation (3) becomes

Tds =

( ¥ k) + dT -

dp (6)
CHAPTER 5: SOLUTIONS 229

and thus

dU
c =
+ k =
C + k ・
(7)

p dT

From Equation (1)

a log
I I
r1 p n dT
= =
1 + (8)
a lo g n T dn
s s

But Equation (3) implies that, when s =


constant,

dT P T T
c - = - =
k -



(c p -

v ) -



(9)

dn 2

Substituting Equation (9) in Equation (8) gives us

(c p -



11 =
1 +



Solution 5.33. From Problem 5.32,

a log p

_


r _

1 a log n

so if Y =
constant,

p =
Kn Y .

For adiabatic changes the first law of thermodynamics is

p+p -l
( )

dp = -
dn = -
+ KnY dn '
n n

or

JL P 2
( ) Kny

dn n

so that the solution is

P K ny-l



+ constant
y_ 1

But when n 0, then pin m, so

KnY


mn + .

y-1
230 PROBLEM BOOK IN RELATIVITY AND GRAVITATION

Solution 5.34. We obtain scalar integrals by dotting various combinations


a v
of u into the vector and tensor integrals for J/l and T/l . Thus
oo

J J exp[(p2+m2)2/kT-OJ-E
41TP dP

_
- -

J /l U/l
_

nd 3 p = 1 '


where g =
2J + 1. Make the substitution P = m sinh X.

Then 00

sinh2x

411gm cosh X dX

(1)
cosh X
n -


3 exp ( {3 -
e) -


where {3 =
m/kT. Similarly, we have




1t
j (u/lU

T/l d P

g/lv)

p =
V .1
(p2+m2)2
00
4 4
sinh X dX



41Tgm (2)
3h
3 exp( fj coshX-O)-E

and finally,


2 d P
/lV
3p g vT m Jl
- =


- =
.!..
/l
(p2 + m 2)

oo

sinh2x dX


41Tgm (3)
h3 exp ({3 cosh X e ) - -

(b) From Equation (2),


oo
4 4


411gm sinh X dx({3 coshX dT/T+dO)exp({3 coshX-O) .

dp =


3h
0 [exp ({3 cosh X -

e) -
E ]2

Now integrate by parts, differentiating the terms sinh x cosh X and

sinh x to get

4 2
4 OOd X[(3 sinh2x+4 sinh x){3dT/T+3 sinh x cosh X de]


41Tgm 1
dp =

3h

{3 exp({3 coshX-O)-E

Substitute expressions (1), (2), and (3) for the integrals, and find
CHAPTER 5: SOLUTIONS 231

dp =
(p+p)dT/T + nkT de .
(4)

(c) From the definition of fl we have


dll =
dp/n + dp/n -

(p+p)dn/n -
sdT -
Tds ,

but

dp =
(p+p)dn/n+nTds ,

therefore

dll =
dp/n -
sdT =
dp/n -

(p+p)dT/(nT) + Il dT / T .

Here we have substituted for s in terms of 11. Comparing this with

Equation (4), we identify


11

kT e ・

(d) For E = 0

fc;inh 4
x e-{3 cosh X dX
3 0
p =


fc;inh 2
x cosh X e -{3 c os h

dX

but

00

f _! 3
sinh4x exp(-{3 coshX)dX =

(3
sinh x exp(-f3 coshx)l

00

J o
s inh3 X coshX exp(-{3 coshX)dX ・

The first term on the right vanishes, so



nkT .

(e) For E =
0, the integrals (1), (2), and (3) can be expressed in

terms of modified Hankel functions, since


00

(2n :)!!I dx sinh ,\-



Kn({3) =
exp (- (3 cosh X)

oo
n-l


f3 2 n-2
e-{3coshX
dX sinh


X cosh X .

(2n-3)!!

232 PROBLEM BOOK IN RELATIVITY AND GRAVITATION

Thus for a =
41Tgm

eOh- 3 we have

2 ({3)/{3
n =
aK

p =
amK
2 ({3)/ {32

3p =
amK
1 ({3)/ {32 .

The exact expression for pin is

p/ n =
m[K 1 ({3)/K 2 ({3) + 3/ {3] .

For kT? m, (3 00. In this limit

.1

(3
[ J

.!L. 4n 1
(j3)
( 2{3 )

K 1


e + + ...

8{3
so

1+3/(8{3)
[ J ( )
3 3 kT
p /n m + =
m 1+
1+ 15/(8{3) {3 2 m

For kT? m, {3 0, K
1 ({3)/K2({3) 0,so pin 3kT. Whew!

Solution 5.35. From Problem 5.32,



(cv + k)/c v = 1 +
du7dT
where, from Problem 5.34

U(T) =
m{ K 1 ({3)/K 2 ({3) + 3/ {3 } , {3 =
m/kT ・

This gives the formal answer. Note that for kT? m, U =


m + 3kT/2,
y =
5/3, and that for kT? m, U =
3kT and y =
4/3.
CHAPTER 6: SOLUTIONS

Solution 6.1.

(a) The analogy with polar coordinates suggests

2 2 2
x =
v cosh u x _

t =

t =
v sinh u x/ t =
c oth u .

Then

dx =
(dv cosh u + du v sinh u)2

dt =
(dv sinh u + du v cosh u)2 (1)
2 2 2 2 2
dx _
dt =
dv _
v du .

(b) Solving Equations (1) for du and dv gives

dv =
dx coshu -
dt sinhu


du =
v- (dt cosh u -
dx sinh u)

and thus for a particle of unit mass,

U 2 du dt dx dt dx
P =
gU U p
= _
v = _
v cosh u + v sinh u = _
x + t .

dr dr dr dr dr

For an unaccelerated particle x =


const. +
: t, and dt/dr, dx/dr are

also both constant, so

dx dt dx
P =
const .

t + t =
constant.

dt dr dr

Now for P use


v'

2 2
)2 (P v )2 (P U )2/v

_
m =
p.p =
gVV(P ) + g UU(P = _

v U

P; p /v2

So, = -
m , which is not constant since in general v varies

along the particle's trajectory. It turns out that whenever the metric

coefficient does not depend on some coordinate (in this case u), then

233
234 PROBLEM BOOK IN RELATIVITY AND GRAVITATION

the covariant momentum of that coordinate (here P ) is conserved. (See


Problem 7.13 or MTW, p. 651.)

Solution 6.2. The metric in Euclidean 4-space is

2 2 2 2 2
ds =
d xl + dx + dX 3 + dX (1)

The equation of a hypersphere of radius R IS

2 2 222
xl + x

+ x

+ x


R .
(2)

By analogy with the 3-dimensional case, introduce coordinates on the

hypersphere:
x =
R cos a




R s in a cos ()
(3)



R sin a sin () cos cp
xl

R sina sin () sin cpo

Then Equation (2) is automatically satisfied; taking the differential of

Equation (3) with R constant, and substituting in Equation (1) yields the
required metric of the hypersphere.

Solution 6.3.

(a) Introduce x and y coordinates on the cylinder by

x =
cp



a tan'\ .
tY

The metric of the sphere in these coordinates is


ds 2 =


dy2 +


dx
(1)
(a

+ y2)

a2 + y2
2 2
Comparing this with ds =
dx + dy2, we see

there is the least distortion near y



0, Le.
near the equator.
CHAPTER 6: SOLUTIONS 235

(b) For the stereographic pro-

jection, it is slightly easier to use





the usual polar angle () =
90 -
A.

Let ((), rp) be the coordinates of a

the sphere, and


point on
(()o'CPo)
the spherical polar coordinates of

the projected point. Clearly CPo cpo=

The distance of the projected point

from the axis is p =


2a tan(()/2), so

we introduce coordinates

x =
p cosc/J = 2a
tan( e) cosc/J
y =
p sin c/J =
2a
tan( } e) c/J sin

and it is easily verified that


( } e) (dx
2 2 2 2
+dy2)
2 4
ds =
a (de +sin edc/J ) =
cos .

Here the distortion is least near () =


0, the north pole. This projection
2 2
is said conformal, because (ds where is
to be
)sphere

g(ds )map' g

some function, in this case equal to cos (()/2). A conformal projection

preserves angles (see Problem 6.7).

Solution 6.4.

(a) Imagine traveling along some curve rp =


rp?()). The compass

bearing tfr is given by


t an 2/r
dcp
()

'fJ

sin .
(1)
d ()

(Note that tfr is measured clockwise from the y-axis.) On the map we

have
(, 1

dy TT
(2)
\tfr

tan + =

dx '2 tan tfr

Now we combine this with Equation (1):


236 PROBLEM BOOK IN RELATIVITY AND GRAVITATION

ax ax dcp

sin


dcp = - dx/ de =
ae+ de
de dy/ de ay ay dcp

ae acp de

ay ay d CP dcp ax

ax dcp
( e

+ SIn = - -

(3)
ae acp de de acp de

ae

Since Equation (3) must hold for arbitrary dcp/ de at the point we are con-

sidering, we can equate coefficients of powers of dcp/de on the two

sides of Equation (3). This gives us

ay/acp =
0, y =
y(e) (4)

ax/ ae =
0 , x =
x(cp) (5)


sin e ay/ ae =
ax/ acp .
(6)

Equations (4) and (5) imply that the left-hand side of Equation (6) is a

function of e only, while the right-hand side is a function of cp only,


and so each side must be a constant which we can choose to be 1. Thus

the map is given by

x =
cp, y = -

J sin
?e e =
log cot 1. e


(7)

(b) Take the radius to be 1 for convenience. Then

2 2 2
ds =
de dcp2
+ sin e
2 2

sin e (dx dy2) +

2 2

sech y (dx + dy2) .
(8)

(c) The great circles are the geodesics of the 2-sphere. The geodesic

equations are easy to solve, since we have two first integrals. Let a dot

denote d/ds; then Equation (8) gives

2 2
sech y (x + y2) =
1 .
(9)

Since x is an ignorable coordinate in the metric (8), xx


is constant

(see Solution 7.13), i.e.


CHAPTER 6: SOLUTIONS 237


(sech y) X =
y .
(10)

When neither x nor y vanishes, we can eliminate s from Equations (9)

and (10):
2 2 2
dy y2 A cosh y
( )
_

= =
(11)
dx 2
x cosh2y

where A =
1/y. Equation (11) is easily integrated by putting z =
sinh Y'.

The result is
sinh y =
a sin (x + (3)

2 2
where a =
(A -

1) and {3 is another constant of integration.

Solution 6.5. We can decide if the space is 3-dimensional or not by


evaluating the 3-volume spanned by dx dy dz

dV =
gZ dxdydz

1.

(i3Y ( !3 ) ( 1 ) OD ( !3)

1- -

= -

( !3 ) ( ) 1-( Y-( ) n ) dx dy dz =
o .

GD ( b ) -

(i3) O) GY 1-

Since this is identically zero for all x, y, z, the 3 coordinates are

always linearly dependent. The space is thus either two dimensional or

one dimensional. We can now throwaway one coordinate, say z, by


taking z =
cons tant; this is allowed because it is an "ignorable coordi-
nate," i. e. the coefficients of the metric do not depend on z. The metric

left is

2 2 l.. dy\
.i.
ds =
dx + dy2 _

( 13
dx +
13).

It is in fact 2-dimensional, not I-dimensional, because

g =
det
1-(!3 )2 -( !3) ( ) /
r= 0 .

(tJ () 1- (13Y
238 PROBLEM BOOK IN RELATIVITY AND GRAVITATION

It is not difficult to find the coordinate


transformation (e. g. by Gram-
Schmidt orthogonalization)

12 3 4
(13 x + y



5" 13

13
11 =

5 (_.i..
13
x + .1. Y
13 J
2 2 2
which gives ds =
dg + d11 , obviously the simplest form of the metric.

The more sophisticated reader will notice that the original metric was of
the form

{3 a{3 Va V{3
= -

where

projects
Va

is the Euclidean unit vector

3-dimensional Euclidean space into


( 1 1 g) '


' This metric just

2-dimensional space

perpendicular to V 1
so it is obvious that there had to exist
g and 11
coordinates which make it a flat-space 2-dimensional metric.

Solution 6.6. Let A be an arbitrary vector. We want to see if P.A IS

orthogonal to u, Le. if u.P.A =


0:

ua( {3+uau{3)A{3 uaA )u{3A{3 -U{3A{3


a a
u.P.A (u O.
+ u
uaA
= = = =
a a

Also it is easy to see that if A.u =


0, then P.A =
A, so A is unaffected

by projection. If n is a unit spacelike vector, we have

(3)A{3

n.P.A n
(ga{3 -na n

(nan ) n{3A{3



Aa -

na Aa

n{3A{3 0 =

Likewise a vector already ortbogonal to n is unaffected by projection.


Suppose now that P is a projection operator orthogonal to a null vector

k, so that k. P.A =
0 for all A. It is easy to see that P + constant x

kk is also a projection operator since k.k =


0; so P is not unique.

(The set of null projection operators is not empty since it is easy to


CHAPTER 6: SOLUTIONS 239

check that for any 4-vector w, one is P w0 k/k.w. However, there



= -

are no symmetric null projection operators at all.)

Solution 6.7. Let A and B be two vectors in the metric space. Obviously
we want the angle to be related to the dot product A.B. We also want it
to be unaffected by scaling A and B, so the natural choice is

A.B/(IAIIBI), which is cos e in the familiar Euclidean case. Now under

{3f(x ')
l1 we have
{3
f(xY )
A.B =
(3AaBf3 f3AaBf3 1
(IAIIBI)

(g A V AVgRPB u
)2 [f( x Y ) g Af1.Av f(xY) g BPB U ]2
pcJ f1.V pu

but the f scancel, giving the same value as before. Null curves remain

null curves because the square of their tangent vector remains zero

o = f.f =

f1.V
f ev f(x Y ) g
/Lv
ff1.fV =
0 .

Solution 6.8. The relative velocity ds between two velocities v and

v + dv is (by Problem 1.3) given by

ds
2 =
(dy)2 ( xdy)2
_

(1- )2

and

dy)2 (dy)2 dy)2



( x =
v _

( . .

Let () and cp be polar and azimuthal angles about the direction of .y (Le.



=vcose, v
x =vsin()coscp, v

=vsin()sincp). Then

(dy)2 =
(dv)2 + v

(d()2 + sin

() dcp2)


dv = !. d(v .,y) = vdv
- -

2 -

so

2 2
2 dv 2
ds =


(de

+ sin e dcp2) .

)2

2 1- v
(1- v
240 PROBLEM BOOK IN RELATIVITY AND GRAVITATION

Introduce the "rapidity parameter" v =


tanh X. Then

2 2 2
dX2

ds =
+ sinh x (de + sin e dcp2) .

Note that for small v, sinh X X v and velocity space is flat, as we

expect it to be in the Newtonian limit.

2 3
Solution 6.9. If we had sin e instead of (e -
( ) the metric would just

be that of the Euclidean 2-sphere. As given, it is obviously some sort of

axially symmetric "warped" 2-sphere with singularities possible at

e =
0, :tl. From the given value of e =
, it is obvious that the range of

e can be extended to 0 < e < 1. We must examine the cases e =


0 and

e =
1 with some care to decide what range X can take. We know that X
must be periodic because, as the metric shows, the coordinates (e =
0,
all X) and (e =
1, all X) each represent just one point.
For e 0, we have
2 2 2
ds de + e dX2 .

If X is periodic with period P, then the proper circumference of a small

circle of radius !1e is r( e)


dX =
(tJ.e) P. If we are to avoid a conical

singularity this must equal 217 tJ.e, so P =


217. But now consider e 1:

The metric is
2 2
ds de + (e -

1)2 (2)2 dX2

so the condition avoiding a conical singularity, is

I o
2( e)
dx =
21T( e) ,

implies that P =
17. Thus we can either take P =
217 and have a conical

singularity at e =
1, or P = 17 and have it at e =
O. These are two

different global extensions of the manifold. (This problem is due to

Gilbert Miller.)

Solution 6.10. A geometrical object exhibits a particular group symmetry

if its functional change (i.e. its change in functional form) under the action
CHAPTER 6: SOLUTIONS 241

of the group vanishes. If g is an infinitesimal displacement, then under


the infinitesimal coordinate transformation x'f.L =
xf.L + g /l, one can easily
show that the functional change of the metric,
ogf.L v
' is

ogflV '"
g v(x) gflV(x)

= -

g/lP

gPV vfl
gflv,P

p. (1)

(See Problem 13.12.) For spherical symmetry,


og/lV must vanish under

the rotation group, a particular realization of which is obtained from the


generators

gO =
0, =

ij


, (2)

where ij ji three arbitrary infinitesimal constants.


E = -
E are Substituting

Equation (2) for the generators /lin Equation (1) and setting the left
side to zero, one gets:

a. /l

v =

ij j j i
E x 0 or x x
gOO,i gOO,i gOO,j
= =

and hence
O 2
goo

gOO(x ,


(3a)


==
(x

)2 + (x

)2 + (x

)2 .

b. /l

0, v 1= 0
g 01 .

ij
+ g . .

ik
xk = 0
OJ, 1

2 0 J
go j

r 1 (r ,

)x (3b)

where r'l is an arbitrary function.

c. /l -1= 0, v 1= 0
kj ei ek x k 0
gij, e
E + E
gej E + =

gik

2 0 2 0 i j
gij


2(r , x ) 0ij + r 3(r ,

)x x (3c)

where r 2 and r

are again arbitrary functions.
242 PROBLEM BOOK IN RELATIVITY AND GRAVITA TION

From Equation (2) one can see that the above coordinate system

(XO, xl ,





) is "cartesian-like." The group theoretically defined
property of spherical symmetry is coordinate independent, however, and
to find g in any other coordinate system one merely transforms the
/LV
metric given in Equations (3) in the usual way.
CHAPTER 7: SOLUTIONS

Solution 7.1. Let the transformation be given bye, =


La, e , then
Il Il a


V ,
e =
rr e' , V (LIla ) L Ve (LIl, e )
{3


, \ , = '

{3


afJ r
Il a
Il
(L{3,e A)


LIla rr,, e r )
{3 '(LIla,/\

L , \ e + ,

Il fL'''

' ,
A r r
LIl ,L rY.. e r ,)
(3 ,(LIla,/\

L L ,
\ e , +
11 ray fL'''

and therefore

r' A /I r' Y A r' II
r , =
, L '
L' ,
L r.. + L '
L Lr, \
{3 {3

a a ,/\
fJ a
y fL''' 11

The second term in this equation would not be present in a tensor trans-

formation.

Solution 7.2.

(a) First consider the line () =


0, r =
s, where s is the affine
2 2
parameter (length). The geodesic equation d x l1 / dr + r ll
rr

0, gives

()
rr
rr

r rr

0 .

Next, for nonradiallines, reparameterize the geodesic equation using ()


as a non-affine parameter:

2 2
d x ll

d ()/ds

d xl1

dx

dx{3 r l1 _

0 (1)
a{3


de (de/ds)2 de de de

The general straight line is

r cos (() -a) =

Ro (2)

243
244 PROBLEM BOOK IN RELATIVITY AND GRAVITATION

where and 2 2

Ro are arbitrary constants. This equation, and ds =
dr
2 2 2
d0
+ r , give ds/dO =

Ro/cos '1', where 'II == 0 -a, and this in turn

yields
2 2
d 0 Ji dO
ds
2/ \ds ) =
-2 tan'll .

The geodesic equation now reads


d x l1
2 tan'll
d x l1

dx

dx{3 rl1 o (3)
_

dO CW dO

a{3

d02

Consider the point () =


a (Le. 'II =
0) and r =

Ro on the line. At this

point (3) becomes


2 l1
d x /I
+ rr =

00

2 ,

d0

which gives r 000 =


0, and rrOO = -
r (where Equation (2) has been
used). Since a and Ro are arbitrary these expressions are true in

general.
Finally, consider an arbitrary point on the line and write out the

geodesic equation in full glory, putting in the r's known so far. The

11 =
r component of the result gives rrrO =

(b) From r




+ y2, and cot 0 =
xl y we get a transformation matrix

l1=x /-L=Y

[ si SinO]

cos
La =

/-L -
0 cosO

and its inverse

{3'=r {3' =

[ Sine]
cos e -r

LI1{3'

sinO r cos ()

(Example: x =
r cos 0 dx = dr cos 0 -
r sin 0 dO first row above.)
CHAPTER 7; SOLUTIONS 245

The transformation law for the r' s (Problem 7.1) is

" ,

ra ,
, La LJL

,rP
,LV + La L/-L , ,

fJ Y P fJ Y /1V /-L fJ , Y

Since rP = 0 in cartesian coordinates, only the second term contributes,


/-LV
and straightforward differentiation and matrix multiplication gives the

desired results.

(c) We now use



_! a/1
( g(3y,/1 )
g +
g/-L(3 ,Y g/1y,(3
_

(3y

2 2 2 2
For the metric ds =
dr + r d0 the only nonvanishing derivative is


2r. Thus r =
0 unless 2 indices are O's and one is an r:
gOO,r (3y


rrOO ( )
rr
-r
g gOO,r
= - =

( gOO,r)
o 00 1
r =



rO

2 r

Solution 7.3.

(a) The geodesic equation is

2 2 2
O dr dO
d dO d 0
( ) ( )( )

+ rr =
0 + 2r rO =
0 .

2 00 ds 2 ds ds
ds ds

Using the results of Problem 7.2, we have

2 2
dO
( )
d r


2 ds
ds



;( ( ): ) 0

Now (dr/ds)2 + r

(dO/ds)2 =
1 follows from the metric so
246 PROBLEM BOOK IN RELATIVITY AND GRA VITA TION

dRo 2

[ ]
sl.. dO d 0 dO
r2 dr
( )( )

= =
r + 2r
ds ds ds ds ds
ds2

[ )]
d 0 2 dr dO
( )(


r + =

2 r
ds ds

ds

and therefore



RO := r =
constant .

(b) From one of the first integrals above:

2 2
dr dO dO
( ) ( )

+ r =

dO ds ds


Now substitute dO/ds =

Ro/r (R o =
some constant):

( &)

+ r =

r4/R .

(c) The equation for a straight line is

r =
L/cos(O-a) y

dr _

sin (0 -a) L
dO 2
cos (0 -a)

dr\2
(d()} + r


L 2f sin 2 (0-a) ...
/)\ ....

Lcos 4(O_a)
\.

...



4 x
1 L r
+ = =

2 4 2
cos
(O-a)J cos (O-a) L

Thus, all straight lines satisfy the geodesic equation.

Solution 7.4. The nonvanishing derivatives of the metric tensor are



gxx, t
= -

gtt, t
= -
2/t . Thus the nonvanishing connection coefficients

are
3 3
r ttt =
1/t ,

xxt

r xtx = -
r = -
1/t ・

txx
CHAPTER 7: SOLUTIONS 247

To find the geodesics it is easiest to proceed directly from their


definition as curves of extreme length. Let a geodesic be x(t). Then
(dot represents d/dt)

J )!
J (1_x2)! dtt

o = 0 (ds = 0 ・

The Euler-Lagrange equation for this extremization is

[ J
d x _


0 .

dt 1
2 2"
t (1- x )

This is easily solved by letting tanh () == x :

sinh () =
cons tant ,

which integrates to
2 2 2
(x Xo ) t + a
- =

Thus the geodesics are hyperbolas asymptotic to the light cones:

/ efJ
)(
II

)(


t=O

Solution 7.5. The calculation of


{:3;y straightforward
is in a coordinate

basis:

f,a f,a a
{:3;y {:3,y ga{3
= - -

ay {:3y

2r
{3
= -

,y ({:3a)y

{3,y {3,y
= - =

248 PROBLEM BOOK IN RELATIVITY AND GRAVITATION

Solution 7.6. In a coordinate frame:

1 lLa
ill ( ) g (1)
2 gav,A +gaA,v-gvA,a

vA

(a) If the metric is diagonal, a must be equal to IL. But since

IL 1= v 1= A, all of the terms in parentheses in Equation (1) obviously vanish.



gAA , a) g/lLl, setting
(b) r lLA1\\ A in Equation (1).

( AA

A A v

2 , ,


rllU = -

gAA,a gp.a
= -

gAA,a (gp.a)-l = -

(gllpF gAA,1l ・

Here we have repeatedly used diagonality of the metric [e. g. gpa =

(gp.a)-l].
-1
ill _! (
(c ) gpA,a ) ( gIJD. )

IL,AgaA,1L
_

pA

(gllll)-l(gllll,A) a (log(lglllll!))

= = .

(d) Setting A =
11 in (c) gives


i IL = --
(log (I glLlL 1 2)) ・

ILIL
axIL

Solution 7.7.

(a) Vy(e e(3)


. =
(Vyea)' e(3 +
ea. (V y e(3)
(3,y


ill e. e + ill e ・

ay IL tJ tJY IL a


i + i
(3ay a(3y

(b) g gl1(3 =
a (3

all

1L(3 + 11(3 0
gal1,y g gall g

,y-

11(3 1L(3
gall g g
1L,y


_

,y

(c) ga13,y = _

gA l1,y gll13,ja =

-erA I1Y +r..


fU
gll13,ja

rallY gll13 r 13
Ay gAa
= - -

CHAPTER 7: SOLUTIONS 249

(Here we have used the result in (a).)

(d) For any matrix II ,811,


(log det II
,8IJ)
,a

Tr II ,811-1II glL v ,a II ,

therefore

(log g) , a =
g/1 gllv
r ,a

and

--E!:. V

g/1 g
/1v,a

And so, by (b)


/1v /1v
gg
g,a g/1v,a gg/1V g
_ _
- - -

,a

(e) In

a coordinate frame,
rlLa,8

gIL (gva,,8 r.v,8,a
+ -

,8,v)'and
since the last two terms cancel ra

,8

gavr.va,,8' Thus, by (d)



r (log I g\2) ,,8
g,,8/ g (log I g\) ,,8
= = =

,8

(f) glLVralLv = -

ga,8,,8 r,8.\,8 gAa



(putting ,8
summing)

y in (c) and


,8 Aa
= -

,,8

(log I g 1 2),Ag by (e)


_gav,v _

(loglglt),v v
changing dummy indices
1 1.
= _
gav ,v
_

I gl2 gav\ gl-


,v

= _2-
1.
(gavlglt) ,v

\g\2

(g) A

,a


,a
+fXl a A,8
,8


,a
+--L(lglt)
1. ,8
A,8

by (e)
Ig\2

--L(lglt A


) ,a
Igl2
250 PROBLEM BOOK IN RELATIVITY AND GRAVITATION

(h)
Aa{3;{3 Aa{3,{3 r{3/L,8 Aa/L p\a{3 A>..{3

+ -

=Aa{3Q+ (lglt)
,11
A/L-rA A\/L
'fJ
all - a 1\

\g\2

--L(\glt A (3)
.1 a 'fJ

P\
all
A\1l
1\

Igl2

(i) A
a{3;{3 =

a{3,{3 ra/L{3 A/L{3 r{3/L{3A
+ + all



a (3 + ra A/-L(3 + --L (I gl t) Aa/-L
, fJ /-LfJ .1, f.l.

Igl2

\ (A {3lg1 ),Q+ra QA/L{3.


a 2

/L

Igl

But r

/L{3

r(/L{3) in a coordinate frame, so if A/L{3 =


A!Jt{3], the last

term vanishes, and

{3;{3 1.1 (I gl t A (3) ,{3


a a
A =

\g12

(j) 0 S =
(S ,a ga (3);{3 =

\ (I gJ !S ,a
lf (3) ,{3 ・

Ig\2
(Here (g) has been used.)

Solution 7.8. Under a coordinate transformation,

A-- =
axil ax v A
f.l. v
ax il a x v /-LV

implies that

aX Il axV
A det
( ) det (A
v) det
( ) J2A
= =

Il
axil ax

where J is the Jacobian, det(axf.l./ax/l'), and where the superscript T

denotes matrix transpose. We want according


A'a,
to transform to the

vector analog of the transformation law for A. (A is called a "density


of weight 2".) Since A'a must be linear in A, set

CHAPTER 7: SOLUTIONS 251

A.a A +
Ka A

, ,a

where Ka must be determined. This can be done by demanding that g,

the determinant of the metric tensor, have vanishing covariant derivative:

o =

g;a

g,a +
Ka g =
2gr t3t3a +
Ka g

by Problem 7.7. Thus


Ka
= -
2f't3
t3a

The generalization to densities of weight W (Le. they transform with a

JW) can be done by considering powers of g. The result is that



= -
wr(3fJa .

Solution 7.9. A geodesic with tangent vector u is spacelike, timelike,


or null according to whether u.u is > 0, =
0, < O. But u.u is conserved

along the geodesic, since

v u (u u)
・ =
2u. V

u =
0 ,

since the geodesic equation is V


UU

O.

Solution 7.10. The integral for length is

f f(
dx(3

dxa d s.
ds -

(3

ds ds

To extremize length take a f ds =


O. According to the Euler-Lagrange

equations
E... (Q' u

) = ! Q' uauy (1)
ds ca(3 2 cay ,(3

where
(- d:: d
t3 )= =
1 and u

;:
d:: have been used. Now since


sL (Q' a dua dx Y a du
+ uY u

u ) +
(3,y

(3 (3,y (3
= =

ds ca(3 ds ds ds
252 PROBLEM BOOK IN RELATIVITY AND GRAVITA TION

Equation (1) becomes

2 a
d 1
uauY g



afJ
_



afJ' Y
_ _

2 c:u.Y'fJ
= o .
(2)
ds

Now, use

uauY g =
uauY 1. (Q .

+ g,., )
afJ'Y 2 C1(lfJ'Y -YfJ,a

and multiply Equation (2) by g(3T to obtain

2 T
d x 1 T
_
+ _

gfJ ( + g,., _
Q uauY

fJ ,Y -YfJ ,a
ds 2 2 caY'fJ
(3)
2 T
d dxa dxY


+ rT =
O.
2 ay ds ds
ds

Solution 7.11. Define a new curve parametrization by the functional re-

lationship s =
f(A). Then derivatives are related by

2 2
d £.. d £.. d


f' -=
f" + f,2 (1)
ciA ds 2 ds
ciA ds2

where £' =

With the new parametrization then the geodesic equation
becomes





fH dx

+ ra
dx(3 dx Y =
0 (2)
fJy

2 ds ds ds
ds £'2

Equation (2) is in the "standard form" (s an affine parametrization) if

the second term vanishes. For s to be an affine parameter, f" must

vanish Le. s and A must be linearly related.

The components of
Solution 7.12.
VpP are

( pfJ +uaporfJ )

(V
p p)(3 = mu

(p(3,a +r(3aa pa) =

dT aa

(1)

In flat space time, one can always find a global coordinate system
(Minkowski coordinates) in which all of the Christoffel symbols vanish.
In that coordinate system, conservation of 4-momentum,
: =
0, can be

written as
CHAPTER 7: SOLUTIONS 253

v p =
0 (2)

by Equation (1), but Equation (2) is a tensor equation so it must be the

correct expression for momentum conservation in any frame. The momentum

vector is proportional to the 4-velocity of particles with mass; for these



particles p'p
= -
m , so the geodesics are timelike.

Solution 7.13. Let A be an affine parameter such that pa =


dxa/dA; then

for geodes ic motion of the particle

a dPl r
a a

O=(Vpp).el=PliaP ciA -Pa alP


and therefore

dPl a a a 1 o
p p ar (aa)l p P gaa,l
= =
= .

ciA 2

Solution 7.14. Start with the geodesic equation


dx dx

d x

raQ



2 tJY
dA

Change variables from affine parameter A to coordinate time t and use

2 i k
o =
dt +
(gij/ goo) dx dx (1)

to get
k f 2 k




+ r
dx

d/ r.
gkf dx dx



t/dA
g.
dx =
0 .

jkf Jk dt
_

gjk 2 dt dt JOO
goo
dt dt
(dt/dA)2
dt

Combine this with the time part of the geodesic equation

2 2
(d t/dA ) _

- -
2l' OkO
dxk/dt
(dt/dA)2 goo

and use the expression for the r's in terms of the metric to get

2 k k f
d 1 dx dx
x 0 (2)
(Yjk,f+Yjf,k -Ykf,j)
_



Y"k

J 2 2 dt dt
dt
254 PROBLEM BOOK IN RELATIVITY AND GRAVITA TION

where
Yjk
== -

gjk/gOO. Notice that this is just a geodesic equation with


affine parameter t in the 3-dimensional manifold with metric From
Yj k'
Equation (1) it is clear then that the solution of Equation (2) extremizes
r dt; Fermat's principle holds. [See also C. Moller, The Theory of Rela-
tivity (Oxford University Press, 2nd ed., 1972), p. 308.]

Solution 7.15.
(a) The geodesIc is the path between two velocities which minimizes

the arc-length between them; but arc-length in the velocity space is just

the magnitude of a small change of velocity. Since a rocket expends fuel

monotonically for the boost it requires, the geodesics of velocity space


are paths of minimum fuel use.

(b) We need to find the velocity geodesic connecting Yl and Y2' In

principle, we could solve the geodesic equation for the metric in Problem

6.8, but this is very tedious. An easier way is to notice that, by symmetry,

a geodesic passing through the origin of the velocity space coordinates is

() =
cp =
constant, X =
s (where s is an affine parameter). To get a more

general geodesic let us view this geodesic from a moving frame. Since

1 2


tanh- V is a monotonic function of p =
yV [where y ==
(I _V )-2],
we can write the geodesics in terms of the non-affine parameter

pes). In

terms of this parameter y



(l+p2)2 and, if we choose the geodesic to

be in the x-direction,

2 2
u =
[(I+p ) ,p,O,O] -oo<p<+oo.

A boost in the x-direction just takes this trajectory onto itself so we

need only find its transformation under a perpendicular boost, say in the

y-direction. Under a boos t by {3 in the y-direction u becomes:

1 1

u' =
[y'(I+p2)2, p, (l+p2)2 y'{3, 0] (1)

2 -2
where y' =
(1- (3) .

The 4-velocity in Equation (1) may be represented now in terms of the

corres ponding 3-velocity


CHAPTER 7: SOLUTIONS 255

1 1

V/(1_V
--
)2 =
pn+(1+p2)2y'{3m
- -
(2)

where nand m are perpendicular unit vectors, and may be manipulated



as if they lived in a cartesian space. We can solve for V and write

Equation (2) as


{32

1 -

V= pn+{3m. -
(3)
1+p2
- -

This equation gives us the general geodesic of velocity space. One picks


{3 (\{3\ < 1), two perpendicular vectors m, n and gets a geodesic

parameterized by p (- 00 < p < + 00). From Equation (3) it is clear that the

general geodesic is a "straight line." (This does not mean the space is

fl at! )

For our problem there are two cases. In case (i), in which either

angle 0 V 1 V2 or 0V

V1 is obtuse, the distance of closest approach
to the origin 0 of the geodesic (straight line) connecting Yl and Y2
\ \ 1or \Y2 \. in which both OV V and OV V
is simply In case (ii) 1 2 2 1

are acute, the distance of clos est approach {3 is (by simple geometric

arguments)
\Yl Y21x

{3 =

IYl Y2\

Y2

o o

Y2

(i ) (ii)
256 PROBLEM BOOK IN RELATIVITY AND GRAVITATION

Solution 7.16. A is parallel transported along the cp-coordinate line (Le.

o =
constant), so


Aa;cP Aa,cP ra{3cP A{3
+ (1)
= =

The only nonzero Christoffel symbols are r° = -


sinO cos 0 and
cPcP
r cP =
cotO, so Equation (1) gives
OcP

sin 0 cos 0 A cP

O,cP -
= 0 (2)


AcP,cP + cotO A =
0 .
(3)

Throughout this problem, 0 is kept constant at 0




Equations (2) and
(3) are easily solved, e. g., by differentiating Equation (2) with respect
to cp:

AO,cPcP AcP,cP

= sinO cosO =
-cos 0 A .

The solution then is


A = a cos (cp cos 0) + f3 sin(<p cos e)

where a, f3 are constants. Equation (2) now gives

ACP =
-a sin(cp cos O)/sinO + {3 cos(cp cos O)/sinO .


At cP =
0, A =

eO' i. e. A =
1, AcP =
O. Thus a =
1, {3 =
0, and


A =
cos (cp cos 0)

ACP =
-sin(cpcosO)/sinO.

At cp =
2", after transport around the circle, the vector is

A =
cos (2" cos 0) eO -

sin (2" cos O)/sin 0 ecp 1= eO '

but the magnitude is unchanged

2 2 2
(A.A)21T =
cos (21T COS 0) eO.eO + sin (21T cos O)/sin 0 e
cP


cP

1 =

(A.A)o'
CHAPTER 7: SOLUTIONS 257

Solution 7.17.

(i) V u (TJa
{3)

Vu(C e(3) a

(VuC a
)' C{3 +
ca' (V u C{3)


AYe .
e + A Ye .

a y {3 {3 y a

Aaf3 +
Af3a

The tensor A must be antisymmetric.

It is conventional to write the transport law in this case as

V e A f3 e ( Ay(3e ) e == Q. e
(1)
(3
= = e .

{3

ua a a a

where n(3y =
A(3y is the 4-dimensional version of a 3-dimensional anti-

symmetric rotation matrix.

(ii) Since u = e

is singled out, decompose Q along u and

orthogonal to u:

f),a(3 =
vauf3 _


v{3 + waf3 (2)

{3 _w{3a, {3u{3
a a
where w =
w =
0 and va is as yet unspecified;
without loss of generality we can take v.u =
O. Now Equation

(1) gives
Q'u=-Vu=-a

where a is the 4-acceleration of the observer. But Equation

(2) gives
Q. u =
-v

so v = a and hence

Q =
a@u-u@a+(a).

a (3
(iii) The tensor w has only three independent components and is

spatial since (a)' u =


O. It therefore represents a purely spatial
rotation of the basis vectors and vanishes if the spatial vectors

are non-rotating.

One often uses an angular velocity vector (a), with (,,). u =


0, to repre-

sent the three degrees of freedom in w af3 . The relation between (a) and


af3 is
258 PROBLEM BOOK IN RELATIVITY AND GRAVITA TION

{3
Ea{3AauAwa' 1 aula
a a
w W -
E lA-I\,
u/lwAa
= =

Note that a spatially non-rotating frame (w =


0) satisfies

V u ea =
(u @ a -
a @ u) ・


Any vector e
a satisfying an equation of this form is said to be "Fermi-

Walker" trans ported.

Solution 7.18. Let the two vectors be x and y. Then the Fermi-Walker

transport law reads


Vux =
(u@a -

a@u).?{

vu y =
(u@ a -
a @u). y

where u is the tangent vector to the curve e and a =


V uu. Using the

product rule we evaluate the change in the dot product along the curve:

Vu(x.y) =

(Vux)' y + x. (V u y)

(a.x) (u.y) -

(a.y) (u.x) + (u.x) (a.y) -

(a.x) (u.y) =
0 .

The scalar product is unaltered.

Solution 7.19. Fermi-Walker transport has the differential equation

Vux =
(u@a -

a@u)'x ,

where u is the tangent vector to the curve and a = V == Du/dr. If the


Uu
curve is a geodesic, it satisfies the geodesic equation V Uu =
0, so the

Fermi-Walker transport law becomes

V x =
0 '

which is just the parallel transport equation.


CHAPTER 7: SOLUTION 259

Solution 7.20.

(a)
;{3u{3TP TP;{3u{3u (VVV). V
Ua =


(b)
VU;{3u{3 TP;{3v{3 VV y VyV [V, y]

= -
""

(c)
Ta{3;yVUW{3UY y. (VVT). W =

v{l;{3v
(d)
(3;yUY WU;{3(v{3;yUY) V(VVy)W
= =

(e) Combine the first two terms of the given expression,

;yUY);{3u{3 TP;{3(w{3;yUY)

(W VV(VV W ) V
(VVW)V [V,VV W ]

= -

Solution 7.21. The paths of light rays can be obtained from the geometri-

{3;{3

cal optics limit of Maxwell's equations F = O. With the identity in

Problem 7.7 (i), these equations can be written as

[gar g{3/l Fr/ g) t] ,{3 -


= 0 .

Using the spatially isotropic, diagonal form of the metric and noting that
k'f

Ei F Oi' Bk one sees that the above equations have the form

Fjf,
= =

v .

(f )


V x (JL- B) =

a(f )/at

where f =
JL

(f/ goo)"2. Thus light moves as if in a medium with an

effective index of refraction

1 1

n =
(fJL)-"2 =

(goo/f)"2

Solution 7.22. For clarity first consider the Newtonian case. Let P (v, n)
be the probability of the velocity being y after n boosts. We want to

find a differential equation for P. To accomplish this note that if the

velocity is ..y. at step n then at step n -


1 the velocity must have been

somewhere in velocity space at a distance t1v from Y: By symmetry


(y, n) has equal contributions from points on the sphere of radius
260 PROBLEM BOOK IN RELATIVITY AND GRAVITATION

t1v about y so we can equate P(y, n) with the isotropic average of P,


at n -

1, over that sphere:

P(v,n)
--
= < P(v+t1v,n-1?

sp h ere

(1/6)[P(y +t1v ,n-1)


x + P(y' + t1v.!y,n-1)+ P(y+ t1v z,n-1)

P(y -t1v ,n-1)
x + PCY. -

t1v y,n-1)+
P(y, t1v z,n-1)] -


P(y,n-1) + (t1v)2 V P/6 .

(1)

The differential equation then, is

ap =
(t1v)2 n2
v p (2)
di1 6-

the standard diffusion equation. Its solution, with


P(y, 0) =

03(y) is

( )
3 2
2 3v
(417)-2 (nt1v /6)-

P(v, n)

exp


2n t1v

In the relativistic problem the same arguments hold except that veloci-
ties do not add linearly; we must use the relativistic velocity addition

formulas. Since t1V? c, however, the proper velocity t1v does add

linearly in a frame momentarily comoving with the rocket ship. Thus, the
diffusion Equation (2) is locally valid in velocity space; we only need
take into account that the global metric of relativistic velocity space is

different. This is done by solving Equation (2) with V representing
the Laplacian in a curved-metric velocity space, with metric (see
Prob lem 6.8)

dlJl2 (dv dv )

dV;roper


+ sinh 1J1 + sin e (4)

where 'P is the rapidity parameter, that is tanh qt ==


Iyl. Using the
relation [Problem 7.7 U)]

P.
,a
;a =
g- t (g t ga{3p (3 ) ,a

(5)

and the fact that by spherical symmetry ap =


ap =
0 we get
av av ()

cp
CHAPTER 7: SOLUTIONS 261

ap v2 1 2


ap
( sinh 'P
)]

6 (6)
an sinh 2 'P a'll a'll'

It is not difficult to check that this has the solution


P(v, n)


(417)-t e- t t-t sinh'll
'II
e-('P /4t)
(7)

where t == n v2/6.This agrees with the Newtonian solution Equation (3)


when t? 1, '11? 1, (no restriction on '1'21 4t).
The probability of 'II between 'II and 'II + d'P is (using the velocity-

space metric) 417 sinh 'P Pd'P which for t? 1, 'II? 1 has the

asymptotic behavior
'II 2
2 -.t(l--)
dP 'II 'II
d\P
(417)-t e-'P /4t-t+'11 =
(417)-t e
2t

tt t

Only the exponential is of significance here, and this tells us that 'I'

has a mean value < 'II > 2t, and a standard deviation 'II ""oJ
V2i, so

'I'
I<'J.I > 0 as t 00. The mean rapidity < 'II > increases linearly with
number of steps, but the average boost per step is v2
13. A sober pilot
would increase his rapidity by v step, 3c/
each v
times as much.

Nevertheless it is amazing that the drunk astronaut does as well as he


does; in the Newtonian case velocity or rapidity increased only as n 2",


not as n. The reason here has to do with the Lorentz transformation: if

an observer moving away from you chooses a random direction to fire a

bullet, the directions look biased outward from you (headlight effect).

Solution 7.23.
(a) If the hypersurfaces are given by f =
constant, then

k oc Vf k =
hf
JL ,JL
and so

(1)
kJL;v h,Vf,JL hf,JL;V


k[,.L;v k,\] h,[v f ,/lhf,'\l + h
f,[,.L;v f ,'\] (2)
= ・
262 PROBLEM BOOK IN RELATIVITY AND GRAVITATION

The first term obviously vanishes and the second term vanishes because

The of this, that


k[/l;vkA] implies that k is
f,/l;v f,V;/l. converse

hypersurface-orthogonal, is called Frobenius' theorem.

(b) From Equation (1) the extra condition


k[/l;v]

0 is equivalent

to k =
Vf for some f.

Solution 7.24. Let k be the tangent vector to the congruence of null

curves. Since k is hypersurface orthogonal,

k =
hf
a ,a

for some scalar functions f and h (see Problem 7.23). Because k is

null,
f f,a =
0 .

,a

From these two equations we have


a,

(3
k(3 =

(3
f + hf
(h "a,a ,
) hf,(3 .

(3

f f,{3 = f f,(3 = 1 (f f,(3).,a =



,a, {3 {3 ,a (3

, 2,

and thus


a, {3

k{3 =
(h f,(3)


,p

a a
so the geodesic equation V kk oc k is satisfied. If k =
dx IdA, we can

introduce an affine parameter A' = A' (A) to write the equation in the



standard form V k k =
0 where kU =
dx IdA'.

Solution 7.25. The general case gives an Euler-Lagrange equation

2 ay

[ (a )]
d F dy dF d
o = + _

CIS dX dX

ds dX dy
dy2
Since dy Ids affine parameter) and since dF Idy 1= 0, this

0 (for s an

is the same as the Euler-Lagrange equation obtained from a fy ds =


o.
CHAPTER 8: SOLUTIONS

Solution 8.1.

(a) The key relations are <d;.a,J/Jx f3 > =h.a.J/Jx p =o ,


(J/Jx ).

(J/Jx P ) =

f3'
and a. f3gaf3.

The answers are:

01
1, 0, gOl' g gOO.

(b) The vector gla a/ax a


==
gla e
a corresponds to 1 since, for

any vector v

--'
la II la II la II s=:-1 1 1
v. g e =
v' e .

g e =
v' g g

v' u =
V = < dX v>
,.
a a /l
/l pn.

Solution 8.2. It is easy to see that f is just the coordinate r, that is


--'

df =
dr, since -" --'

< dr, e"" >r
= < dr, e

> =

--'
-1
<dr, 0
eO> r
<dr,ee>
= =

-0 then
Now suppose there exists a g such that dg =
CA)

<dg,

o = < d g , e",,>

= < dg, e

> =
ag/ar
--
-1 -1
1 <dg, <dg, eO> ag/ao.
eO>
= =
r =

Clearly the conditions ag/ ar =


0 and ag/ ao =
r are incompatible.

Solution 8.3. In a cartesian coordinate basis the condition is that


Vi,j

v. .. This is equivalent to the basis-free requirement that the curl of tI


3,1

(the vector equivalent of a) vanish.

263
264 PROBLEM BOOK IN RELATIVITY AND GRAVITATION

Solution 8.4. Choose a basis c:; 1'"


N'
Any p-form can be written as a

sum

1 A..

lJ
...

((;,.,,
1 J
....) in which there are exactl y P elements in each
1,j

term in parentheses. Since the distributive law holds for wedge multipli-

cation and addition, it will be sufficient to prove the identity for monomial

p and q forms and to evaluate e.g.

((;) 1 ";"2 " ...

"c:)p) (;"1''';''2''''.' " q')


" .

Wedge multiplication is associative, so the parentheses are not important.


First move
(;,1' to the extreme left by interchanging it repeatedly with
its left neighbor. Each interchange brings in a minus sign, a total of

(-l)P. Now move


(;,2' left to the second position. This brings in p
more minus signs. And continue... .
By the time we are done with
;"q"
the sign will have been changed pq times, hence the desired identity.

Solution 8.5. We show that the two definitions are equivalent by showing
that dg has the same components in an arbitrary coordinate basis when

calculated from either definition. Start with the definition in terms of the

covariant derivative. Let the com p onents of g be 0 = O


ap"'Y [ap".Y ]
(p indices). Then the components of dg are

r O
O[a{3.. .y;o] O[a{3' ...y ,0] [ao {3 f... y]
= -

(1)
rf Of -...-r
[{30 [yu a{3o fl-Or 'y ,u ]

la{3'

- ・

...y f J
. . .


(The r terms vanish since


rp.[va]

0 in a coordinate basis.) Now

start with
g = 0 "J;{3
a " ...
. y
ap' ..y

By property (ii), d does not disturb the summation convention, so

dQ =
(dO a{3.. 'Y) A (d; ch{3 A A ...
A d;Y)
(2)
+ {} d ( d;a " d;{3 " ...
" d;Y )
ap"'Y

CHAPTER 8: SOLUTIONS 265

where we have used property (iv) with p



0 since
fJa{3...y is simply a

function (zero form). Now from property (i) we know that

-" -


df =
f dx
,a

for any zero form f. From property (iii) ,


< df, a/ a x f3 > = <f d;.a , a / a x f3 >
,a
cll
f thus
af3 llaf3...y ,8 ;h8 The second term in Equation (2)

,f3;


...

is zero: According to property (iv) it can be reduced to a sum of terms

each of which contains a ddx, which vanishes by property (v). Thus


Equation (2) becomes
-"0 -- -
-"

dg =
Q dx "dx

" dxt-' ・ ..
"dxY .

at-' Y ,u
. . .

Since dg is a (p + 1) form (property (i)), and the wedge product is anti-

symmetric, the components of dg are


n[a{3...y,o]
as in Equation (1).

Solution 8.6. Define

so that =
g;h2

and
==

[Xl.[l f x2 ,'"
( xl,
xn)d ] ,

2 -3 -"2
d it =
g ,1 . " i 2 -1
g ,1 dx = "dx + g
,3
dx "dx + ...

Now

g,l

f.l f'( xl,x2,...xn)d ]


[f.\( xl,x2,...xn)d ]+[xl The
where f' indicates a derivative with respect to the first argument.

second term above can be written as

1 1

I xl

f'( xl,x2...)d =

1 0
: f( Xl,x2,"')d

f( xl,x2,"')I -

I f(o
xl,x2,...)d ,
266 PROBLEM BOOK IN RELATIVITY AND GRAVITATION

where we have integrated by parts; we have then

g, I

f ( XlX2,

...


)I = f (Xl, X2, ...


) .

Next, da =
0 tells us that

3 1 2 4 1 2
da
f,3 dx "dx "dx +
f,4 dx "dx "dx + 0
= ...

which can only be true if f






" 4



= ...

,n

0, and hence

g,3

[Xl r. fJ xl,x2,...,xn)d ]
0, that finally

0 ・

Similarly g 4 ,

g 5

= ...
g , n

so




Ad;2
l f (x ,... x
n -1-2
) dx "dx
g,l a
= = =
, x .

Solution 8.7. The 2-form is F =


F d;/l" d;v so that
/lV

o =
dF =

/LV ,A
A AA d;v
d;/l.

gives the Maxwell's equations O. Similarly d*F 0


us
F[/lv;A] gives
= =

*F[Aa;v]

0, but this is equivalent to (see Problem 3.25)

o = E/lvAa *F =
2 **F/l

= _
2F/l

Aa;v ;v ;v

and so we have the remaining Maxwell's equations


F/l v O. =

Solution 8.8. At any point on the surface choose a local orthonormal

tetrad e \. ,x
e"" e"" e",. If the surface is spacelike, we can choose the
y z

tetrad so that e'f is the unit normal. The 3 orthogonal, linearly inde-

pendent vectors lying in the surface are then spacelike


(ex' e

, e
z )'
Now

any other orthonormal tetrad we might have chosen is related to the first

one by a Lorentz transformation, which does not affect whether a vector

is spacelike or not. Hence the three vectors are always spacelike when
the surface is spacelike.
CHAPTER 8: SOLUTIONS 267

In the null case 1 choose ei +


ex to be normal to the surface. A null
vector is orthogonal to itself 1
so the 3 orthogonal vectors in the surface

are
e}}
and e
z (spacelike) and e


ex (null). In the time like case,

choose to be normal. Then in the surface, e is timelike; e and


ex f y

e'2 are spacelike. Again these results are unchanged by Lorentz trans-

formations.

Solution 8.9. The volume element is

, x{3 XU)

3 1 a(x
d ( a, b ) = E d a d b dc

Il IlV{3U

a(a, b, c)

3!

If we change parameterization from a, b, c to a, (3, y then



a( a, b,
I a(

da db dc da d{3 dy .

a, (3 y),

Provided (a, (3, y) has the same orientation as (a, b, c), the Jacobian is

positive (definition of same orientation!), so we can drop the absolute


value signs, and

d 3 ( a"c
b ) .1 E
a(xv, xP , xu) iJ(a, b, c) fJ d y
da d,Q
Il 3! IlVPU a(a,b,c) a(a,{3,y)
v P xu)
= 1. E
a(x , x ,
da d{3 dy
3! IlVPU a(a, (3, y)

d3If1.(a,{3,y)
which completes the proof. With differential forms, the invariance is

obvious since d I can be viewed as a 3-form
f1.


d 1 = 1. E hV A Ah
Pu .

f1. 3! f1.vpu

Solution 8.10.

(a) The 2-form e and its exterior derivative are

k 2 At m
e = f d S

= fk 1. E
klm

de =
fk,n 1. E nA + fk 1 (dEklm)
lA m A lA ・

2 kl m 2
268 PROBLEM BOOK IN RELATIVITY AND GRAVITATION

Define the symbol [klm] to be + 1(-1) if k, I, m is an even (odd) permu-


tation of 1, 2, 3, and zero otherwise. We then have



-- 1

g,n dx II gl2 dx / g.
dE klm =
d(1 gJ2[klm]) =
[klm]


g,n

2 k1m 2

Thus we have

1 (1

k --1'-" m

1 n
de
\f g,n /g dx dx Adx
= + E A
2 ,n 2 k1m

1 1 ......,.-.. --
1 k n 1 m
\ g\ 2"
(I g\2" f dx

"dx "dx
),n
= E .

k1m

-1
--

m 1m 1
"""'2 3
But in three dimensions, dxn" dx "dx =
\ g\ 2 En dx " dx "dx and
n1m n

klm


20 k'
so



-'1 -2
de =
f' k \gl2dx "dx "dx 3 .

Thus, in ordinary vector notation, the theorem becomes

f V
v. i dV =

Ii'!!S
dV

(b) e -

flLd3 =
f/l E (:} ,, a{3
,, Y
IL 3! IlflfJY

de =

(f/L,X Ep.af3y +f/ldEp.af3y)Ad;aA f3


Ad;Y
g ,'\
= 1
3!
(flL ,\
\,
E (:}
/lUfJY
,\
+ f/l 1.
2 g
E (:}
/lUfJY )
,\
" ,, a{3
,, Y


\ gl t flL) ,\
g\-t(1 E
/lUfJY
,, ,,
a {3
,\ "d;Y
3! '


-1
(lgltf/l)X,/lUfJY EAaf3YlgltdxOA lAd;2Ad;3
E t:).

3! 1 g\ 2

(\ g \ t flL) , ,\ 0'\/l 3! I g I t d; " ,, 2
1 0 1 3
= "

3! \g\2"

-2 -3

0 -1
dx dx
f/l;/L I gl2 dx A dx
= A A .

The theorem then reduces to

I \g\ t

4 3
f/Ld
V.f d x
I/L'

n ao
CHAPTER 8: SOLUTIONS 269

(c) 8 =
FILV d2
IL V

FILv !. E d;a A d;{3
2 IL va fJ

1 v -._
de = -


d(FIL E
IL va fJ


dx
) A dx fJ

Igl-t(lgltFflV),,\ElLVa,8;G'\A aAd;,8
where we have taken d E as in part b. Now
lLva {3

Eflva,8J;A A a
AJ;,8 8 J;KA
J;a Adx T

Eflva,8 (1/6)

= -
(1/6) E EyAa{3 E YKur J;K d;u J;rA A
ILvafJ

= -
E EyAaf3d3 y
ILva {3


2oyAd3
v IL y'

and by Problem 7.7 (i) we have

de =
Igl-t(lgltFILV)\oyAd3 ,/\ IL V Y


2F ILv d3
;v IL'

and finally


an
FILV d2
ILV


1 n
FILV d3
;v IL


(d) Let e =

Ak dx = .

!if, and let n be 2-dimensional; then the

exterior derivative is
.........k
-.


dx
de =

Ak ,j dx . A



k,J
.0[koj] m n
mA n



k'

rkj E J;m A n
2 ,J rmn



(VxA)rd s
_ r

which completes the derivation.


270 PROBLEM BOOK IN RELATIVITY AND GRAVITATION

Solution 8.11. Suppose there were such a tensor. What would be its com-

ponents at some arbitrary point in a freely falling local orthonormal frame?

In this frame all the vanish (because all the r


{3/l vanish),
and
{3,/l a

{3l1a{3°

Thus the tensor must be some algebraic combination of
l1a{3'
"zero," and possibly

f3yo.
oaf3 and E You can very quickly convince

yourself that the only nontrivial one is '1 0 TJ


!H10.'. (the a only renames


f3yo gives
indices, e. g.
oaf3 Tfay Tff3y
and the E zero by its antisym-

metries, or else just gets some of its indices lowered). Since we chose

an arbitrary point, the conclusion holds at all points.

Solution 8.12.

(a) The general definition of the r's is

r V e
e/l.

1Jil{3

{3 a

In a coordinate frame

r/l{3a e/l' (V (3 Va e(3)


r - =
e -

1Jil{3 a


e/l. [e{3' e


0 ,

since in a coordinate frame basis vectors commute.

(b) In an orthonormal frame

r + ra V + e V
e/l. {3ea {3e/l

1Jil{3 /l{3 a

V ) V 0
(3(e/l' e
(3(T//la)
= = =

Solution 8.13.

(a) We use the fact ("usual rul for a derivative operator") that for

any vector field v

x<A,v> < =

xA,v> <A, xv> + .


(1)

Thus for the components of the I-form xA have we


CHAPTER 8: SOLUTIONS
271

(x A)a ==



= <

A ' ea >

x<A,ea>
-<A' xea> (1)

(3 A, [x, ]>
Aa,{3x < e
= -


a,fJ
x{3 AY'
< [ x{3e


fJ'


]>



a,fJ
x{3-A

< Y,(x{3c
fJa
°e _x{3e
,a fJ u



a,fJ
x{3 x{3,a AfJ
+ -



x(3cfJa Y .
(2)

The last term vanishes in a coordinate frame. An alternative form is,

from Equation (1),

( xA)a Vx<A,e =



-<A,(Vxea-V e

x?
- -


<V

A, e

> + < A, Vex> a

Aa;f3 xf3 xf3;a Af3 (3)


= + ・

The connection coefficients do not really enter into the defini tion of the

Lie derivative; they cancel out of Equation (3) to leave Equation (2).
(b) For simplicity, work in a coordinate basis:


T =

x (Ta e (l;;;,{3 )
fJa

(!!.x -ra(3) e

f3
+ -ra
f3(!!.x e
a) it,f3 + -ra a
f3e (!!.xit,f3)
xY e a ;,{3 xY + rz x{3
;,(3 Ci,Y


rz -


(3

Y Y (3


fJ , Y ,a ,


(T

xY -
T /1 x

+ rz xl1 e ) ;,{3
(3 ,y (3


,11 l1,fJ

This is often written

rz =
Ta xY -

TI1 xl1
xa+ TIl1,fJ
x fJ fJ,y fJ,/1

and the commas can be replaced by semicolons as in part (a).


272 PROBLEM BOOK IN RELATIVITY AND GRAVITATION

Solution 8.14.

1\ R
., r-l

a(P I ) a(P 3 )

P3
PI

=I
=O (PI)

At the points
Po and P

consider two tangent vectors A(P 0) and

A(p 1) each extending from A =


0 to A = =
aCE).
A (Note: In the figure
Ataken to be unity so that the
is A vectors are large and the
"difference vector" K is exaggerated for clarity.) Suppose that at point
P1 the vector B(P 1) connects P

and
PO. Let us compute the amount

by which the vector field B fails to connect points of equal A along the

congruence A, i.e. let us compute the vector K:

K =
[B(P 1 )+A(P o )] -

[A(P 1 )+B(P 3 )]

[B(P 1 )-B(P 3 )] -

[A(P 1 )-A(P o )]
T 2
= ATBY
,T


+ (9(A2B) -
B AY
,T


+ (9(B A)

[A, B] + (9(f3) if IAI (9(f) \8\

Thus in the limit of infinitesimal vectors A and B,

K =
[A, B] =

fAB .

i.e. if B is "Lie-dragged" along A.


Thus K vanishes if AB
0, =
CHAPTER 8: SOLUTIONS 273

Solution 8.15. Contraction may be thought of as multiplication by the


constant }(ronecker-delta tensor
oflv. But the Lie derivative of the tensor

oflv is zero, as we can easily prove in a coordinate frame,


(oIL ) = oIL \ x + all.. xA

_
oA xlL A =
0 + xlL v -
xlL


0 ,
x v
V,/\ A,v v' "

and therefore the contraction operator


o /
can go on either side of
x.
Solution 8.16. If the operator identity holds for scalar functions and for

vector fields, it will hold for arbitrary tensors. This is because the

action of the Lie derivative on arbitrary tensors is determined by its

action on scalars and vectors (see Problem 8.13). Let f be a scalar

function, then

u vf- =

v uf vVuf
uVvf- [u,v]f = =

[u,v]f.
Now let w be a vector field, then

u - vw
v - uw
[u,v]w [u, [v,w]]
= -

[v, [u,w]] -

[[u,v], w] =
0 .

The vanishing of this last expression (the Jacobi identity for commutators)

can easily be checked e. g. by writing out all terms. This completes the
proof for scalars and for vectors, and hence in general.

Solution 8.17.
(i) First the scalar case:

(I)(P N) =
(I)(P 0) + [xlL(p N) -

xlL(p 0)] (I) ,fl(P 0) + ...

(I)(P 0) -

g IL <I>,fl '

and (I)(P
N) =

<I>(PN) since <I> is a scalar, hence

<I>
<I>,/l /l

(ii) Since x lL(p N) =


xlL(p 0) =
xfl(p N) + g fl, we have for vector fie Ids



fl(PN)

(ax / axIL)
AIL(P N )

(oafl- a,fl)Afl(PN) .
274 PROBLEM BOOK IN RELATIVITY AND GRAVITATION

Now we combine this with

A/L(P N )

A/L(P 0)

A/L,f3 f3
to get

A/L
A/L(P o)

/L(P N )
= -

A/L(P o ) Aa,f3
(aa/L - a,/L)(Aa(Po)- (3)

A/L,f3 f3 Aa a,/L +

(iii) The tensor case is handled similarly:

(iJxNiJx )(ax f3 ;ax )-raf3 (P N )


a V
T/Lv<PN ) =


(a/L + a (a f3 /L,a) v

f3,v)[Taf3(p0) Taf3,ye] -


Tilv (P o ) -

T/l
v,y
e -

T /l
{3 {3
,v
+ T

v,a
/l

T/l


T/l
v,y
+ T/l

{3 (3
,v
_

v,a
/l .

Solution 8.18. The parallel transport law u



v{3.,a =
0 requires only the

affine connection r The Fermi-Walker transport law
{3y'

v v =
u(a.v) -

a(u.v) a == V u
u u

requires, in addition, a metric for the dot products. The Lie transport

equation V v -
V Vu =
0 requires u to be a vector field defined off the

curve, not just a tangent vector along the curve, because the gradient of
u in the v-direction must be defined, and v is not in general along the
curve. Although the (ormula above includes V's, a connection is

actually not needed, because the l's cancel out due to the antisymmetry

of the equation. Obviously no metric is required for Lie differentiation.

Solution 8.19. If can be returned to A unchanged, then in principle

it can be extended into a vector field which is both parallel transported

), and consider
x Z
and Lie transported from A to B. Let u


(u , u

, U

the identity
CHAPTER 8: SOLUTIONS
275

V v




u =
[u, v]
- -

2. u v = -
2.v u .
(1)

Setting the appropriate terms to zero in Equation (1), i.e. V vu =

2.vu =
0,
and noting that the Christoffel symbols vanish, we have



v = 0 =
UiV k .

0 (2)
,1

Y X z a -

-u =
U =
au z =
0 .


So !. can be nonzero (and have U 1= 0) only if

a=O. (3)

Solution 8.20. Let the vector field be u(p). If it is parallel propagated


along itself, we have V UU =
O. This is just the geodesic equation, so the

field must be a field of tangent vectors to a space-filling congruence of

geodesics.
If the field is F-W transported along itself, we have

V u =
(u a-a u).u (1)

where a =
V uU' Thus

a =
u(a.u) -

a(u.u) .
(2)


Now dot u into this and get a.u =


(a.u) -

(a.u)u =
O. From (2) we


have that either u =
-lor a =
O. The first possibility means that any

field of properly normalized 4-velocities is F -W transported along itself.

The second says that the tangent vectors of any congruence of geodesics

are also, even if they are not normalized to -1.

The Lie transport law is

o =
2. u u == [u, u]

which is satisfied by any vector field.


276 PROBLEM BOOK IN RELATIVITY AND GRAVITATION

Solution 8.21. Consider a I-form U =

UadX

. First calculate
d xU:
fiJ =
(xaU(3 ,a + U
axa,fidxf3
d(fiJ) =

(xaUf3,a+Uaxa,f3),Y.hY A;G{3 (1)

Now calculate
x(diJ):

----

dU Y Adxa
Ua,ydx

- -.. ---

x(dU) x(Ua,y)dxY

= Adx

Ua,y x(dXY)Adxa ........

Ua,ydx
+ Y
x(dxa) A

........ - .-....,I -- - -

dx Y A dx
T T

xTU dx Y A dxa + U xy dx A dx

+ U x

a,yr a,y ,T a,y ,T


A;G{3
[exTU(3 ,T + Ua xa,(3) ,y Ua xa
,{3y] ;G

= -


(2)

Equation (2) is obtained merely by relabeling indices. By symmetry of


partial derivatives and antisymmetry of wedge product, the last term in

brackets on the right hand side of Equation (2) vanishes. Comparing


Equations (1) and (2), we have for I-forms:

d( xU) x(diJ) =

The generalization of this method from I-form to p-form is straightforward.

Solution 8.22.

(i) (\IS/- =
c:,i.VS =

hid;i.VS =

hiaS/ax

(ii) The general expression for the curl must be

(VXV)i =

!gl-tEijkVk" ,J

Ig\-tEijkV k ,j

since it is a covariant relation and it is true in a locally flat


(cartesian) coordinate basis. (See Problems 3.20 and 3.21 for
ijk
the relevant properties of the antisymmetric tensor E .) In
the orthonormal basis
CHAPTER 8: SOLUTIONS
277

" "

(V X V)i = 6)i. (V x V) =

hi(V X V)i = h (h h h )-1 E ijk a(hk Vk)lax-i


i 1 2 3 .

(iii) Using the result in Problem 7.7 (g), we have

1 1.
V V I gl-z [I glzV
l 1

],i

(h 1 h2 h 3 )- [(h 1 h2 h 3 ) hi Vi],i



(iv) With the expression in Problem 7.7



(j), V S becomes



S =
Igl-t(!glt ga l3s,I3),a =

(hlh2h3)-1[hlh2h3hi2S),i'
Solution 8.23. We use results (ii) and (iii) of Problem 8.22 with xl =
r,
2 3
x =
0, x =
cp and with h


1, h 2 =
r, h3 =
r sinO. (For simplicity
we follow the usual notation here and omit the carets indicating physical

components.)

2 2 1
V ・



(r sin 0)-1 [r sin 0 h:-

A.]
1 ,1



(r sin 0)-1 [(r2 sin 0 A)
r ,r
+ (r sin 0 A
O), 0
+ rArt.. rt..]

(VxA)r =
(r sinO)-l [(sinO Aq),O-AO,q)
(VxA)O =
(r sin 0)-1 [Ar,cP-(r sinO AcP ))

(V x A)CP =
r- [(rA O ) ,r -

r, 0
] .

Solution 8.24. If forms and exterior differentiation are used, this problem
- -

is trivial. The first equation gives F =


dA so, dF =
ddA =
0, since dd
is always is exactly the components of dF,
zero.
F[p.V;A] so we are

done. The trick is that the total antisymmetrization implicit in the d's

keeps any connection coefficient from entering the problem. If components


are used, the proof is straightforward but more tedious.


Solution 8.25. In general, ds dxfldx13 Since xi =
constant along
rul3

2 2
any of the geodesics, ds =

goo dt along any of the geodesics. But


2 2
along the geodesics ds dr -1 everywhere. Let e be
goo
= - =
, so

278 PROBLEM BOOK IN RELATIVITY AND GRAVITATION

the coordinate basis vectors, and let u =


d/ dr be the tangent vector field
to the geodesics (Le. u =
eo)' Since u is orthogonal to
SI' then u. e

eo' e

gOi
= 0 at r = 0, and

d(u.e.)/dr = V (u.e.)


0 + u. V e. =
u. V U
1 u U 1 e.

since the curves are geodesics (V Uu =


0) and since e

and u form a

coordinate basis ([e i u],



0). This last result vanishes

u .
V u = 1. V (u.u) =

e.
1 2 e.

and therefore u.e. =


go 1 .

0 everywhere.

Solution 8.26.
-A
(a) From the point of view of modern differential geometry,
r and
Il V

r v
are the component description of "machines" which map two vectors

and a form into a number; Le.

A V
(nu m b e r) =
r a

U ll v == < a,
t7
v u >
Il V v

where the covariant derivative is taken using the metric corresponding to

whichever r is used. A tensor is a "machine" which is linear on all

"slots" (Le. in all its arguments). By itself r or r is not a tensor

since
f<a, Vvu> 1= <a-, Vv(fu)> =
f<a, Vvu> + <a,u> Vvf

and likewise with "bars" over the V's for g . If however we subtract

the two equations, the linearity-spoiling < a, u> (V v -



v) f term is

identically zero since all covariant derivatives acting on a scalar function

give the same result, i.e. are just directional derivatives. Thus
S v
IS

a tensor.

(b) We cannot compare the geodesic equations of g and g directly

since they would involve different affine parameters rand T. If t is an

arbitrary function of r, a change of variable makes the geodesic equation


CHAPTER 8: SOLUTIONS 279

2 A \ v A 2
r dx dx dx d r dt
+ =
0 ・


dt 2
dt2 dt dt
dt
dr


To eliminate the dr / dt term, multiply by dx / dt and antisymmetrize on

a and A to get


a du A du

A a
PI
A V

dt
_

dt
+ (r u

_

/lV
u ) ulLU = 0 '

a a
where u == dx / dt. If the world lines ua(t) are geodesics for both g
and g then the above equation must also hold with r replaced by r.
Subtracting these two equations gives

(oa SA _oA Sa )uau uv 0





Va IlV

which implies that

oa SA
(a Il)v

oA(a Sa =
0 ・

/l)v

When this equation is contracted on a and v the result is

A A A
a a

a(l)/l +
(I)a

all


(I) -
S /l

5 all

[See also L. P. Eisenhart, Riemannian Geometry (Princeton University

Press, 1962), Section 40.]

Solution 8.27. The difficult way to do this problem is to compute connec-

tion coefficients in a coordinate frame. Here we will use the method of

"rotation I-forms." (See MTW, Section 14.5 for details.) Choose an

orthonormal frame as follows:


" -

lOOt a
CA) =
e dt


;e{3J;
wO =
eYcW

;"cP =
e Y sin (] d4> ・
280 PROBLEM BOOK IN RELATIVITY AND GRAVITATION

The key formulas the definition of '6,a


are

dc;"a = -

waS A c;"S

and the relation of the forms to the Christoffel symbols


v-" ;,X =


/iA- CA) .

We now use these formulas for the given metric; let dot denote a/at

and prime denote a/ ar :

d =
teaa'.h dt A =
a' e-{3 "; A t (1)

d6;; =
e{3 dt ch e-a ;;.,t 6;"; A =
A (2)

de::. e =
eY y;IT A.de + eY y' d; A dO
,," ,," (3)


-a


A ;., () + e -(3 y' r

()

dw4> =
eYy sinO dtAdif> + eYy'sinO d;Adif> + eY cosO deA.ij
=e-a y C;/-A;;,4> + e- f3 y'c;,?Ac;,4> cotOe-y eA;;,4> + (4)

But we also know that dc';)C2 =

-c';)aS c';)S A and


c;,aS -c;,Sa'
= where

indices raised and lowered with For a t have


TfUS'
are

we

dCA)

t -

t A CA)


-CA)

t"
A CA)

0 -CA)
....
t"
A CA)....
-CA)?

() cp

Comparing this with Equation (1), we guess that


""
(3 CA)-t

t r , r
(5)
- .... - -

CA) e + K CA)

= =
CA)



" "

....
t"
....

() -

()
CA)
()

CA)
t =. K 2 CA) (6)
" "

cp
Co)rP't
". ..

t...
K3 Co) (7)
Co)
4>
= =

where K 1 ,

2, K3 are functions we must find. Similarly, Equation (2)

implies that
CHAPTER 8: SOLUTIONS
281

;.,t -


e-a c;,;+ K

wt (8)

-,;" -0 -

CA)
() -CA)
KsCA)
= =

r (9)

-; _2 _:1..
'f'
CA)
'f'r K6 CA)


CA) =

(10)

Equation (3) implies

c:,0t =

-a
yc:,O (11)

c:, e-{3y'c:,O =
(12)


0 - :;.. _:1..
CA)
-CA)'f'O K7 CA)'f' (13)
= =

and Equation (4) gives

,c:, t e-ayc:, =
(14)
" "

-(3 y' c;,CP
;"CPr =

(15)

."

c;,CP 0 =
cot () e -Y ["cP .
(16)

Comparing Eguations (5) (16), -

we find

'" "

-'t {3 CA)t (3 e-aCA)


- -
, -r

CA)" =
a e +

i;)to =
e-ayc:,O


e-ayc:,CP

-;" - , - ()
CA)
()

-e CA) y

';-e-{3y,


-cote e-Y c:, .


" "

c::,>.
But
rv/lX =

i:JViL' so the nonvanishing connection coefficients are


282 PROBLEM BOOK IN RELATIVITY AND GRAVITATION

" "

,.
{3
r\ t -T"""-
r -


.L -

ae
tt

" " .

rt rr

rrt; =
(3 e-


t""


o" r
t " "

J.." -a
()()

t()

cpcp

tcp

ye

r "" 0"
[' ?() r
? " "


2 " , -
tJ
?cp
= - = = -
y e
= -

()() cpcp

rO;p -r = =

e-cotOe- Y .

Solution 8.28. Start with l+z =


I1T A /I1T .
TQ show that this is equiva-

lent to the other definition, recall that p is proportional to k, the wave

vector, which is the gradient of the surfaces of constant phase:

k =
V() .

Since k is a null vector,

o k. k k. V () V
= = =

k()

i. e. () =
constant along the rays. Consider two rays differing in phase by
11 (). Obse A
ver computes 11() as

!!..O =
O(T+!!..T A) -

(J(T) =
(!!..T A) \l 0 =
(!!..T A)uA.k ・

UA

A B

8 =80 +68

TA
CHAPTER 8: SOLUTIONS 283

Similarly observer B finds

11() =
(I1r B) u B k

where k is now evaluated at B. Since 11() is the same for A and B,


we have
I1r (u 'P)B
A =
(U.k)B =

rB(u.k)A (u'P)A

so the two definitions are equivalent.


CHAPTER 9: SOLUTIONS

Solution 9.1. If we construct a coordinate patch from geodesics we can

then bisect that coordinate box with a geodesic diagonal, forming two

geodesic triangles. The angular excess of a triangle made from great



circles is 11 [Areal a ]. Since the sum of the interior angles of the coordi-

nate box will be the sum of the interior angles of the two triangles, the

angular excess of the coordinate box is 11 [Areal a ].
The best coordinate patch constructed of lines of longitude and lati-

tude is that near the equator. Clearly there is no angular excess, and
- -

AC =
BD, but

8.
2 -/18 CD -
AB =
al:!cp-aI:!<f; sine ; -MJ)
al:!cp ( (})2 ,

B =
(Equator)
so

- -

Area
1. ()21.
CD-AB

cp 2
AC a

In both these examples, the departure from flatness is measured by



11 a , the curvature of the sphere.

Solution 9.2. The antisymmetry of (a{3) and (yo) in


Ra{3yo
means

that there are M =


n(n-l) ways of choosing nontrivial pairs (afJ) and

similarly M ways of nontri vially choosing (yo). Since the tensor is

symmetric with respect to interchange of the pairs (a{3) and (yo) there
are M(M+l) independent ways of choosing afJy8 when the pair sym-

metries are considered.

284
CHAPTER 9: SOLUTIONS 285

The Riemann tensor also possesses the cyclic symmetry

R 0
Aa{3yo +
Rao{3y +
Rayo{3
= =

{3yo

The pair symmetries guarantee that A is totally antisymmetric so



{3yo
that the constraint Aa =
0 is trivial unless a, (3, y, a are all distinct.
{3yo
The number of added constraints is then the number of combinations of 4

objects which can be taken from n objects =


n!/(n-4)! 4! (This formula
gives zero, and is therefore valid, if n < 4 in which case the permutation

symmetry gives no additional constraints.) The number of independent

components is then

2 2
M(M+l) -

n!/(n-4)! 4! =
n (n -1)/12 .

Solution 9.3. The problem requires ordering 21 independent components

in some canonical way and assigning a number N::; 21 to each nonequiva-

lent quadruple I, J, K, L. The following program. accomplishes this and

reads the value of R(I, J, K, L) from R(N), 1::; N 21.

READ I, J, K, L

SET SIGN =

IF 1= J OR K =
L SET RIEMANN = 0 AND RETURN

IF I > J EXCHANGE I, J AND SET SIGN =


-1

IF K > L EXCHANGE K, L AND SET SIGN = -


SIGN

SET N1 =
(5*1 -

1*1)/2 + J
SET N2 =
(5*K -

K*K)/2 + L

IF Nl> N2 EXCHANGE N1, N2

SET N =
(13*N1- N1*Nl-12)/2 + N2

SET RIEMANN =
R(N)*SIGN
RETURN.

Solution 9.4. According to the discussion in Problem 9.2, there is only


one independent component. We can choose it to be

r() ra
R(}cp(}cp g()(}R(}cp(}cp g(}(}{r(}cpcp ,() r(}
(}cp ,cp r(}(}a cpcp
+ ra
cpa cp(} }

= = -

286 PROBLEM BOOK IN RELATIVITY AND GRAVITATION

2 2 2
From the metric r r sin () find that the only
cp we
= =
, non-
g()()
vanishing Christoffel symbols are

r()cpcp sin () cos (), rcp cot ()


= -

()cp

and, therefore,

2 2
()
R()cp()cp r sin
Rcp()cp() R()cpcp() Rcp()()cp
= = = - = -


Solution 9.5. The only nonvanishing Christoffel symbols are r = v
uu

u u 1
and r uv
= r vu
= v- ,
so that

R =


= r




+ r

va
rauu -

ua
PIuv
vuvu uvu uU,v vu,u

=1-0+0-1=0.

Since there is only one independent Riemann component in two dimensions

conclude that 0, and the spacetime is flat. (This also


we
Ra{3yo was

shown by a coordinate transformation in Problem 6. 1.)

Solution 9.6.From the diagram, two orthogonal infinitesimal displace-



dcp2

ments have length adcp and (b+a sincp)dO so ds a + =

cp )2d()2 cp)2, gCPCP



O. The
(b+ a sin or
()(b+ a sin a =

cpgCP()
= = =

formula for the Christoffel symbols now gives the nonvanishing r's
directly
rCP()()
-1
= -
a (b+ a sincp) cos cp

r()()cp r () = = a cos cp (b+ a sin cp)-1 .


I I I II

III

----1-----
I b
II

---- -----




CHAPTER 9: SOLUTIONS 287

In 2 dimensions the Riemann tensor has


only one independent com-

ponent. Take it to be then the standard formula gives


Rcp()cp()'
Rcp()cp()

a sin cp (b + a sin cp) .

Solution 9.7.

(a) In one dimension there can only be one component R 1111 , and

by the symmetries of the Riemann tensor, this component must vanish.

(b) In two dimensions the relations, due to symmetries,


among the

components of the Riemann tensor may be summarized in any coordinate

system as

( g{3o
y of
g{3y)

{3yo

but

{3a{3 (g!la g{3{3 ga{3 g{3a)f



R =
R = - =
(4-2)f =
2f .

Thus, in any coordinate system

Ra{3y8

}( y
g{38

g{3y)
8R ・

(c) In three dimensions there are six independent components of the


Riemann tensor. Since the independent components of the Ricci tensor

represent six independent linear combinations of the Riemann components,

we should be able to invert such a relation to get the independent Riemann

components in terms of the Ricci components.

To have the correct symmetries, we must have

Rp.VAa a(gpA gvA Rp.a- gp.aRvA + gvaRp.A)


a-


b(gp.A g",a gp.a gvA)


Now we find a and b by contracting:

Rva

a(3R va -

Rva- Rva+ aR)



b(3g",a- gva) R .

So we must have

a=l b=--.

288 PROBLEM BOOK IN RELATIVITY AND GRAVITATION

Contract again to check:

R =
a(R+3R)+ b.6R =
(4-3)R .

Solution 9.8. We give the proof for a tensor of arbitrary rank. Choose
locally flat coordinates so that all Christoffel symbols (but not their

derivatives!) vanish at a point. At that point

A A
R =
r rA: = -
2r

Y[V,K]

YVK YK,V YV,K


and

. a. . ・ ・ ・ .a
r fJ
・ ・ . . ・

(T a. fJ
) " VK -

(Ta. fJ + T +..._ T fJ r _00'


), K
va. av

a. av
・ ・ ・ ・ . . ・ .

... a... ... a



T fJ + T r fJ +..._ T fJ r _....
a. . ・


VK a. . .
a v,K a.. ・
av , K

It follows that

... a... ... a


2 Ta... R fJ T fJ R
(Ta...
fJ ...+ +...

;[VK]

aVK-

a... aVK

Since this relation is covariant it is true in any coordinate system.

Solution 9.9. The second derivates of a scalar are

(S ,a);{3 S S ra
S;a{3
= =

,a{3 a{3

,a

This expression is obviously symmetric in a and {3.


To investigate third derivatives use local flat coordinates as in

Problem 9.8. We have then

S;a{3y (S,a{3 Sar (3),y S,a{3y Sa I ,aa{3,y S;(a{3)y


= - = - = ・

From this expression and from the result of Problem 9.8 we also have

S;a[ y]

S;aRaa y ・

Solution 9.10. Form the tensor



{3 =



{3 _


{3 ・

IlV ;Ilv ;vp.



CHAPTER 9: SOLUTIONS 289

From Problem 9.8 we have

a {3 {3 Ra

p.v
= _


a p.v
_

aa
R{3 a
p.v
so that

cILV/LV = -

AUVR/LU/Lv A/LURvU/LV - = -
AuvR uv +
A/LURU/L =
0 .

Solution 9.11. Choose locally flat coordinates at the starting point P.


u 8
Thus r 0 at P but ra
f"lt3y,8 u after displacement When

t3y u.

t3y
we displace A from P by u we find that its components become

[A (displaced by u)]U = AU + A

u t3
,t3
a a
ut3 Y u t3 a
A + A
f"lyt3 A A
= - =

t3

When we displace, again, by v the new components are

[A (displaced by a
t3 t3
f"lYt3 AY

u then v)]a =
A + A
it3
v -

Aa-f"lYt3AYvf3 Aa-f'O-Yt3,8AYvt3u8. =

We could have reached this same point by travelling around the other side

of the parallelogram to find, similarly

f'O-Yt3,8AYut3v8

[A (displaced by -v then _u)]a = A -

The change in A in making the complete circuit is our first result minus

our second

8 8
AYv t3 u AY u t3 v
8 AYvt3u8
a a a a
8A = _r + r _R
YfJ, 8

YfJ, 8

Y fJ

..

-u

..


290 PROBLEM BOOK IN RELATIVITY AND GRAVITATION

Solution 9.12.

(a) First consider the linearity in A. It is clear that

R(A 1 +A
2 , B) C =
R(A 1 B) C , + R(A 2 B) C , .

Furthermore,

V fA VB VB VfA V[fA,B]
- -

fVA VB fVBV A (VBf) VA


Vf[A,B] AVBf (1)


- - - -

The two "extra" terms cancel and hence

R(fA, B) C =
fR(A, B) C .
(2)

It is easy now to prove that the value of R(A, B) C at a point P depends


only on the value of A at P, not on the way in which A varies: Con-

sider f(P) =
0; then Equation (2) tells us that R =
0 at P whenever the

first argument vanishes at P. Now let A and A be two vector fields


1 2
which are the same at P but different elsewhere. From (1) we know

R(A 1 ,B)C -

R(A 2 ,B)C =
R(A 1 -A
2 ,B)C =
0 .

This completes the proof for A and, by symmetry, for B. The proof for

C follows similarly.

(b) Since only the values of A, B, C at point P are important, we

can arbitrarily choose all three to be covariantly constant in a neighbor-


hood of P; then we have

(R(A,B)C)a =

(VAVBC-VBVAC)a


2A [a (B ] Ca. A)'
a , ,

2A aBACa. [A' , ,
a ]

According to Problem 9.8, this is equal to

RL CILAAB a

p)..a

and the proof is complete.


CHAPTER 9: SOLUTIONS 291

Solution 9.13. Let xll =


xll(A, n) be a family of geodesics with affine
parameter A. The parameter n labels the individual geodesics. The
a a
tangent to the geodesics is u == ax / aA and the differential vector which

connects points of equal affine parameter on adjacent curves is

a a
n =
ax / an .

The solution to this problem is most transparent in abstract notation:

Write
u =
a/a'A, n=a/an.

Note that [u, n] =


0 since the partial derivatives commute. (The vanish-

ing of this commutator in fact is the definition of n as the connecting

vector, according to Problem 8.14.) Now use the Riemann operator (see

Problem 9.12):

R(u,n)u ==
VuVnu VnVuu V[u,n]U VuVnu VuVun
- - = =

where the last equality follows from the vanishing of [u, nJ. In com-

ponents our result, according to Problem 9.12 (b), reads

D 2 na
cL\2

(V u V u n)a = _
Ra,Q
fJ y
ou{3nYuO .
292 PROBLEM BOOK IN RELATIVITY AND GRAVITATION

Solution 9.14. {t is convenient to transform the spatial coordinates to

polars and to write the metric


pherical as

2 2 2 2 2 2
ds = -
(1-2M/r) dt + (1+2M/r) (dr + r [d0 + sin 0 dcp2]) .

For
r O
a circular equatorial orbit, u =
u =
0 so

Du I dr


0 = u

Fa{3u{3 =
(u

)2 Foo (uCP)2 rrcpcp

and therefore

(dcp/dt)2 (,)2
-rroo/Fcpcp
= =

The Christoffel symbols are easily calculated and we find

(211/Period)2
2 3
(U = =
M/ r .

This is the same as the Newtonian result. In Problem 9.13 the equation

of motibn is derived for n, the vector joining points of equal proper time

on two geodesics. We are required in this problem to deal with



g, the

separation of points on the two geodesics at a given coordinate time.


From the diagram, however, it is clear that the fractional difference

between nand g is proportional to the relative velocity of garbage and


--

the Skylab, and hence can be ignored to lowest order.

Since all the Christoffel symbols in the t, x, y, z system are of



2 1
order M/r and did, is of order (U
, (M/r) "2 r- '"
, we can approximate

D .Q. ru d

_

dr

dr dr

Furthermore

.£.
[ ( )] d.£.t


.!l = u =
1+a '
dr dt r

2 2 2 2
so we can approximate D /dr d /dt .
Using these approximations
and the equation of geodesic deviation we have

2 i
d g i 0 2 j


+ R

0.0 (u ) g =
l=X,y,Z.

dt
CHAPTER 9: SOLUTIONS 293

Skylab
/ /
Garbage

To lowest order the Riemann components are

i j

( )
i i i 1 M 3X X
R 0'0 1 rO = -
g OO = -
0.. -

00

. . .. .

J , J J ,
0 2 , lJ 3 lJ 2
r r

If we choose to describe the Skylab orbit as

x =
r coscut, y =
r sin cut ,

then the equations of motion for become

z+cu2 z=0

x2 gx
+ cu =
3cu cos cut (cos cut gx + sincut Y)

2 2
gy + cu gy =
3cu sincut (cos cut gx + sincut gy) .

The relative motion in the z-direction is obviously

gz oc sincut

if the garbage was jettisoned at t = O.


294 PROBLEM BOOK IN RELATIVITY AND GRAVITATION

To find the relative motion in the xy plane introduce new variables


1 2
11 ,
11 defined by
1 2 2
,x =
11 cos cut + 11 sin cut, gy =
111sincut -

11 coscut

and the equations of motion become:

22cu =
10

+ 12cu _

23cu 111 =
0 .

These can be solved quite easily to find the four independent solutions:

1 2
[17 ,
17 ] =
[1, wtJ, [0, 1], [cos wt, 2 sinwtJ, [sinwt, -2 cos wtJ .

The linear combination of solutions which correspond to the garbage

jettisoned at t =
0 (i.e. gx =
gy =
0 at t =
0) is

,x =
A[cos 2cut-3 + 2 cos cut + 3cut sin cut] + B[4 sin cut -
sin 2cut]

gy =
A[sin 2cut+2 sincut -3cut coscut] + B[cos 2cut+3-4 coscut] .

The constants A and B depend on the x and y components of the

velocity with which the garbage was jettisoned. The nonperiodic terms

in the solution correspond to the fact that the two orbits are of a slightly
different period, so the relative distance will exhibit a secular growth in

ti me.

Solution 9.15. Choose locally flat coordinates so that

a a
R =
2r
{3yo {3[0,y]

In these coordinates then



2Il{3[0,y]] 2Il{30,y]
= =

[{3yo]

Since the Christoffel symbols are symmetric on their lower indices the

right side must vanish, and by lowering the first index we have
CHAPTER 9: SOLUTIONS 295

O. The Riemann tensor is antisymmetric in its last two indices,


RU[{3yo]

so this is equivalent to the cyclic identity.



From Problem 9.8 we have for any U that

U a a
2U = -
U R
[{3y] {3y

; a

Take UU to be fixed at some }Joint and parallel transported along geodesics



to all nearby points in some neighborhood, then at the point U ;{3 = 0 for

all {3, and hence


U a u
2U -
U R
;[{3y]0


{3y;o
and
u a u

2ifl;[[{3y]0] 2U U R
[{3y;o]
= = -

;[{3yo]


From Problem 9.8 we have for the tensor U that
;{3
U U
Ra
U {3 ayo if;a {3yo
2U -
R +
;{3[yo]
= ・

Since
UU;{3
= 0 at the point we have
if;{3[yo]


0 and hence
UU; [{3yo] =
O.

We could have chosen the direction of U at the point in any way so it

follows that

R = 0
[{3y;o]

Since the Riemann tensor is antisymmetric on its last two indices, this is

equivalent to the Bianchi identities. (The Bianchi identities can also, of


course, be derived by the straightforward but tedious process of differenti-

ating and manipulating the formula for Riemann components in terms of

Christoffel symbols. This is best done in locally flat coordinates.)

Solution 9.16. By contracting the Bianchi identities,

u a u
R + R + R -0
{3yO;f {3cy;o {30f;y

first on U and y then on {3 and a we get


296 PROBLEM BOOK IN RELATIVITY AND GRAVITATION

R{3o;f R{3f;o + +
R{3of;a =

R. (

R{3c t:l
,fJ

RIca ,

0 .

Thus the divergence of the Einstein tensor

( i 8 (VR);v RV(;V

GV(;V =
R =

R;(

must vanish.

Solution 9.17. Choose A, B, C, D to be mutually orthogonal at a point P


and to satisfy A.A =
-1, B.B =
C.C =
D.D =
1. Since spacetime is

Riemann flat, parallel transport around a closed curve does not change a

vector, so we can define A, B, C, D away from P by parallel transport.


If we do this we have e.g.
aA

-r Y
a{3Ay

ax{3
therefore
aAa

aA{3
ax{3 ax

so we can choos,e Aa

cP ,
a'
and similarly for B, C, and D. We can

summarize by defining

A=W(O), B=W(l), C=W(2), D=W(3)

and we can define four functions <1>(0), <1>(1), <1>(2), <1>(3) such that

w(ll) =
V (I)(Il) .

Since inner products are preserved by parallel transport

W(Il) w(v)・ =
[W(Il). W(V)]p =
111lV ,

and this tells us that

(J<I)(a) (J<I)({3)
gllV V

.,.,a{3
axil ax
CHAPTER 9: SOLUTIONS 297

and hence there exists a coordinate transformation, namely

xP- = c}) (Il) (x ll )

which takes the metric to the Minkowskian form.

Solution 9.18. If the Weyl tensor vanishes, the metric can be written in

the conformally flat form


ds =
e2 ds 2 o

e2 11
IlV
dx ll dx


where is a function of x . The null geodesics

dt =
n.dx, n. n =

--


show that they remain null
of ds remain null curves for ds . If we can

geodesics, then clearly a cone of null geodesics, will remain a cone;

there will be no preferred direction for squashing of the cross section.

To show the curves dt = n.dx are geodesics we need only show that


d x


dpa _



2 dA
dA

for null geodesics. Note that the Christoffel symbols are

al1
[-cP ,11 TJyl3 +cP ,13 TJYI1 +cP ,y TJI1I3]

r =
TJ
Y13
so that

pyp13 01Y13 =
2 pa(VcP' p)
and

dpa (3 pY dpa 2 pa(vcP'p)


o +
01I3Yp +
= = ・

dA dX"

dpa/dA 0 'and the straight null lines of


By rescaling A, then we have =


ds are null geodesics of ds .

the
Notice that, while beams of light have undistorted cross section

factor 2cP
sectional area, like all areas, is affected by the scaling
e .

cross
298 PROBLEM BOOK IN RELATIVITY AND GRAVITATION

Since is in general anistropic, beams propagating in different directions

will grow in cross section at different rates.

Section 9.19. In a conformally flat spacetime the Weyl tensor vanishes,


so the Riemann tensor can easily be computed from the Ricci tensor

1. 1.
r {j
a a
(log \ g\ 2) ,/lV + r + (log \ gl 2),a r
R/l
= - -

V /lv,a /LV /lU vf3'

The expressions we need are



y {3

- ,al1y{3+ ,{3oay+ ,yOa{3


V 2 cp
raytJ,a + 2cp ,tJy

TJytJ

raytJr pa
Y =

6cp,/lcp,tJ

2(Vcp)2 TJ/ltJ

-g

e 8, log \ g\2 =


a 2
(log \ g\2),a r y{3 = -
4(V )
l1y{3
+ 8
,{3,y

(log \ g\2) ,y{3 =



,y{3

In the above expressions raising and lowering have been done with l1/l v ' e. g.


cp
,a ==
TJpa cp ,/l; V cp ==
rF' cp ,pa; (Vcpi ==
rF cp ,/l cp ,a ・

The Ricci tensor and scalar are then


/lv
= -
2 II v
'r

2 II 'r'

1JlIv[V2 +2(V )2]



e- 2cp 6e- 2cp [V cp+(Vcp)2]


R/l/l TJ/l R/l
= = -
==

and the Riemann tensor is


CHAPTER 9: SOLUTIONS 299


Ra{3yo +
( R{3o

{3yo R{3y
o g{3y Rao + g{3o Ray)
- -


( g(3o
y g(3y)
oR


1 2 rP

R(3y 11(30 Ray)

(3y Rao
_

l1ay R(3o l1ao 11 +



- -


1 4rP (l1
l1ao 11(3y) R

11(30

6 ay

2CP

[TJay(CP ,(3 cP ,0 cP ,(30) TJao(cp ,{3 cP ,y cP ,(3y)
= - - -

11
(3y (rP ,a rP ,0 rP ,ao) 11(30 (rP ,a rP ,y rP ,ay)


+ -

(V cP i(
+ TJ TJ + TJ
ao TJ (3y)]

ay {30

Solution 9.20. We use the same orthonormal tetrad as in Problem 8.27:

'"
'" '"

'0
- -






dt , w

= e (3 dr, =
eY dO , rP =
eY sinO d1> .
(1)
'"

In Problem 8.27, we com puted the 6 connection 1-forms c;)p..v from the

equations '" '" '"

d 1L _wJ1",


' V CA)
'" '" = -
CA)
'" '"
(2)
IL V

v VIL

'"

The Riemann tensor is most easily found from the 6 curvature 2-forms
lLv:
'" '" '" '"

a ",
IL",

dwlL",


+ wlL",

A t7J
v (3)

'"


/l =


1. R v"' (3 a
'"


aA
i:J(3 (4)

The advantage of this method is that only non-zero components of the


Riemann tensor are computed. Using the results of Problem 8.27 in

Equation (3), we find

'" '"

()
'\
9{. =
d(a' e
(a- (3) dt + e{3-a J;) + c::, % A c::, r + C;/- A c::, cP1


[a"+(a')2 -a'(3']e -(3J;

A dt + ( +-a )e(3-adt
2 J; A


[e -2acfl + _

2 )e- 2 (3 (a" +a,2


a _ -

a'(3')] 6)tAW; .
(5)
300 PROBLEM BOOK IN RELATIVITY AND GRAVITA TION

""

Here a dot denotes a/at and a prime a/ are Notice how the term d t""is
"" r

computed by writing c:)tr in the coordinate basis and


using dd ==
O. The

remaining curvature 2-forms are

9t to y dO) t-r i;)'; 0 + c:,t i:J if> 0


-a
d( eY + c:, A
if>

""

(y+y2 -ay)-e- 2 {3a'y']w t ()


2a

[e- A

+ e -(a+{3) ( yy .,

-a
, .

y+y -y
., ,

fJ

) CA)-
';
A CA)


(6)

9tt
if>

d(ey-ay sinO dcp) + w A
w';i> i}o ;/)i> + A


[e- 2a (y+y2 -ay) _e- 2 {3 a'y']i:Jt A

""

+ e-(a+{3)(yy'_a'y+y'_y' ) A ';
6:J (7)

i:J';if> i;)i> 0
""

d(ey -{3y'dO) c:,\ to


CD r ""
J\()=

+ A c:, + A

-0 -';
[ e -2{3 ( y

H ,2 ' ' -2a
{3 y ] CA)


+y

(3 y ) -

e A CA)

""
. ""


e-(a+{3)(yy'-a'Y+Y'-Y'{3)w A t (8)

9t'; if> =
-d (ey -{3y's in o.ij)+ c:,\ A
c:,tif> ;)0 + A i:J0
-2 fJ H ,2 , -2a .
- -
';
[e

(y +y ) e fJY ] CA) A CA)


= -

fJ y

""
. ""

+ e-(a+{3) (yy' -a'y+y' y'(3)6:J -


A t (9)

"" "" "'"
"" ""


9tO = -
d(cos 0 .ij) + c:, °t A c:,
tif> + c:, r
A c:,r
i>
"" ""

-2 y -2a .2 -2 '2 -() - (10)



(e + e y


fJy ) w A W .

We can now read off the non-zero components of the Riemann tensor from

Equation (4):
CHAPTER 9: SOLUTIONS
301

"",..

R tr"""", =

tr
"" ""
"" ""

RtOW ""

RtCPtif> =

"" "" ""
"" "" "" ""

to"" trl.. rO r'rl..


R R 'f' "" ""


""

R C
'f'tr/J

';r/J
- -
';0

to
= =

ROif>Oif>
",,""


""
r',.I.. "'r0

""

'f';cp R
""

'rO

E (11)

where
{.:l

-2a
( fJ + fJ 2

-2 t-' (" + a ,2 ,
(.:l'

afJ ) )

- - -
e \a


fJ

2a 2
B =
e- (y+y2 -uy) _
e- {3a'y'
C =

-(a+{.:l)
fJ
.,

(y + y y
.,

,.
Q '


y fJY
- -

D =
e- 2y + e-
2a
y2 _
e-

{3y,2
E =

-2a
fJY


e -2{3 (Y H
+ Y
,2 -
Q'
fJ Y

) ・
(12)

The Ricci tensor is obtained by contracting the Riemann tensor:

"" "" ""


Rya
"" ""

R - ""

{3 y{3

This gives
""

R t"" =
A + 2B

""

R t"" =
2C

""

R = A + 2E

ROe =
Rif> if> =
B + D + E
""

t t '; ; o

""


""


"" "" ""

0- R r/J- R 0- R r/J- (13)


- - - - -

r/J-

The scalar curvature is


""

a ""
R =
R =
2A + 4B + 2D + 4E .
(14)

302 PROBLEM BOOK IN RELATIVITY AND GRAVITATION

The Einstein tensor is found from

- - -

a - a 1 a-



R u R
{3 {3-

2 {3
Le.

Gtt' =
-(D+2E)

G t", =
2C



G _


-(D+2B)
'" '"

O-



GcP '"

-(A+B+E)
- -

t t r- r - o
G G



'"

G G (15)
0=

o = = =

Solution 9.21. Choose an orthonormal basis e- and construct a complex,


null tetrad basis oriented so that the wave vector Vu == k is one of the

tetrad legs: 1
k =
(2)- 2 ( +
e;)

} =
(2)-2 (t -

e"')


m =
(2)-2 (e-x + ie-)

m =
(2)- 2 (e'"

ie"')

(1)

and consider the components of the Riemann tensor in this basis: e. g.

YIO
Ra{3yo lam
R 1mm1 = m etc. Since the Riemann tensor is a function

only of retarded time and since the only nonvanishing dot products of the
basis vectors in Equation (1) are


k.} =
m.m =
1 , (2)
one finds


Rabcd,p (3)

where (a, b, c, d) range over (k, I, m, Iii) and (p, q, r.") range over

(k, m, m ). Now, consider the following subset of Bianchi identities for


the Riemann tensor:
CHAPTER 9; SOLUTIONS
303


Rab[pq,l] (4)

where I is the I index of Equation (1). Using Equation (3), we get


Rabpq,l (5)

from Equation (4), and Equations (5) and (3) imply

R =
R =
0 (6)
abpq

pqab

aside from a trivial, nonwavelike constant. Consequently, all nonvanish-


ing components of the Riemann tensor must have the form R
p1q1
and,

taking into account the symmetries of the Riemann tensor, we see that

there are only six independent components. These six components corre-

spond to the number of degrees of freedom in the most general gravita-

tional wave in the generic metric theory of gravity. Einstein's theory has
only two degrees of freedom in a gravitational wave.

Solution 9.22. In a local Lorentz frame, the acceleration of any particle



is related to F/l by the Lorentz force law:

2 u
d x




U {3
dr
2 in {3


It is convenient to think of the 6 components of F as the vectors
{3
and In
. the comoving frame of one particle, it has zero velocity and
so feels no magnetic forces. Measuring its three components of accelera-
tion gives E .
Measuring the acceleration of a second particle as seen in

that frame gives only two components of B, since the components of B -


along the direction of motion of the second particle produces no accelera-

tion. Thus at least three particles are required to meas ure all components


of F
{3.
The reader is invited to figure out the answer for the Riemann tensor

from the geodesic deviation equation


304 PROBLEM BOOK IN RELATIVITY AND GRAVITA TION

D2 a

= -
/Q
fJ yo
u(3 yuo .

dr

Solution 9.23. Choose a coordinate system geared to the surface. Let


1 2 3 4
x , x vary, and x , x be constant on the surface. The only contra-

variant components of A and B are then the 1 and 2 components. We

are interested then only in the Riemann components with indices 1 and

2, but there is only one such independent component, R 1212 . The non-

vanishing Riemann components of this type are related by symmetries


which can be expressed as


ij kl
ex
(gik gil

gil gj k) i,j,k,l =
1,2.

It follows immediately that in this coordinate system K is independent


of A and B. Since K is clearly coordinate independent the desired
res ult is proved.

Solution 9.24. Let the interior of the circuit be subdivided into infinitesi-

mal parallelograms, and consider the change in angle for one small rec-

tangle. If u and v represent the sides of one rectangle then from


Problems 9.23 and 9.11 we have that the change in A after transport is

Circuit -

approxima ted with rectangles


CHAPTER 9: SOLUTIONS 305

oA

= -

afJl1-v
A{3ul1-v v

K ( Q: g

gav g A{3ul1-v


11-fJV fJl1-

Note that AaoA




o(A.A) =
0 so the length of A is unchanged. The

change in angle 0 between A and B can be calculated from

BaoA


o(AaBa ) =
IAIIBlo(cosO) =
IAIIBlsinOoO
and

BaoA


K[(B.u) (A. v) -

(B.v) (A.u)] .

Now choose a locally flat coordinate system on the surface with u "'J e

and v "'J e

then

1 2 2 1
IIAllBlsinOoOI =
K(B A _B A )uv

K IAxBI uv =
K sinO IAIIBI. (area enclosed) .

Hence, for transport around a small rectangle 1001 =


Kol. If we now

consider a more general circuit, the change in A due to transport around

that circuit will be equal to the change in A due to transport around all

the rectangles into which the circuit is divided, and the area of the circuit

is the sum of the areas of the rectangles, thus I OI =


l.

Solution 9.25. Define

W R (1)
K( {3 g{3y)
==

ryo o ay {3o
- -

ay{3o

and note that W has the symmetries If K is in-


ay {3o
same as
Ray{3o'
dependent of the vectors A and B at the given point, then by the
defini tion of K (Problem 9.23)


ay{3o
Aa A{3 BY BO = 0 (2)

for all A, B, thus


ay {3o
+ W + W
ao {3y
+ W
{3yao

0 (3)
{3oay

306 PROBLEM BOOK IN RELATIVITY AND GRAVITATION

The symmetries of W allow us to rewrite Equation (3) as


ay {3o

W =
W (4)
ay {3o aoy(3

Permute (y{3o) cyclically:

W =
W (5)
aoy{3 {3oy

Substitute Equations (4) and (5) in the cyclic identity

W + W + W =
0 (6)
ay{3o aoy{3 a {3oy
and find

W =

ay {3o

which is the required result.

Solution 9.26. Since the metric is always covariantly constant


K,A (gay g(3o g{3y)

(3YO;A gao

Now subs ti tute in the Bianchi identities

o = R + R + R

(3YO;A a {3Ay;o a (3oA;y

and contract and (3, a to find 0 i.e. K is constant.


on
K,A

a, y

Solution 9.27. By contracting we see

R/lK 3Kg/l

R =
12K.

The definition of the Weyl tensor is

C>"P.VK R>"P.VK ( v
RP.K g>"K Rp.v gp.v R>"K gP.K R>"v)

- -
= -


(g>"v gP.K g>"K gp.v) R

so that
CHAPTER 9: SOLUTIONS
307

CAP.VK

K(gAv gP.K -

gAP. K)
- ・

3K(2gAv gP.K -

2gA
p. K)


12K(gAv g/lK gAK g/lv) 0

- =

Solution 9.28. Since n'u =


0 we have immediately


(n{3u{3);a =
o =
n{3;a ua u + n{3 U ua
(3 {3 ;a

-Ka{3u u{3


+ 2n'

Solution 9.29. The metric has the form

2 2 i
ds = -
dr + g.. dx dx j
lJ
where

(xk)

g I].. =
a (r) y I] .. .

The normal vector to the r = constant surfaces is n =


alar. Thus
-a
1 ----L
K..=-e..V.n=n.V.e.=r g.. =
gij'
"=-
lJ 1 1 J J n]1 2 lJ , n a

Solution 9.30. From the definition of the Lie derivative (Problem 8.13)

we have

nPa{j P {3;y nY


+ P
y{3n

;a
+ P nY
ay ;{3

nY
(ga{3 na n(3);y +
(ry{3 nyn(3) n +
a( nany) n {3

= - - -

Y + (1)
(na n(3);y n + n

n(3;a

a ;{3

where we have used


n ( a
nY);a O. If e are three basis
= =

vectors in the hypersurface, then

K.. =
-e.' V.n = -no .
= _1 (n. .+n.o') (2)
lJ J 1 l;J 2 l;J J,l

where we have used the fact that K.. is symmetric. If we are using a
lJ

general coordinate system (i.e. not one in which 1, J lie in the hyper-
surface), then Equation (2) can be rewritten
308 PROBLEM BOOK IN RELATIVITY AND GRAVITATION

Ka{3
= -

(n,,;O+no;y) PY a

°{3 (3)

Le. as the symmetrized covariant derivative of n


projected into the

hypersurface. Using the explicit form of


pYa in Equation (3), and com-

paring with Equation (1), we find

Ka{3
= -

nP
a{3

(xl,
3 2 3
Solution 9.31. Let the surface be x =
0, where x , x ) are Gaus-

sian normal coordinates (see Problem 8.25) based on this surface. Then

the area is

J gt 1 2
A =
dx dx (1)

where g is the determinant of the two-dimensional metric in the


gij

surface. Vary the area by displacing surface elements a distance ox

along the normal n =
a/ax :

J gt 1 2
oA = ° dx dx ・

In Problem 21.1 it is shown that

o gt =

t gij 0 = 1. g
t gij g.. 3 a x

gij 2 lJ ,

But we have chosen Gaussian normal coordinates, so


K.. = - -

g 1J..

1J 2 , 3

f gij K ij gt
1 2 3
oA = -
dx dx ox


Since A is a minimum, oA =
0 for all ox , and hence we must have

ij
K =
g K.. =

lJ

CHAPTER 9: SOLUTIONS 309

Solution 9.32. Let e



be a coordinate basis in ;
the equations to be

derived are tensor equations and so will be valid independent of the basis

used to derive them. The vectors e


i along with n form a basis for the

so
spacetime,
(4)V.e, 1

a..n + fJ
k..e


(1)
J 1J 1J

The coefficients in Equation (1) can be found by dotting in nand e


(n.e i =
0):
(4 ) V (4 ) V ( 4 ) V. n 0 + K..
,(n. e,) e.
・ =

E a.. = e, = -

n .


lJ 1 J 1 J J lJ

(4)V.e. (4)r k (3)r k J 1


{3k lJ e

= = ..
= ,.

J1
'.

k 1 J

The last equality follows from the fact that the Christoffel symbol can be

computed directly from (3)gij. With these coefficients Equation (1) be-

comes the "Gauss-Weingarten equation."

(4) V . e. = E K.. n +
(3 )


.. e .
(2)
1 J1 k
J lJ

The Riemann tensor can be computed from derivatives of the r's; it

is somewhat quicker to use the curvature operator (see Problem 9.12):

R =

ea' R(e y eo) e(3 (3)


{3yo

where
[V Vo ] (4)
R(e y eo) V[ey,eo ]
= -

, '

Evaluate Equation (4) for e



and eke
The last term vanishes because

[e j , e

] =
0 for a coordinate basis, and

(4)V. (4)V k e.1 =


(4)V.(£K.1k
n + (3)r m

1k

m )
J J

m (3) m
= E K' .n -
E K' K e + r . .e
k J m
1 k ,J m

1k J 1,

(3) n

(3)

lk (
E K, n + r . e
n)

(5)
Jm

mJ
310 PROBLEM BOOK IN RELATIVITY AND GRAVITATION

Evaluate Equation (5) with J and k interchanged and subtract the two

equations. The result is

R ( e.,e
k) e. =
E(K.1 k\ J.-K'. )n + e [E(K..K m-K.1 k K.m)+(3)Rm.. k ].(6)
J 1 lJ \k m IJ k J IJ

Dotting Equation (6) with n and en respectively gives the Gauss-

Codazzi equations.

The components of the Riemann tensor with two n's can be found

from R(e k n) n. , Since n has been chosen to be a coordinate basis vector

[n, e

] =
0 so the last term in Equation (4) vanishes and thus

( 4 ) V' ( 4 ) V (4 ) V (4 ) V
R ( e k' n )n =

k n
n -



k'

(4) V
But

n =
0 because n is tangent to a geodesic (Gaussian normal
coordinates), so that

(4) V V
e R(e k n) n e
n(Kkme ) Kki,n Kkme
. .

nei

= -


i i m m

Now [e, n] =
0 im plies that

(4 ) V e. =
( 4 ) V. n = -
K.j1 e .

n 1 1 J

so finally
R.In k n =
ERn.In k =

k 1, n

+ K.j

K'
J k

Solution 9.33. The Riemann components were found in Problem 9.32. Con-

traction gives

(4)R =

gik?(4)Rnknj+(4)Rmkmj)
i i

(3)R iJ .
+ E (gik K.
kJ , n
+ 2K




_


K) (1)


where K == K

.. Now gik ,n
= -
gim gks Q
cms,n
(see e. g. Problem 21.1) and

g ms,n

2K in Gaussian normal coordinates, so
ms

im ik
gik,n =
2g gks K ms =
2K .
CHAPTER 9: SOLUTIONS 311

Equation (1) can now be rewritten:

(4)R iJ . =
(3)R iJ .
+ E (gik K. +g
ik
K .
_



K)
kJ, n ,n kJ J



(3)R iJ .
+ E

(K J,n

_
K .

K) .

(2)

For the other components of the Ricci tensor we have:

(4)R n

E(4)Rana J _(4)R n i
E(K I J._Ki..) (3)
J \1
. . = =
. .

J 1 J

(4)R nn = E(4)R inl '


= E gij(K..lJ +K. K

.) =
E(K -K. K
mi
). (4)
, n 1m J , n 1m

The Ricci scalar is then

(4)R =
(4)R n


+ (4)R i .



(3)R+E(2K -K.
1m
Kim_K2) (5)
, n

and the components of the Einstein tensor are:




(4)R n
_! (4)R = _!. (3)R + ! E(K 2 _
K. Kim) (6)
n n
2 221m

(4 )Gn (4 ) R n K '.

(K 1 J \ ) (7)




E .- .

J J J 1

(4)G



(4)R i J _1 oi.J (4)R


im 2
(3)G -Ki.K-10i.(2K
i i
= .
+E[K .
-K. K _K )]. (8)
J J ,

J 2 J ,
n 1m

Equating (6) and (7) to the corresponding components of the stress-energy


tensor gives "initial value equations" for the gravitational field.

Solution 9.34.

(i) Since there is no favored direction in the spherical surface the

extrinsic curvature tensor must be K.. ex 0.. in an orthonormal


lJ lJ

basis; this implies that any vector is an eigenvector. From the


definition of K as the rate of change of n, the eigenvalue

must be -l/sphere's radius =


-l/a.
312 PROBLEM BOOK IN RELATIVITY AND GRAVITATION

We can give a mathematical development of these intuitive results by

going to the usual spherical coordinates r, (), cpo These are clearly
Gaussian normal coordinates and

2 (2 i k
)gik dx (d()2 + sin2 () dcp2)

ds =
dx =
r .

Now K so
gij,r

gij,n
= = -

ij

1 1 1 2

( ) 1/ a
""""

K = - = - - -
r = -

()() ()() 2
g()() r2 ,r

1 1 1 2

() 1/ a.

K ,/..,/.. (1)
""""

K ,/..,/.. = - = -
r sin = -


'fJ'fJ
cp
'fJ'fJ 2
sln2 ()



(ii) Again we can argue intuitively: The "special" directions must

be the axial and circumferential directions. From the definition

of K the values of the curvature (rate of change of n) in these

directions should be 0 and -l/a.

Mathematically: Again the usual coordinates are Gaussian normal and

the 2- geometry is

dcp2
2 2
ds = r + dz .

The orthonormal components of K are then

1 1

( ) II a
"" ""
= =

gc/>c/>,

C/>C/> 2 r

c/>

.l
K""""
(_l2 g
) 0 (2)


zz zZ,r
gzz

and our intuitive answers are correct.

(iii) In cartesian coordinates


2 2
dy2

ds =
dx + + dz (3)

the surface is

f( x, y, z) =
0 (4)
CHAPTER 9: SOLUTIONS 313

where

f = -
(ax +2bxy+ cy 2)+z. (5)

The unit normal is

n =
Vf/!Vfl =

[-(ax+by)ex-(bx+cy)ey+(e z
)] (6)
where

N _

[(ax+by)2 + (bx+cy)2 + 1]2. (7)

Note that since n =


e at the origin, and e can be used as basis
z ex y

vectors in the surface there. Thus

K.. =
-e..V.n = -n.. = -n..
lJ J 1 J ;1 J ,1

since the connection coefficients vanish for the metric (3). Because

N =
N =
0 at the origin, we have
,x ,y

The secular equation is



ij

[: :] ・

a-K b
o =
det(K.. -

Kg..) =
(8)
I] IJ
b c-K

where is found by res tricting (3) to the surface (5). Since dz =



gij
at the origin, g..

0.. at the origin. The principal curvatures are found
lJ lJ

from Equation (8) to be

2 t

[ ]
c+ a c- a 4b
K+ =
:!: 1 + .
(9)
2 2
(c_a)2

With these eigenvalues, the principal vectors are found from

a-K+ b V

0 .
(10)
b c-K+ V
314 PROBLEM BOOK IN RELATIVITY AND GRAVITATION

The solutions are:

V:t =
(e x +a:t e y)/(3:t

a+ ==
y [1 :t (1+ y-2) 2]

2 2

(3:t ==
(l+a:t)
y ==
(a-c)/2b .

Solution 9.35. From the Gauss-Codazzi equatio (Problem 9.32), we have

(3)R ffi
. .

(2)R ffi
. . + E (K.. K kffi -
K'
1k K.ffi)
lJ k lJ k

lJ J

Contract on m and j and on i and k, and set (3)R ffi


'.

lJ k
= 0 because

the 3-space is flat. Thus find

(2)R = _
E(K) K.i_K.iK))1 J 1

(1)

At any point P on 2, choose the principal directions as axes and let

xi (2
the coordinates measure lengths away from P. Then
)gij = a
ij
at

P and K.. is diagonal, with the principal curvatures as the diagonal


1J

elements. Hence

[ ( )J
(2)R = __
E -1. + 1. - 1. + 1 = .-L ,

pi p Pl P2 PI P 2

where we have set E =


n.n =
+ 1.

For 2 3-dimensional in a flat 4-space, Equation (1) gives

[pi ( )J
(3)R = -
E -1. + l + -1 _ -1. +-1 +
P P Pl P2 P3

2 2 2

( + +


PI P 2 P2 P 3 PI P 3
CHAPTER 10: SOLUTIONS

2 2
dcp2;

Solution 10.1. The metric is ds =
d0 + sin 0 the connection

coefficients are r ifJ rifJ cot (); r() sin () (); all others
ifJ()
= = = -
cos
()ifJ ifJifJ
are zero. Now:

'a;(3 +
(3;a =

a,(3 {3,a 2rlla{3 11


+ -

0 ・

a =
(3 =
cp: cp,cp = -
sinO
O cos 0 (1)

a =
(3 =
0 :
0,0

0; :.


f(cp ) (2)


0,(3 cp: O,cp cp,O + 2 cotO
cp (3)
= = =

Substitution of Equation (2) in Equation (1) gives cp,cp f(cp) sin 0 cos 0
= -

which implies

cp =
-F(cp) sinO cosO + g(O) (4)

where F(cp) ==
f f dcp. Next substitute Equations (4) and (2) in Equation (3):

:J :
2 2 2
+ -

F(cos () -
sin ()) =
2 cot () g -
2 cos () F

or

( CW-2cotOg(0))
df dg
+ F(cp) = -

dcp

Since the left side is a function only of cp and the right side only of 0,

each side must be a constant:

df/difJ +

f fdifJ =
b , (5)

dg/ dO -
2 cot 0 g = -
b .
(6)

315
316 PROBLEM BOOK IN RELATIVITY AND GRAVITATION


Equation (6) has an integrating factor exp[-2 fcotOdO] =
sin- 0, giving


( )
g =
-b
CW 2 2
sin 0 sin 0


g(O) =
(b cot 0 + c) sin 0 .
(7)

Equation (5) is solved by differentiation:

d2 f + f = 0
drp2
f =
d cos rp + e sin rp

F =
d sin rp -

e cos rp .

(The integration constant is included in g(O).) Substituting this result in

Equation (5)

d sin rp + e cos rp + d sin rp -
e cos rp = b

shows that b =
0, and hence from Equations (2) and (7) that


go =
d cos rp + e sin rp =


cP

cP c sin 0 sinO cosO (d sincP coscP) sin 0





- =

Thus


(d cos rp + e sin rp) 1- + [c -

cot 0 (d sin rp -
e cos rp)] 1-
ao arp

is the most general Killing vector. Note that it is a linear combination of

the three Killing vectors


arp

( cos rp .£
ao

cot 0 sin rp 1-
arp )
sin rp 1- + cot 0 cos rp 1-
ao arp

which just the usual angular momentum operators and


are
Lz, Lx Ly'
the generators of the rotation group.
CHAPTER 10: SOLUTIONS
317

Solution 10.2. It is straightforward to prove this statement in


component

Alternately, "dot"
fe g
notation. with two arbitrary vector fields A

and B:

o =

A.f g.Bf (A. B) -B.f :A Atf B


= -

V (A.B) B.(V A-VA A.(V


) B-VB )
= -

B,VA +
A,VBe 2AaBf3 (a;f3)'
= =

Since A and B are


arbitrary, g(a;(3) must vanish. To interpret this
result geometrically recall that the Lie derivative of any geometrical
change of that quantity, ocp A

quantity, cP A' is the functional ==
cp ew
(x )
cp X1d(x ) under


a displacement of coordinates by T
.his is equi va-

lent to the change in


CPA upon moving it in the manifold along .or

the Lie derivative of g along to vanish, therefore, requires that the


geometry to be unchanged as one moves in the direction, Le. repre-
sents a direction of symmetry of spacetime.

Solution 10.3.

(a) We use the equivalence proven in Problem 10.2. If u and v are

Killing vectors then


fug fv g = =
0 and, with the result of Problem 8.16,

[u,v]g (fufv-fvfu)g
= = 0 ・

Thus [u, v] is a Killing vector.

(b) If a and b are constants, then

(aua +bv a bV
);(3 aU +

;(3 ;(3

a a

Thus if u and v
satisfy Killing's equation, so does au + b v.

Solution 10.4. The three Killing vectors describing rotations are just the
an g ular momentum operators, Jz =
x a

yax , etc. (See Problem 10.1.)


At a point (x o Yo' '
zo)
318 PROBLEM BOOK IN RELATIVITY AND GRAVITATION

Jz =
( -

yo' xo' 0)

Jy =

(zo' 0, -xo)
Jx =
(0, -

zo' YO)

so Jy =

-(zo/Yo) Jz -(xo/Yo) J x and the three vectors only span a

2-dimensional surface at the point.

Suppose, however, that for some constants a and b

a Jx + bJ y + Jz =

were true at every point, then we would have

a [J x
' J y] + b [J y J y], + [J z' J y ] =
0 ・

This requires Jx ' result which is obviously false. the


Jz Thus, over
oc a

whole sphere, Jx ' Jy '
Jz span a 3-dimensional space. The explanation

is that 2-spheres are 2-dimensional, but orientations of 2-spheres are

3-dimensional (e. g. the 3 Euler angles).

Solution 10.5. The commutator must be a Killing field according to

Problem 10.3. By hypothesis any Killing vector must be a sum (wi th con-

stan t coefficients) of and :


(t) (cp)

[ (ct?' (t)] a (ct? b (t) +



At infinity (cp) a/act> and


(t) a/at so the commutator O. This

means that a and b must be zero, Le. that the commutator vanishes

everywhere. (B. Carter, [Commun. Math. Phys. 17, 233 (1970)] has proven

that
[ (cp)' (t)] 0 under very general conditions even when there are

other Killing vectors.)

Solution 10.6. For the commutation of second derivatives of any vector

'Il;VA 'Il;AV RllaAV,a


- =

CHAPTER 10: SOLUTIONS 319

Now we use the Killing equation


gll;v -gv;1l and contract on and A

11

g v; A. A

( g/l: ,,);


+ R
vag = _

,,-

The term on the right vanishes since


1l;1l 0 by the Killing equation,

and we are left with the desired result. The variational principle is easy

to write down if we note that g11,'V is antisymmetric, just like the electro-

magnetic field tensor F We know that the Lagrangian F F/lV


/lv' /lV
gives a term FIlV. v when we write F

Av'"

A".v =

Av "

A" v
and
, 11,,.,..,.,..,,,- '-'

Thus the Lagrangian density be taken to be


vary
All' can

f =

Il;V
gll;v _1


Il V
1l V



and a r f \g\2 d x =
0 will give the correct equation. This can be verified

by finding the Euler-Lagrange equations for this Lagrangian.

Solution 10.7. For any vector

a; P /l

a; /lP

RAa P /l A (1)

by the definition of Ra{3yo' Add to Equation (1) its two permutations and

use the cyclic identity for the Riemann tensor

ap/l /lap + +
PW

to obtain the identity


a;PIl a;IlP gll;ap +
Il;pa p;lla p;all'
+ (2)

= - -

For a Killing vector, Equation (2) becomes

o = -

ga;IlP -

' (3)
a;PIl Il;pa

and substituting Equation (3) into Equation (1), .one obtains

/l;pa RAap/l A (4)


= ・
320 PROBLEM BOOK IN RELATIVITY AND GRAVITATION

Solution 10.8. We first show that "stationarity" is equivalent to the

existence of a time coordinate for which O. Choose the time


ga(3, t

a/at, then (see Problems 10.2, 8.13)


coordinate such that =

f =

g0, or

(3,y +
Y y g(3y (30a + =

Since =
(1,0,0,0) this
y proves that
(3,t

O.

of
{3 independent
Time reversibility (definition (i)) means is t

and O. To show equivalence of the definitions it remains to show


gti

that
gti

0 is equivalent to the hypersurface orthogonality of =
a/at.
If gti =
0, then =

a {3
= = 0
tunless a = t. Hence

is

proportional to t
, a
i. e. is orthogonal to the surfaces t =
constant. If

is hypersurface orthogonal (see Problem 7.23),



[a; {3 y]

this becomes
With Killing's equation
(a;(3) 0,

a;{3 yy;a (3f3;y 0a


+ + =

Dotting with and


a using Killing's equation on the first and third terms

gives

( a;{3- {3;a)e +
2.a {3 2.{3 0a - =


==

that is

( -2(3);a ( -

-2a);{3 =
0 .

Thus ,-2 'a is a gradient and

'a =
h 2
,a

for some h. Since ,2 =

gtt and

a we have
t'

gtt
t h,a.

Putting a =
t we see that
h,t

1 or h =
t + f(x ). By choosing a new


time coordinate t' = t + f(x ), we have
CHAPTER 10: SOLUTIONS 321

gtt h gtt t' i 0

git'
= =


1 ,

also
/' ,

a a
JX
ga' =
g{3 =
JX _

(1,0,0,0)
ax{3 at

so
{3
is still independent of t

Solution 10.9. Choose a coordinate system in which the metric is


7J{lV
(all of the Christoffel symbols vanish). Then Killing's equation for a

Killing vector becomes


(11; v)

11, v
+ gv, 11 =
0 .
(1)

We now enumerate and classify linearly independent solutions of

Equation (1):

(i) Translation Killing vectors:

Sl
.Il =
a.1l

i=1-4 (2)

where a.

are constant vectors.

(ii) Rotational Killing vectors:

1km
g =
0, g =
E X

k =
1- 3 .
(3)

Check:


g =
x E.
k ( Il,v )

m, ( J
. .

k ( 1,3 )
1 )k m


0mO Ei)km -Ek(ij)
= = =

(iii) Boost Killing vectors:

Sil
k=O[Ok]
11
x k =
1, 3 .
(4)
Check:


[o[Ook]+o[Ook] ]
k o.
S = =

(11, v) 2 11 v v 11
322 PROBLEM BOOK IN RELATIVITY AND GRAVITATION

There are ten Killing vectors in Equations (2) (4), given -


in the coordinate

(-1, 1, 1, 1). In any other coordinate system


system in
which. gllv one

can find the components of the gf, for i =


1-10, by transforming them
as vectors.

Solution 10.10. The change in u. along the geodesic is given by

v u(n .

)(Vuu). + u .

(Vu .

Now V
un

0 since u is tangent to a geodesic, and u' Vu =
0 since

is a Killing vector. In components,

a f3 a f3
u V u u
a;f3 =
u u
(a;f3)


・ =

Thus u. =
constant along a geodesic.

Solution 10.11. The divergence of J is

JIl;1l

(TIlV gV);1l

gv TIlV
TIl 1l +
gV;1l

TIlV
0 +
g(V;Il) 0
= =

When =
a/at, then JIl =

Tllo = -
TIlO at infinity where goo =
-1.

Thus J is the negative of the energy flux vector for a stationary observer.

Solution 10.12. From Problem 10.11, the integrand .f ==


Taf3 f3 is a

conserved quantity:

Ja'a



O. The result then follows from the identity

J ,a

(\gI2Ja) ,a \g\-2 and from Gauss's theorem:

f Ja.,a \g\ t d
f (\g\ t j ) ,a d
f Jad31
4 a 4
o = x =
x =

L L
a 3 a 3

j d Ia -

j d Ia ・


2 Fl
CHAPTER 10: SOLUTIONS 323

Here we have assumed that J 0 sufficiently rapidly at infinity that the


contributions to the integral from the edges can be neglected. Thus the

integral is independent of the spacelike hypersurface F.

Solution 10.13. From Problem 10.11, we know that for each Killing vector

T l1
JI1
, ==
v,v is a conserved vector, i. e.

1 1

JI1.,11 =
\ g\- 2" (I g\2" J ) ,a =
0 .
(1)

From Problem 10.9, we know that in flat spacetime there are ten linearly
independent Killing vectors
i)' i =
1 10. Thus there are ten con-

served vectors
TI1V
JI1(i) ==
(i)v .
(2)

From Equations (1) and (2), one may construct ten globally conserved
quanti ties


1.
0 3
Q(i) ==
\ g\ 2
J (i) d x, i =
1 -

10 (3)

since

dQ(i)
J J
1. 1.
0 3 k 3

dt

(\g\2 J(i) ),od x = -
(lg\2 J(i) ),k d x


1.
k 2
= -
\ g\2 J (i) d
Ik .

This last expression represents the flux of the


J(i) current through the
(2-dimensional) surface bounding the integration volume. If T falls off
fast enough at large distances, and the integration volume is infinite,

this flux term vanishes and


dQ(i)/dt =
O. [Alternative proof: apply Solu-

tion 10.12.]

The four Q's derived from translation Killing vectors correspond to

conserved energy and momentum; the three derived from rotation Killing

vectors correspond to conserved angular momenta. To see the meaning

of the three derived from boost Killing vectors, write (e. g. in flat

cartesian coordinates with




[x, -t, 0, 0])
324 PROBLEM BOOK IN RELATIVITY AND GRAVITATION

Q =

f TOV vdxdydz

f xTOOdxdydz f TOXdxdydz t

(C.M.) t)
M tP M
(C.M.) (X (C.M.)

X =

(C.M.) (C.M.)
- -

x(C.M.)

which is a component of the conserved "origin of uniform motion" for the


center of mass (C.M.) of the system. (x(C.M.) is defined to be

f X TOO dx dy dz/ f TOO dx dy dz.)

Solution 10.14. The components of u are




ga /(_ySl:Y)2

(the minus sign is chosen to make the term in parentheses positive), so

a;fJ a
Y;fJ Y fJ
a =
u QU
fJ =

a 1 3
a;fJ 1-

(_ y (-
Y)2 Te)2(_ f1 f1)2

The second term vanishes because


Y (Y;fJ) fJ
Y;fJ fJ Y O. Since
= =

ga;{3 g{3;a' the first term gives


= -

fJ;a fJ 1 ( fJ fJ);a
i [log (- fJ fJ)]
= = =

aa

2 ,a
+gy gy (g gy)y

Solution 10.15. Consider a particle of rest mass JL and conserved energy

E where p is the 4-momentum and is the time Killing vector.


-P'

Not all values of E/ JL are possible for trajectories through a given point
in spacetime. For example, particles at radial infinity must have E/ JL 2: 1.

To find the bound on E/ JL for a general point, pick an orthonormal frame

at the point. The 4-velocity of a particle has components u =


(y, y.i)
CHAPTER 10: SOLUTIONS 325


(1- v ) -"2;

with v a 3-vector and y

the time Killing vector has com-

ponents =

(go' f), with


{ a 3-vector. The particle's ratio of energy

to rest mass is given by

Elfl. -u' (t) y(go-!'{) (1)


= =

where the dot denotes the scalar product in the local Euclidean 3-space.
Evidently, a necessary (but not a sufficient) condition for an extremum

(hence a bound) on EI/l is


v.g=ivg.


(2)

Here v ==
Iyl, g ==
1£\. Now we distinguish two cases: If is spacelike
(e. g. in the ergosphere of a Kerr blackhole), then we have
go < g; and

inspection of Equation (1) shows that all values of EI fl. are possible,
that is -
00 < EI fl. < + 00. This is the answer for regions in which is

spacelike. The infinite limits correspond to v 1 with the two signs of

Equation (2). Jf, instead, is timelike (e. g. at radial infinity), so that

go > g, then the right-hand side of Equation (1) is always positive, and

there is a nontrivial lower bound on EI fl.. Writing Equation (1) and using
Equation (2) with the upper sign, we obtain

(g2 + E2/fl.2)v2 -2 go (g _E2/fl.2)


v + =
0 .

The extremum in EI fl. is obtained by setting the discriminant of this

equation, a quadratic in v, equal to zero; this gives


o =
(EIfl.) [(EIfl.)2 _

g2 + g ]

The root EI fl. =


0 is spurious, and the lower bound on EI fl. is

2 2 2 ,:'
(EI fl.) =

go -

g = - ・

We see that the allowed range of EI fl. at a point depends only on the

norm of the time Killing vector at that point:


326 PROBLEM BOOK IN RELATIVITY AND GRAVITATION

( -

) 2" < E/11 < + 00

for timelike.

Solution 10.16. For a test electromagnetic field, all we have to check is

that Maxwell's equations are satisfied in the given background metric. If

All, the vector potential, is a Killing vector, then

AM =

;11

Le. the Lorentz condition is satisfied, and the wave equation

AIl;Vv _
Rilv A V =

is satisfied by Problem 10.6 and the fact that


RlLv =
0 in vacuum.

If in Minkowski space in spherical polar coordinates


'"
A ex
a/ acp, i.e.

only A cp =
constant is non then
ero, A cp ex r sin e, or in conventional

3-dimensional notation,


a r sin e (a =
constant) .

Hence 0 and V 2a (cos e sin e 2a Thus


E = =
x =

r- eO) =

.£ .
there is a uniform magnetic field parallel to the z axis. (This problem
is due to Robert M. Wald.)
CHAPTER 11: SOLUTIONS

Solution 11.1.

dJ/dr d( x) dp dp d( x)

(i) p + lix p
= _ _

dr dr dr dr

=u p-p u
1 1
P P-P o.
m P=

(ii) Consider times immediately before and immediately after the


collision. Since all particles are at the same point at the

collision,
x =
x = x
( k )1 before ( k )1 after ,

(where isxthe displacement to the point) and thus

( )
J (k) =
x p p (k) I x
I after

I after (k) after


(k) (k) (k)

( )
x p x
P(k ) I before

(k) I

before
(k) (k)

I J(k)lbefore '

(k)

since total momentum I p is conserved.


(k)
(k)

Solution 11.2.

(a) For the' 'angular momentum density" Ja{jy defined by

Ja{jy =. 2x[a T{j]y =




T{jy -
x{3 Tay

327
328 PROBLEM BOOK IN RELATIVITY AND GRAVITATION

we have

Ja{jy,y = oa

T{jy + x

T{jy,y -
0{3

Tay -
x{j
,y

T{3a
y -
rz{3 =
0 .

Thus J

{3 ==
J J {3y d Iy
a 3
is a conserved quantity, by Gauss' theorem.

{3(x
J (aaT{3Y-a{3TIY)d3Iy+Ja{3(xu)
a u U
(b) J +a ) =

Thus Ja{j is not invariant under coordinate translations (different angular

momentum about different points).


(c) Consider the time derivative of the spin:

J{3y duB
( )
dS

= _!


a{jyu
So' i3 + .

dt dt dt

The first term on the right vanishes from part (a). The second term vanishes

because the system has no forces acting on it (duo /dt =


0). Thus

dS /dt =
O.

(d) From (b),

Ja{j(x a
+ aa) =
aa p{j _
a{j pa + Ja{j(x a
) .

and thus

{3yB(aap{3-a{3I?a)pB/ 1P1
U U
Sa(XU+a ) = -



Sa(x ) .

The first term on the right vanishes (E is totally antisymmetric and p/l
a a
occurs quadratically) so
Sa(x + aa) =

Sa(x ).

Solution 11.3. From the definition of Sa'





= _!


afJYu
So' J{jy uou a

O a
by antis ymmetry of E and symmetry of u u .

Solution 11.4. Consider an observer in the local inertial frame comoving

with the center of mass of the gyroscope. Because there are no torques,
CHAPTER 11: SOLUTIONS 329

he sees no precession of the spin axis, so


d dt =
O. But in this frame

the 4-velocity of the gyroscope is u =


(1, Q), so we can write the condi-

tion of zero torque as



uS =
gu

where g is some constant of


proportionality. We find g from the fact

that S. u =
0 :
o =
V (S'u)


( Vu S). u + S. (Vu u)

gu.u + S.a

g + S.a
= -

Thus

V S


(S. a) u

which is the equation for Fermi-Walker transport

V S =
(S.a)u -

(S.u)a

with S.u =
O.

Solution 11.5.

(a) About the center of mass in the center-of-momentum frame:


l 00 3 .0
l 3
r x T d x =
0 (center of mass) and fT d x =
0 (center of momentum).

Thus we have


3 oO iO
JiO =
d x (xiT -
t T ) = 0 ,

or, in frame independent notation {lf3 uf3 =


O.

From the definition of


(b) So:


a {3yo S u =! E
a {3yo E ]ILVuGuS-'
y a
_


2 YILVG

Now perform the summation over Y [see Problems 3.27, 3.28]:

o JILVuUuo

- = -
jI-f3 u
uo

Jaf3 saf3 == .

[Only the J

{3 terms are kept in the sum; by part (a) we cannot have a

a index on J due to the presence of u



']
330 PROBLEM BOOK IN RELATIVITY AND GRAVITATION

Solution 11.6. First we calculate the total angular momentum. We work

in the center-of-momentum frame and take as our origin 0 the event at

which the center of mass of A and that of B would collide. About this

origin both systems A and B have only intrinsic angular momentum

(see Problem 11.5), so



{3(O) Asa{3 sa{3

+ = -


{3JLV (S u + S u )
A+B B If K #- If

and by conservation of angular momentum this is equal to J



{3(O). We

can now use this result in the definition of the spin vector to find


J a{3

Sa = - -



c 2 aafJ C C


E E a{3JLv (S U + S U


v)

a{3A

2 a JL v =u.
A A B B C

According to Problem 3.28 (see also Problem 3.27)


afJrlIv -2u JLV
Eaa{3A

aA
sot ha t

a JLVA (S )u
S U +
I.L u
= -


a a JL Av v
A B B C

-Sa(U'U) Sa(U'U) +
ua(S.u) +
ua(S.u)

A AC B Be A AC B Be

We can now substitute (P+P)/IP+PI for U to complete the solution.


A B ABC

Solution 11.7. The basic equation (see Problem 11.4) is

dS/dr =
u(a.S) .

In the lab frame, the world line of the particle is

x =
r cos wt



r sin wt
CHAPTER 11: SOLUTIONS 331

(where cu and r are constants) and hence

o x dx Y Z
u 0

u =
y, =


= -
cur y Slncu t ' u =
y cur cos cut, U =

dt

2 2-"2
where y ==
(l-r cu) . Now a =
du/dr so


o x dux Y 2 2 Z
a =
(y =
const), a =
y '
a = -
y cu r sin cut, a =

dt

and hence

ry2 ry2 sincutS Y


x 2 X 2
S.a =
SXa + SYaY =
_cu coscutS -
cu .

We have therefore

O O
dS dS 0
dr


dt
= u (a.S) =
y(a.S) (1)

X x
dS dS
dr
_


dt
_

uX(a. S) = -
cur y sin cut(a. S) (2)

dS Y dS Y Y

y =
u (a. S) =
cur y cos cut(a. S) (3)
dr dt

dS

_

0 .
(4)
dr

Introduce radial and tangential components:

SX =
Sr cos cut -
Sf} sin cut

SY =
Sr sin cut + Sf} cos cut

Equations (2) and (3) imply:


dS / dt = cu Sf} (5)

f} 2 r
dS /dt =
-cuy S .
(6)

Equations (5) and (6) imply:

2 2
y2 Sr

/ dt

d S = -
cu Sr =
A cos (cu y t+a), A, a const. (7)
332 PROBLEM BOOK IN RELATIVITY AND GRAVITATION

and from Equation (5)

sf} =
-yA sin(cuyt+a) .
(8)

From Equations (4), (7), (8) we obtain

sX =
A[cos cut cos (cuyt + a) + y sin cut sin (cuyt + a)] (9a)

sY =
A[sin cut cos (cuyt+a)- y cos cut sin(cuyt+a)] (9b)

sz =
const. (9c)

Consider initial conditions SX = n (2)-t, sy =


0, SZ =
11. (Although
we are not treating the electron spin quantum mechanically, we want


S2 =
311 /4.) These conditions imply a =
0, A = 11 (2)- t; and Equations

(9) can be written as

2- t 11 [e -i(y-l )cut
icut
SX + i SY =
+ i(l-y) sin (cuyt) e ]. (10)

The first term on the right hand side of Equation (10) and Equation (9c)
indicate a preces sion around the z-axis with angular velocity

(y l)cu 1. v

cu (11)
cuThomas
= -

while the second term in Equation (10) is small (1- y _! v



) for an

electron in an atom.

Solution 11.8. Do the computation in the local comoving Lorentz frame

of the center of mass of the body. Treating the center of mass as the

reference point in the equation of geodesic deviation, the relative accelera-


......

j is
tion of a mass element at position x

2 j ":'


-RJ..................x (1)
2 OkO'
dt

-EiijipRTokOxk,
th
The i component of the torque per unit volume is then
......


where p is the mass density at x . The total torque, which is just the

time derivative of the intrinsic angular momentum, is


CHAPTER 11: SOLUTIONS
333

dSr

":'
l' k 3
-EiljRJ 6£6 px d (2)

x x
dt

'"

if we approximate RhkO as constant over


'"
the body. Because of the

symmetry properties of Eilj Rj ako' Equation (2) is equivalent to


dt -EiljtfkRjOkO (3)

where


1 k 2 Th 3
8 )d x
t'ik ==
P (x x -
r ・

taf3
If one now defines the quadrupole tensor, to be such that
taf3uf3 =

i. e. to have only spatial components in a comoving local frame, then the


invariant tensor expression of Equation (3) is

DS K K{3all a A l1

t{311 R (4)

uJiu
_

aaA.

(iT

Note that either in the case of a spherical body, or a body small enough
so that Riemann x (size of body)2 0, Equation (4) reduces to

DSK/dr =
O. (Cf. Problem 11.4 with a =
0.)

Solution 11.9. We shall use two coordinate systems: the XYZ coordi-

nates are the spatial coordinates of a local Lorentz frame with spatial

origin at the center of mass of the Earth. The ecliptic plane is the XY

plane, and we shall assume that the Sun and the Moon move in circular

orbits in this plane as seen from the Earth. The xyz coordinates are

the spatial coordinates of a similar Lorentz frame, but with the z-axis

parallel to the Earth's angular momentum J. Choose the x-axis parallel


to the X-axis. The relation between basis vectors for the two coordinate

systems is
e =

-x -

1/"

.....y

cos 'fJ ev- -
sin tfr e z
i/fIIetItI

zsin tfr + cos tfr


y z (1)
334 PROBLEM BOOK IN RELATIVITY AND GRAVITATION

J.



Here tfr 23 is the angle between and the Z-axis, which remains
2 1
constant if we ignore the small nutation. As time goes on, J precesses

about the Z-axis with a period T which we wish to find.

Let.£s be a unit vector pointing to the Sun. If the sun has polar

0, cp) with respect


coordinates (r s ' to the xyz coordinates and (r s '
; '
cps)
with respect to the XYZ coordinates, then

cos 0 =

-z


-s


(sin 0/ + cos
Y 0/ (cos CPs
z). x CPs
+ sin e
y )


sino/ sincps

and similarly,

,I,.
sin 0 sin rf...
'P



e =
cos 'f' sin rf...
'Ps
_y -s

sin 0 cos cp =

£x '.£s

cos
CPs ・

From Problem 11.8, the equation of motion of the spin vector J of the

Earth in an external gravitational field is

dl =

dt
CHAPTER 11: SOLUTIONS 335

where the torque is

N.1 = -
E ..


t. R
kO m O

lJ Jm

Here the "reduced quadrupole moment tensor" is



p(xjxm_ o jm )d

t =
r x
jm

Take the Earth to be spheroidal, with moment of inertia C about the

rotation axis and A about an equatorial axis. Then in xyz coordinates


2 3
c =
p(x +y2)d x

f f
2 2 3 3
A =
p(x +z )d x =
p(y2+z2)d x ・

The nonzero components of t.Jm are


t = -
(C -

A)
zz


t =
t =
(C -

A)
xx yy 2
so that

Nx = -

(C -

A) R zOYO

Ny

(C- A)R zOxO

N = o .

In the weak-field, slow-motion limit,


a (I)
R =

zOiO
az axi

where (I) is the Newtonian gravitational potential:

(I) (I)

SUN

(l)MOON ・
336 PROBLEM BOOK IN RELATIVITY AND GRAVITA TION

Since

(l)SUN(X, y, z)

[(X_X )2 + (y_y )2 + (z-z )2]2


S s s

we find


a (I) =
3Ms sin () cos () cos cp
az ax 3
0 r

2 3Ms sin () cos () sin cp


a <I> =

az ay 3
0 r

Thus has
SUN (x, y, z) components

!i SUN = 3a (C -

A) sin () cos () (sin cp, -

cos cp, 0)

3a (C -

A) sinfjl sin CPs (cos tfr sin


CPs' -
cos
CPs' 0)

where a == M ,/r s3. Since the Sun's angular velocity about the Earth is

much greater than the precession angular velocity we wish to find, we can


the Sun's orbit. The term while
average SUN
over sin
fjJs gives ,

sincps coscps averages to zero. Thus the only nonzero component of

!!SUN is UN
N UNNi
= =
a(C -

A) sin Ifr cos Ifr .

A similar expression holds for the average torque exerted by the Moon,
with a replaced by b =

MM/r .Thus, at t =
0, there is a torque in
the X direction of magnitude

N =
(a+b)(C-A) sinlfr coslfr .

In a small time dt therefore J changes by an amount dJ =


Ndt perpen-

dicular to itself. Thus J rotates about the Z axis by an amount

dJ Ndt
dX = =

J sin fjI J sin fjI


CHAPTER 11: SOLUTIONS 337

The precession period is the time for X to change by 217:

T =
217 J sintP .

Put J =
Cw, where w is the angular velocity of rotation of the Earth.
Then
417 C 1 (i)
T =

3 C A -
cos tP a + b

Putting
C/(C-A) =
305.3

tP =
23.45°
5 1
(i) =
7.292 x 10- sec-




1.989 x 1033g
13


= 1. 496 x 10 cm

MM =
7.349 x 1025g
10



3.844 x 10 cm

we find

T =
25,600 years.

This agrees with observation to better than 1%; some error in the calcula-

tion arises because we treated the orbits as circular and because the

Moon's orbit is not exactly in the ecliptic.

Solution 11.10. The observers have 4-velocity u =


II I, where I I =

( _ .

) 2" . Thus


eo [ =
,u] =


[V (- . )-2"]
where we have used the fact that [ ,] =
O. But

V (

(. a )a);f3 2 f3a;f3 a f3 = =
0 ,


since
g(a;f3) 0 by Killing's equation. Thus O. Also
ej
= = =

eo
338 PROBLEM BOOK IN RELATIVITY AND GRAVITATION

since
ej connects points of equal t along any observer's world line.

(See Problem 8.14.)


(2) Q =

ft;(Qa...j3 ea '" ej3)

(ft;Qa...j3)ea "' ej3Qa...j3(ft;ea) "' ej3


'"
'"

+ ...
+ if'" f3 ea '"(ft; ej3) .

Only the first term on the right-hand side is nonvanishing by part (1), so

(ft; Q)a...j3 ft; (Qa...13)= = V


t;
Qa'" 13 =

:t Qa... 13
(since Qa...j3 is just a scalar function). Measured in units of proper

time, the rate is

.E.. Qu'" =
V if...j3 = -L V Qa...j3 = -LJi Qa...j3 .

I I dt

dt I I

(3) In the observer's local rest frame, the precession equation is

dS "'

" '"

J k 1
E",=,"''''Ct) S
Jkl
- =

dt
'"


where S is the gyroscope spin vector. Now u so
Ejkf EOjkf EUjk1'
= =

the 4-dimensional version of the precession equation is

'"

(3
'"
" "",


U'" E
a {3yo Ct)'" S'"
dt

yo
that is

13 '" '"

dS u {3yo
dt

g",a E Ct)"
yo'
S'" (1)

We wish to derive an equation with the form of Equation (1) and read off
w. We know that S is Fermi-Walker transported (see Problem 11.4), i.e.

v S


(S.a)u (2)
and
CHAPTER 11: SOLUTIONS 339

S.u =
O. (3)
Equation (2) becomes


v 5 =
(5. V ) I /
1 (4)

and Equation (3) implies

S. =
0 .
(5)
From part (2),
dS Cc>
S S V
= =
V -

s (6)
(it

Since plays the part of a 4-velocity (unnormalized), it is convenient

to decompose V analogously to the decomposition of Vu (see Problem


5.18). Since is a Killing vector, V is antisymmetric, so we write

V =
w + A -

A (7)

where w is anti symmetric and w. = O. Since

(V )


V = - .

(V )
we must have

A =

(V )( .

. )-l
Thus Equations (6) and (4) give

dS -1
= _
(S V.

) (

) + S. V = S. w .
(8)
dt

(In the first equality we used


VS =
(V )'
S -S.V

.)
We now want to

express w in terms of a vector as in Equation (1). It is convenient to

work in component notation to keep track of which slots are contracted

into which. Equation (7) is

ga;{3
= w +
Aa g{3 ga A{3

a {3

and from this we can get a vector independent of A:


340 PROBLEM BOOK IN RELATIVITY AND GRAVITATION

A Aa f3y Aa {3y
B :=
a;{3 y E W
y (9)


{3

Express w in terms of B:
a {3


A a{3y
E -w
{3SyOpaK

ApaK a


E B S
_

w oa{3y S K
{3 Sy

ApaK

a paK

Since
Wa{3 {3 0, =
the only non-vanishing terms occur when y =
K, so

EApaKBA K -2wpa K K =

Substitute in Equation (8),

'"

dS{3 =
s J;S _12

s EraS K ByS'K
J: e: 8)-1
dt a a Su

which is of the form of Equation (1) with

wy = ! BY (g
2 a
a)-l

Eyaf3a
and, by Equation (9) this is equal to
a;{3 012 aa.
(4) The vanishing of w oCCUrs if and only if
ga] O.
[a;{3

But this

is the condition that be hypersurface-orthogonal (Problem 7.23) i. e.

that the metric be static. (See Problem 10.8.)

Solution 11.11. The equation of transport for the gyroscope spin is


DS a

(S. a) u , Sa
ua

0 .
(1)
dr

th
The locally measured time derivative of the j component (relative to a

local Lorentz frame) of the spin vector is

£. .i (S. D D
&-

dr J )

dr
(S'eJ ) S. dr = e

dr
'" '"


s.rloea =

r osar1j6s =
(2)
CHAPTER 11: SOLUTIONS 341

where we have used Equation (1) and the definition of the Christoffel
symbols. Now, using the fact that the Christoffel symbols, in an ortho-
normal frame, are antisymmetric in the first two indicates, Equation (2)
may be written as ;A..

dS j
EjkfDk Sf (3a)


dt

where
n;A.. = ! E """, r ",...,..,;A..

(3b)
ltk klj IjO'

We must compute the Christoffel symbols,


now
rljo. The approximate

metric of the earth is


4h. dx j dt

ds =
-(1+2cP)dt + (1-2cP)0'J k dxidxk -



(4)
Here

cP = -
M/r =
0(£2)

Jxr 3

h = -=--=



0(£ ) (5)

where J is the angular momentum of the earth. We are working in the


Newtonian limit, where velocities are (9(£), and will keep terms of order

£3. A coordinate-stationary observer in the metric (4) has orthonormal


basis I-forms
'7 ...........

wO =
(l+cP)dt + 2h.

J;j, C:;J =
(1- cP) dx


(6)
- -

The dual basis vectors, found from the relation < a,



{3
> = o
a{3 are

eO

(1- cP) a/at, e


(1+ cP) l..: -
2h. a/at


(7)
ax J

If the gyroscope has coordinate velocity v., related to the 4-velocity by



J 0 O 2
u = v. u , u =
1-cP + v (from u'u =
-1), (8)
J 2

then in the stationary observer's frame


342 PROBLEM BOOK IN RELATIVITY AND GRAVITATION

j -7" 0
< wJ

v = =
, u > /< W , u
> =
(1- 2cP ) v.J .
(9)
J 0

The basis vectors in the orthonormal frame comoving with the gyroscope
are obtained from those of Equation (7) by a Lorentz transformation, a

boost -
\17:

e = Aa e-
a a a

where
":'

2 o
ADO 1 + v A ' -
AJ 0

Y ==



, . = v!

J J


(10)
Ajk = ojk + (y -1) v



/v

This gives

)a/ at (1- I/> )v


2 k k

(1- I/>

eo
= + v + + v a/ax

(11)

[ 31/>+- )V -2hJa/cft +[Ojk(1+1/?+ vjvk]a;aX


2 k

e:J
= ・

The Christoffel symbols in an orthonormal frame can be found from the

formula 1
r = -
(c /lva + c -
c )
/lva 2 flaV vall
(12)
c =
[ e e ]
va'


/lva /l'

Using Equation (11) and working to a((3), we find

(- I/>,j Vj,k)a/ at
2 2

[9' eo] + V v v
vj, t

- - -

,j j , t k

2 2

1 1
+ V .-cP
k ,j,j
.v







k+ 2 v v
k,J ,+v,vk
J, t

Vk),m] a;ax

+ v V
j m Vk,m

(Vj ), t V



ml/> ,mO jk -

m (Vj .

Since (a/ax ).

(a/ax{3) =

{3'
we get
CHAPTER 11: SOLUTIONS
343

[ ,e6] k e

(1-1> i V Vk,j i vjv




+ + v + v v +
k vj, t j k, t m vk,m + v
m vkv j,m

111
(vjvk),t


vmVkVm,j VmCPf Ojk (vjvk),m


2 m


Thus

[ej,e6]' [ef,e o ]' 0-1> i ) (vk,j


e 2
k vj,k)


+ v -

(13)


2 vm(vkvj,m-VjVk,m+vkVm,j-VjVm,k)
and

[( v2\
1 1
[e"'J ,ek]


1- 2cp + 2 v + v,v -

31> ,j,v k + 2 v2. v 2h k



. -

k ,J k t

J, ,j k ,j


i vjvmvk,m] aNt
[1>,lkm i (vkvm),j v/vkvm),t] ajax

+ + + -

/j kl
so


[J e
k] ( 1 cP + 2 v2 (v.J, k
) .) + 4(cP ,j v k cP k J) 2(h k ,J h.

eo

v v +

k)
' -
- -

k ,J
. .

, J,
(14)

+ v
k vj,t- Vj v k ,t+ 2 vm(VjVm,k-VkVm,j+vkVj,m -vjvk,m) .

The Christoffel symbols are then


jk6

i ([ ,ek]'o [ej,e o]' ef- [ek,e O]' ej)


e +

dv.
(15)

( )
J dv k


2( cP ,J V k

cP k
v .) + h
k ,j


h.

+ v


V '

, J J, 2 dr J dr

where
dv. avo avo

== ---2 + v =
-cP , J .
+ a. .
(16)
dr at m
m J
ax

Here a is the acceleration produced by non-inertial forces; we get


a =
0 for geodesic motion. Thus

rjk6 =
(1) ,j v k -1>,k V j) +
hk,j hj,k (17)

344 PROBLEM BOOK IN RELATIVITY AND GRAVITATION

and Equation (3b) can be written

Q = h x

<I>xy, (18)

Had we not set =


0, there would have been an extra term x

which is the Thomas precession.

The term <I> x


Y is called the "geodetic precession." For a

particle in a circular orbit of radius r,

("\ 3 M
( V fJ fJ )
__ __

l£ = - -
e -

V e
cp

-geo d etlc 2 2

where
1.

2 2
M 2 (sin2fJ
( )
v =
_

sin a)
(J r
sin fJ


M 2 sina
v- =
( ) r
sinfJ

are the spherical components of the particle's velocity and a is the in-

clination of the orbital plane to the polar axis. To this order, it does not

matter what we use, indices with or without carets. In order of magnitude,

3 M 2
( )
("\ M "
£. "'"'W - -
"'"'W
8 per year
g 2 R 2

where R is the radius of the earth. The term independent of v is the

Lens- Thirring preces sion:

Q :YL


1- J

L T
= -

J+ "'"'W "'"'W
0.1" per year.

..
3 -
2 3
r r R

This term could also have been obtained from Problem 11.10 since, to

ord er f ,
jOkl

[k,
"'-

oj _
oj R:: 1]
2 Y y
CHAPTER 11: SOLUTIONS 345

Here
0 =
1, =


-(1+2cp) and g.J = -
2h.

so

'"

oj O=Vxh.
Ejkl h1,k'

CHAPTER 12: SOLUTIONS


Solution 12.1. Equating gravitational acceleration Gm/ r to centripetal



acceleration (U

r gives Kepler's law: (U =
(Gm/r )"2. The average


density inside the sphere of radius r is p =
3m/417 r , so (U =
(417Gp/3)"2.
This result means, for example, that a grain of sand would orbit a steel

ball bearing at the ball bearings surface with about the same 90 min.

period as a satellite orbiting just above the earth's surface.

Solution 12.2. Spring tides occur when the sun and the moon are in the

same line as the earth (new moon or full moon); neap tides occur when the

sun and the moon are at right angles. Let the height of the tide be h.
For a rough estimate, take the sun and the moon to be in the equatorial

plane. An ocean element on the equator at high tide is in equilibrium



between the earth's gravitational acceleration g(r) = -
M m /r at a height
r =
r B +h (M B =
mass of earth, r B

radius of earth) and the tidal accelera-

tions of the s un and moon relative to the center of the earth:

sun moon
O =
g ( r B+ h) + r B R 2020 + r B R 2020 .
(1)

An element 90° away in longitude experiences low tide:

sun moon
O =
g (r h)-+ r B R 1010 + r B R 1010 .
(2)

tidal bulge
,/ \j
Ji;u- "

_
--J '-


e -")
...."')
I J('\ ,..
C'
'--
-'..

Earth Moon Sun

346
CHAPTER 12: SOLUTIONS
347

The difference of these two


equations gives us

o =
2h g'(r B ) + sun
(R 2020 R
sun

moon moon
r + R ) (3)
_


1010 2020 1010


Note that if we had put in centrifugal forces,


they would have cancelled
in Equation (3). Compute the Riemann tensor in the Newtonian limit (see
Problem 12.12):

2 un


sun

a <1>s = --
a M0
1010 2 2 2
)2] t
a x ax
x=y 0
[(X-X s un
)2 + (y-y sun
)2 + (z-z sun

and similarly for the other components. This gives


sun 3
1010 (x sun-
-z -0 y
sun- -R)
sun- ,



/R

sun 3
1010 ( Ysun Zsun -0
R -2M
-R) 0 /R



- - -


sun

R O(xsun=zsun=O,
Ysun=R) = -
2M
0/ R3
sun 3

2020 ( Ysun-zsun-
-0-



sun
-R)
- =


/R

and similar expressions for the moon. Spring tides occur when the sun

and the moon are in a line, say both on the y-axis. Equation (3) then
gives 4

( )
M M r
3 moon .-!
h .

+ =
39 cm .

s prln g
4 33M EB
R R
o moon

Neap tides occur when e. g. the moon is on the y-axis and the sun on the

x-axis. This gives


( )

h =
3 moon
_
M0 =
15 cm .

neap 4 M EB
R3moon R30

The actual tides are, of course, considerably larger than this in many

places because of hydrodynamical effects (like" sloshing" of water in

shallow seas and against irregular coastlines).


348 PROBLEM BOOK IN RELATIVITY AND GRAVITATION

Solution 12.3. To good approximation the solid earth responds linearly

to the tidal driving force of the Riemann tensor. (This is not true for the

hydrodynamic ocean tides.) Thus the fourier spectrum of the earth tides
will be the same as that of the Riemann tensor at earth. Since (in Newton-

ian order at least) the tidal force is linear in its sources we can treat the

sun and moon separately. The gravitational potential of a mass M at a

position x, y, z in geocentric coordinates is




U =
M/(x + y2 + z2)"2 .

The Newtonian components of the Riemann tensor are thus

(7-1)
2 2 M 3X
R xOxO =
a U /a x =


(1)
3XY

(7)
2 .


R =
a U/CJxdy =

xQyO 3

etc. for YY, zz, yz, xz. In the frame of the rotating earth, the apparent
orbit of the sun or moon is given by the standard formula of Keplerian

spherical astronomy (see e. g. MTW, p. 648)

x =
r cos (w + v) cos cpt -
r sin (w + v) cos ( sin cpt



r sin (w + v) cos f COS cpt + r cos (w + v) sin cpt

z =
r sin (w + v) sin f . (2)

Here w and f are constants, the longitude of perigee and inclination to

the equatorial plane, respectively. The angle cpt is the hour angle of

the rotating earth,


2"
cpt =

1 sidereal da y

r is the "radius vector" and v is the '(true anomaly," given by

r =
a(l-e cosE), cosv =
(cosE-e)(l-e cosE)-l

where a is the semimajor axis, e is the eccentricity and E is the

eccentric anomaly defined by


CHAPTER 12: SOLUTIONS
349

217
E -
e sin E =
t -
Ot

and where T is the period of the orbit. Working only to first order in the

(small) eccentricity e, we compute

E =
Ot + e sin Ot


cos E =
cos Ot -
e sin 0t


(3)

r- =
a- (1+3e cosOt)

cos v =
cos Ot + e cos 20t -
e .

Now substituting Equation (3) into Equation (2) and substituting the re-

sui t into Equation (1), we get

R =
x (constant terms + time varying terms) ;
jOkO

the generic time-varying term has the form (here scos means either cos

or sin and an exponent mN means any integer between zero and m

incl usi ve) 2


sin f or

2 N
3(scos 0t

2e scos Ot + 2e scos Ot scos 20t) x sin f cos f scos
cpt or

2N 2
cos f scos cpt .

This expression is analyzed into individual frequencies by repeated use

of the trigonometric identities for products of scos's in terms of sum

and difference frequencies. The following components result:

Angular Frequency Amplitudes (leading order in e)



(i) 2 :r
COS2N(

(ii) sinE COS (

(Hi) 20 t 2c/J II"
COS2H(

(iv) 20t sin ( COS (


2H
(v) 2G } sin ( and COS (

2H
n t 2c/J 3e COS (
(vi)

(vii) 0 t 6e sin ( cosH I


2 2N
(viii) n 6e(sin ( or } cos t}
3 e COS 2H (
(ix) 30 t 2 7"


(x) 30 t. 3e sin f COS


30 3e sin (
(xi)
350 PROBLEM BOOK IN RELATIVITY AND GRAVITATION

For the Earth-Sun system: 23to,



e =
.017, Q =
211/1 year, and rP =

211/1 sidereal day. For the Earth-Moon system e =


.054, n =

211 /1 sidereal month, rP



211/1 sidereal day, and varies between

23to :t 5° due to the 18.6 year nutation of the moon's node. (This induces
further splitting of the spectrum that we have neglected.) With these

values, and using the fact that (M/ a ) is 2.2 times larger for the moon's
orbit than for the sun's, we get the following estimate for the ordering of
former components by strength (M =
moon, S =
sun):

M(i), M(v), M(ii), M(iii), S(i), S(v), M(iv), S(ii), S(iii), M(vi), M(viii)'" .

Solution 12.4. Laplace first realized that there must be a difference in

positions. The position of the sun in the sky is displaced by the finite

velocity of light; in other words, is aberrated by the velocity v of the

earth in its orbit, by an angle () =


v/c. The Coulomb gravitational field
is not aberrated: If the force of gravity pointed in this direction, there

would be a component of solar acceleration (GM 0 /r ) (v/c) in the direc-
tion of the earth's motion, so the earth's energy would increase at a rate

given by
d(E/M(B) =
GM
0 v

dt 2 c


but E/ME!) GM the energy loss rate implies dr/ dt 2v / c.
0 / r,
= -
so =


Since GM this is easily integrated to give
0/
v =

c 2 2
t -

to =
(r -

ro ) ・

4GM

13
In particular, the earth's orbit has r =
1.5 x 10 cm, v =
30 km/sec, the
10 4
radius of the sun is 7 x 10 cm, so () 10- and t
to
ro
= -

10
1.3 x 10 sec 400 years. This is much less than the known geologi-

cal time during which the radius of the earth's orbit is known to have been

cons tan t.
CHAPTER 12: SOLUTIONS
351

Solution 12.5. Light pressure within the s tar is given by P


ra d .

(1/ 3)U ra d .

where
Urad. is the energy density of radiation. Since the light diffuses
outward in many scatterings, U satisfies a diffusion equation and
ra d .

the radiative flux F is proportional to a gradient in U


ra d .

F='=£'VU rad.
3Kp

where K is the opacity (given here by the Thompson value K



aT/m p

/gm). The


.4 cm relation L =
417 r F between flux and total luminosity

L of the star gives


KP Ldr
dP = --

ra d .
417C 2

which is the net light pressure on a unit area slab of matter of thickness

dr. We equate this to the downward gravitational force -

GMp dr/r to

get the Eddi ngton I imi t

38
417,?Mc
(MM )
L = =
1.38 x 10 ergs/sec.
T 0

Microscopically, we could also get this result by noting that since the

scattering cross section is forward-backward symmetric (oc 1 + cos 0), a

photon of momentum p deposits on the average a momentum p to the

electron per collision, and a radial component The energy of the


Pre

photon is pc, but its contribution to the outward luminosity is reduced


by the factor v

/c =
p r /p. Thus the radial momentum deposited to an

electron at radius r per unit time is just proportional to the net radial

luminosity f. (constant at all radii)

La

(momentum)
( )
L x
x =

aT

( tim e) 417 r

(1iUJ) 417 r

equivalent radial cross section momentum


photons per area per photon
per time
352 PROBLEM BOOK IN RELATIVITY AND GRAVITATION

Momentum per unit time is a force, and we equate this to the inward gravi-
tational force on the proton which is associated with the electron:

LUT GMm


2 2
417 r c r

which gives the same result as above.

Solution 12.6. Let cpg and cpe be respectively the gravitational and

electrostatic potentials. The potential energy of an electron is then

(I) =
mcpg + ecpe ,

(m, e =
electron mass, charge) and the force on an electron is F = -
V(I).

In a static situation there can be no force component tangential to the con-

ducting surface, and hence (I) must be a constant on the inside surface.
2 2
Now there is neither mass (V cpg =
0) nor charge (V cpe =
0) inside the
2 V2(1) 0
container so V (1) =
O. The solution to Laplace's equation =

ins ide the conductor equipotential is then (I) =


constant. If an electron is

then introduced into this field, it will feel no force. [Note: Since the
electron is. introd uced at the center of the container, image-charge forces
can be ignored by s yIhmetry .]

Solution 12.7. As the contents are heated, mass-energy is added accord-

; ;

dM kN dT ( since air is diatomic; N is the total number
ing to c =

of molecules in the cylinder). However the center of gravity of the gas

rises which acts to decrease the weight (distance to earth's center in-

creases). The position of the center of gravity is:

h h

i PZd1 pdz p Po exp (- :; z)


Zcg
= ,

so the chan g e of z with temperature is given by


cg
CHAPTER 12: SOLUTIONS
353

dz

[l-( YJ
cg mgh
dT

:g Sin ' 11 ==
2kT

where m is the average mass of an air molecule.

The weight change is thus

) ( J)
dz

( [
dM Cg 7 gkN 2g l 1-
dW =
g + M dT = _
mN dT.
dT dz dT 2 2 Rm mg 2
sin h

cg c Q7
11

The right-hand side is negative when


11 7 g R(B 10-

1 _
> =
.122 x .


sinh211 c2


This implies that 11 > 6.05 x 10- and that h =
2kTl1/mg > 218 cm.

Solution 12.8. Since Poisson's equation is U,kk



417
Po
we have


T '

Jk k ,
= _
(U U
417' J'k , k
+U .U k k-U n 'U
"J " J
n) =

PoU , J . ・

Thus the Newtonian equation of motion can be written as

dv. at
J jk au-a
Po -
= _
- -

Po

--:

(T.k+t. k) k
J,

(1)
dt J
axk ax J axk

Now use the Newtonian equation of mass conservation

apo/at +
(POV ),k = 0 . (2)

From Equation (2) and the expression for the convective derivative

write
d/dt =
a/at +
vka/ax ,
one can

( )
dv. av.
J J k'
Po dt

Po dt
+ v
vJ,k

(POv j ), t +
(po vjv k) ,k

and thus Equation (1) can be written as

(p 0 v j) + (T j k + t j k + Po v j v k) k

0 ・ (3)

t ,
354 PROBLEM BOOK IN RELATIVITY AND GRAVITATION

Solution 12.9. Consider a virtual displacement £; the energy must be

stationary in .£. If we include in the energy, the gravitational


potential
energy of the work done on or by forces, then the change in energy for any

one force is
dE. = -
F. .
a + (dE.) (g/ c )
2 .
x.
1 -1 -
1 _ _1

and the change in energy from raising the object mass is

dE
M £'
= -

Thus


g..x.
I F. Mg. a
_1
a 1-
dEtotal

= - .

-1 -
2 _

for any a and hence

g' x.


F.
_1

( 1- =--=


) = -
Mg
_

Note that by taking the gravitational potential relative to the center of

gravity, we avoid having to add the gravitational potential energy of the

gravitational potential energy. (This argument is due to K. Nordtvedt.)

Solution 12.10. From the previous problem


F.
_1
(1-g.x./c ) _
_1

-Mg;

If p is pressure and dA is directed surface area, then E =


pd and

the above result becomes

or
-f p(x)(1-!'!ic2)d =

-M

f f
.!

pdA .--..
+ pg.

dA = -
Mg .......,

for g constant. Now with



= -

G r
r'.!

dr ,d = :t
dA r'we
have
CHAPTER 12: SOLUTIONS
355

JP
GM
-A[p(r+dr)-p(r)]e
-r

dr e
22-r
dA =
-M g =
Adr p
GM

c r
_
2- r

and thus
dp GM

( )
= _
p +
.E. .

dr 2 2
r c

This result holds in any theory of gravity that satisfies TflV. v =



(such theories are called metric theories) and is independent of the field

equations. Each theory, however, gives its own prescription for finding
M(r), which is defined such that the local acceleration of
gravity is


= -
GM/ r .

Solution 12.11. The preferred coordinates of Newtonian theory are

"universal time" t and Galilean space coordinates x



. The equation of

motion
2 j
d x
= -
a (I)
....----


dt ax j

can be written in 4-dimensional form:


d t =


dA

2 j 2
d a (I)
(d )

+ = 0 .

2 d/\
dA ax j

From this we read off the Christoffel symbols:

a (I)
rj all other
r{3y vanish.
- .

--:-,

00

ax

The standard formula for the components of the Riemann tensor gives


a (1)
. .

RJ

R JOkO - -

all other R vanish.


{3yo


OOk .



ax ax
356 PROBLEM BOOK IN RELATIVITY AND GRAVITATION

Suppose these were derivable from a metric, then we would have



a a cI>
gja R

jOkO OkO

g. ,
Jm m k
ax ax
but



gOa R o -R
= =

OjkO jkO jOkO


This violates the antisymmetry property has if it is derived from


Ra{3yo
a metric.

Solution 12.12. Let the Newtonian test-particle trajectories be

xj =


(t, n) .

Here n is a parameter telling which trajectory we are considering. The


vector

a ax a k a
n =
=---=n
an an k k
ax ax

is a connecting vector between neighboring trajectories. The relative


acceleration of neighboring trajectories is

2 j 2 2
axj a2xj
( ) ( )
a (1)
( ) ( )
a n a a a a (I) k a a (I) k
= = -
n n
_

= -
= -


2 2
at2 at an an at an ax j axk ax j axkax j

2 j 2 j
where we have used the equation of motion a x /at =
-acI>/ax . The

equation of geodesic deviation is

2 a
D n
= -


ny u f3 i>
f3y8


dr

In the Newtonian limit (velocities""'" (, gravitational fields,....., (2)

uO =
l+(:J(f), u
j =
(:J(f), r

f3y

(9(f2)

and n, which connects events of equal proper time, connects events of

equal coordinate time to (:J(f). Thus, 2 Jt =


+ (:J(f) and so
CHAPTER 12: SOLUTIONS
357

2 j


RJ

-=


OkO
n + aCE) .

dt

The Riemann components are then


a (1)

RJ -

OkO
ax ax j

in the Newtonian limit. Note that, unless the velocities involved approach
the speed of light, the other components of
R{3yo do not enter into the

equation of relative motion of test particles.

Solution 12.13. Newtonian gravity is characterized by instant "action at

a distance," where simultaneity is defined by the slices of a "universal


time" field t. We want a
relativistically invariant expression for the

gravitational acceleration at a point in terms of the matter density Tfl
in the rest of the universe. Spacetime is taken to have the Minkowski

metric everywhere. Only TOO should enter, so we first dot into two
universal time vectors:

Vt T Vt t T

{3 t
(or
,(3)
・ ・

,a

Thus the inverse square law acceleration of a particle at position x

becomes


Vt.T(x'). Vt 3 '
a =
G I d x
grav 3


(I 1)"2


constant


where I is the spacelike vector connecting x' to x and d x' is the

proper volume element on the constant t slice. This is not yet con-

sistent with special relativity, because a is not necessarily orthogonal


to the particle's 4-velocity u. We must apply a projection operator,
giving a final answe r


grav


j Vt.T'3Vt U+(u.l)U]d3x,.
(1.1) "2
358 PROBLEM BOOK IN RELATIVITY AND GRAVITATION

This is manifestly consistent with special relativity because it is (i)


written in terms of geometrical objects, and (ii) defines a geometrical
object (a with a.u =
0). Signals can be sent faster than light by changing

the distribution of masses at a point. The gravitational field then changes


on surfaces of constant t. Since these surfaces are spacelike, they are

outside the lightcone of every observer. Nevertheless the theory is not

acausal; since every observer's worldline is timelike, t is always in-

creasing in his future direction. Likewise, t increases along any light


ray he emits. So any signal can at best connect events of constant t; no

sequence of signals can return to an observer at an earlier t than they

started. This is to be contrasted with the case of tachyons (Problem 1.6)

which are acausal. The reason for the difference is that this problem has

some' 'pregeometry" in the scalar field t, which slices up spacetime in


a universally time-ordered fashion. Tachyons "slice" spacetime in a

manner that varies from observer to observer.

Solution 12.14. Take the separation as ."



(separation) -z
e ; then due to
_

gravitational forces

d 11z R

u {3 l1Yua
{3 Ya
- = -


dt

Since the particles start from rest, if we confine our observations to short
j so that
time, u 0,

d 'YJz
., z z
= -
R Oz 11


dt

The Riemann component for a weak field is easily calculated

L 2M
( )
Z M
R Oz 0
= -


= _

2 3
aZ Z

Due to the E field, there is an acceleration of the lower particle


a = -
Ee
-z

CHAPTER 12: SOLUTIONS 359

Thus


d 7JZ 2M q


7Jz -



dt z

It may disturbing at first that the two terms look as if


seem
they can be

comparable. But suppose our laboratory is limited to a chunk of space-

time xll L. Then at the end of the experiment the gravitational forces

will have caused a change in 7Jz:

1. 2M 2
07JZ '-


7JzL .

z3

But l1 < L so

B(L3)

0l1 = .

The change in 7Jz due to the electric field is

q !. 2
B(L2)

0l1 '- _



E .


L =

so, if we sufficiently limit the "size" of our experiment we can ignore

gravi tational effects.

Solution 12.15. The geometrical object which tells about the charge is a

scalar, vector, 2-tensor, etc. for spins 0, 1, 2, etc. For example, the
charge density of a particle of rest-charge qo moving along a worldline

xll =
zll(r) with 4-velocity u is


U 4
p(X ) =

qo 0 [xlL -

ZIL(T)] dT (scalar)



JIL(xU) =

qo u li 0 [XU -
ZU (T)] dr (vector)
(1)



TILV(x )

qo ulLu 0 [XU

ZU(T)] dT (spin 2)



TW"'P(x )

ulLuv ..
.u P 0 [XU ZU (T)] dT (spin s)
qo

= ・
360 PROBLEM BOOK IN RELATIVITY AND GRAVITATION

The charge of a particle in a laboratory volume, measured by the

"Coulomb" part of-its interaction in the lab is


00".0 3


T d x

(la b frame)

J rzl3"'Y u!;AB uhAB...




(_I)S AB
d x .
(2)

Substituting Equation (1) into Equation (2) gives the measured charge of
a single particle

J J

q= qo d x dr (_u.uLAB)s 8 4 [xa_za(r)]


3 4

qo d xdt (_u.uLAB)s
( i) 8 [x

_za (r)] ・

LAB
Integrating out the delta function, and using u.u =
-y and dr/dt =

s-l 2
(1 +T )
s-l
1/y, we get q=qoY qo v f or v?c. Now f ora

perfect gas we have < v > =
3kT
/mo' So when we sum the charges of

all the partic les in the gas we get

[1 ]

e )

Q =

nqo + J

and from the temperature dependence given in the problem we now read

off s =
5.

Solution 12.16. Suppose, to the contrary, that there were another s


pin-2
field. Since it is massless, and has infinite range, it must admit weak,

plane waves as solutions. Also since the field is spin-2 it must couple

to a symmetric, traceless "charge" tensor Jfl .
Moreover, for weak
fields and slow velocities, "charged" particles which are much smaller

than a wavelength must couple predominantly through their JOO (Le. the
"Coulomb" limit).
CHAPTER 12: SOLUTIONS 361

Consider a plane wave incident on two particles (initially located at

the same point) of charge JOO =

ql' JOo =

q2. For a sinusoidal plane

wave of a definite linear polarization the particles must execute linear

sinusoidal motion whose amplitude is proportional to their charge and

inversely proportional to their mass. Hence, at an instant in time, we

can define a polarization vector (analog of the electric field E) for the
field -1
ql q2


==

( -



- -



) (x 1
--
-x
2 )

which is independent of the numerical value of ql' q2' m


1 ,


. But this

is a contradiction! Since the field has spin-2 structure, a polarization


unchanged by 180

state should be

rotation, but under 180 rotation





implying V


O. What is wrong? We assumed that the particles

responded differently to the field. We now see that .!1 =.!2 for all

charges and masses is the only possibility. This is equivalent to saying

that all particles have a fixed charge-to-mass ratio: q/m =


constant. By

choice of units we can make q



m. Hence, in the weak field, plane-wave
OO
limit J =
TOO, the stress energy. But by Lorentz invariance we must


have JflV =
Tfl (e.g. we cannot introduce Tfl since this is not spin-2).
fl

We have shown that our "arbitrary" spin-2 field is coupled to Tfl .

Hence, it is just a piece of gravity, i.e. whatever we measure experiment-


ally as gravity. (No claim that it is general relativity is made.) R. P.

Feynman has pointed out that the assumption of a "point" particle, with
no intrinsic polarization vector, has entered crucially. Thus it is possible

to have the field couple to individual particles with spin; but for un-

polarized bulk matter, there can be no net force.

Solution 12.17. In c.g.s. units we have

10
c =
2.998 x 10 cm/sec
2 20
c =
8.998 x 10 erg/ gm
2 28
G/c =
0.7425 x 10- cm/gm


G 2" =
2.582 x 10- esu/gm .
362 PROBLEM BOOK IN RELATIVITY AND GRAVITATION

(The value of G 2 comes from the fact that in Coulomb's or Newton's


law,
2 2 2
force =
e /r or Gm /r .) Thus, in gravitational units, the values are

1r=1J' G\ (J\ 1rG (ergsec)(cm/gm)(cm/sec) 10-66cm 2


(a) c =
2.611x

&\c J \c2J c

(erg/gm)

G .1... (esu)(cm/gm) eG2 1.381x 10- 34 cm
(b) e e

( ) ( Gt)


2 (esu/gm) c

e e (esu) 21
(c ) 2.042 10
'-

_



(gm) (esu/gm)
G2m

GM

M0 : (gm)(cm/gm) 1.48x 10
<:><
(d) = <:>< cm

(erg/sec)(cm/gm) 0l7x10- 26
( )(c2Jl\ ( )
G l
()

L0

L0 C
<:><


2 (erg/gm)(cm/sec)

( )

(300 K) k
0 c COK)(erg/OJ()(cm/gm) 63
(f) 300 K =
<:>< <:>< 3.42x 1O- cm


2 (erg/gm)

17
(g) (1 year) =
(1 year) c =
9.460 x 10 cm

(erg/esu)(esu/gm)
( ) Gt ( 1.)
1 erg
(h) (1 vOlt)= 9.58X10-28.
299.8 esu 2
(erg/gm)

Solution 12.18.


33
L =
G/c3)2 =
1.616 x 10- cm


44
T* =
{trG/c

)2 =
5.391 x 10- sec



M =
(1rC/G)2 =
2.177 x 10- gm.

These combinations were noticed by Planck, immediately after he dis-

covered his constant. They are now called the Planck length, mass, and

time. They are uniquely defined up to factors of order unity (e.g. using h

instead of 1r), since by dimensional analysis there is only one way of


CHAPTER 12: SOLUTIONS 363

mapping three dimensional quantities into three specified dimensional


results (Le. there are no free dimensionless combinations).

Solution 12.19. Force balance gives us

( 2 y:\
2 2 2 1
N/
Gm r
mN(U

According to Bohr quantization the separation radius must satisfy


2mw
G9 = n1r .

From these two equations, the separation is


r =
2n n2 /Gm .

The lowest energy level is the n= 1 level. In this level the separation is

24 6
r =

21I2/Gm =
6x10 cm 6x10 light years!
CHAPTER 13: SOLUTIONS

Solution 13.1. According to the Bianchi identities the Einstein tensor

R is divergenceless that the divergence of generalized


gtLVR so our

tLv
field equation is

( -a) R,tL =

81TT':;v ・

On the other hand, if we contract our field equations first and then differ-

entiate we find
811
(1- 4a)
R,11 T,11

so that the equations of motion must be

z-a

,:;v K
T,11 K


== .

1-4a

In the Newtonian limit, and for a fluid with density p and negligible

pressures, the 11=0 component of this equation is

ap V.
ap


(Pi) =
K -

at at

where v is the velocity of fluid flow. If K were not zero (Le. if a

were not ) this would differ from the Newtonian continuity equation,
and violate the conservation of mass in Newtonian order.

Solution 13.2. The vanishing of the Weyl tensor allows us to write the

metric in the conformally flat form


e 2cp 11
gflv 11V

where cp? 1 in the Newtonian limit (nearly flat spacetime). We have

then (see Problem 9.19)

364
CHAPTER 13: SOLUTIONS 365


R _6V cp .

For nonrelativistic stress energies


Tilfl TOo -

p and the field

equations become

fxP ,a(3 Tfa(3 =


K T = -
K P .

In the Newtonian limit time variations will be slow


compared to spatial
variations (in c =
1 units) so we have

ij
6cp ,1J a
..

K P .

For K =
2411 this is the usual Newtonian equation for the Newtonian

potential cp. The solution to Problem 12.11 implies that Newtonian tra-

jectories are geodesics in a geometry with


goo

(1 + 2cp). Thus in the


Newtonian limit the proposed theory agrees with Newtonian theory and cp
plays the role of the Newtonian potential.

For the metric of a massive object like the sun the function cp must

falloff far from the object where spacetime approaches flatness. To see

that light is not deflected we need only notice [see Problem 9.18] that the

null geodesics of
gp.v

e2c/>Tfp.v are identical to the null geodesics of

11
Il V
, which are certainly not deflected by the massive object. Thus when
we compare, in the asymptotically flat distant regions, the directions of a

photon's motion before and after it interacts with the massive object, we

will observe no deflection.

Near the earth's surface the metric is of the form

2 2cp (z) 2 2 2
ds =
e (_ dt + dx + dy2 + dz )

where z corresponds to height above the surface. If particles are to

fall with the proper acceleration we must have cp -

gz. From the

geodesic equation we find that for the energy of a photon moving


vertically
dpO o
dz
_
_r
oz pO =
-c/> ,z pO.
366 PROBLEM BOOK IN RELATIVITY AND GRAVITATION

Thus a photon loses energy at the same rate as a particle and the proposed
theory agrees with Pound-Rebka experiment, as any theory based on

geodesic motion must.

Solution 13.3. The static field equation for the Brans-Dicke scalar field is

t72,/.. 811T

'P
= ・
(1)
(3 + 2(1))

Now, for a spherical shell of mass M, radius R,


Mo(r- R)

-p =
(2)

411 r

so that the cp field inside and outside the shell is

cpI =

cpl ' r < R (3a)

,/.. -,/.. 2 M
'PII

'P 00 +
3 + 2(1) r'
r >R (3b)

where cpl and CPoo are constants. Matching cpI and CPII at r =
R gives

2 M
cpl =
CPoo +
3+ 2(1) R
(4)

Now, we also know that in the Brans-Dicke theory, the local gravita-

tional constant, in terms of cP, is (Weinberg, Equation 9.9.11)

-1 4+ 2(1)
G =
cp
( 3+ 2(1) ) .
(5)

Thus

,/.. =
4 + 2(1) 1
'Poo 3+2(1) c;-
oo

[1 cPooR :+2w)J

G1 =

cPll G:t) G:) cP:


and finally
G1 =
Goo
[1 G;.M (2;(,))J


(6)
CHAPTER 13: SOLUTIONS 367

This solution is correct only to lowest order in M/R, not only because
of the terms neglected here, but also because in higher order one would
have to account for the changed metric g due to the mass and use the
f1.V
curved space V in Equation (1) rather than the flat space one.

Solution 13.4. If there is to be no preferred frame for the vacuum, the

stress-energy of the vacuum must look the same in any Lorentz frame. The

tensor looks the same in any Lorentz frame that the stress-energy
l1f1.v so

or in general coordinates does not single out any


Pvac l1f1.v' Pvac gf1.V'
frame. [The uniqueness of can be seen as follows: if there is no
gf1.V
preferred frame there can be no preferred vectors, hence no preferred
eigenvectors. This is only possible if a 11 vectors are eigenvectors, but
if
S,:V KV for all
K, then must be proportional to
S,: 0':.]

v f1.
The field equations with vacuum and matter sources are

Rf1.

!. f1.V R v
8 17 ( Tf1.
Pvac )
f1.V
g + g

_


matter

When we compare this with the field equations including the cosmological

term
Rf1.

_ 1. g
f1.V R + A f1.V


817 Tf1.

2 matter

we see that we can identify the cosmological constant with the vacuum

energy density according to A = 817


Pvac'
If the vacuum contains particles of mass m at a separation of
A I"OtoI
ir/mc then the energy density would be of order m(mc;1r)3. This is
4 17
of order 10 grams/cc for electrons and 10 grams/cc for protons:

both absurdly large.

If the vacuum energy is to be considered as arising from the gravita-



tional interaction, of energy I"OtoI
G '
of nearby particles, the equivalent

2 2 3 6
mass density would be c- (Gm /A)/A =
Gm c

/fr4. For protons this is
22 41
of order 10- grams/cc and for electrons 10- grams/cc. The
23 22
density of our Galaxy is 10- grams/cc so the 10- grams/cc
368 PROBLEM BOOK IN RELATIVITY AND GRAVITATION

associated with protons would have measurable consequences even for


41
Galactic dynamics. The 10- grams/cc associated with electrons, on

the other hand, is small even compared with cosmological densities


31
(10- grams/cc) and would have no measurable effects.

These arguments should be considered only suggestive and somewhat

ad hoc. On dimensional grounds must be of form


Pvac


4 3 2

( )
m c Gm

3 cn
11

For n =
1 we get somewhat reasonable mass densities. The Zel'dovich

argument is an attempt to give a physical justification for n =


1.

Solution 13.5. If we contract the Einstein field equations we get a rela-

tion between the trace of T and the Ricci scalar: R 811


T,f. In
= -

/lV
the local Lorentz frame in which
T': is diagonal, T,1
= -
P + Px + Py

Pz'
where p x , py , pz are the (principal) pressures. For all known equations
of state P 3p so
T,f should always be negative and R should be

positive.

The electromagnetic stress-energy is traceless (see Problem 4.16) so

R =
0 if the only stress-energy is electromagnetic.

Solution 13.6. Given T! in his own orthonormal frame, 1ihe observer


can find its four eigenvalues and eigenvectors:

T!W{3 =
'AWa ・
(1)

We are given that one eigenvector is timelike. Normalize it so that


a j
W wa

= -
1. If its components are W =
(y, yv ), then perform a Lorentz

transformation with velocity v to the rest frame of W. In that frame
Ok



(1,.Q) and Equation (1) gives TOO =

-Atimelike ==
P, and T = O.

The 3 x 3 matrix
Tjk can be diagonalized by a spatial rotation, and
from Equation (1) we see that the diagonal elements are the remaining
CHAPTER 13: SOLUTIONS 369

eigenvalues Ai Pi with i 1, 2,3. Now let be arbitrary


= =
u an
""

4-velocity, with components j



u =
(y, yv ) in the frame of the observer for

TafJ
whom

tha t is
is diagonal. The weak energy condition is that
-rafJuaufJ 0,

-2 -2 -2
P + v 1 Pl + v
2 P2
+ v
3 P3 > 0 (2)

where j
arbitrary and

v is v 1. The necessary and sufficient condi-

tion that Equation (2) be satisfied is

p O p +
Pi 2: 0 (3)

a test which can be made by the original observer after he has solved

Equation (1). For the case when T



{3 has no timelike eigenvector, see

Hawking and Ellis, p. 89.

Solution 13.7. For n =


0 the statement reads u.u::; 0 and is trivial. For

n =
1 the statement is u.T.u 2: 0 for all timelike u. This is just the

weak energy condition. For n =


2 the statement is that (u .T). (u .T) 0

or that u.T must be nonspacelike. But for an observer with 4-velocity u,

u.T =
(-p, [Energy flux]i) so, by the dominant energy condition, Ipl >

I Energy fluxl and u.T is indeed nonspacelike.

Now consider any n and the statement

( _l)n u

T T ・ ・ ・ ・
T. u ::; 0 .

Since u.T is nonspacelike, the statement for any n is equivalent to the

statement for n -
2. But we have shown that the dominant energy condi-

tion implies the n= 1 and n= 2 cases so it follows that the dominant

energy condition implies the statement for all n. The converse follows

immediately, any stress energy satisfying the statement for n= 1 and

n= 2 satisfies all the requirements of the dominant energy condition.

Solution 13.8. Yes. Write down metric such that for


l1f1.v
a =

gf1.V gf1.V
t < 0 (this is clearly empty space) and =
(arbitrary functions of your
gf1.V
370 PROBLEM BOOK IN RELATIVITY AND GRAVITATION

choice) for t O. The only constraint on the arbitrary functions is that

they be twice differentiable and match smoothly to the flat-space values

at the t 0 surface. Now from this metric we can calculate the


gflv

Ricci tensor and scalar and we can define a tensor


Tl1v=(877)-1(Rl1v- gfLVR).
This tensor is obviously symmetric, and by the contracted Bianchi identi-

ties is divergenceless. If we fill spacetime with a stress-energy equal to


this, then is a solution of the Einstein field equations. (However,
gflv
for a physically meaningful solution there Inust be further requirements
for T should be everywhere nonnegative. This
flv' e.g. energy density
will not in general be true for the procedure above.)

Solution 13.9. We use Killing's equation (a;{3) 0 and the "hypersurface-


(see Problem 10.8) of staticity


orthogonal" statement
[a;{3gy] 0, to

derive


(g[a;{3 gy]);y

( a;f3 Y y;a f3f3;y a);Y


+ +

(RAYf3a A RXay
Y A RXYf3
f3 A a)
+ +

where we have used the result


fl;a{3 Ry{3afl Yfrom Problem 10.7. The

first term above vanishes by symmetry and the remaining terms give Ricci

components, so we have

o =

ARA[a f3]877 A(TA[a


= -

gA[a

) f3]

877 A(p+P)UAU[a (P-P)gA[a) f3]


+ ・

U[a g{3]
The last term vanishes trivially so we can conclude =
0 and

hence u oc .

A less rigorous proof is possible if the equivalent "stationary and


time reversible" condition for staticity (see Problem 10.8) is used. In
Oi
this case we see immediately that if u is not parallel to then T 1= 0
Oi
in the "time reversible frame." But this implies G 1= 0 which is in-

compatible with time reversibility of the metric.


CHAPTER 13: SOLUTIONS 371


Solution 13.10. From the Bianchi identities G
J1.;fl == 0 we have

1..... G
VO
== _

vi

_

a fl


_


va
rJ1. ・

at ,1 afl afl

Terms the right cannot involve third derivatives of the metric with
on

Vo
respect to time, hence G cannot contain second time derivatives. It
VO VO
follows then that the four equations G =
811 T serve as initial value

equations, Le. constraints on the data on the Cauchy hypersurface, while

G ij =
811 Tij are the dynamical equations.

An examination of the constraints on initial data is obscured by the

arbitrariness in the metric associated with the arbitrariness in the coordi-

nate system. To eliminate this confusion we can choose at the outset

four constraints on the metric. A convenient choice is Gaussian normal

coordinates: = -1 and =
O. We now appear to have six field
goo gOi
ij Tij which
variables
gij'
six dynamical equations G = 811 we can solve
2 2 VO VO
for a g /at and four initial value equations G =
811 T giving
ij

relations among the initial data goo


1)
and ago1)./at on the initial hyper-

surface.

We can now differentiate the four initial value equations with respect

to time and use the dynamical equations to eliminate terms of the form
2 2
a g.. /at . This generates four new relations among the initial value data.
1)
[These new relations must be independent of the original four because
vo vo vo
aG fat =
811 aT fat, and we can specify aT fat independently of
any previously considered constraints.]
We have now eight constraints on the twelve initial value functions

g..
lJ
and agoo/at.
1)
It follows that we can independently specify four initial
value functions on the hypers urface.
If we think of these four initial value functions as the values of two

field variables and their time derivatives on the cauchy hypersurface,


then we can picture the time evolution of the gravitational field as being
described by the dynamics of these two fields. For this reason we usually

speak of the gravitational field as having two dynamical degrees of freedom.


372 PROBLEM BOOK IN RELATIVITY AND GRAVITATION

Solution 13.11. The components of the Landau-Lifschitz pseudotensor

are given by the rather awesome expression

{3
[ {3 A f.l

(-g) t L-L

(161T)-1 9-


A 9- ,f.l

9-
aA

A 9-{3f.l,f.l + 1

ga{3 g\nf.l 9-AV,p 9-Pf.l ,

_
gaA gf.lV 9-{3v,P,
9-f.lP A g{3Agf.lV 9- ,p 9-f.lP A
av -



gA J1. gVP 9-
aA
,V,p
9-{3f.l

(2gaAg{3f.l_ ga{3gAf.l) (2rz



1 VT
+ \ 9- pa
8 P
g aT -

ga

rz
T )9- , /\ ,f.l

where

9-
a {3 ==
(_ g) t ga{3 .

For the metric we are considering


.1
3 1 2 Oi
9-
00
= -

(1- 2c/> ) 2" (1 + 2 (1))-


2"
, 9-
ij =
(1 _
4 (1)2) oij ,
L1
7T


0 .

{3L_L

Notice that all terms in the expression for t involve the product
of the derivatives of two 9-'s, so to lowest order t
a {3 will go as the

product of the first derivative of (I). We need to keep therefore, only the

terms proportioned to (I) i when we calculate the derivatives of the 9-'s:


gij,k 4c/>c/> ,k oij


o0
9- 4c/> ,1 .
_
O.
,i

00
If we keep only the 9- ,1

type terms, only the underlined terms in the


a {3
expression for t
L-L
survive and we find

[ ]
a {3 1
(1611)-1 _gi j 9- '9-{30'+ 8 (2galg{3m_ga{3g1m)9-00 1 9-o0,m
aO
t = ・

L _
L ,1 ,J ,

{3 00
From this, with ga{3 l1

and 9- .

,1

4c/>.
,1
we easily find

[ ]
oo -1 ij00 00 1 00
lm 00 -7 ij
m m

_

(16 11 ) -
u .

9- .
+ -

9-
_

- -
u 'V. 'V .

9- 1 9-

L -
L ,

, J 8 "

811 '
1 ,j

Oi
t =

L-L

(I),m\.
[1. (20ilojm_ O ij
] (411)-1((1) 1. 0 (I)
00
t ij
L-L

(1611)-1 8
Olm9-009-
,I,m

\..

,1
(I) .-

,j 2
..

lJ,m )
CHAPTER 13: SOLUTIONS 373

Note that the spatial components tiL_ L agree with the stress tensor

given in Problem 12.8.

Solution 13.12. At a given point the value of a scalar is unchanged so

cpnew (xnew (P)) cpold(xo1d(p)) cpold(x


new
= =
(P) _

(P))

cpold(x
l1eW new
cpnew (x ) ) _

cp a
a,

For a vector field


old

new
= a(x ) Vold(xold)
f1. v

a(x
new
)f1.

(OVf1. _

)v
(Vold(xnew)

_
VoId
v, a
a)
,11

Vold(xnew)
f1.
_

v,f1.
_

vV
f1. ,

a.

For a tensor field, similarly

Tnew(xnew)
f1.v
= a(xo1dt a(xOldl TO (xold) v a fJ
new f1. new
a(x ) a(x )

V)
(T (x

- (oft
new

a{3 ,a
) )
( /l) T

= -

T -

T/l{3
d -


v{3v -

/l
T/lV,a a ・

Notice that

a ( a
+ ara ) =
ga +gar f1.O.v
gp.a,v
gf1.a \s
_

gf1.a S
_

s S

;v ,v av
f1.;v

and that

(/l;V)

ga(/l

a(g/lv,a)
V) ・

For the metric tensor then


376 PROBLEM BOOK IN RELATIVITY AND GRAVITATION



- 1k X
V V a
o =
h '

Re [ik A e ]
V V
O a
1k
aX

a a
o Re [ -ka k ]
h v,a A ve
= =

So the 4-vector k must be null and must be orthogonal to A.

The gauge transformation of is given in Problem 13.14


h v
as

ld c
bh ew = ho 2 +
ca
,v) l1 vs,a

s(

v v
O a
1k X

Clearly the gauge transformation must be of a plane wave e form.
ik x


With
g iC e we have
= -

Caka

A W (C/l
d CVk/l)

+ +
"'/lv

If this equation is contracted with k we see that

V V
Anew k = -
k C k + k Caka = 0

so A remains orthogonal to k. We can now apply the transverse-


ne w
traceless condition, (Anew) = 0 and A


0, to our equation to get

o =
(A old)Il + 2C.k

o =

A -

d C/lk o )
(COk/l + +
"'/lO
C.k ・

These can be combined into

old 1. old Il
o =
A -

(C 0 k + C k
0)

11 A
o 2'

o .

(2korl(A d(A0 ):).


1d
0 have The spatial
Taking /l = we
Co =

components are then easily found from

o =

A -

di
(Cok +
Cik o ) .
CHAPTER 13: SOLUTIONS 377

Solution 13.17. For a beam of light moving in the x-direction nonzero

components of the stress-energy tensor are T


00

T = -

T From the
xx Ox'
form of the linearized field equations

oh == h , Y = -
1617 T

(3 (3
a,y a
(3

and from the fact tnat all components of h should be retarded integrals

(3
over the source, the solutions must be related in the same way that the

sources are:

hoo =

hx x
= -

ho x

with all other components zero. Since the trace of h IS zero, h =


(3 a
(3
h and the above relation is also true if we remove the bars on the h's.

(3
For a photon moving in the x-direction with affine parameter '\,


dx (3
d a 2
2 dx dt
ry
( d'\ ) (rY 00 + r y xx + 2r y Ox'

= _ _

dA 2 dA dA a(3

But the right hand side vanishes

2r y ox ry ry _!. (h +h 2h 0
o x,y )
+ t = - =

oo xx
2 O,y
0 xx,y

2 2
so that d y/d,\2 =
0 and similarly d z/d,\2 =
O. Thus the photon con-

tinues to move parallel to the beam of light and therefore two thin beams

of light initially moving parallel will not attract each other to the lowest

order in each of their mass-energies. (In fact, it turns out that there is no

attraction even in higher order.)

Solution 13.18. From linearized theory we know that in the Lorentz gauge

oh = -
1617 T
(3 (3

a a

Since the source, and hence the field, is stationary this becomes a

2-
Poisson equation V To calculate need
ha(3=-1617Ta{3' hOY we
TOy.
For a spherical mass configuration of mass density p rotating (in the
378 PROBLEM BOOK IN RELATIVITY AND GRAVITATION

positive sense about the z-axis) with angular velocity 0, we have

TOy

r Op sin () cos cP. Notice that TOy is proportional to the real part

of the spherical harmonic Y 11 ((), cP) and hence


hO Y will also be. We

therefore write h
OY

f(r) sin () cos cP and the Poisson equation becomes

£ r2 .Q.
dr [ dr
f(r)lJ -
2f(r) =
1617 r Up .



r2

If the source is a spherical shell of mass M, radius R, then



p =
Mo(r -

R)/4TT R and the equation can easily be integrated to show

r/R for r < R
4MO
fer) = -


(y for r> R

and hence inside the shell

40

r.

hOY hOY Sln() coscP
= = -

gOy 3 R

By symmetry


o J...
g"
'fJ

go y I cP= 0
= -


OM
( )
1.

sin ()

and

(V = -
g°cP =
g°cP =
go =
40M


gcPcP 2
r sin () 3R
( cP)
Inside the shell then locally inertial frames will rotate relative to inertial

frames far away. Mach's principle states that the inertial properties of
spacetime depend on the motion of distant matter. The dragging of in-

ertial frames certainly is an example of the influence of matter on the

inertial properties of spacetime. The constancy of (V suggests that the


influence of distant matter on inertial properties might be viewed as a

law, so that the inertial properties are not primarily determined by nearby
matter, but rather the influence of all the matter in the universe must be

considered. (But aside from a few idealized examples such as this one,

no one has ever been very successful in "deriving" Mach's principal from

the field equations of general relativity.)


CHAPTER 13: SOLUTIONS 379

Solution 13.19. The equations of motion T/l =


0 are clearly gauge in-
,v

variant, but in the Lorentz gauge (see Problem 13.15) T/l ,v


,v

"V v

(1617)- 0 hr =
0 rather than T =
O. The difference between T/l
,V /lV ;v
and
T/l v is of order Tr. But both T and r are of the order of h
a (3
so the inconsistent tenTIs are of second order in the deviation of the metric

from flatness.

Solution 13.20. By the principle of equivalence, the acceleration of a

body is independent of the magnitude of its mass, and depends only on

other masses with which it gravitates. Thus a positive mass attracts both

positive and negative masses, and a negative mass repels them both.
Here, then, the acceleration of the negative mass is toward the posi-

tive mass, and of magnitude GM/e .
(Since e? M, we are in the

Newtonian limit.) The acceleration of the positive mass is in the same

direction and of the same magnitude: the two masses try to chase each

other.

The problem is more complicated once the particles are moving, be-
cause each sees the retarded field of the other.

Since we are given M? e, we shall solve the problem using linearized

theory which will be justified as long as their distance in a frame comoving

with either remains much greater than M. The equation

o 1617 T
h/l (1)
= -

V /lV

has the retarded integral solution


/lv(t', ')


h (t, x) =
4 o (t'+ \x-x'\-t)d x'dt' .
(2)
/lV
\ \
- ' - -

x-x
- -

In linearized theory the T (that is the h/lv's) of


we can superpose
/lV's
each particle. For a particle of mass m moving along a world-line

x/l =
z/l(r) (3)
we have
380 PROBLEM BOOK IN RELATIVITY AND GRAVITATION


,.w(t,.?!)

f u u 8 4 (x z(r)) dT =

mUlluV8\.?!. (t))/uO (4)



ll v


where we have used dr =
dt/u to do the integral. Substitute Equation (4)
in Equation (2):
o3 [x '

oCt'+ I.! !.'I- t) d


(t')]

x' dt'


ulluV
- -

4m
hllv(t,!.)



I.!-.!'I
(5)
uin (n8(t'+I.!- (nl-t)dt'

4m
f uO(t') \x-z(t')\ - -

By symmetry everything depends only on one spatial coordinate, say x.

Let
R(t') =
\ x-z(t')\ = f [x-z(t')] (6)

where f = :t 1 for x z. Then

ulluV oCt' R-t)


+ dt'


hllv(t, x)
_

4m .
(7)

uOR

The 4-velocity u is determined from the equation of motion

dull /dT = -

r {3uau{3
Le.

dull /dT + rfv uau{3( a,{3 +


{3
,a

{3,v)/2


= 0 .
(8)

Here the ha of particle 2 evaluated at the position of particle 1 is


{3

used to find u ll of particle 1 and vice versa. By symmetry uY = U = 0

for both particles. Write out Equation (8) for Il =


0 and Il =
x, using

hoo =

hxx (easily verified from Equation (7)):

2 2
O 1 h 02 X
h 00 0 x x
(10)
du ld r =

2 0 0 0(,
u -

u ) +
,x
u u + h
o x,x

1 h 02 2
h 02 h 0
du

/d r =
(U -



) -

Ox, 0
u -

00,0
u U


(11)
2 00, x

Now evaluate the derivatives of the from Equation (7):


ha{3
CHAPTER 13: SOLUTIONS
381

! uOo'(t'+R-t)dt'
(t')8(t'+R(t')-t)dt'


h =
4m -4 m
00,0 at R(t') R

where 0' denotes differentiation with respect to the argument of the

delta function. Let


f(t') =
t' + R(t')

df dz

1 -

(v(t'), v =
....-:-? .

d? dt
Then

uO

d ;J dt'

hoo,o 4m o(f df
t)J
= - -

R df df


[R df] o(f-t)df
d u dt'

4m
df

[ )
dt" .!L dt"



4m
df dt' R df
f=t

[ ] t'=t-R(t')'
1 u

4m (12)
l-(v dt' (l-(v)R


Since duO /dt' ,....,
m, to linear order in m we need not differentiate u

and v in Equation (12). Thus

[ J


( vu
h -


4m (13)
OD,D 2
(1- fv)2 R
ret.

where rete means evaluated at the retarded time t' = t -

R(t'). But

vet') =
v(t) + (dv/dt)(t'-t)+...

vet) + (9(m)

o 2-
and similarly for u =
(1- v )-2, and

R(t') =
([x -

z(t')]

([x -

z(t) -

vet) (t' -

t)] + (9(m)

f [x -

z(t)+ vet) R(t')] ,


380 PROBLEM BOOK IN RELATIVITY AND GRAVITATION


/lit,.?!)

f u/l 84(xz(r)) dr mU/luV83 =


(t))/uO (4)

where we have used dr =


dt/uo to do the integral. Substitute Equation (4)
in Equation (2):

ulLuv03[.!, .!(t')] oCt' + I.! -.!'\- t)d x'dt'


4m
hlLv(t, )

uOI.!-.!'I
(5)
u/l(n (n8(t'+I.!- (nl-t)dt'

4m
f uO(t') \!. (t')1

By symmetry everything depends only on one spatial coordinate, say x.

Let
R(t') =
I x-z(t')\ = f [x-z(t')] (6)

where f = :t 1 for x z. Then

UILUVO(t'+R-t)dt'


IL vet, x)
_

4m .
(7)
uOR

The 4-velocity u is determined from the equation of motion

dull /dr = -

r {3uaul3
Le.

dull/dr + rfv uau{3( a,{3+ {3,a-


{3,v)/2 h

= 0 .
(8)

Here the h of particle 2 evaluated at the position of particle 1 is



{3

used to find up. of particle 1 and vice versa. By symmetry uY = U = 0

for both particles. Write out Equation (8) for IL =


0 and IL =
x, using

hoo =

hxx (easily verified from Equation (7)):

d u O ld r 1 h 02 X

h 00 0 x x2 (10)

2 00 °
(u

u ) +
,x
U U + h
o x,x

1 h 02 -u X 2 h 02 0
d u /d r

(u ) X
(11)
ox,ou hoo,ou U
= - -

oo,x

Now evaluate the derivatives of the from Equation (7):


ha{3
CHAPTER 13: SOLUTIONS
381


(t')8(t'+R(t')-t)dt' uOo'(t'+R-t)dt'


h =
4m -4 m
00,0 at R(t') R

where 0' denotes differentiation with respect to the argument of the

delta function. Let


f(t') =
t' + R(t')

df 1
dz
dP
= -

fV(t'), v =
"';'7; .

dt
Then

uO

d ;J dt'

hoo,o 4m o(f df
t)J
= - -

R df df

uO
J [R df] o(f-t)df

4m
d dt'
df

[ )
dt" L


4m
u dt"
df dt' R df
f=t

[ ]
1 A. u

4m .
(12)
1-fv dt' (l-fv)R
t'=t-R(t')


Since duO /dt' m, to linear order in m we need not differentiate u

and v in Equation (12). Thus

1fVUO
h 4m
[ (13)

00,0
(l-{viR2jret.
where rete means evaluated at the retarded time t' =
t -

R(t'). But

vet') =
v(t) + (dv/dt)(t'-t)+...

vet) + (9(m)


2",

and similarly for u =
(1- v ) and

R(t') =
f [x -

z(t')]

f [x -

z(t) -

vet) (t' -

t)] + B(m)

f [x -

z(t)+ vet) R(t')] ,


382 PROBLEM BOOK IN RELATIVITY AND GRAVITATION

thus
R(t')(l-fv) =
f[X-Z(t)] .

Since ho o, ° is being evaluated at the position of the other particle, we

have °
f vu
hoo , °

4m (14)
(zl -

z2)2
where the quantities are evaluated at the instantaneous time t. Similarly

x -



hOx,o

° hoo,o (15)


( )
_ _

hxx,o

hoo,o' (16)

The spatial derivatives can be evaluated by noting that x occurs only in


R in Equation (7). Thus

uOo(t'+ R-t) dt'




h -

f -
4m
oo,x aR R

4ml' f U08(t': -t)dt'f 8'(t'+RR-t)dt']




= +

[f ]
°


°
dt' dt' d
=4mf -

!!.-o(f-t)-df+
df
!L--o(f-t)df
df df

R2

[ }]
uO uO

1 d
= -
4m f + '7:"'?

v) R
2 (1 -
f v) d t (1 -
f v) R
(1 _
f re t.


°

[(1-l'v)R2
u u fV
= -
4 ml' +

(1_l'v)2 R 2 ret.


4mfu
- -
(17)

(zl -

Z2)
CHAPTER 13: SOLUTIONS
383

Similarly x


=--h (18)
Ox,x o oO,x

hxx,x
( : )2 h (19)

oo,x


Note that the gauge condition h =
0 is satisfied by Equations (14)-
p., a
(19). Write out the equations of motion, Equations (10) and (11), for

particle 1 letting

u =
y, U =
y v, g =
m/ (z 1 -

z2)2. Take particle 1

to be the positive mass particle at > so that f +1 and the


z2'

zl
source mass in Equations (14) (19) -
is negative.

dYl /d r = -

gY2 v 2 (1 + v +
)2gY2(1 v yi) + v

4g Y2 v2 yiv (20)

d(Yl v 1 )/dr

gY2(1+ v

)4gY2 v YI +
2gY2 v

(1+ v )YIvl .
(21)

Now the equations for


dY2/dr2 and
d(Y2v2)/dr2 have the same right-
hand sides as Equations (20)-(21), with v




(Em keeps the same

sign.) Since v




at t =
0, then v




for all t. Thus
zl

z2
is constant for all t, i.e. zl

z2

E. We can drop the subscripts 1
and 2 in Equations (20) and (21) and get, after some simplification

dy/dr =
gv/y (22)

d(yv)/dr -

g/y (23)

independent, because of the relation


2 -"2
These equations are not y

(I-v) .

We find then that the differential equation for the motion is

2 2
d v / dr =
g( 1 -

v) , (24)
and hence
2 -1
2gr =
v / (1 -
v ) + tanh v (25)

where r =
0 when v =
O. Thus
384 PROBLEM BOOK IN RELATIVITY AND GRAVITATION

dz dz dt dr -5/2

(1 v )
_
= - -

dv dt dr dv

implies
1 -t
[(1 ]

z2
= -

v) -

3g
(26)
+ e
z2

zl

and similarly

dt dt dr -5/2

_
- 1. (1- v 2 ) (27)
dv dr dv g

implies

[ J

v 2-"2 v 2- 2

"3 (1- )
t =
(1- v) + v .
(28)

Equations (26) and (28) are parametric equations for the trajectories with
v as parameter. Note that while the coordinate separation of the particles

remains constant, the proper separation as measured by an observer co-

moving with one of the particles increases, approximately as


ye. Proof:

Suppose particle 1 has velocity v at some instant t



at position

zl (t 1 ). The Lorentz transformation to the locally comoving inertial


frame is
x' =
y [x -

zl (t 1 )

vet -


1 )]

t' =
y [t -



v {x -

zl (t 1 )1] ・

where
Thus the trajectory of particle 2 in these coordinates is z'2 (t'2)

Z'2 =
y [z2(t 2 ) -

zl (t 1 )

v(t 2 -

1 )]

t'2 =
Y [t 2 -



v { z2 (t 2 ) -

1 (t 1 )}] ・

The distance to particle 2 is -

z'2 (t'2 =
0). Let

(3 =

z2(t 2 )-zl(t 1 ).
CHAPTER 13: SOLUTIONS 385


Now t'2 =
implies



-=-


+ v{3
so

(3 =

z2(t 1 + v(3) -

zl (t 1 )

v 2{32 d z

z2(t 1 ) + v{3 dz / dt
2 t +

Zl(t 1 )
t=t
1 dt t=t


= -
E + v {3 + a(m/L) .

Solve for {3:




{3 =


(1- v )
Thus

z'2 (t'2

0) = -
y(/3 -

v (3) ye .
CHAPTER 14: SOLUTIONS


Solution 14.1. First we determine what is the appropriate Tfl for a

point particle, say of mass m. In the momentarily comoving frame it

should have only a TOO component, and this should be a a-function of

position, so

T/lV
J 84(x
a V
oc
-xa(r))u/lu dr

where xa(r) is the particle's trajectory in spacetime as a function of its



proper time r. Now we want Tfl to transform like a tensor. The product

04(x

already transforms

uflu as a tensor, but -

xa(r)) is not a scalar;


04(x a -

xa(r)) (- g)-"2, however, is a scalar:

j J 84.1 (_g)t
0 4(X
a a 4 4
1=scalar= _x (r))d x= d x .
(1)
(_ g) 2

Thus, normalizing to mass m, we have


u/lUV
84(X


Tfl V = m

xa(r)) dr ==
p ufluv dr (2)
.1
(_ g) 2

where p contains all the terms except the u's. Now



o =

Tfl v ,

[(puV).vufl +

(PU ) u ,
v]
dr ・
(3)

Dotting with u fl gives

0=
J [-(PUV);v +
p(Vuu).u]d r ・
(4)

The second term vanishes because 4-acceleration V u



is orthogonal to

4-velocity. We conclude therefore that the first term vanishes and hence,
from Equation (3)
386
CHAPTER 14: SOLUTIONS 387



o =
(PU ) u/L;v dr ・


This implies that u ull


0 wherever the (delta function) P IS nonzero,



that is, wherever the particle is! But u
ull,v

= 0 is just the geodesic

equation.

Solution 14.2. If a system is in thermal equilibrium, then no net energy

will flow between two locations A

Tat g80 and B if they are allowed to ex-

change energy. In particular,


imagine a "light pipe" between
the two locations which transmits

light pipe a thermal flux of photons. Since

the pipe and the system are static,


there is no change in a photon's
energy when it reflects internally
TAtg o in the pipe. There is a change in

energy between A and B due to


1 1

the gravitational redshift: (hv)A/(hv)B (- gOOB)2/(- gOOA)2.


= Now use

the fact that a black-body intensity Bv(TA) at A is changed by any

kind of redshift to precisely another black-body spectrum with a tempera-

ture T B =

[(hv)B/(hv)A] T A' If thermal equilibrium holds, this must be

the ambient temperature at B, so that the fluxes up the



pipe and down

the pipe are identical. Thus TB =


[(- gOOA)2 /(- gOOB)2] TA' and



T( -

gOO) =
constant

throughout the system.

Solution 14.3. For a perfect fluid the stress-energy tensor is Tllv =


(p + p) ullu + pgllV, thus the equations of motions read

ulluV (p + p) (ull.vu

uIlUV v ) gllV
o =

TIl v (p + p )'



",
+ .
+ p


388 PROBLEM BOOK IN RELATIVITY AND GRAVITATION

To get the Euler equation, project this equation


perpendicular to u
using
the projection tensor
P u +
u{3

a (3 {3

The result is

o =
P Tllv
all ;v


0 + P
all (p+p)ull.vu '
+ 0 + p
,v
gllV p
all

(p+p)ua.vu + p + p uvua
, ,a , V

which is the Euler equation.

To take the Newtonian limit (see Solution 14.8), write the Euler

equation as

PO(l+1T+p/PO)(uj,v uV -r jUauV) -P,j = -




dp/dr .


Set u =
1, U


-1, u.


v.

and get



PO(dvj/dr +
Oj ) p.

,J

But

O 1
r -r = -
cP
OOj 2 go 0 j

Oj , , j

Thus

dV
j 1

dr

-cp ,
'-


-P'
P o,J

Solution 14.4. The Euler equation for a perfect fluid is

(p+p)Vuu =
-Vp -

uVuP .

Hydrostatic equilibrium implies the existence of a time Killing vector .


The fluid 4-velocity, according to Problem 13.9, must be parallel to this

Killing vector, so
u =
/1 I ・


(In components, this means that only u =
0, where =
a/at.) From

Problem 10.14 we know


CHAPTER 14: SOLUTIONS 389


V u


V log Ie. I .

We also know that ap/at 0, VuP 0, and therefore


or
Vep
= ex =

Vp =
-(p+p)V log 1 .eI2 .

1 1
But e. e =
(a/at). (a/at) =

goo' so I ・

e \2 =
(- goO)2, from which the

result follows.

In the Newtonian limit, p? p and -

(1 + 2 (I)) where the


goo
Newtonian potential (1)? 1. Thus

ap a 1
-p

log (1+ 2(1)) -p ..
ax v
ax

ax

Solution 14.5. Using p =


p in the equation of hydrostatic equilibrium

(Problem 14.4) p A


(p + p) [log (- goo) 2] ,
A we get


.1
1 dp

log (- goO)2 =

4 p'

This integrates to p =
constant x (- o)-2.As long as
goo is finite,

therefore, p cannot vanish and thus there is no free surface.

Solution 14.6. A static uniform gravitational field has a line element

2 2 2 2
ds =
g tt dt + g
zz
dz + dx + dy2 (1)

where gtt and


gzz depend only on z. The nonzero Christoffel symbols

are easily computed (Problem 7.6). The weight as measured by a scale at

z =
0 is W(O), where

W(z) =

f T;; dxdy =

f TZZ dxdy
z ・
(2)

tl
We wish to show that W(O) depends only T T tt and on no other
on

We have .
390 PROBLEM BOOK IN RELATIVITY AND GRAVITATION

o =

'fO-{3;{3 =
I gl- t (I gl t 'fO-(3),{3 + ra T{3y .
(3)
{3y

Take t and integrate over using the fact that


{3 indepen-
a =
x, y, is

dent of x, y and t:

o =

J Ttt
, t
dx dy +

J Ttx dx dy +
,x,y

J Tt dxdy + Ig\-t (\gl t


J Ttz dxdy)
,z


tz

2r\z T dxdy ・
(4)

tl nd
The 1 st term vanishes because T is time-independent. The 2 and
rd tx
3 terms give terms proportional to T and Tty on the boundary of the

integration region. These vanish for a


boundary outside the container.

Equation (4) can be put in the form


1 1
-1 tz
o =
I g \- 2 gtt (\ g I 2 gtt T dx d y
), z

(5)


tz
Thus \ gl2 gtt .r T dx dy is independent of z. Since this term vanishes

above the top of the container, it vanishes everywhere.

Now put a =
z in Equation (3) and integrate over x and y:

0=
JTzt,tdXdy+J TZX,xdxdy+ JTZY,ydXdY+ Igl-t(lglt J TZZdxdy),z
JTttdxdy J
z ZZ
+ r

+ r T dxdy ・
(6)
tt zz

st nd rd
The 1 term vanishes as above, while the 2 and 3 give boundary
contributions which vanish. Using Equation (2), we find





(\gtt I2W ),z =
1 gtt 1 2,z T dxdy. (7)

Integrate Equation (7) from z =


0 to the top of the container, and use the

fact that W(top) =


0 to get

top

i I
1 1""""
tt
W (0) =
1 gtt I 0 ! dz dx d y \ gt t 1 2 , z
T , (8)

,...,...

tt
which depends only on the component T .
(Solution due to W. Unruh.)
CHAPTER 14: SOLUTIONS 391

Solution 14.7. Since the gravitational field is


stationary, there is a time-

Killing vector =
a/at. Dot this into the Euler equation (Problem 14.3)
for the flow of a perfect gas:

(p+p) .Vuu - =

.Vp- .uVup. (1)

But


V u =
V ( .
u) -

( Vu u = V (
). .

u)
u u u

since by Killing's equation V is antisymmetric. Also, Vp


. ap/at=

for stationary flow. Putting . , find





u we
(p+p)duo/dr =

o dp/dr
[Equation 2]. For adiabatic flow of a
perfect fluid, the first law of thermo-

dynamics (Problem 5.19) is dp =


(p + p) dn/n. [Equation 3]. From Equa-
tions (2) and (3)
duo =
dn _
d(p+p)
U n
o p+p

and hence U


constant x .

p+p

Solution 14.8. The Newtonian limit of relativistic hydrodynamics is ob-

tained by choosing a global, nearly-Lorentz, frame in which


po(l + 17), 17? 1

goo
= -
(1+ 2rp), Irp\ ? 1



p/po ? 1, v << .

Here
Po

nm

is the rest-mass density (m B =
mean baryon rest mass),
17 is the specific internal energy, cp is the Newtonian potential and v

IS 3-velocity of the fluid. Since


'" 1

o 2 1 2
u =
(1_v )-2" 1 +

v ,

and
392 PROBLEM BOOK IN RELATIVITY AND GRAVITATION






(- goO)2 u

1 + y2 + cp

the Bernoulli equation becomes

(1 y2
cp)
constant x n
+ + =

po(l+1T+p/po)
tha t is

(1 + y2 +
cp) (1+7T+p/po) =
constant

and finally

y2 + cp + 1T +
p/po =
constant .

Solution 14.9.

(a) The condition that be a


connecting vector between the world

lines of neighboring particles can be written

fu =
au (1)

where a is a scalar function. In Problem 8.14, a was zero since

connected events of equal proper time; here we find a from the condition

, = ・
u =
0 along the world lines:

o = V ( u) ・
= ・
a + u ・
V ・
(2)
u u

Compare this with Equation (1) dotted with u:

u'(Vue-v u) -a

and find a = ・
a. (Recall that u.

u V (u.u) =
0.) Thus we have

(3)
Vu V u .a)u
+.(

The condition for rigid body motion, V u( ・

) =
0, is equivalent to


Vu =
0, or by Equation (3) to

f3(u
a a ;f3
+ u
a a(3)

0 .
CHAPTER 14: SOLUTIONS 393

Since this must be true for any it follows that

+ 0
u(a;{3) u(a a(3)

and hence (see Problem 5.18 for definitions) that



aa{3

(}Pa{3 u(a;{3)

u(aa{3)
= =

Rigid body motion therefore can occur if and only if a


a{3

() =
o.

(b) The symmetry aa{3 =


a{3a reduces the condition aa{3 =
0 to 10

equations. The 5 identities



u u and pa{3u =

u =
0 leave 5
a {3 a {3
independent equations. With the condition () =
0, we have 6 independent

constraints altogether.

The rigid-body conditions do not constrain aa or wu.{3. Each of

these has 3 independent components (aa ua 0, =




{3 = -
w{3a) so there

are 6 degrees of freedom as in nonrelativistic rigid-body motion.

Solution 14.10. From the definition of () =


V .
u we have

d(} =
u{3(u a
). =
u 13 ua =
u 13 (ua -

Ra uy )
dr ,fJ a

fJay
. .

,a ,fJ ,afJ
(1)

u{3u a

R u13 u
Y .

;fJa fJY
But


13 (3 ) ua a {3
a ( u.,fJ, u
u u u
_

,fJ''a

'a
. .

,fJ

aa;a

(wa{3+cfl{3+ ()fP-{3-aa u
{3)

(w{3 a
+ a13 a
+ 1.

(}p{3 a

a13 u

) (2)

of
where we have used Problem 5.18. Using the symmetry properties

w and P and the fact that they are orthogonal to u, Equa-


a{3 a{3

a{3
, ,

tions (1) and (2) imply

d(} =
aa. _

wa w13 J1- a13 _


_

()2 -


QU

u{3
dr ,a fJ a fJ
a 3 fJ


2w

2a
2 1 £)2


u {3
a{3u

a +
- .
= -

;a 3
394 PROBLEM BOOK IN RELATIVITY AND GRAVITATION

Solution 14.11. Since the fluid flows on geodesics aa == u{3u =


0,

;{3
and since the flow is shear and expansion free () O. The decom-
aa{3
= =

position of Vu in Problem 5.18 then gives

u =
cu
;(3 {3

a a

But CA) is antisymmetric, so 0 and hence satisfies Killing's


uCa;(3)

equation.

Solution 14.12. In geometrical language the observer sets up his local



coordinates x as follows: If u is the 4-velocity of the center of his

l1a{3 along
box along its worldline, he chooses =
u and his
eo e{3

ea'
world line. From each point peT) on his world-line (T =
proper time), he
sends out spatial geodesics orthogonal to u, with affine parameter equal
to proper length s. If n is the tangent vector to the spatial geodesic
through the point P near his world-line, then he assigns to P the
o k k
coor d 1na t es

x =
T, X =
sn .

The equation of motion is now


d x


a dx{3 dx Y

{3y CiX"

0 (1)
dA2

where we can take A to be the proper time of the particle. The 4-velocity

)-2.

(y, yy), where

of the particle is dx IdA =


(1- V First change
d/dA to d/dt in Equation (1) using d/dA=yd/dt:

d 2 xa + 1 dy dxa + r
a dx{3 dx Y = 0 (2)
{3y <It

2 Y dt dt dt
dt

Putting a =
0, we get

1 dy + r
0 dx{3 dx Y =

{3y dt

y dt dt

Substitute this in Equation (2) and get, for a =


j,
CHAPTER 14: SOLUTIONS
395

dv j
(IT
+ (-vjr

+ rj ) dx{3 dx Y
O.
{3y

{3y dt dt (3)

To first order in v, Equation (3) is

d vj j 0

J J k

v r + r + 2r 0
v (4)

dt 00 oo kO

Thus, to first order in


.! and we
, have

dv j j 0

k'J k'
(IT

v r 1
00 x-O

rJ
00 1 -
x r oo , k l -

2v
rJkol x=O .

(5)
.,!-o
_
_

x=O

The Christoffel symbols at x =


0 are found using a result of Problem 7.17:

r {3 e =
V e = -
o{3 e
a O (3 u a a (3


Q/3a uAw u
aa u/3 Ea/3Au

= == +
a/3ua

/3ao


Since u =
(l,Q), ua =
(-l,Q), aa =

(O, ),cua (O, ) =


we get

r = 0 if (3 =

{3a 0

r = -
r = -

OjO jOO j


r r
EjkOmCUm EjkmCU (6)
= - = =

kjO jkO

The r's with no O-index are found from the condition that the coordinate
a k k
lines x =
(T, sn ), where T and n are independent of s, are geodesics.
Thus,

o =
d x

+ r
a dx/3 dx Y

ds
2 {3y ds ds

O + r
a j nk
jkn

This is satisfied at x = 0 for arbitrary so


r .
= 0 ・
(7)
aJ k
396 PROBLEM BOOK IN RELATIVITY AND GRAVITATION

We find r j from the expression for the Riemann tensor:


OO,k

a a a a
R r r r

/lr/l{3o raO/lr/l{3y

{3yo {3o,y

{3y,o

This gives

Rj
rjoo,k =

OkO

rjok,o rjk/lr/lOO + rjO/lr/l

Ok

At x =
0:

rjok,O

cjkmw
= -

,0

rjk/lr/l OO


l1 rj rj
'O/l r
J m
r =
r + r
Ok 00 Ok Om Ok

wI


cmjnw

aja k c
km1

and thus

dv j k j m n 1 m k
(R OkO-Cjkm W ) 2
,0+ajak+cmjnW
-a -x c w +
_


cjkmw

dt j km1

. . .


-RJOkOx
-aj(l+ .!.) + (x x [
,o)1- x )x2 (v x]J)1
+ .

The first term is the" inertial force" due to the acceleration of the refer-

ence frame. The 1+ a'x


--
is a relativis tic correction factor (see MTW

Exercise 37-4). The second term is the "true" gravitational force. In

the weak-field limit, a Newtonian physicist would separate into a

contribution from the local acceleration of gravity, a (I)/ax., plus an


"absolute" acceleration 2.a bs. Thus,



(1)/ax j ax l x- 0 xk




RJOkOx =
(a J.)abs + a /axj
I x



+ a _

- -


(a.) + a (I) / ax j I
x =x
Jabs
-Partie Ie

_

The terms involving are the same as in nonrelativistic mechanics;

the second and third are the centrifugal and Coriolis forces respectively.
CHAPTER 14: SOLUTIONS 397

Solution 14.13. Setting the divergence of the stress-energy tensor to zero,

we have
o =
411 T v. =
(I) <I> ,V + (I) <I> ,v. _!. a v
(2 <i>a;v<I> ,a )
11 ,v

,11,V ,11 ,v 2 11

1(1)

II'V
(I)
V'II )
(1) ,V m
+ '.I:'
m,V
11'.1:' .

,,...., ",.... ,,....,


But second covariant derivatives of a scalar commute (cI> [ (1)


] +

,11;V [ ,p.,v ]
(I)
,ara v]0)

so only the last term in our equation survives and the

equation of motion must be

0(1) =

(I)'v = 0 ・

;v

Solution 14.14. In the proposed Equation (2) the term ReI> is indepen-

dent of the size of the laboratory in the same way that the p term is. In

principle then, if we measure the <I> field we can measure the Ricci

curvature; the conformally invariant Equation (2) then violates the spirit

of the strong equivalence principle.

To find the anomalous R-term forces between two particles, let us

go to the local Lorentz frame in which one of the particles is stationary.

The (I) field due to this stationary particle of charge 111 satisfies


'.I:' .
,j _ !. m
R '.I:' =
11 1 u
(r ) .

,j 6

In writing this equation we have assumed a global (or at least laboratory


sized) inertial coordinate system. That is, we have ignored aspects of

curvature other than the Ricci scalar. Pres umably (and in fact) these

other aspects of curvature do not violate strong equivalence.

The equation for (I) has the solution

eI> = -
exp 1 {- r / a , I a

R-2

so that the force on a particle of charge 112 is


398 PROBLEM BOOK IN RELATIVITY AND GRA VITA TION

112(1),r /l1112/r2 [1

F = =
+ r/v/6a] exp {- ria v/ 6 1

111 11 2/ r2 [1+ r /12a


2 2 3 3
+B(r /a )] .

The anomalous R-term force 111112/12a2 is then independent of parti-


cle separation, clearly violating the strong equivalence principle.
From the Einstein equations it is clear that the magnitude of the

Ricci scalar is of the order of the density of mass-energy, so that the

ratio of anomalous forces to "ordinary" scalar forces is


r 2
"J
r p mas s
2 -energy
12a
14
"J
[r(cm.)]2 [P/Pnuclear]' 10-

Thus, even if the scalar forces are being measured inside matter at
14 3
nuclear dens ities, P nuc 1 ear

10 g/ cm , a very large (r "J
1 00 km)

matter filled "laboratory" is necessary before the R-term forces become

important. In any practical experiment the R-term forces would be

miniscule.

Solution 14.15. According to Problem 7.7 (i) and Maxwell's equations

411}11 =
Fl1v. = -L (I gl t F/l V )
,v ,v
Iglt

41T O/llglt) ,11 -.J:, (lgltF/l




411
}11;/l
= =


V /l

Igl2 Igl2

Since F is antisymmetric this must vanish.

Solution 14.16. In flat spacetime we have

Fl1v
,v

411}11



Av,11
= -

I1 V I1 ,v
CHAPTER 14: SOLUTIONS 399

and hence

v ,11

AI1,v + A =
411}11 .
(1)
,v ,v

If we interchange 11 and v differentiation on the second term, as is

allowed in flat spacetime, and then apply the "comma to semicolon" rule

we get


AI1;v + A ;11 =
411}11 ・
(2)
;v;v

If, on the other hand, we apply the rule directly to Equation (1) we get

V ;11

AI1;v + A =
411}11 .

;v ;v

The derivates on the second term can be exchanged by introducing a

curvature term (see Problem 9.8)

V a

AI1;v + A ;11 + R l1 A


411}11 .
(3)
;v ;v

Equations (2) and (3) have, in principle, measurably different consequences.

As of 1975, there is no
experimental evidence to tell which is correct.

Solution 14.17. If the curvature coupling term is of the sort found in



All (Rl1v A )
Problem 14.16, we have (in a Lorentz gauge i.e.
AIl;11 0) 0 +


411 }11, where the term in parentheses mayor may not be present. Since

the Ricci tensor vanishes in vacuum, the only hope of detecting its term

would be by experiments in matter (e.g. in glass). The first term is of

order All [minimum ( e)]-2.


, The magnitude of the Ricci tensor is (by
2 28 2 p
Gp/c
( -3)-
the field equations) of order .74 x 10- cm- x

gm. cm

The fractional correction of the second term is thus


)t y
( (t ,E)(P/gm.. cm-
minimum
oAI1 .


-.,

14
All 1.16 x 10 cm

Since a,n earthbound processes is presumably no larger than the size of



the earth -.,
6 x 10

cm, this is bounded by (0 All /All) .¥ (5 x 10- )2 for
400 PROBLEM BOOK IN RELATIVITY AND GRAVITATION

p =
1. One can also dream up fancier curvature coupling terms like

KRa{3YORa{3yoA/l where K is a dimensional constant. Because there is

no natural value to assume for K, there is no a priori way to estimate

the magnitude of the effect resulting from such terms. Since the equations

are similar to the Proca equation 0 All + mAil =


0, e to
periments measure

the photon rest mass do put some limits on K, but so far they are not

useful ones.

Solution 14.18. For the electromagnetic stress-energy

4 7T Tllv = _
(FIl
\1 a

av
+ !. gllVF Fa (3
4 a{3 ) (1)

{3;Il
( 1.

av a
T llv
4 7T.V -
F Il Fav + F
FIla.v + F F =
0 (2 )
{3
= ・


a'v , , 2 a

Now, the first and last terms above equal

_Fa{3gp.r( Fa{3;r+Fra;(3) = -

Fa{3gllT(Fa{3;r+ F
ra;{3-Fr{3;a)
= -


{3gfr (Fa{3;r Fra;{3 + + F
(3r;a)
oc F

{3glff F [a{3;r] = 0 .

If F is derivable from potential A in the usual way, then


F[a{3;r]


and we are left with


F Il F
av
O -


4 7T T llv -


,.V a ,.v'

This implies that F


av


0 except in the case that the determinant of



the coefficients
Fila vanishes. But det (Fila) = -
(E . )

Solution 14.19. First note that H has been normalized to have dimen-

sions of (mass)2; this means that Hamilton's equations are

dx ll / dA =
aH/ a7T (1)
11

a7T /dA = -
aH/ axil (2)
11
CHAPTER 14: SOLUTIONS 401

where A is an affine parameter (= r /m for a particle of mass m where

r is proper time). If H were divided by m, one would replace d/dA by


d/dr in Equations (1) and (2); d/dA has the advantage of being valid for

massless particles as well.



Since the particle's 4-momentum is pf.1 =
dxf.1 /dA, and since gf.1 and

functions of
Af.1 are x not
TTa' Equation (1) gives


pf.1 =
gf.1 (TTV -

eA

) .
(3)

Equation (2) gives

dTTa 1 /lV eA ) + gf.1 eA



- = -
g (TT -

eA ) (TT -

(TTv

eA ) ・

dA 2 ,a f.1 /l
v v f.1 ,a v

But from Equation (3) we get

dTTa d fl + )
( f.1p eA
_


CiA dA

dplL p{3
=!!
cafl cIA
+!!
caf.1,t-J
p{3plL + eA

,t-J

(where we have used d/dA =


p{3 a/ax(3 ). Multiplying by gay, we find

dpY p{3pIL 1.2


aY
e (A Aa f.1 ) pf.1
gay!!cafl,t-J + gay rfv,a pfl p v =. g


dA. f.1 " a

Now rearrange the derivatives of to form a Christoffel symbol.


f3
Since

o =
(gf.1f3 gt-JY ,a
)=
gf.1f3,a gt-JY + gf.1f3 gt-JY, a

we have on multiplying by gYv

gYv gf.1f3 g

gf.1 ,a
= _

t-Jy,a

Thus

dpY + gay is 1
\ p f3 p fl -

gaYeF Pfl .

\ f.1,f3 gf3fl,a)

CiA 2 af.1
402 PROBLEM BOOK IN RELATIVITY AND GRAVITA TION

Since

(j11-!.
p ( )P {j11
gal1,{jP 11,{j {j,11 P

2 + ,

we
get, finally

dpY eFY I1
dX"
+ r
Y{jl1 pfJpl1 =

I1P

which is the equation of motion.

Solution 14.20. For an uncharged test particle, p' is conserved. We

can guess that for a charged particle, 1T. is conserved, where 11' is the

canonical momentum p + eA (see Problem 14.19). One way of seeing


this is to note that corresponding to there is an ignorable coordinate,
Le. we can choose coordinates so that the Hamiltonian H is independent

of one coordinate. The conjugate canonical momentum is then a con-

served quantity.

Alternatively, we can verify directly from the equation of motion

v p


eF.p
that

v (TT ) (V V (V eA V
pp) + p + e
pA) +
. . .
= . .

P p p

=e .F'p+O+e(VpA)' -ep'VA .

Here we have used Killing's equation to set the second term to zero and

to rewrite the last term. Note that the order of vectors in the dot products
in the first line is immaterial, but F has two "slots" and we must keep
the vectors in the correct positions. Now the fact that the electromagnetic
field admits an ignorable coordinate is expressed by the relation

o =

fA =
V A
VA

Using this in the last term, we get


CHAPTER 14: SOLUTIONS 403

v (11' =
)e ・

F.p + e ・

(V A) ・


ep

(V A) .



e .F'p-e .F.p

where we have used the fact that F is VA antisymmetrized. Thus TT.

is a conserved quantity along the particle's trajectory.

Solution 14.21. Under the conformal transformation the four Maxwell

equations -


F[aJ3,v] F[aJ3 ,v]
= =

are clearly invariant. The other four Maxwell equations are

1 1
Fllv. \ g\ 2
[FIlV\ g\2] ,v JIl

= =
411 .

,v
Since

vQ 2 -lIn-vQ

2 "V

"V IIn
Fr
_

gr- g
fJ F =
f gr- g fJ F =
f F r

J3 J3
_

a a

g ==
Det( t3) f4g

11-
it follows that \ g!2FIlV =
\ gl2 Fllv so that

2 1

FI1V. = -
[I g1 2 FI1V] ,v =
411 JI1 =
411 gJ1fl Ja
1 gl t
,v

2-

417 (fgl1a ) fJa =
411 f JI1 ,

and therefore

FIlV =
411 JI1 .

;v
CHAPTER 15: SOLUTIONS

Solution 15.1. When the particle moves in the equatorial plane, we have

L =

p , which is obviously a conserved constant since


Pcf>
itself is.

( =
a/acp is a Killing vector so 'p
is conserved; see Problem 10.10.)

But by spherical symmetry the motion is always in the equatorial plane of

some rotated coordinate system. If we could write this


P as an invariant

quantity (under rotations) and then evaluated the invariant in the original

system, we would be done.

At some instant when the particle is at radius r the covariant

p has components Consider the "reduced"


4-velocity (Pt' Pr' PO' Pcp).
4-velocity p ed (0,0, rpO' rpcf?
= whose construction from
Pa (e.g. by

projection) is independent of 0 and cp. Now when the motion is equa-

torial
Po

0 and 0 =
71/2 we have

2 a{3-red -red cpcp r2 p 2


L =
g Pa P{3


cp

p =
conserved .

But in general


gOO r2p gcf>cf> 2 p
L = +
r2p =

PO +

sin

So this must be conserved in general, q.e.d.

Solution 15.2.
orient the coordi-
(a) Using the spherical symmetry of the metric .we
nate axes so that the particle is at 0 =
71/2 with 0 =
0 at r =
0; then

the unique solution of the geodesic equation

i (r

0) =


sin 0 cos 0 2
is 0 =
71/2 for all r.
404
CHAPTER 15: SOLUTIONS 405


(b) Using the constant of motion L of Problem 15.1, we have

(L .p )

() (gee pel 1.. 2


(1)
= =

'\
_

4 2
r s1n 0

Suppose that the unperturbed orbit is 0 11/2, L =


K. Let the
pcp
= =

particle be perturbed out of the plane of the orbit Le. 0 =


11/2 + 00,
L K + oL, pcp K + oPcp' To first order in oL and
oPcp and second
= =

order in 00, Equation (1) gives

[d<:)T :4 [2K8L

= -

2K8pcp

K (8e)2]
and hence

2 2
d (00) K
= _
00 .

2 4
dA r

This shows that 00 oscillates about 0 =


7T /2 and does not grow in time.

Hence the orbits are stably planar.

Solution 15.3. We have two constants for radial fall: u.u = -1 and


a/at =


= -
(1-2M/r)uO. From these we get

-(1-2M/r)(uo)2 (1_2M/r)-1(u )2

u.u = -1 =


[_ (1-2M/r)+ (1-2M/r)-1(dr/dt)2] (uo)2

[- (1-2M/r)+ (1-2M/r)-1 (dr/dt)2] (u o )2 (1-2M/r)2 .

Solving for (dr/dt)2 we find

(dr/dt)2 =
(1-2M/r)2 [1- (1-2M/r) (u o )-2] .

"" 1

A stationary observer measures time intervals dt =


(1-2M/r)2 dt and

radial distances dt =
(1-2M/r)-2 dr so he measures velocity

dr d
"""" = (1-2M/r)-1 ..l
dt

dt
406 PROBLEM BOOK IN RELATIVITY AND GRAVITATION

Notice that independent of the value of U this locally measured velocity


approaches the velocity of light as r approaches 2M.

Solution 15.4. From the first integrals of the geodesic equation (see
Problem 15.3) -

U = -
E =
constant

gOOu grr(u )2

+ =
-1

we get
O dt E
u = =
(1)
dr 1 -

2M/ r

d 1

r 2

d (E -1+ 2M/r)2 (2)

u =

The minus sign for the square root in Equation (2) corresponds to an in-

falling particle.

Case (i) . For this case dr/dr = 0 at r= R implies 2M/R = 1- E2 ,


i.e.

E < 1. Thus Equation (2) can be rewritten

dr
dr =
(3)

(2 2: t

This can be integrated in closed form to give

! t
fr
[2 ) l)J
R3 2

( ) (
r -1 2r
T = -

+ cos -

(4)
8M
\R R
2 R

where the constant of integration has been chosen so that r = 0 at r =


R.

It is convenient to rewrite Equation (4) in parametric form by introducing


the "cycloid parameter"
2r
( 1)
-1
11

cos -

so that we have

r = R (l+cos71) (71=0 at r=R) (5)



T =

( :) (71 + sin 17) .


(6)
CHAPTER 15: SOLUTIONS
407

Equation (1) gives


f (1 J
Edr

t dr 1

E dl1
2r d11 1_

1-
3 2

( 8M )

t (1+ cos 11) dl1


2M

(1- ) R
1- 4M [R(l + cos 11)]-1

From a table of integrals, we find

(7)

where the constant of integration has been chosen so that t = 0 at 11 =




i.e. at R. Note that t when tan (R/2M 1) 2

(11/ 2 ) i.e. when
= 00
-+ -+ -

r -+ 2M.

Case (ii) . Here E =


1 and Equation (2) gives

1-
3 2

( )
r =
2 L
- -

+ constant (8)
3 2M
and since

dt _


dt/dr
dr dr/dr
we get
1-

t = -


r3
( )
2M

4M
( 2M )


+ 2M log ( 1



+ cons tant .
(9)

(2 t -

Case (iii) .
By analogy with case (i), choose R such that

- -1 v
2M 2 2 00
- =
E -
1 =
(1 -

v) -
1 =

R 00

1 -v
oo

Then changing the sign of R in Equation (4) gives


_( : )t r ( :) 1)]

T =
+ -
cosh-
( + .
(10)
408 PROBLEM BOOK IN RELATIVITY AND GRAVITATION

Note that T = 0 at r =
0, and T = - 00
at r =
()(). Put


.,.,

cosh-
( +
1)
and get

r =
(cosh.,., -

1) (r =
0 at 11 =
0) (11)

.1

R3
T = -

( ) 8M
(sinh 11-11) (12)

The equation corresponding to Equation (7) is

R 2


11
+ 1 + coth
2" \t
( [
2M R R.
log +
1) 11+ (slnh 17-11) (13)
= -

.1

2M 4M


R 2
11

+ 1 -

coth -

2M 2

Note that t goes from -


00 to + 00 as r
goes from 00 to 2M.

Solution 15.5. Choose the equatorial plane to be () =


7T /2. Then u() =
O.

One first integral of the motion is the normalization of the 4-velocity

which implies

gOOu +
gCPCPu +
rur2 = -1 .
(1)

From the Killing vectors a/at and a / a,p two more firs t integrals are

L =
cons tant
u,p

u = -
E =
constant

- -

where Land E are respectively the angular momentum per unit rest

mass and energy per unit rest mass. (1)


quation now implies

,I..,p L 2

2 00 2 'P
dr 1 E

( )
g g
- - -


(2)
dT r

or with the explicit metric functions:


CHAPTER 15: SOLUTIONS 409

L2 i\ t
-1

{[ ( ) J( })

2 2
dr/dr =:t -1+E 1- 1- (3)
-"l

From u,p =
g,p,pL we have d,p/dr =
L/r 2 .
Combining this with Equation
(3) gives us finally


2{[ ]<1-2M/r)}
dr 2"

r 2 -1 L

:t -1+ E (1-2M/r)
-"l

d L

Solution 15.6. Choose 0 =


7T /2 and pO =
O. If A is an affine parameter

such that pr =
dr/dA and p =
d /dA =

r-2p ,then
dr =
pr
(fi

d,p p

From p'p
= 0 we get a relation amongst pr, pcP, and po which can be

solved for dr/d,p:



(dr/d,p)2 (1_ 2M/r) [y (1-2M/r)-1


r -

1/r ] (1)

where y ='
p /pconstant.

If we now introduce u ='
M/r, Equation (1)
becomes
2 2 2 3
(u,)2 (1-2u)-1 ( )


(1-2u) [yM _
u ] =
yM _

u + 2u

differentiate this equation get the very


where denotes d/ d,p. If we we

simple second-order equation


(3)

u + u =
3u .

In the limit M/b? 1 the zeroth order solution can be taken to be

the "straight line" solution r sin,p = b or




(M/b) sin,p .

Now write u = U + u +... with u ?1 so that Equation (3) is


o 1 1

approximately

ui +


3u =

3( Y sin2 =

( ) (l-cos 2 ・

410 PROBLEM BOOK IN RELATIVITY AND GRAVITATION

This can be solved by inspection, and we get


( ) sine/> +
( y (3+cos 2e/? .

Now we can find the total deflection angle by calculating the two angles

at which r = 00
(u= 0). These angles must satisfy:

2 sin cf> -

(M/b) (3 + cos 2cf?

cf> -

2(M/b) and cf> 7T + 2(M/b) .

Thus the total deflection angle is 4M/b.

Solution 15.7.

(a) The orbital equation was derived in Problem 15.5. If we introduce


- -

u == M/r and two new parameters f and U



in place of E, L, we can

rewrite the orbital equation as

2 2 2 2 2 3
(du/dcf? +(u-u o ) -f U


6u (u-u )
o o +2(u-u o ) ・

(See also MTW, Equation 25.47 or Weinberg, Equation 8.4.29.) The terms

on the right are the relativistic terms; in Newtonian gravitation theory,

the right s ide would be zero.

The lowest order solution, the Newtonian solution, is

u =

o (1 + f COS cf> )

where f is the eccentricity. Now note that the first correction term is

(m 2\

2 '" 3 2 '"
6uo(u- u o ) =
u(u o ( ) =
u (

\r )
(u-uO)3
(:; (3)
and the second is =
(9 so we can ignore it! Thus, first

order corrections can be found from

(du/dcf?2 + (1- 6u o ) (u -


)2 =

u f2
CHAPTER 15: SOLUTIONS 411


Now define tfr ==
(1- 6u o )2" </>, /1 =
u -


so that

(du/dtfr)2 + /12 =

u f2 /(1- 6u o ) .

Note that the solution is periodic in tfr so that r is periodic in



2"
tfr =
(1 -
6u o ) </>. One orbit corresponds therefore to tfr = 277 or

</> =
277(1- 6M/r o )- 2" so that the change in the periastron is given by

o</> =
3M
277 .

ro
per or b 1 t

[For another way of doing this, better suited to large eccentricity, see

A. S. Eddington, The Mathematical Theory of Relativity (Cambridge Uni-

versity Press (1922), Section 40).]


(b) From the Newtonian orbital equations, derive

du

2(1)(u) 2Eoo
( d<t>)

+ u + = .

2 2
L L


With (I) = -
M/r -

AM/r , this becomes

3 2Eoo
;Y
2amu

2 2mu
+ u --- =

2 2 2
L L L

Put this in the form

( +

(1-c)(u-u )2
2 (u-uO)3 O


constant.

2 2
conclude that 6au o m/L For
By comparing terms of order u c = .


nearly circular orbits L mr so that c =

6a/r .From our experience

know that the solution is periodic in the variable


with part (a) we

(1+ c)2" </> so that the angular spread from periastron to periastron is

277 277
</> =



2" 2


(1 c)

e ta

412 PROBLEM BOOK IN RELATIVITY AND GRAVITATION

and therefore
ocp =
3a
271 2
per or b 1 t

rO

(The periastron advances if a > 0 (oblate) and regresses if a < 0

(prolate). )

(c) In G 1 units 1.5 km. For Mercury .58x 10 km.

M0
= = = =

ro
So the relativistic advance is

ocp =
.105 seconds of arc per orbit


42 seconds of arc per century

since the orbital period is .241 years. If this advance were due to the

oblateness of the sun, then for the other planets we would have,

ocp =
(42 sec/century) (r Me rc. /r)2 (Period of Merc./Period)


(42 sec/century) (r M erc. /r)2.
Numerical answers:


Merc general relativity oblateness
r sec/ century sec/century

Mercury 1 42 42

Venus .536 8.8 4.7

Earth .386 3.9 1.51

Mars .245 1.25 .3

Solution 15.8. We solve the problem in two stages: First, compute the

laser frequency as measured by a static observer at r ("SOR"). Second,


compute the redshift between this observer and a static observer at

infinity (SOl).
Let DR
be the 4-velocity of the rocket and PR
the photon momentum

at r.

V D
SOR SOR PR


(1)

o DR PR
CHAPTER 15: SOLUTIONS 413

If v is the proper relative velocity between the rocket (R) and SOR
observer, then in the rocket frame Equation (1) &ives us


SOR yv o (l+v cosa)

V V
o o

or


SOR

yv o (l+v cosa) (2)


-2

where y ==
(1- v) . To
com,pute v, recall that circular orbits have an

angular velocity
o dcp
(IT

exactly the same as Kepler's Third Law in Newtonian gravity:


C )t (3)
= ・

(See Problem 17.4.) Thus

d t
rdcp
[ J

v= =
_


(4)
dt r(1-2M /r)
(1-2M/r)tdt
The observer at infinity then measures a frequency


SOR 1
v =

SOI 0


SOR (1- 2M/r)2

SOR


SOI

yv o (l+v cosa)(1-2M/r)2

where v is given by Equation (4).

Solution 15.9.

(a) From the first integral of the motion, one has the equation

(dr/dt)2 + V

(r) =
E2 (1)
414 PROBLEM BOOK IN RELATIVITY AND GRAVITATION

where V is as defined in Problem 15.11. If one now uses

dcp/dr =
pep =
L/r2 (2)

and defines the variable u == M/r, then an equation relating u and cp


may be obtained:

du E2 (1-2u)(1+L
2 2

(d)

u )

(3)
L2

where L == L/M. Differentiate Equation (3) with respect to cp:

2 2 -2
2u' u" = -
(2u L -

2 -
6 L 2u ) u' L
or
" 1 2
u + u =

-2
+ 3u ・
(4)

For large impact parameter, the right side of Equation (4) is small. Let

u =


+ fV, with the right hand side of Equation (4) =
B(f), then




A cos cp ,
A =
constant, (5)

for suitable choice of x, y, axes.

Substitution of U

into Equation (4) yields

-2 2 2 -2 2
(6)

v + v = L + 3A cos rp = L + A (1+ cos 2<1? .

The solution to Equation (6) is

A;
-2 2 -2 2
v =
L + A -
cos 2rp = L + 2A _A2 cos

rp

and hence
2 --2 2
u =
A cos cp -
A cos 2
'fJ + L + 2A . (7)

The asymptotes of the orbit are given by u =


0 i.e.

2 2
A cos cp -
A coscp -
B = 0

1. 2
B == + 2A
-2


2 2 2
A -

(A +4A B) B
(8)
cos cp = -


-.--
_


2A
CHAPTER 15: SOLUTIONS 415

Since B/ A is a small quantity, the solution to Equation (8) is

cp 17/2 -

B/ A .
(9)

Since there is equal deflection at both asymptotes, (cp 17/2, -

17 /2) the
total deflection angle is
cp 2B/A


(10)

If b is the distance of closest approach, then by the definition of A in

Equation (5)
A =
M/b .
(11)
Also,
2 2

2 =



b (E2 _


) =
b {32
2 2
M2(1_ 2)
2 2
M m M m

2 2
B =
M (1_{32) +
2M


M (1+{32) .
(12)
2 2 2
b (32 b b (32

Combining Equations (10), (11), (12) one has

1'11> gra v = 2M{1 + (32) .


(13)
b(32

(b) The differential equation describing the trajectory is


d. 2 Ze

(myr) =
ymr(c)
--
(14a)
dt 2


myr (c) == L =
constant (14b)


Ze
my
- -
== E =
constant ・ (14c)


From Equation (14b), d/dt =
(L/myr )d/dcp so Equation (14a) becomes

( )
d2 Z2e4 u=
Ze2E (15)
-u+ 1-
2 2
dcp2 L L

where u == l/r. The solution to Equation (15) is


416 PROBLEM BOOK IN RELATIVITY AND GRAVITATION


!. rcp
Ze E
u =


cos +

' (16a)
L _
Z2 e 4

which reduces, for large L,


T =

to
(1_ Z )2 4 , (16b)


1 Ze E
u = -


cos cp +

(17)

Now, the asymptotes are given by the equation


o = !. cos,l.. +
Ze E
b 2'

or r/> bZ 2E , so that the total deflection angle is


2bZe E
CPEM =


(18)

From the relations,



2 -"2
E =
m(l -

(3 )


132

2 b m
L =

1- (32
Equation (18) becomes
1.
2 2 2
2Ze (1-13 )
CPEM =
(19)
mb{32

Equations (13) and (19) differ because of the difference in the tensor

nature of gravity and vector nature of electromagnetism: the fields

Lorentz transform differently and this induces a different dependence of

the deflection on {3 when {3 is not small.

Solution 15.10. It is convenient to work in "outgoing Eddington-


Finkelstein coordinates": Define r* by dr* /dr =
(1- 2M/r)-1, that is
CHAPTER 15: SOLUTIONS 417

r* =
r + 2M log (r/2M -1) + constant

and define a retarded time coordinate


u =
t -
r .

The Schwarzschild metric in these coordinates is

2 2 2 2
ds =
-(1-2M/r)du -2dudr+r d0 .
(1)

Putting ds =
0, we see that outgoing radial photons travel along lines

u =
constant. The redshift is calculated by comparing the time between

reception of photons and emiss ion of photons:

Aoo ( t)oo ( u)oo ( u) du


l em.
em
= = = =
(2)
A ( r) em (r)em ( r)em dr
emitter

Note how we can relate times at


infinity to times at the emitter by
using u =
constant along the rays.

We must therefore find the component

du/dr =
U of the emitter's 4-velocity

U as a function of t.

Now u and t are both ignorable

coordinates and

ut =



constant = -
E (3)
observer observer
falling along r=r('r') atc::x:>
and U.U =
-1 implies



-2
U =
-E -

(E -1+2M/r)2 (4)

1 2M/r

therefore




gUUU U + gurU r

0 + [E+ (E 2 -1+ 2M/r)2] (1-2M/r)-1 (5)

and
418 PROBLEM BOOK IN RELATIVITY AND GRAVITATION

E [-E-CE2 -1+2M/r)!]

u =
gurU u
+ grrU r
= +

1. (6)
-2
= -
(E -1+ 2M/r)2 .

Thus


r 2
dr U -

(E -1 + 2M/r) 2" (1-2M/r)
(7)
= =

du u
- -


l'

E+(E -1+2M/r)2"

Near r =
2M, Equation (7) gives

du -2(1- 2M/r)-l dr -2(r/2M -

l)-l dr
u -4M log (r/2M -

1) + constant

1 -

2M/r "J
exp (-u/4M) .

Thus Equation (5) implies that, as r 2M



"J

+u/4M .

But u =
t + constant for an observer at fixed (large) r, so finally

Aemitter/Aoo exp (-t/4M) .

- -

S oluti on 15.11. F or particle motion u == E and == L are both con-



Ucp
stants of motion, so u.u =
-1 gives us

(dr/dr)2 + V

(r) =
E2 (la)
_

2 2
V ==
[(1- 2M/r) (1 + L /r )]2" .
(lb)

(See also MTW, Equation 25.15 or Weinbelg, Equation 8.4.13.) By setting


2 2
aV /ar =
0 we can find the maximum of V


V max =
[2 + 36 + (1,2 -12)(1-12/[,2)2" (2a)
54

L == L/M .
(2b)
CHAPTER 15: SOLUTIONS 419

If a particle has E > V
max'
it is captured. Thus, the limiting L for
capture is found by equating


E2 =

max

(3)

(a) For large E (large L), Equation (2a) reduces to


2 [2 + 36 + ([2 -12)(1-6/[2) =
[2 + 9
max
54 27

and Equation (3) becomes

-2 -2
L =
27E 9 (4)
crit

Corresponding to L
crit
is a critical impact parameter, b crit ' given by

L crl t ' L
crl t

b = =

crit 1
p 2
)2

(E _

where m is the particle mass. Capture takes place for impact parameters
b < b
crit
' Thus the cross section for capture then becomes

2 2 2
17L 17M L

. ,

crl t crlt
a =
17b = =

cap crit 2 2
(E2_m ) (E -

1)

..1-

17M
1 +
) (27E2 9)

-2 -2
E E

or

( 2)
2 2 2
17b
crl t

2717M 1 + ・
(5)
3E
- -

2 2
(b) For E 1, (small (3) we have E 1 + {3 and Equation (3)

can be approximated by

2 2
18 + 54(32 L + (L -

12)"2 / L .


We can solve for L to first order in {32 to find
420 PROBLEM BOOK IN RELATIVITY AND GRA VITA TION

L rit =
16(1+ 2{32) + fJ(f34)
and

2 17M2 L 2crlt .

1617M

a =
17 b =
(6)
crit

(3

capt
(E2 _

1)

Solution 15.12. Choose Paul's orbit to be in the () =


17/2 plane. Since

the orbit is circular u = 0 and from the r-component of the geodesic

equation, Du /dr =
0, we get

2 2

r t


cp )
tt(u) +
cpcp(u

and hence

rrtt
( t ( )2


M 1
w = = = =

rr 3 2
r 64M
cpcp

for Paul's orbit. Since dcp/dt =


1/8M and he traverses 2017 radians
between meetings, the coordinate time elapsed between meetings is


t16017M .

From gtt(U

)2 +
cp(ucp)2 =
-1 we get


dt 3M -2

( )

u == = 1 _


dr r

for Paul's orbit. Thus Paul measures an elapsed proper time

I'1T =
I'1t =
8017M 251.5M
Paul

For Peter's orbit we use the equations describing radial fall (Problem

15.4 ). The time for Peter to fall from R to r =


4M must be 1'1 t =
8017M.

We must find the values of Rand 11 corresponding to a lapse in t. of


8017M. We can then find Peter's proper time from Solution 15.4, Equation (6):

1-

R3
r 2
( ) (1)

Peter

8M
(11 + sin 11) ・
CHAPTER 15: SOLUTIONS 421

The equations for Rand 11 are Equations (7) and (5) of Solution 15.4

(X-1)t + tan TJ
4017 log + (X -l)t
[TJ + (TJ+ sin
TJ)] (2)

(X -I)! -

tan 1 11

4 =
X(l + cos 11) (3)
where X == R/2M.
Now use physical intuition: Peter travels outward for a considerable

time while Paul orbits. Therefore X should be quite large and 11 17.

Since logarithms are slowly varying functions, ignore the logarithm term

and approximate
xt
4017
0+ X17) ・

This gives X2 8017 and X 18.5. From Equation (3) then,

1 + cos 11 .216 implies 11 =


2.47. More accurately,

11 =
2.46029 X =

2 =
17.91737

Tpeter =
468.72M ・

Paul measures a much smaller (about half) proper time interval than

Peter. This makes sense because Paul remains in a highly relativistic


orbit deep within a gravitational well -
both factors make his clocks "go

slow." Peter spends most of his time at fairly small velocities and not

very deep in a gravitational well (remember R/2M 18).

Solution 15.13. We clearly need a transformation such that




e 2IL r2 (la)

2A 2 2IL ctr2
e dr =
e .
(lb)

When these are combined, we get a differential equation for the transforma-

tion r =
r (r) in terms of the known function A:
422 PROBLEM BOOK IN RELATIVITY AND GRAVITATION

dF

eA dr

This can immediately be integrated, and our solution is

r =
constant x exp

(J e dr) (2a)

211 2
/r2
e =
r .
(2b)

For the Schwarzschild metric e =
(1- 2M/r)-t and we have from

Equation (2a)

which gives
r =
constant x exp

(f (r
2 _
dr.!
2Mr)


r =
r(l + M/2i)2 (3)

if we choose the integration constant so that r r as r 00. Equation


(3) can also be expressed as

r =
[r -
M :t (r (r-2M))t] .
(4)

From Equation (2b) we have

21l (1 + M/2r)4
e =

(5)

From this it follows that the area of a sphere with rand t cons tant is

A =
(1+M/2r)4r2
f de sinedc,b =
41Tr2(1+M/2r)4. (6)

In constructing the embedding diagram note that (i) accord ing to

Equation (4) the mapping from r to r is double valued and (ii) the

coordinate r only describes the region of the Schwarzschild geometry

with r 2M (In Equation (4), r is complex for r < 2M.)


The embedding diagram for a surface of constant t and r requires a

function z(r) such that


CHAPTER 15: SOLUTIONS 423

2 2 2 2
(dz/dr)2] dr dcp2
2 2
ds =
dz + dr + r dcp2 =
[1 + + r


(1- 2M/r)-1 dr dcp2


+ r .

The solution is

Z =
[8M(r-2M)]Z .
(7)

This parabola is plotted


in the accompanying Z r)
figure. The embedding
surface is the paraboloid
generated by rotating this
curve about the r axis.

Solution 15.14.

(a) Let the boost be in the e'"



direction. For a velocity parameter

tfr =
tanh- {3 the transformation is given by

'" '" '" '"

At""t =
AX""x =
cosh I/r
'P
At"" x

AX""t =
sinh';'
so that


R"',"' ,"', = A a""A{3 ""AY""A ""R", (J"'
t x t x t x t x a fJ yu

cosh 41/r cosh 21/r 21/r


'P R"'''' + 'P sinh 'P
"'''' '" '" '"

txtx txxt

+ sinh 21/r 21/r


'P cosh 'P R"'''''''''' + sinh 41/r
'P R"''''''''''
xtxt
xttx

4 2 2 4

(cosh tfr -
2 sinh .;, cosh .;, + sinh tfr) Rt t
2 2

(cosh tfr -
sinh tfr)2 Rt t

R'" '" '" '"

txtx
424 PROBLEM BOOK IN RELATIVITY AND GRAVITATION

(b) In the Schwarzschild geometry the nonvanishing physical com-

ponents of the Riemann tensor are


2M/ r
""""

R R
t t-;
= = -

OcpOcp

R R .;}..." M/ r
" " ""

R R
""
= = - -

tJrO

t{ftO tcptcp

?cp";cp

and the components related to these by symmetry. To show that all physi-

cal components are invariant for a radial boost we can explicitly calculate
20 independent, boosted components in a straightforward manner. Another

way is to define, in the Schwarzschild geometry, the vectors


" '"
+ m == + 1
== e
t e-; n== -e-; eO ecp

and to observe that the Riemann tensor can be written as a sum of products

of the vectors:

1 M.
R =

2 3
{- (n A I) (n A I) + (m A m *) (m Am *)

+ Real [en A m) (I A m *) + (I Am *) (n A m)] I .

For a boost in the r-direction with velocity parameter tfr, clearly the

components of invariant. From cosh tfr sinh tfr and


m are
e't' e;,
= -

e-" =
cosh tfr e'" -
sinh tfr e" ' it follows that
r r t

1 =
e':' + e" =
e -tfr (e", + e",)
t r t r

n = -

e-;

etfr (et , +
e-;,)
so tha t

='
l OV'
, 1j1 nv ・

From the form of R then, it is clear that all components are invariant.

To see that this same conclusion does not apply to boosts in other

directions, notice that for a boost in the cp direction, with velocity


parameter tfr
CHAPTER 15: SOLUTIONS 425

R"""' p....,

t :i.., =
A a ""A {3 :£. R'" ' '"

r r t
rarfJ

sinht/J cosht/J (R;t;t +


R ; )


= -
(M/r ) sinht/J cosht/J

1= R'" 2
"''''


tr

Solution 15.15. In Kruskal coordinates, the Schwarzschild geometry is

given by

2 32M 2 2
e- r/2M (-dv +du )+
2 2
ds =
r d0 (1 a)

where
r/2M
(2
2 2

e = u _v .
(lb)

Setting v =
constant, and () =
11/2 in Equation (la) and equating the line
element to a Euclidean surface of revolution gives


-r 12M 2 2
32M 2
dcp2 [1 + (dz/dr)2] dr dcp2

e du + r = + r (2)

or

2 3 2
dz 32M du
( )
12M
( )
-r
1 + =
e .

dr r dr

We can calculate du/dr from Equation (lb) to arrive at

2 r 12M
dz
( )
r e
1 + = ・
(3)
2M 2
dr
(r/2M _1)er/2M + v

For embedding, z must be a real function, so we must have

r/2M
r/2M e
> 1 .
(4)
r/2M + 2
(r/2M_1)e v

From Equation (4), we conclude


r/2M log v .

"rhus, for Ivl > 1, there will be a minimum value of r inside which the

embedding breaks down.


426 PROBLEM BOOK IN RELATIVITY AND GRAVITATION

Geometrically, the necessary condition for embedding is that

d (circumference) < 2 17.
d (proper radius)

We now consider the general condition on dv /du which allows Euclidean

embedding: Suppose we have a slice described by v(u). Then

32rM3 [1 ( YJ
2 / 2M 2 2
difJ2

ds = e- -

du + r

(6)

32rM3 [1 ( fJ
[ (u-v ) :J
r/2M
e-
r / 2M (r / 8M ) e dr


2 2


dcp

and we require

[1 ( )2J: (u-v Y
r / 2M

2 e > (7)

at each r.

Solution 15.16. To avoid the suggestive connotations of coordinate names,

relabel rand t as follows: r z, t w. The metric then takes the form

2 2

[( ;:J
dz2+[(9/2)M(z-w)2]3d02.

ds =
_dw2+ (1)

Now, define a new coordinate, r, by






r = M (z -

w) ・
(2)

(We are led to do this because Equation (1) is manifestly spherically sym-

metric and therefore, the coefficient of d0 is a geometrically defined
curvature radius.) With r thus defined we have

(r + z =
w (3a)

dw =
dz -

(2r dr ・
(3b)
CHAPTER 15: SOLUTIONS 427

Substituting Equations (2), (3) into Equation (1) gives:


2 2M 2 2
( ) dz2 2 2
ds 1- 2
( ) ..L dz dr _..L dr d0


+ + r
r 2M 2M

(4 )

2 2
Next we want to try to diagonalize the dz , dz dr, and dr terms: Define
a coordinate t and a function F(r) by

z =
t + F(r) , (5)

substitute this into Equation (4):

-(1_ 2)(dt2+2F'dtdr+F'2dr2)

ds =

2 2
(2r dr (dt+ F'dr) 2 dr

+ 2 -'
+ r d0 , (6)

and choose F so that metric is diagonal, Le.

(1_ 2)F' ( 2 f =

(7)

With this choice for F', Equation (6) becomes

(1_ )-1 ( 2 )dr2


2M 2M
ds 2 2
= -

(1- )dt r

-1
2M 2 2
( )( )

+ 2 ..1:.. 1- dr2 _
dr + r d0
2M r 2M

or

(1_ 2 )dt2 (1_ 2 rl dr


2 2 2 2
ds = -
+ + r d0 (8)

which is the Schwarzschild metric in curvature coordinates and is, of

course, a static spacetime. The original coordinates of Equation (1) are

called "Lemaitre coordinates." Its time coordinate w measures the

proper time of freely-infalling observers; each observer moves along a

line z =
constant.

Solution 15.17. A coordinate-stationary observer in Problem 15.16 satis-

fies z =
constant (see solution to 15.16). By Equation (5) of Solution
15.16, this implies
428 PROBLEM BOOK IN RELATIVITY AND GRAVITATION

t 2M
-1

( 2M ) ( )
, r
dt = -
F dr = -
1- dr (1)

according to Equation (7) of 15.16. From this equation we get

(dr/dt)2 =
(2M/r)
(1- )2 M .
(2)

Comparison of Equation (2) with Equation (3) of Solution 15.3 (with U




for a particle falling from rest at infinity) shows that Equation (2) repre-

sents a radially falling particle, with unit energy-to-rest-mass ratio at

infinity.

Solution 15.18. The relativistic Bernoulli equation (Problem 14.7)

-1

( p ) (poon+oopo<)
p (1)

can be expressed in terms of the fluid's proper radial velocity

v =
(1- 2M/r)dr/dt, by using the relationship between U

and dr/dt

0 2 r 2
goo ( u )

grr(u) = -
1 ,

to yield



= 1 -
(1- )( ・
Y (2)

oon+: Y

One can now use rest mass conservation (uan \ g\2) ,a =


0 to obtain (after

again expressing u in terms of v)



v(1-2M/r)2 nr

constant ==


(3)

477

(1_v )2

Incidentally, in this solution n is a rest mass density, not a number

density: n
here
==
mp nusual-
CHAPTER 15: SOLUTIONS 429

Now, we would like to express n in terms of the sound speed a (using




Kn Y ; compare Solution 5.25):


dp yKny



_ _

dp 1
1 yKny

+ l(y-1)
This implies
2 2


1 a a (p + p)
yKn =



(4)
1 -
a /(y-1)
or


l!..-l

[YK(l_:2 Y (5)

1(y-l)

For the purposes of finding a self-consistent solution for some constant M

and some vCr), Equations (2), (3) and (5) can be thought of as defining M

as a function of r and v. We can then find the radius at which the flow

becomes supersonic by setting v =


a and setting dM =
O. First we must

express r in terms of a. Using Equations (3) and (4), (with v= a), we

get
2M
r =
(6)
/(y_1))2 (1-
2 2
1 -

(1- a a )r 00

where


00

(P ) 00

Substituting Equations (6), (5) into Equation (3), we get



2 2

( ) [yK(l- JFI
2 a a
4M a 1 -

M y=l a

/(y-l) (7)
_

417 2 2 2 2
[1- (l-a ) (1 a l(y-1)) roo ]

Now, we expand Equation (4)





00
1+
y-1

and work to lowest order in a. From d log M/da =


0, we have


2a 00

a =
, y 1= 5/3 (8a)
5-3y
430 PROBLEM BOOK IN RELATIVITY AND GRAVITATION


a =
2.3- 2

oo ' y

5/3 .
(8b)

Using Equations (8) and Equation (6), we conclude finally that the radius

at which the flow becomes supersonic is

5 3Y
( ) a2
2M

rs

Y 1= 5/3 (9a)

00

32 M
rs



5/3 (9b)
4a oo

Solut ion 15.19. The form of 0 (I) for a diagonal metric is

1 1

0(1) =
V.(V(I)) =
(_g)-2[(_g)2gzl'-"(I) {3 ] ,a ,
(1)

1 1 1
OO

n (- g) 2 g I {(-g) 2 grr (I) }

o (I) =
(- g) (I)
, 0 , 0


r ,

(2)

tJO(I) 1 ,1.,,1.,
+ {.( g) 2 g
_

,O} ,0 + {(-g) 2 g'f''f'(I) ,cp} ,cp] .

In the Schwarzschild metric, curvature coordinates,

gOO _(1_ 2 rl, (1_ 2)




i =
(3)

and gOO and gcpcp have their flat space form. It is clear that the third

and fourth terms of Equation (2) have their flat space form (with flat space

angular operator L ) and thus, Equation (2) has the form

-1
L2(1)
[ ( ]
2M 1 2 2M
0(1)
( 1-
) (I) + r 1-
) (I) + (4)

00
= - - - -

' 2 r 2
r r,
r ,r r

The field equations 0 =


0 then imply that

)( ],r
2M 2M \m L
(1-
2M\m
o =
(I)
,00
_.1

2 (1-

r2 1-
r/ '

_


2 r)"Z'

(5)

Let <I> have the form


CHAPTER 15: SOLUTIONS 431

l/1(r,t)Yfm?()'cP). (6)

Substituting Equation (6) into Equation (5), we have

1/1,00

(1_ 2) [(1- 2 1/1,rJ.r +


Vf(r)1/1 =
0 (7a)

where
2M 2M fef + 1 )

f(r) (1 )[ +

] (7b)
= -

r 3 2
r r

Solution 15.20. The Brans-Dicke field equations can be put into the form

GIlV + FIlV1(1) (I) 'a ) =


T/l v (la)

{3

.a' , ,

o (I) =
Til (lb)
11

where every term of F/lv is proportional to derivatives of the scalar

field (I). In vacuum, a solution of Equation (lb) is

(I) =
constant .
(2)

Substituting Equation (2) into (la), one obtains

GIlV =
0 ,

which are the vacuum Einstein equation. Consequently, the Schwarzschild


metric is a solution of the vacuum Brans-Dicke equations. (The most

general static, spherically symmetric vacuum metric in Brans-Dicke theory

has two arbitrary parameters corresponding to mass and scalar charge.


A spherical black hole has zero scalar charge and is the Schwarz schild

solution. )
CHAPTER 16: SOLUTIONS

Solution 16.1. Since an observer's time axis points along his 4-velocity,

if he is stationary in the (t, r, 0, cp) coordinate system then his time axis

must be along a I at (and normalized properly). Accordingly, take



f alat, where f is to be determined by normalization:
et)

2 a a 2 2 2(1)
1 '" '"
f f -
f e
eo. eo
' =
= = =

go 0

dt

at
-(I)
Thus f = e and
-<I> a
eo

e ' (1)
at

The three directions, alaO, alacp, alar are all orthogonal to alat and

choosing basis vectors along them, with unit normalization (same tech-

nique as above) yields

a -II a -1 a
e -Il r sin v
-1
(2)
L1
8"=e -Il )


'"

e'" err =
"37}
ecp



or

() 00 acp

The basis vectors in Equations (1) and (2) constitute an orthonormal


'"

proper reference frame for a stationary


observer. Dual I-forms, a,
are

found by the requirement '"

-a a
(3)
,eS>
<w =

(3

and are easily seen to be

6J
0 =
ecf>d't, w"; elLfu,
= 6Je =
elLr dO ,
6J = elLr sin 19 J;p. (4)

Solution 16.2. The observer makes measurements in his local orthonormal

frame -A -1 -1
_(I) L1

eO' ecp= (
'" '"

et=e



=er , eO=r
e r Slnu
ecp;
'"
Ih
'"


0 -:i.. -

c:,t =
e'¥dt, c:,r =
e dr, c:, =
rdO, w = r sinO dcp ・

432
CHAPTER 16: SOLUTI0NS 433

2A
Here the metric is as given in the introduction, except (1- 2m/r)-1 == e .

The volume of a fluid element which he measures is

" "

-r -() -,/..
V =
W AW AW
.- -- -

A2

e r sin () d r A d () A
dcp
A 2

e r sin () dr d() dcp .

The pressure at opposite points on the vertical walls of the fluid element

are the same, but because of the radial pressure gradient, the top and

bottom pressures are not:

-A-
I Ptop -

Pbottom I

Ip,,
,r
rl I pew
,r
I = .

Thus,
I F buoy.! =

\(Ptop Pbottom)X areal



Ip ,r e->"()Cl}w 1

Ip ,
re-AVI .

Since the direction of the buoyant force is radial,

-A 2m 2
Fb
uoy.

e p
,r
V&--r =

( 1- -

r, r ) p Ve.....r .

Using the OV equation for p ,


r'
we find


F =
-F bu 0 = _
(p+p)(m+41Tr p)Ve;- .

gr a v .
y . 1-

(1_ 2:nY

The Newtonian result is

pmV
p ,r Ve"r = --e"
2 r

Solution 16.3. A spherically symmetric metric can depend only on t, dt,

dcp2:

r, dr, and d(}2 + sin ()

ds
2 =
-A(r,t)dt

+ B(r,t)dr

+ 2C(r,t)drdt+ D(r,t)(d(}2+ sin 2(}dcp2).
434 PROBLEM BOOK IN RELATIVITY AND GRAVITATION


Choose a new radial variable r' D"2 (r, t). Dropping the prime, have

we

2 2 2 2
ds -E(r,t)dt F(r,t)dr 2 2
+ + 2G(r,t)drdt (d0 0dcl>2).

+ r +sin

Eliminate G by defining a new time variable

dt' =
H(r,t)[E(r,t)dt-G(r,t)dr]
where H is integrating factor chosen
an to make the right-hand side a

perfect differential. The metric then takes the standard form


(with the
prime dropped)

ds 2 = _
e2 <I?(r , t) dt 2
+ e
2 ..\(r, t) dr2 + r

(d(P + sin 2 e d<t>2) .

From Problem 9.20, the Einstein tensor has nontrivial


components

2e -2"\..\ (1- e -2..\)/r



Gt t ,/r + (1)

'" 2e -((I)+A) A /r (2)

t r , t

r r
'" (e -2A
= _

1)/r

+ 2e
-2A
(I) /r (3)
,r

2A
GEE
00

G22 =
e- +(1) 2 -(I) A +(1) /r-A /r)
((I) ,rr,r,r,r,r,r
'P'P

(4)
-2<1?

(..\, tt +..\,/ <1?, t"\' t)


When the field equations 0 imposed, Equation (2)


vacuum
Gap are

shows that A is independent of t and Equation (1) becomes

2dA =
dr
2A r
1 -e

from which we find

(l_ rlM
2 ..\
e =

(5)

The constant of integration has been chosen as 2M for convenience.

Equations (3) and (4) are now equivalent equations determining (I). (They
CHAPTER 16: SOLUTIONS 435

had to be

way to find
equivalent because of the identity
(I) is to add
G{3a;{3 =
0.) The quickest
Equation (1) to Equation (3). This gives
(I)


= -


so e
2 (1)

f(t) e -2A. By defining a new time variable
l'
dt' =
f2 (t) dt, f can be eliminated, so the final form of the metric is

-(1_ 2)dt2 (1_ 2 rl dr


2 2
(d{P+sin2 0d<t>2).

ds =
+ + r

This is the Schwarzschild metric which is static.



Note: The r coordinate used here is defined by 471 r =
surface area

of spheres defined by the spherical symmetry. If r were not a monotonic

function, it could not be used as a coordinate. However, a detailed in-

vestigation of this case (MTW, p. 845) shows there are no other solutions

Solution 16.4. From Birkhoff's Theorem (see Problem 16.3), we know that

the line element in vacuum for a spherically symmetric system is the

Schwarzschild solution. Inside a self-gravitating hollow sphere we there-

fore have a Schwarzschild solution, but here the point r =


0 is not a

singularity; therefore, the mass M (which is actually an integration con-

stant in the solution of Einstein's equations) must be set to zero to avoid

infinite values of the M/r terms in the metric. The metric inside the

hollow sphere is therefore a flat space metric and consequently a test

particle there experiences no forces.

Solution 16.5. The vacuum field equations for the scalar field tI> and

metric in the Brans-Dicke theory are


g/lv
!. ((I) (I) _!. (I)'Y)

afJ

Q'
2 cafJ
R =

2 (I)
(l)2,a'fJ fJ, Y
(1)

+ <p- (<p
,a;{3

o<P)
(3

0(1) = 0 .
(2)

If the standard form of the spherically symmetric static metric


2U 2 2A 2 2 2
(_ e dt + e dr + r d( ) is used, 0(1) = 0 becomes
436 PROBLEM BOOK IN RELATIVITY AND GRAVITATION


(e -Ar 2 (1) , r
) =
0 .
(3)

The general solution is


00

A-u



<P =
a dr + b .
(4)


2U 2A
Near r = 0 the metric functions e and e approach unity. (The
metric is Lorentzian near r =
0.) The integral in Equation (4) therefore
blows up near r = 0 and (I) takes the form

(I) air + b (5)

near the origin. But if a is nonzero, the right hand side of Equation (1)
2 2
is of order a Ir (This is not a coordinate effect; the trace of the right

2 2
hand side is of order a /r so by Equation (1) the scalar curvature blows
2 2
up as a Ir .) For a solution regular at the origin, then, a must be zero,

Le. (I) must be constant. For (I) constant, Equation (1) reduces to the

vacuum static spherically symmetric field equations of general relativity.


From Birkhoff's theorem we know that the only solution to these equations

which is well behaved at r 0 is


l1/1v'

g/1v

Solution 16.6. Examine the components at any point in an orthonormal

tetrad [e t e;, eO'


ec¥]. Spherical symmetry implies the components are in-

variant under rotations:

sina
e{j cosa
eO+ e
-sin a
e eO+ cosa e

and under rotations. The


The components Ttt, Tt; T;; are invariant

transforms like 2-dimensional vector under rotations


(TtO' Ttc¥)

pair
0 O.
and invariant. Thus
TtO:::: Tt and similarly T;O T;
= = =
so is not

The only 2-dimensional matrix invariant under rotations is a multiple of


the identity matrix, so

CHAPTER 16: SOLUTIONS
437

:).
TOO
T"'....... 1


()cp
TOe


"'.......

T 0
.......'"

cp() cpcp

Thus, there are four independent components:


Ttt' Tt;, T;; and

TOe T if;'

Solution 16.7. The standard form for the metric of a static, spherically
symmetric star is
2 2 <I> 2 2A 2 2 2
ds = _
e dt + e dr + r d0 (1)

The equation =

f3
is most easily evaluated by recalling that it is

equivalent to the first law of thermodynamics

p+p dn
dp/dr =

n dr
(2)

(see Problem 5.19) and the Euler equation

(p+p)ua ,.fJ u(3 =


-p
,a, fJ
-p u(3u a (3)

(see Problem 14.3). For a static star, both sides of Equation (2) vanish

identically. The only nonzero component of u is u and U'U =
-1 im-

P lies ut =
e-(I). Since only p ,r is nonzero, the only nontrivial compo-

nent of Equation (3) is

(p+p)u ;f3 uf3 -(p+p)r rf3 ua uf3



-P,r
= =


= -
(p + p) r u u
t t =
(p + p) (I) ,

Solution 16.8. Let mer) be a solution to the Newtonian equations for a

polytropic star, namely 2


dm/dr =
471 r p (1)

Gmp
dp/dr (2)

= --



KpY. (3)
438 PROBLEM BOOK IN RELATIVITY AND GRAVITATION

When y =
4/3 the star is neutrally stable; this is seen in the fact that

mer) =
mea r)

per) =
a p(a r)

per) =
a pea r)

is also a solution to Equations (1), (2), (3) and has the same total mass

m(oo) and adiabat K.


The relativistic version of Equation (1) is identical; Equation (3), the
given equation of state, is also identical by hypothesis. The relativistic
form of Equation (2) is [Weinberg, Equation 11.1.13 or MTW, Equation
23.22]
p\


471 -l
dp Gmp P r

( ) ( )

1+ 1+ 1- (4)
Jr.
= _

dr 2 P m

Since the star we are considering is nearly Newtonian, the terms in

brackets are all very close to unity. Hence, one can find an exact solu-

tion to Equation (4) which is nearly equal to the Newtonian solution to

Equation (2) [e. g. by substituting into Equation (4) to find a small correc-

tion to p and by iterating]. Call this solution mer), per), per). From
this generate -

mer) =
mea r)

per) =
a p(a r)
4"

per) =

pea r) .

Equations (1) and (3) remain equalities, but the ratio of the right hand
side of Equation (4) to the left hand side is, to lowest order,


'"
""
rig ht

[ ]
2m
left

1 + (a _

1) + 471(ar)3 + .
(5)
p mar

For a > 1, which corresponds to shrinking the star to smaller radius,

the right side is greater than the left side, hence the pressure gradient is

everywhere insufficient to support the star and it must go to a smaller


CHAPTER 16: SOLUTIONS 439

radius yet. This cannot be an


equilibrium because it would still be almost
Newtonian and the same argument would apply. Therefore, y =
4/3 is
unstable, and the range of stability must begin at 4/3 + c .

Solution 16.9. Increasing the of


mass a degenerate star makes the material

more compact until the Fermi


energy of the gas (electron gas in case of
white dwarf; neutron gas in the case of neutron star) rises to a relativistic

level. If A is the number of baryons in the star, and R is the radius of

the star, then the Fermi energy is


approximately (in the relativistic regime)

At
1Tc
EF '"
(1)

(In this equation we have ignored such details as the fact that there are

twice as many pressure


supporting fermions per baryon in a neutron star

as in a white dwarf.) The gravitational mass-energy per fermion is

approximately

GAm

EG (2)
'" -

where m is the mass of a baryon.


Note that both the compressional and gravitational energy depend in

the same way upon R. The decisive question is therefore the sign of the

total coefficient of R- . When it is positive, R will increase, the Fermi

energy will become nonrelativistic, the energy of compression will then



fall faster than R- and a stable equilibrium will be achieved at some

finite R. When the coefficient is negative, collapse will set in and con-

tinue. The critical number of baryons at which the star becomes unstable

is thus obtained by equating EG and E




1Ic 2 57

( )
A .

crlt.

10 .
(3)

Gm

Equation (3) corresponds to a critical limiting mass of


440 PROBLEM BOOK IN RELATIVITY AND GRAVITA TION

M crl t .





A '

c rl t .
2M
0 ・
(4)

and limiting radii. of


-1



for white dwarf

(me c ) -1

crit
"'-J
1rc A rit. x

(mBc ) for ne utron star


5 x 10 cm for white dwarf

2.7x 105cm for neutron star

Solution 16.10. At a particular point P let an observer construct a

sphere S passing through P and defined such that the geometry is the

same at all points on S. Then, from the line element, the measured area

of this sphere will be



A(r) =
411r .
(1)

The gradient of the scalar function A(r) is a I-form, I.e.

--

V A(r) == dA =
811 r dr .
(2)
Thus,

22-- 22
dA .
dA =
1611 r dr. dr =
1611 r (1- 2m/ r) (3)
and
1 ----

(A/11)2
(1 )
dA dA .

m r ( ) =


1611A'
(4)

Solution 16.11.

(a) Under the transformation, the metric

(1_ 2)dt2 (1_ 2 rldr2


2 2
dQ2
ds = _
+ + r (1)

becomes
2 2M 2 2 n2
ds
(1 -7 ) du -2dudr + r d li (2)
= - .

(b) We wish to compute the Einstein tensor G


a {3
when M =
M(u).
The Christoffel symbols are most easily found from the Lagrangian for

geodesics (see Problem 7.25):


CHAPTER 16: SpLUTIONS 441

2M

.2
12
(1



..
2 2. 2
2 ur + r () () 'P (3)
= - -
u -

+ r sin

The Euler-Lagrange equation

aL aL
ds ( ) ar
_

a r
= 0 (4)

gives the geodesic equation

u -

(M/r )u
2 2
+ r0 2 2
+ r sin () =
20 (5)

from which we read off the nonzero Christoffel symbols

u M U u 2
r r r

= _

r , =
r sin ()
uu


()() cPcP

Similar ly, Equation (4) with r replaced by u, () and cP gives the re-

maining Christoffel symbols

M 2M M' M
(1 ) (1 2r )

r =

rOO = -

_ _ -

uu ' ,
2 r r

() 1
(1- 2)
2 r

rcpcp r sin 0 r = r
= _ =-

ur

r () r'


0cpcp = -
sinO cos 0 ,
r cPr
cP
=!.

' r
CPOcp =
cot 0 ・

These all have their Schwarzschild values except r


ruu' From the

formula for the Ricci tensor (MTW, Equation 8 .51b)

rY
110 Y
rY
Ra{3 (log IgI2),a{3 + (log
IgI2),y a{3-r {3or

{3,y


ay

that all the will have their Schwarz schild values (i.e. zero)
we see
Ra{3
except 2
Ruu = -
2M'/r ・

The Ricci scalar is

R =
gUUR uu
= 0

so the only nonzero component of the Einstein tensor is


442 PROBLEM BOOK IN RELATIVITY AND GRAVITATION


G uu = -
2M'/r
which implies that

T uu = -
M'/411 r

and that all other components of T are zero. Since the vector k =
Vu,
.I

that is (1, 0, 0, 0),


ka =
is null, the stress-energy tensor

M'
T=--k k

417 r

corresponds to a pure radiation field. Thus a physical interpretation of


this solution (called the Vaidya metric) is the exterior metric of a spheri-
cal star whose mass is decreasing because of the energy carried off in

radiation.

Solution 16.12. The equations of stellar structure are (MTW, Equation 23.5
or Weinberg, Chapter 11.1)
-1
2111 2
ds 2
(1- 2)

r2d02 (1)

= _
e dt + dr +


m =
47Tr P dr (2)


dp (p + p) (m+ 411 r p)

(3)
dr r (r 2m) -


d(l) m + 417r p

(4)
dr

r (r -

2m)

Since P =

Po' a constant, Equation (2) gives

m =

411r3po/3 (5)
and
M =

411R3pO/3 , (6)

where R is the radius and M the total mass for the star. Substitute

Equation (5) in Equation (3) to get


CHAPTER 16: SOLUTION 443

dp =
rdr

471(PO + p) (tpo + p)

1- (871/3) r Po

Integrate and solve for p:

1 1


= (1 -

2Mr /R3)"2 -

(1 -

2M/R) "2
(7)
Po
3(1-2M/R)t _

(1-2Mr /R
2 3
)t
The constant of integration has been determined by demanding p

0 at

r =
R. The quantity (I) is most easily found from the equation:

d(l) _


d(l)/dr _



dp dp/dr Po
+ P

Integrate to find

(I)
e =

constant/(po+p)
3 .1 1 23t


(1 -

2M/R) 2 -

(1- 2Mr /R ) (8)


where Equation (7) has been used to fix the constant of integration so that

(I) (r R) 2M /R) t

e =
(1 _

(That is, so that the metric will match smoothly to the Schwarzschild

metric outside the star.)

We thus have a I-parameter family of stellar models, conveniently


parametrized by the central pressure


Pc 1 -

(1-2M/R)"2

(9)
Po
3(1- 2M/R)t -

The central pressure becomes infinite when


0 2: Y 1 =

- -

Le. the limiting value of R/2M IS 9/8.


444 PROBLEM BOOK IN RELATIVITY AND GRAVITATION

The dominant energy condition demands < so that the right


Pc Po'
s ide of Equation (9) is < 1. This implies R/2M > 4/3.

Solution 16.13. A relativistic zero-temperature Fermi gas has the equation

of state p =
1/3 p, so the equations of structure (see Problem 16.12)

read

4p(m+ 417 r p/3)
dp/dr =
(1)
r (r 2m) -


dm/dr =
417 r p .
(2)

Substitution of the suggested solution mer) = 3 r/14 into Equation (2)

gives per) =
(3/14)(417r

)-1; substitution of this p and m into Equa-

tion (1) verifies that they are a solution. From the equation of state

(1/14) (417 r ) -1.



per) = To find n(r) we use the relations for a relativistic

Fermi gas (see Problem 5.24)

[( 2 2 817 3

) 417 P dp
PF (3)
= - = -

3 3
o h 3h
fF

( )
2 2 817 3
417 P dp (4)
3 PF
= -





o h 4h

Eliminating (the Fermi momentum) from Equations (3)


and (4) gives
PF

3 4

( )
4h P
n(r) = k
817
3h3

and substitution of per) gives



3 4


817 4h 3

K (5)
n(r) K ' ・



3 871 14 471

2 3h

The total number of particles inside radius r IS


CHAPTER 16: SOLUTIONS 445

N(r)
L n(r)d (proper volume)


A 2

n(r)e 47Tr dr

1417

Kr (6)

which is finite for all r. [Note: In deriving Equation (6) we have used

rl 7/ .
2A
e == =

r (1_ 2 (r) =

The 3-geometry of the t =


constant hypersurface has the metric

2 2
(3)ds 2 =
g rr dr + r

d0 =
(7/4)dr

+ r

d0

This obviously has a conical singularity at the origin, because a 2-sphere


of circumference 217r will have a radius of (7 / 4)"2 r which is greater

than r.

The embedding equation for a radial cut is

(3 )ds 2 2 2 2

(7/4) dr =
dr + dz

which implies

Z = :t (3/4)"2 r

conical singularity
446 PROBLEM BOOK IN RELATIVITY AND GRAVITATION

Solution 16.14. The metric cannot be continuous across the shell in

curvature coordinates, because grr



(1- 2m(r)/r)-1 and mer) changes

across the shell. Therefore we use isotropic coordinates, in which the

metric will be continuous and (matching flat space inside to mass M out-

side) has the form


ds
2 Y dt2 + a 2 2 2
= -
e e (dr + r d0 ) (1)

2 -2
M M

eY =
(1 -

2r ) ( 1 +
2r ) r> R
(2a)
2 -2

M M
(1 -

2R ) ( 1+
2R ) r < R




(1+ ) r> R

(1+ 2) r < R . (2b)

In Equations (1) and (2) we have used the fact that the geometry is

Schwarzschild outside the shell and flat in the interior (Birkhoff's


Theorem -
see Problem 16.3). Also, the surface of the shell is located

at r =
R (in isotropic radial coordinate distance).
""

Next we define the integrated stress, A


as in the following invariant

manner R +(
R +(


""


""
""

(3)
TIS
za
dr dr
Aas TIS LlM e
= ・
== LIM

(-+ 0 (-+ 0
_( R-(

The components of A may be evaluated by using the Einstein field


to Equations (2) and (3), only
equations and by noting that, according
terms in Gfl with second derivatives of the metric potentials can

contribute.
(see
The components of Gflv for the metric of Equation (1) are

Problem 9.20) o d -a
(4a)

" -a

G o"""J ae """J--


(e
dr



"""J
0 (4b)

CHAPTER 16: SOLUTIONS
447

()
G 1>
1 1 d2

-at , d


G '
-a
) (e

-a
() e

1> +Y (e

,a
2 y)
- -


- -

2 dr
dr

where denotes the process of


discarding all terms not involving
second derivatives. Using Equations (3) and (4), and the Einstein field
equations, we get


O O RH






-(877)-1 LIM
f O
R-f
ea / 2
dr
d: (e-a)dr ......
-(877)-1 LIM

ea/2

J/ e-


R-f
or

AD", -M
I\.
(4c)
_



417R (1 + M/2R)3
'"

Ar....=Ar=O
r r (5)

'" '" R+f

()O' 1>:;..
l [
/2
A =
A = -
(817)-1 LIM !.e a
(e-

) _.!!. (e
-a
y ,) dr
'-P 2 2 dr
f 0
R-f
dr

J (+£

[ i (e -a)
/2
= -

(817)-1 LIM


dr

e-ay'
f O R-f

or

O M


M/2R
)( )

A 1
'"

()

+ (6)
817R
2 1- M/2R 2R

To have proper interpretation of Equations (4) to (6) we must remember

that R denotes the radius of the shell in isotropic coordinates. In

curvature coordinates, the radius is


R(l + M/2R)2 .
(7)

The proper surface mass density is just

M M


(8)
417 2 417R (1+ M/2R)4
448 PROBLEM BOOK IN RELATIVITY AND GRAVITA TION

The Newtonian limit of Equations (4) to (6) is


just

ADD -> _

41TR

'"

Ar"...r =

O 2


A ->

161TR

Solution 16.15. The dominant energy condition requires

ITOOI > ITkkl .
(1)

Using the results of Problem 16.14, and integrating Equation (1) through
the thin shell, we have the inequality

-3 -3
M M/2R M
( ) )

1+
2R

2 1 _

M/2R ( 1+
2R
(2)

where R is the isotropic radial coordinate. Letting x =


M/2R we have

2 -

2x > x or x < 2/3, giving R > M, or for the curvature radial

coordinate

9t =

(1+ 2Y i > M , (3)

about 1.04 Schwarzschild radii.

Solution 16.16. In isotropic coordinates (see Problem 16.14), the redshift


to radial infinity is

\nfinity (- goo)
Z =

-1 =


-1 =
(e- y /2) s -1
surface
(- gOO)J


1+ M/2R -
1 _


M/R
1 M/2R

1 -,M/2R

The integrated energy density and transverse pressure are respectively


CHAPTER 16: SOLUTIONS 449

-3
_AOo M


1+

_

2 \ 2R
41TR


A() =ACP =

IvI
M/2 1+
() cp
81TR2 ( 1-M/2R )( 2R ) .

Hence the ratio of integrated pressure to energy is

Ae
()

-=
M/4R = !.z
1 M/2R 4
_ADo

so the redshift to radial infinity is

= -4A
e /AO

e o .

If the dominant energy condition is satisfied, IA eel :S 1AOol, so z < 4.

Solution 16.17. The non-vanishing components of the fluid 4-velocity are



an d u
4> =
nt
£u,I.e.

1-
( )

u =
u + 0 .

at acjJ
Let
a a

+ n =
+ 0
at aif;
(t) (if; )

Then is a Killing vector, since 0 is a constant. By Problem 10.14,


we have 1



u =
V log I .

\2

where, since u.u =


-1, I It
. =
l/u


Thus, the Euler equation (see
Problem 14.3) for the fluid becomes

t -1
(p+p)V log(u) =
-Vp -

(V t p)u.
(u )

Since p is independent of t and 4>, it follows that V p =


O. Thus,


(p+p)V log u =
Vp .
450 PROBLEM BOOK IN RELATIVITY AND GRAVITATION

Solution 16.18. First, note that



dp =
(p+p)d log u
(hydrostatic equilib-
rium; see Problem 16.17). Next take the exterior derivative of this
equa-
tion, and use dd p =
0 to get


o =
d(p+p) A d log u


d(p+ p) A
(p+ p)-l dp
which implies that

dp A d p =
0 ,

and therefore that the surface p =


constant coincides with the surface

p =
constant.

Solution 16.19. The surface is a surface of constant p (that is p



0).
By Problem 16.18,

it is a surface of constant p, and also of constant u ,

since

dp ex du .

But, in the notation of Solution 16.17,

1 1 i
t 2
(u )-


1 ' =
12
Igtt+2gtcpO+ cpO1
so on the surface

gtt
+ 2g tcp O +
cp02 =
constant .

Problem 16.20. If P is the 4-momentum of a P hoton and U , u


00
are
em

the 4-velocities respectively of an emitting point on the star and of the

observer at 00, then the ratio of emitted and observed frequencies is


em p'u em


oo p'u oo

But U has only t and cp components, so


em

p'U em

Ptut +
PcpuCP =

Pt ut (1+0e)
CHAPTER 16: SOLUTIONS
451

where (} '" u jut is the angular


velocity of the star and e '"
p jPtis the
impact parameter of the photon relative to the axis of rotation. Note that
and
pcp Pt are conserved along the photon's trajectory. Also, p'u oo

Pt

since u =
1 for an observer at infinity and thus

t 1 + oe
z =
u (1+0e) -
1 =


_



I gtt + 20g t 4> +0
I:
4> m

Since gtt +
2(}gt (}2 + is constant on the surface of a rigidly rotating
star (see Problem 16.19), the variation in z across the surface of the

s tar is
= z
o e


I gtt + 20g tcp + 0 I :urface
4>

All photons reaching an observer on the axis must have 0, I.e.


pcp

e =
0, Le. = e
O. Thus = z
0 for such an observer.

Solution 16.21. The configuration of perfect fluid is convectively stable

if its mass (Le. rest mass + energy) does not change under a redistribu-

tion of baryons. We first calculate its change in mass, oM, due to an

addition of a A baryons from infinity.

A distant astrophysicist drops a total mass-energy OA + W onto


/lB 0
the fluid, where is the average rest of the baryons and is
/lB
mass
Wo
additional energy to be used for injecting the baryons into the fluid. A

static observer at radius r catches the baryons and energy and measures

a total mass-energy of

W =

uOpo =
e-(I)CI1 B
oA+ W o ) (1)

-(I)
where e is the redshift factor and comes from the metric

2 2
ds = _


<Pdt 2 + (1-2m/r)-l dr + r

d0


(2)

With energy W, the local observer must heat and compress the baryons
to local thermodynamic conditions and then open a space for them in the

star
452 PROBLEM BOOK IN RELATIVITY AND GRAVITATION


OA

(p/n)oA (3a)
(local conditions)


open

p8v =
(p/n)oA (3b)

where n is baryon number density. (Note that, since we have assumed

that the star is already at an extremum of total


mass-energy, the change
in energy of the star due to readjustment of its structure as mass is dis-

placed for the oA baryons can be neglected.) The excess energy left

over is

Wex(r) =
W -

(WOA +W open )

(p+p) SA

e-<I>V-tBSA+WO) -



(4)

The local observer then converts a fraction (1 -


e (I)) of W
excess
into

kinetic energy so that he can throw the rest to the observer at infinity,

who then catches energy

W W
<I> <I> (p+p)
ex
(00) =

ex
(r)e =
IL B uA+ W o -e

oA .
(5)

The observer at infinity, therefore, measures a mass increase of the star,

oM, of
(P+P) OA ap
oM =
e<I> n

e(l)
( )
an s
oA (6)

where the second relation follows from the first law of thermodynamics.

For convective stability we require oM to be independent of r, I.e.

e(l)(p+ p)/n =
constant. (7)

We can easily show, using the Euler equation,

dp d<P

dr

-(p+p)-
dr
(8)

that Equation (7) is equivalent to adiabatic gradients of thermodynamic


variables. Taking the derivative of Equation (7) with respect to r gives
CHAPTER 16: SOLUTIONS 453

(p + p) dn dp dp d<I>
_

+ + + (p + p ) =
0 .
(9)
n dr dr dr dr

Using Equation (8), we can put Equation (9) in the form

dp (P+P)dn ap
( )
dn
dr

dr

dr'
(10)
n an s

From this equation, it is obvious that the fluid must be isentropic.

Solution 16.22. Begin with the Euler equation for a rigidly rotating star:

(p + p)-l ap/axll =
a log uO /ax ll (1)

and compare it with the equation for constant injection energy

(p + p)/nuo =
constant. (2)

Take the derivative of the logarithm of Equation (2) and substitute in

from Equation (1), to obtain


(p + p)-l ap/ax ll = n- an/axil .
(3)

Equation (3) is the first law of thermodynamics specialized to isentropic

systems.

Solution 16.23. Since TIlv vanishes outside the star, we can take the

volume integral to be over all space at time t. The field equations imply

81T(TIl -1-0 11 T\ =
RIlv
\ 2 v)

so the mass integral is

1= -

f (2TIlv _0IlT)

Vd3
Il


-(477)-1
f RlLv Vd3IIL

(477)-1
f 1L; vd3IIL
f IL;V


(877)-1 d I
ILV

454 PROBLEM BOOK IN RELATIVITY AND GRAVITATION

[The third equality follows from Problem 10.6; the last equality follows

from Stokes' theorem, Problem 8. 10 (c).] If we choose coordinates (t,r,O,cp)


which reduce to spherical polar coordinates at infinity, then

d2 = -1 E
a(x

,x(32 dOd<t> =


sinOdOd<t>
Il V Il va {3
2! a(O, cp )
and thus


/l;Vd2 /lV2e;rr =
sinOdOd<t> .

The asymptotic form of the metric is

ds
2 =

_(1_ 2)dt2
_
4J
s n20dtd<t> +
(1+ 2)(dr2+r2d02+r2sin20d<t>2).

Since the only nonzero component of is = 1
(t)

gt;r gt;r
ct t t

S ,

+ r
ar
ga =
0 + r
tr


gtt,r

M/r ・

For the mass integral then


2M
(811)-1

I =


r sin 0 dO dcp =

as required. Similarly, we have

S =

f T/lv V(<t?d3 /l
J R/lv V(<t?d3 /l (1671)-1 fR


(871r -

/l(<t?d3 /l'
Since
d3 <t> 0 =
the second term vanishes and

f /l; (1671)-1
f d2 /lV

S (871r vd3 0 /l = -
/l;V

= -

f 2e;rr

= -
(1671)-1 sinOdOd<t> ・
CHAPTER 16: SOLUTIONS
455

But

g tjr ,r
+r

ar
t cpr
a=o+rt

gtt r t<l>r + gt<l> r<I><I>r
1 -2J
( )

- -

g r/... + - -

gr/...r/...
2 t'f-' , r 3 2 'f-''f-' ' r

2 2
= -
3J sin 0/r ,

and thus



S =
(1617)-1 61 sin 0dOd<l> =
1 ・

16.24. V 3
Soluti n In curvature coordinates,
v(t) 8 =


and d
IfL

0 gl2
1 dr dO dcp, thus the integral is

J (2TfLv -8 fLv
T) v(t)d3IfL
= -

J (2T\-T)e<l>+Ar2sinOdOd<l>dr.
= -

But

T\ =
(p+p)utu t +
po\ =
-p

T = -
(p + p) + 4p =
3p -

p .

Since everything is independent of 0 and cp, the integration over angles


gives 417 and therefore

I =

[ (p+3p)e<l>+A417r2dr .
(1)

We can transform this by integration by parts. Since

1r

mer) =
417r pdr (2)

we have
R R

L pe<l>+A417r 2 dr [me<l>+A]
-1 m(e<l>+AYdr (3)

456 PROBLEM BOOK IN RELATIVITY AND GRAVITATION

where a prime denotes d/dr. The stellar structure equations tell us p'


so we write

R R

I 3p e<l>+A4r r [p e<l>+A47Tr3]


dr = _

417 r3(p e(l)+A)' dr .


(4)
o 0

The first term on the right in Equation (3) is meR) =


M, since m =
0 at

r =
0 and (I) = -
A at r =
R. The first term on the right in Equation (4)

is zero since p

0 at r =
R. Thus, I =
M provided the sum of the re-

maining terms in Equations (3) and (4) vanish. This sum is

R R

L L

m(e<l>+AYdr+ 471r (pe <l>+AYdr

L <I>+A[m(<I>'+A')+ 471 3

dr e r { p'+ p(<I>'+ A ')1] ・
(5)
Now
2A
e =
(1- 2m/r)-1
so we get

A' =
(417r p-m)
r(r 2m) -

If this res ult and the relations



m + 417 r p
(1)'=
r(r -

2m)




(p+ p) (m+ 417 r p)

r(r 2m) -

are substituted in the expression in Equation (5), all terms cancel and it

red uces to zero as required.

Solution 16.25. Let (), cp and the radial coordinate R be comoving

with the fluid of the star. We can then write the metric as

2 i
ds gtt dt2 +
gtR dRdt +
gRR dR2 +
gti dtdx



gRi dR dx + dxidx j (i,j (), cp)
gij

CHAPTER 16: SOLUTIONS 457

where i, j =
(), cp. If the metric is spherically symmetric, then for dt =
dR

0 we must have

2 j
gijdxidx (R,t)(d()2+ sin 2()dcp2)

ds = =
r .

Furthermore, gti and gRi define vectors in the 2-dimensional (), cp


space, so by the isotropy of spherical shells
gti and gRi must vanish.
Finally, we can choose a new time coordinate T =
T(t, r) to remove the

gtr term, without affecting the comoving status of the radial coordinate,
and we have
2 2 2 2 n2
ds
gTTdT +
gRR dR + r ( R, T)d

Thus in general we can have both comoving coordinates and a diagonal


metric.

N ow consider the motion of a fluid element. From the fluid 4-velocity

n =
(- gTT)-Z e

we have the fluid acceleration:


T 2 RR

du T 2 R 1
V r )
dT+(u)
e e (u

g gTT,R.
= -

nn R TT 2 R

It follows then that the fluid acceleration is zero (and hence the pressure

gradient vanishes) if and only gTT is a function of T only. But if that


is so, we can define a new time coordinate r(T) from the equation

dr/dT =
(- gTT)Z

representing proper time for the fluid. Thus the three nice properties are

all satisfied if and only if the pressure gradient is zero; since the pressure

must be zero on the surface of the star, a vanishing pressure gradient is

equivalent to zero pressure.


458 PROBLEM BOOK IN RELATIVITY AND GRAVITATION

Solution 16.26.

(a) The first law of thermodynamics (see Problem 5.19) in the comoving

frame is
--L=n
p+p n

where n is baryon density. The number of baryons in a shell of thickness


dR is (411 nr

e )dR, so the law of conservation of baryons in this shell

2 A
a(nr e )jat =

and thus

-L = _
2i _
A .

p+p r

We can now combine this with

t 2 -(I)-A ., ・ '


",

0 -


e (r -

r'.i'
m'

r A)

(see Problem 9.20) and with

T{3R;{3 =
0 =
p' + (p+ p )<1>'
to arrive at

2r r' rp'
( )

-p

(p+p) -+ -,
r r
+ --,

This relation can now be substituted in the expression for m,



2 2
m =
411 (2rir'p+r i'p+r r'p)dR

= -

fR(2rriP+r2i'P+r2iP')dR
411

-411I


(r rp )'dR.


Since r rp = 0 at R =
0 it follows that


2 .

m = -
411 r pr .
CHAPTER 16: SOLUTIONS
459

"-

(b) From G\ = -
817P (see Problem 9.20) we have


817pr =
1- r,2 e -2A + i
2 -2<1>
e + 2ri e
-2<1>
A + 2r e -2A(r'A' -

r") .


N ow use G =
0 to eliminate A, and note that 8m pr' is an exact

differential with res pect to R

2, , , 3 -2A .2, -2(1) -2<1>


8 17P r (r )

.,


'm'
r r 2 rr (r

e + r r e + e r ':i'

+ 2rr' e -2A(r'A' -

r")

[ r-e -2A r 2 2 -2(1)


, '




(r ) +r r ( ) e .

Integration gives the desired relation


2m =
r(1- e -2A(r')2 + i- e -2(1))
2 2

r(1 -

r + U ) .

Solution 16.27. From Problem 16.26 we have

dr/dr =
U (1)


dm/dr = -
417 r p U (2)


2:
U = =

: :!:
[r2 (2 1) ] + -

(3)

The first two equations show that dr/dr and dm/dr have opposite signs,
so for the shell to pass from 2m/r < 1 to 2m/r > 1 it must be that

i- O, m>O

upon passage through r =


2m. From Equation (3) then

:-[r2+ e -1)J2: ・

( 2;0 -1)

The radius therefore will continue to fall as long as r + > 0,
but as long as the radius is decreasing, the mass must be increasing and
2m
the factor -

1 continues to increase, so the collapse cannot stop.



460 PROBLEM BOOK IN REIIATIVITY AND GRAVITATION

Suppose the shell has reached the point 2m/r -


1 =
c > O. Since this
factor continues to increase, then

.1-
dr/dr :s c 2


and the shell will reach r =
0 in a proper time :S 2m/c"2. [This solution
is due to J. M. Bardeen.]

Solution 16.28. In Problem 16.25 we saw that we can choose <I> =


0 for

pressure-free collapse. From G =
0 (see Problem 9.20) with (I) 0

we have then
o =
2r'r + 1_r,2 e -2A + i-

2 2

2r r + 1 -
1 + U

and therefore (see Problem 16.26)


d r M
r = =

dr2 r

Solution 16.29. From the dynamical equation for mass (see Problem 16.26)

2.
m = -
4 1TP r r =

mass is clearly time independent, as is


physically obvious. By differenti-

ating the 1 equation in Problem 16.26 we get

2rr 2m; 2i 2ifr+ m




+ =

r 'r2

and using the result of Problem 16.28 we see that 1, too, is time

independent.

The dynamical equation for r(R, T) is now a simple first order equa-

tion and can be solved parametrically:



Case (i) 1 -
1 < 0:

m(b)
r =
(1+cOSl1) (1a)
1_r2
CHAPTER 16: SOLUTIONS 461

m (b )
T =
3 (11 + sin 11) + F (b ) (lb)

(1-r2)
where F(b) is an arbitrary function.

Case (ii) r2 -
1 =
0:



3 2"
r =


(m(b)) [g(b) -T] (2)

where g(b) is an arbitrary function.

Case (iii) r 2 -
1 > 0:

m(b) (cosh 11- 1 )


r =
(3a)
r2 -1

m(b)
T =

(sinh 11-11) + H(b) (3b)

cr _1)2
where H(b) is an arbitrary function.
Note that there are three free functions -
i.e. functions that we can

choose by choosing the physical situation. By choosing m(b) we can

choose the distribution of mass at some initial time and forever after. The

integration "constants" F(b), g(b), H(b) correspond to the choice of

the r value of each fluid element on an initial hypersurface, Le. the

choice of r (b, t =
0).

The choice of r (b) corresponds to the choice of the velocity of a

fluid element initial hypersurface. By comparison with the equation


on an


for a radial geodesic, r 1 can be thought of as the conserved

"energy at 00" for the fluid shell. Therefore, it should not be surprising

that there are three solution regimes: we can choose to give a fluid shell

less than the escape velocity (a), precisely the escape velocity (b),

or greater than the escape velocity (c). Note that the sign of t can be

reversed in the above solutions so that, for example, solution (c) can

correspond to shells falling inward. Note that in general if the three free

functions are not chosen carefully, mass shells are going to cross!
462 PROBLEM BOOK IN RELATIVITY AND GRAVITATION

Solution 16.30.

The stress tensor is The assumptions of spheri-


(i)
Ta{3 pUa
u{3'

cal symmetry and uniform density are equivalent to assuming

isotropy and homogeneity, so the metric must be a Friedmann

solution:

2 2 2 2 2
ds = _
dr2 + a

(r) [dX + l2(X) (d0 + sin 0 dcP )] (1)

where

sinx ,
k = 1

l= X k =

sinh X ,
k = -1 .

The surface of the star is at some constant value of the "radial"

coordinate, X

Xo say. From Equation (1), the proper circum-

ferential radius of the star is R =


a(r) l(x 0). The Einstein field

equations (s ee Problems 19.16, 19.18) for press ure


zero are

(a:)2
k 817p

--+- (2)

2 3


pa

constant. (3)

Equation (2) can be rewritten as


kl2CxO)

= l..
817
[( R ) +


] . (4)

Thus, if R =
0 at any finite value of R, k must be +1 so
,r

that p is positive. Conversely, if k =


-1, R ,

is never zero.

The case k 0 corresponds to R 0 as R 00 .



(ii) The Euler equation for zero pressure implies V nn =


0, Le. each

fluid element moves along a geodesic. By spherical symmetry


the geodesics are radial.
CHAPTER 16: SOLUTIONS 463

(iii) We shall do the matching for k =


+ 1; the cases k =
0 or k =-1

are similar. For k= +1, Equations (2) and (3) give

(a
,r
)2 =


/a -
1 (5)

where the constant in Equation (3) has been fixed by setting


a =

am

maximum of a, when a

= O. Equation (5) can be in-

tegrated as it stands; it is convenient, however, to introduce a

new time parameter by


dr =
ad 11 .
(6)

Equations (5) and (6) then give


a =

2 am(l+cos 11) (7)

am (17 + sin 17) (8)


T =

The constants of integration have been chosen to make a =

am
and r =
0 at 11 =
0, and to make a =
0 and r = 11a /2 at 11= 11.

The intrinsic 3-geometry of the surface of the star, as measured

from the interior, is found by putting X in Equation (1):


Xo

2 2
(3)ds 2 = _
dr +
sin2xo d0


(r)

(9)

(11) (- d11 + sin2xo d0 )


a .

The exterior metric is

2 2 -1 2 2 n2
ds = -
(1- 2M/r) dt + (1 -
2M / r) dr + r d .
(10)

The surface of the star is at r =


R(r), and from the equations for
radial geodesics (see Problem 15.4), R(r) is given by

R =
R (1+

cos 17) (11)
464 PROBLEM BOOK IN RELATIVITY AND GRAVITATION

.1


r =

(8) (11+ sin 11) (12)

t dt (1- 2M/R i )2
u =
(13)
_

dr 1 -

2M/R

Thus the geometry of the surface is

( 3 )ds 2 2 2 2


_dr + R (r)d0
(14)

( sZ ) }
R. R.
2 2 2 2
= -
(l+cOS17) d17 + (I+COS17) dO ・

Comparison with Equation (9) shows that the 3-geometries match


provided we identify

Ri =

am sinxo' 2M =

am sin3xo .
(15)

C ).In
compute t h e
.. -

Th e
. .

N ow ex t rlnSlC curvature K . .
t h e Interior.
lJ

normal to the surface is


o =
a- a/aX (16)

(recall that 0'0 =


1), while the vectors u =
a/ar, alae and
.a / a lie in the surface. Let the indices i, j range over

r, e ,
. Then


K..=-e..V.o=-e..ra.e
1 J 1 nJ a =_g.la r nJ

lJ

1 1

-r
inj
= -

2 (gin,j + gij,n gnj,i)



= -

2 gij,n (17)

-1
since g In


a g.1
= O. From the metric we see that


K =

)K O Kcf? =

q} 0=
(IS)

) 2

OK

sin- 0 = -

am (l+cOS17)sinxo COSXo' (19)


CHAPTER 16: SOLUTIONS 465

In the exterior metric, the 4-velocity is


t r
U u e + u e .

t r

The normal vector

n =
nte t + nre r

satisfies

t 2 r
n.n =
1 =

gtt(n) + )
r(n (20)

t r
n'u =
O =
n u

+ n u

. (21)
Since

u.u =
-1 =
gtt(u t )2 + grr (u )2 r
(22)
and
rr -1 tt -l
g ( ) r (g ) 1 -
2M/ r
gtt
_ _ -
- - - -
- =

the equations above imply that

t r
n =


' n=-u.

(23)

Let i, j range over " (), cp as before. Then Equation (17)

holds for the exterior metric, since g.In =


n.e. =
O. Equation (18)

holds for the exterior metric because ere,



U'U =
-1, ere() =

O. Also,
e,. ecp e()' cp
= =

(+ ) _!. (r 2 ) nr
K 1K

2 1 2
(r )


rf....rf.... sin () = - - =

'P'P 2,n 2 ,r


rUt
= -
R (1- 2M/R i )2

= -
R i (1+ cos 71)(1 -

2M/R i ) t

am sinxocosxo (1+ cos 71)


= -

where we have used Equations (23), (13), (11) and (15). Thus

KlJ-r) =
K lJ :-)and the proof is complete.
CHAPTER 17: SOLUTIONS

Solution 17.1. One finds the mass and angular momentum of an asymptoti-

cally Minkowskian (flat) metric by finding a coordinate system in which it

takes the form

goo
= -

2 +(.')(r-2))
_ (1)


-kx -3 (2)
7(.')(r )}'

\4 Ejkl

gOj

The constants M and 'Sk which occur in the expansion are then the mass

and intrinsic angular momentum (see Weinberg, Section 9.4 or MTW, Sec-

tion 19.3).
Expanding the Kerr metric in Boyer-Lindquist coordinates in powers of

r- gives the leading terms

(1_. 2+..) dt (4 M 0+") dtdcp


2 2 2
ds = _ -

Sin

+ (1+...)[dr2+r2(d02+sin20d<p2)] .

to cartesian coordinates by sin 0 <p, sin 0 sin <p,


Transforming x == r cos y == r

z == r cosO gives

_(1_ 2 +")dt2 _( +..) M



ds =
(xdy-ydx)dt
(3)
2 2
+ (1+ ...)(dx + dy2 + dz )

and comparison with Equations (1) and (2) gives at once

M=M

S =
aM

466
CHAPTER 17: SOLUTIONS 467

Solution 17.2. Suppose the car has mass m and length L before crush-

ing and length L' and "lumpiness" h afterward, and the hole has mass

M. Suppose the internal stress per mass that steel can support is

( '"oJ
0.1 electron volt per nuclear mass; see Problem 5.6). Comparing

gravitational pressure forces to internal stresses, we see that the condi-

tions for crushing are

( )L > ( (condition for crushing to begin) (1)

GM (condition
( L,2) h > for lumps no larger
(2)
than h after crushing ends).

Since cars typically crush to .1 of their original dimension, we see that

the two inequalities are equivalent for h/L' =


.1, which is a reasonable
9 18
value. Taking L' '"oJ
100 cm and '"oJ
10 ergs/gm gives M.2: 10 gm.


The time to crush wreck is set by the free-fall time (L /GM)"2

'"oJ

5 8
10- sec, so on the order of 10 can be processed per hour.


L L
. . . t

. V . V

before affer

Solution 17.3. For the rocket ship to be moving along a timelike world-

line, its 4-velocity must satisfy (from the Schwarzschild metric)

1 =
-U'U =

(1- 2rM)( / (1_ 2 rl ( Y:_r2( )2


_
_r

Sin20( / .
468 PROBLEM BOOK IN RELATIVITY AND GRAVITA TION

Inside the horizon all of these terms are negative, except the one in

(dr/dr)2, so

1 2

(2 -1)- ( ) : > 1 .

We also know (e.g. from the Eddington-Finkelstein or Kruskal picture) that

the sign of dr/dr must be negative for a "future directed" (i.e. physical)

observer and thus,



2"
dr < -

(2 1) -

dr

and

-t

2M

max
= -

( -




) dr

2M

[ )J
.1 .1

(2M-r)2 +Mcos-
( -1


r = 17M ・

2M

Solution 17.4. We derive the case for a Kerr black hole and then obtain

the Schwarzschild case by setting a =


O. The Kerr metric is (in the usual

Boyer-Lindquist coordinates)

r (4Mar Si O)

(1_ 2 )dt2

ds = _
dtd4> -
+ dr

(1)

(r2 sin )O
2 2

d4>2
2 r
+ I d0 + + a + 2Ma sin 0

where
2 2 2 2

L.1 == r


2Mr + a ,
1 == r + a cos () .

The geodesic equation is



X IL /dr

+ r lL

uf3 0 (2)
{3u

but the first term vanishes since u



and uCP, the nonvanishing compo-

nents of u, are both constant. The r component of Equation (2) is

o r

u{3 (r rtt dt2 + 2r
rt4>dt d4> + r
r4>4>d4>2) (dr)-2 (3)
ra{3u
= = .
CHAPTER 17: SOLUTIONS 469

Since the metric coefficients depend only on rand (), the relevant l's
in the equatorial (() =
TT
12) plane are

1 2
lrtt = -

2 gtt,r

MI r


21rtcp 2Ma/r (4)

gtcp,r
= = -

rr
= - =

M ,r
2r
-r .

So if n ==
dcp/dt, Equations (3) and (4) give

( j
2 Ma 2Ma n M
o =
0 _ _
+ , (5)
2 2 2
r r r

a quadratic whose two roots (corresponding to direct and retrograde orbits)


are
1 3 1
n =
M2 I(:!:: r
2 +
aM2) .
(6)

2 3
For a =
0 this becomes 0 =
M/r which (by coincidence in these

coordinates) is exactly the Newtonian Kepler's law.

Solution 17.5. The Reissner-Nordstrom metric is

2 2 2
dcp2)
1 2 2
ds = _
Adt + A- dr + r (d()2 + sin ()


where A == 1- 2M/r + Q2 Ir . We first find the Keplerian frequency
o ==
dcp I dt of circular orbits, in the same manner as in Solution 17.4. Here

1 M Q2
lrtt = -

gtt,r
= --

2 2 3
r r


21rtcp
- =

gtcp,r


1 -

gcpcp,r
= - =

rcpcp 2

so the quadratic equation for 0 becomes


470 PROBLEM BOOK IN RELATIVITY AND GRAVITATION

Q2

and the orbital frequency is


02r _

( r
_



) =


Q2
( )

{} = :!: _

34'
r r

The proper velocity of the orbiting observer relative to a coordinate

stationary observer is



Q2

v = =

dcp =
Mr _

df At dt r

-2Mr+Q2

(Carets indicate orthonormal components in the stationary frame.) In the


stationary frame the local electromagnetic field has only the component


E " -

-"2

Lorentz transforming this with the standard relations for E-M fields

gives, in the orbiting frame, all components zero except


"2
r2 Q2
( )
t ..2

-2

2Mr +
EAr =
(1- v ) 2

2 2 2
r r r -3Mr+2Q

"2
Q2
( 2Q2)
,,2

t "
Q Q Mr -

B (1 -v )


_

= - - -

()

2 2 2
r r r _

3Mr+

Solution 17.6. The specific angular momentum a and charge e of a

Kerr-Newman black hole of mass m cannot be arbitrarily large, but must

satisfy the inequality


2 2 2
a Ge G 2
-+- < -M (1)
2 4 4
c c c

(Here the G's and c's have been put in for clarity.) Equation (1)
follows from the requirement that there exist a horizon, located at
CHAPTER 17: SOLUTIONS 471


2 22-2



M + (M -
e -
a ) .

If Equation (1) is violated, one has a "naked singularity," which also

turns out to be acausal. Measurements of the electron indicate that its



spin, charge and mass give the result that the a term is of order
22 2 2 68 2 2
10- cm the ,
e term of order 10- cm and the m term of order
110
violently violated and the

10- cm .
Thus, the inequality is rather

electron is not a black hole.

Solution 17.7. Since we have three relations for the components of the

L) the orbital equations



E, can be
4-momentum (p'p m
Pt
- =

Pc/>
= =

reduced to the form


(dr/dA)2 =
V(E, L, r)

where A is an affine parameter and V is some effective potential. The


condition for a circular orbit is that dr/dA remain zero, Le.

V (E, L, r) = 0 (1)

V'(E, L, r) = 0 (2)

and
where V' == av lar. By the implicit function theorem, Equations (1)

(2) can be solved for


E =
E(r) ,
L =
L(r)

provided
aV av
dE dL
-1= 0 . (3)
aV' av'
aE aL

be
(This condition is in fact satisfied.) Then dE/dr and dL/dr can

found by differentiating Equations (1) and (2):

dV av dE av dL + V' (4)
o = = +
dr aE dr aL dr

dV' av' dE aV' aL V" (5)


0=- = + + .

dr aE dr aL a r
472 PROBLEM BOOK IN RELATIVITY AND GRAVITATION

Now consider an orbit at r =

ro perturbed to r + . The perturbed


ro

orbit equation is

(dr/dAi =

V(ro) + fV'(r o ) + f2 V "'(r o ) + ...

0 0
where V(ro) =
and V'(r o ) =
(condition for unperturbed circular

negative if

orbit). The V" term then governs stability and V must be

the orbit is stable. For a marginally stable orbit we have V" = 0 in

addition to V =
0 and V' =
0, so the only solution to Equations (4) and

(5) is dEldr = 0 and dLldr =


O. It is physically clear that these extrema

are minima.

Solution 17.8.

(a) From u.u = -1 and n == uCP lu o we have

-1 =
goo(u

)2 + 2g ocP u
o cP
u +
cP(ucP)2 (1)
0 2 2

(u ) (goo + 20 goCP
+0
CP) ・

and hence
o 2
20 -0
<p)-

u =
(- goo -

go<p

where, in the () = TT 12 plane,

3 2 2
2Ma/r )/r
r/> (r
1- 2M/r - - = -
+a r+ 2Ma
gocP

goo



, ,


The other components are easily found in terms of u :

u<P = Quo

o CP O n
( goo go<p )
u + u u + lt
go<p


goo


cP

gocP
uo +
cP'ucP uO(gocP +0

cP) ・

(b) From Equation (1) we have


Y + 20 + 0 < 0 (2)
;:
goo gocP cP

CHAPTER 17: SOLUTIONS 473

The observer can be coordinate


stationary (0 =
0) only where goo < 0,
i.e. only for r >
ro

2M in the equatorial plane.

(c) The discriminant of Y defined in (2) is


quation

2 2 2
2Mr

gocp ) googcpcp
+ a (r r+) (r r_)


r -
= - -

r+. If the discriminant of Y is negative, then


and is negative if r_ < r <

Y has the same sign for any n, Le. the same sign as which is
gcpcp
positive. This violates Equation (2), which assumed only timelike motion

and constant r. The observer therefore cannot remain at constant r.

{}
Note that as r
r+, then goes to
gocp/goo a/2Mr+.
- =

Solution 17.9. The conserved energy of a particle is the dot product of


its 4-momentum with the time Killing vector e Ct ) =
a/at:


(%t) -Pt (1)

-p
= =

The Kerr metric is of the form

ds
2 = _

2v
dt

+ e
2 r/1 (dcp -wdt)2 + e 21L1dr2 + e 21L2d{P .
(2)

The contravariant components of ga{3 are

tt

-2v tcp =-cue
-2v

cpcp =e -2t/J -cu


-2v

_
_


_

(3)
rr -2111 ()() -2112
g =e g =e ・

If /l is the mass of the particle, then

2 -2v 2 -2v
e 2 cu
p'p e
Pt Pt pcp
= = - -

-11
(4)

_2 'fJ
2 r
-2 v 2 2 -2111 2 -2112 2
+ (e -
e cu ) + e
Pr + e
P().
pcp

Solving the quadratic equation (4) for E Pt' we get


1..

-2111 2 -21l2 2
,,2 )]
E + [ e 2v-2t/J

+ e
2V
(e Pr + e
p() +r (5 )
pcp
cu P

cp
474 PROBLEM BOOK IN RELATIVITY AND GRAVITATION

The sign of the square root in Equation (5) is positive so that E =


+ /1

for a particle at rest at infinity. If E is to be negative, we must have

Pcp negative
and

2"
.,/r
2 v- 2 'fJ 2V -2/1 1 2 -2/1 2 2 2
)]

[e (e + e
PO + /1 < (6)
+ e
Pr wPcp

P rt..

'f'

The boundary of the region of negative E can be found by letting

Pr

Po

0, and /1 0 (i.e. highly relativistic particle). This gives

2v -

2tjJ < w

which is equivalent to
gtt> O. That is, the orbit must be

inside the ergosphere. (This result also follows from Problem 10.15.)

When a rocket ship fires a bullet we have conservation of 4-momentum

Pafter +
Pbullet (7)
Pbefore

Dotting this into the time Killing vector gives

E =E
after
+E
bullet'
(8)
before

fall into the ergosphere from infinity, have


Since the rocket ship can we

E b e f ore > II. In the ergosphere it can fire a bullet sufficiently fast with
,....

negative PcP so that Ebullet


< O. Then, by Equation (7), E after >


b e f ore
and the rocket coasts off to infinity with increased total energy.

bullet's negative
Note that since both the rocket's trajectory and the

are timelike, the bullet can be fired from the rocket with
energy trajectory

a locally measured velocity less than c.

Solution 17.10. In Problem 17.8 we saw that 0 a/2Mr+ when 0 =


1T/2.

To 0 we write, in the notation of Problem 17.9:


prove this is so for any

e- 2tjJ p
gcPcP pcP gcP Pt

pCP

2v cp
0= =

w _

gtt Pt gcP pcP Pt + w P cp



pt +

the metric function


-2I/I is finite
(that is, on the horizon)

At !1 =

but e
2v
vanishes, hence {} = wand at 6. = 0
CHAPTER 17: SOLUTIONS 475

w =
gCP t =

gCPCP 2Mr

Solution 17.11. Start with the rand (J equations for orbital motion in
the Kerr geometry (see e.g. MTW, Equation 33.32):

22 1

(r + a cos (J)dr/dr =
:t V "2 (1)


22
(r + a cos (J) d(J /dr =
:t V
(J"2 (2)
where
2 2 2
Vr =
[E(r + a )_ La]2 - [r + (L- aE)2 + Q] (3)

2 2 2 2 2

(J

Q -

cos (J [a (1_ E )+ L /sin (J] (4)

A 2 2
Ll == r -

2Mr + a

and where E, L, Q are constants of the motion. To have (J motion

across (J = 0 and (J = 17 we must have V positive at both (J = 0 and


(J
2 2
(J =
17. By Equation (4) this implies L = 0 and Q > a (1 -

E ). Equation
(3) then is



E2(r 2 +a 2 )2 _ [r2+a2+I] (5)
where
2 2
I ==
Q -
a (1- E ) > 0 .

The condition that the orbit be at constant coordinate r IS Vr = 0 and

dV r /dr = 0 (condition for a turning point and a perpetual turning point).


The condition on the derivative gives

2 2 2 2 2 2 2 A A
o =
2E r(r + a ) -

(r -

M) E (r + a) /Ll -
r Ll (6)

where V r =
0 and Equation (5) have been used to eliminate the constant

I. This result can be rewritten


2 =


[(r

+ a

) (r
3 _

3Mr

+ a

r+ a

M)]-1 .
(7)
476 PROBLEM BOOK IN RELATIVITY AND GRAVITATION

For very large r we have E R::


(1- +
..) which is the Newtonian

binding energy of a circular orbit. As we decrease r, Equation (7) tells


how E changes. Clearly, since the numerator is positive, the only way

Equation (7) can fail to provide a solution is for the denominator to go to

zero (its going to 00 is obviously impossible outside the horizon). Thus


there exist circular polar orbits down to a radius r given by

3 2 2 2
r -
3Mr + a r + a M =
0 .
(8)

This is easily solved with the cubic formula to give the minimum radius
of a polar orbit

[ (3-f)t { (3t(1 a:)) }]



r =
M 1+ cos cos-l
3 2 2
(3_a )2


where a == a/Me Note that E 00 in the limiting case, so the limiting
case is a photon orbit. The limiting radius decreases from r =
3M for

a=O to 2 for a=M.


r=(1+2 )M

Solution 17.12. Let e be the timelike Killing vector present because the

black hole is static. Then Killing's equation tells us

-{3;a (1)
a;{3

and the requirement for a static geometry (see Problem 10.8) is

(2)
[a;{3 y]0

A static observer has 4-velocity parallel to ,


1.e.


U =
/V2 (3)

v =

_ a .

a (4)

For a photon of momentum p, the infinite red-shift surface occurs when


CHAPTER 17: SOLUTIONS 477

.1.
o =
(p .u) 00 /(p .u) emitter -



emitter

where we have used the fact that


p. is constant along the geodesic
trajectory of the photon and v is normalized to unity at infinity. Thus,

the ergosurface is the surface at which e becomes null. We now prove

that the ergosurface, v =


0, is a null hypersurface, Le. that its normal

vector is null: We use Equation (1) to write Equation (2) in the form

ga;{3gy gy;a g{3 gy;{3 a


+ 0 (5)
- =

then we dot y Equation (5):


into

ga ;{3 (6)
,[a g{3]
v + v = 0 .

Equation (6) shows that v


,a
is parallel to,a wherever v =
O. But v

is the normal to v =
0, Le. the normal is a null vector.

The proof has to be modified in the degenerate case when v




0,

since it is possible that v =


0 in a finite region, not just on a hyper-
surface. However, a detailed investigation [B. Carter, J. Math. Phys. 10,

70, (1969)] shows that this is not so.

The theorem proven here corresponds to a Schwarzschild black hole,

where r =
2M is both the horizon and infinite red-shift surface for static

observers (in fact, for all observers). A Kerr black hole is an example of

the case where the ergosurface does not coincide with the horizon.

the metric with dt dr 0


r+)

At the outer horizon


Solution 17.13. (r=

becomes
2 2 2 2 2
dcp

2 2 2 2 2 (r+ + a ) sin e
ds =

(r+ + a cos e) de +
2 2 2
(r+ + a cos e)

The area of the horizon is then

JJ gtded<p
JJ (r +a2)sineded<p 41T(r +a2). =

A = =
478 PROBLEM BOOK IN RELATIVITY AND GRAVITATION


Since is the larger solution of Ll 2 2
r+

0 we have
r+

M + (M +Q2_a )2
from which the result follows.

Solution 17.14. The two initial black holes have the same area, so the
total initial surface (Q 0 in Problem
area =
17.13) is


22-
Ai =
161TM 1 [M l + (M 1 -
a ) 2] .

The final black hole is a Schwarzschild black hole, so

Af =
41T(2M 2 )2 =

161TM .

The inequality becomes


Af 2: Ai

2 2 2 2-

M2 2: M 1 + M
l (M 1 -
a ) .

If a =
M1 , this means M
2 2: M l . The total initial mass was 2M l , so

the final mass must be at least half of this. According to Hawking's

theorem then, up to 50% of the original mass can be radiated away!

This is the most efficient of the collisions without charge. Suppose

two black holes of masses M1 and M


2 , and angular momentum parame-

ters a

and a coalesce to form a black hole of mass M ,
and
2 3

parameter a

; then Af 2: Ai implies that

1 1 1

M1 [M 1 +
(Mi ai)2] -
+ M

[M 2 + (M a )2] -


[M 3 + (M a )2] -

One gets the most energy out when the equality obtains. Moreover, for

given M
1 , M
2 ,

l ,


the smallest
M3 (hence the largest radiation of

energy) will for O. Similarly, for given M3 (with a 0),



occur a =

3 3

the largest M

+ M
2 (hence the largest radiation of energy) will occur

for I all =
I a21 =
Ml =


. Since a


0, conservation of angular momen-

tum implies that a = -


a . This is just the situation we worked in
1 2

detail above. The conclusion, therefore, is that when two Kerr holes

collide and coalesce, no more than 50% of their original mass can be

radiated away.
CHAPTER 17: SOLUTIONS 479

Solution 17.15. The area A and "reduced area" A of a Kerr black hole
are
_ 1
2 4
A == 81TA =
81T[M +(M _J2)2] (1)

or
equivalently,

A2 -

2AM 2 + J2 =
0 .
(2)

Taking the first variation, we have

(A-M 2 )OA =
2AMoM-JoJ .
(3)

The left-hand side is nonnegative by the second law. Let a particular


wave mode have the t and cf> dependence exp (- iwt+ imcf?. All scalar,
electromagnetic and gravitational waves satisfy

oM =
OJ (4)
so we have

( 2AM -

] ) 8M 0 .
(5)

For an amplified wave conservation of energy requires that the mass of

the hole decrease, so we want oM < 0; thus the term in parentheses in

Equation (5) must be negative. This condition can be rewritten as

2Mr+
---<0

(6)
a w

where r+ is the coordinate radius of the event horizon, or (if we take into

account that m and w can have either sign) the condition is

o S w S
ma
(7)
2Mr+

where S means "lies between." Note that


a/2Mr+ is just {} the

"angular velocity of the black hole" (see Problem 17.10).


480 PROBLEM BOOK IN RELATIVITY AND GRAVITATION

Solution 17.16.

(a) Using the results of Problem 7.7,

1 1
0(1) =
(- g)-"2 [(- g)"2 ga (3 (I) a] ,

, {3
.1.
2 2 2
and (- g) 2 =
(r + a cos 0) sin 0, we have

sin2e1 a2 <J>2
[ (r2 + a 2 l + a 2 2

( ) acp2
o = _
Mar a <J>

1 a

a2(1)

_
_

A. at A. at acp sin 2 e A.
(1)
1.<1>\


.a + .1 0 (sin 0 acI?

ar ar sin ao
\ ao

(b) In Equation (1) we take the t and cf> dependence to be

<I>(t, r, 0, cf? =

-iwt
eimcf> R(r) SeO)

and we divide through by (I). This gives us

1. .!! dR
w2 (r
2 2
22 4Marwm
2 2

( )
+ a a m
+ +
R dr dr

(2)

-1 d

dS


L) 2 2 .
2 L} m

Slnv + a w sin v + .

S sinO dO dO 2
sin 0

Since the left-hand side of Equation (2) is a function of r alone and the

right-hand side must be a function of 0 alone; each side must be a

constant A, thus

( )

1 d dS
( )
2 2 .
2 L) m
sinO -A 0

- -



a w sin v + S =
(3)
sinO dO dv 2
sin 0

and
2 2


2 2 2 2

[ (r )
dR
( )
w +a -4Marwm+a m
+ -A R=O. (4)
dr dr

Since S must be regular at 0 = 0 and 17, Equation (3) is an eigenvalue

equation for A. The function S is in fact a "spheroidal wave function";


in the simplest when aw 0, we have S
Pem(cos 0) and A e(e+ 1).

case = =
CHAPTER 17: SOLUTIONS 481

(c) It is convenient (but not


essential) to introduce a "tortoise"

coordinate r* which satisfies dr* /dr =
(r + a2)/ .Note that the interval

(r+, (0) in the r-coordinate is stretched to


(- 00, (0) in r*. Equation (4)
now becomes

2 2 2
d R


2r


dR 2 a m -

4Marmw - A
+ + w + R 0 (5)
2 2

*2 2


) dr

dr (r + a
(r

+ a2)2
As r 00, we get

d R 2 dR 2
+ + w R 0

dr* 2 dr*

which has solutions

-1 +iwr*
R """-'
r e-

corresponding to ingoing and outgoing waves.

(d) As 0, Equation (5) gives

d2 R


2 2


2 2amw a m

+ w _

+ R =

dr*2 2Mr+ (2Mr+)2


and hence

R """-'
e:tiCW-mw+)r*
where

w+
==
a/2Mr+ .

(e) Since all physical observers at the horizon are related by Lorentz

transformations, they will all agree whether a particular solution repre-

sents ingoing or outgoing waves. Thus we can calculate with any con-

venient observer. We choose an observer at constant r just outside the


horizon. Since he is ins ide the ergosphere, he is dragged around in the

positive c/J direction with some 11 =


dc/J/dt > 0 where, from Problem
17.10, 11 =

+.
This observer sees the local (t,r) dependence of the

solution
(I) =

-iwt
eimc/J e:tiCW-mW+)r* See) (6)
482 PROBLEM BOOK IN RELATIVITY AND GRAVITATION

as


e -i(w-mw+)t e:l:i(w-mw+)r* eim S(O) .


e- "()

Thus c:u-mc:u+ r
where we have put cp =
cp -c:u+t. corresponds to in-

going waves.

(f) The stress-energy tensor for a scalar field is

* 1
41TT
afJ
= cI>

(I)
(a,fJ )
- -

2 C(lfJ

1(1) ,Y(I)'YI .

(The complex conjugation is necessary because the representation of (I)


is complex.) The energy flux vector is J {3 = -

Ta{3 a,
where is the

time Killing vector a/at (see Problem 10.11). The energy flux into the

horizon is found by integrating the radial component of J over the

2-surface r =

r+

Since
cIJ; =

f T: I gl t dO dcp ・

( )
2 2
r r + a
41TT Re((I) <f>*,r)
= =
Re (I) (1)* *
t,t ,t,r I

2 2Mr+

c:u(c:u -

mc:u+) S (0)

we have
2Mr

f S2(O)sinedOdcp

d =

w(w-mw+) 417+
and dE/dt is negative, Le. energy flows out of the horizon if c:u -

mc:u+
< 0,

i.e. if 0 < c:u/m < in agreement with Problem 17.15.


c:u+

Suppose charge oQ and energy oE dropped


Solution 17.17. are into

the hole. We then have

0(Q2 _M2) =
2QoQ -
2MoE . (1)

Not all values of oQ and oE are allowed, however. The radial

"effective potential" equation (see e.g. MTW, Equation 33.32) for a

charged particle falling in the Reissner-Nordstrom geometry is


CHAPTER 17: SOLUTIONS 483



-!(r aE-QOQd -L\V-t

r =

r2+L;+ )!2 (2)

where E, Lz and are conserved quantities, and


110
is the particle

rest mass. A
particle which crosses the event horizon into the black hole

Ll
r+. Since
has dr/dr:S 0 at the event horizon 0 at
r r
r+, (2)
= = =

becomes
2 dr 2

dr
= _
(r+oE _

)<
QoQr+- 0 (3)

and hence oE Combining this with Equation (1)



QoQ/r+. we see that



0(Q2 _M2) <

20E(r+ -M) =
20E(M _

Q2)2 .
(4)


As Q2 M the right side goes to zero as a square root, so no sequence

of adding oE's can make the integrated left side positive.

Solution 17.18.
1 1 -'

,.:t -t
--

2 2 2 2
1 gtt 1 1 gt t gCPCP 1
(a) W

(a) W d t. W dt
cp
= - -

2 tt
-(gtt- W )g
cp
= .

The
[The minus sign is chosen because
gtt

(j)2 cjJ oj < Kerr metric

has only the gtcjJ tet mt


the diagonal.
f Inverting
f3
we find gtt =


r'.:J fA -1. The calculation of other inner products
gcjJcjJ anE

(j) =

gtt

to verify wfi: .;;,f3 =


l1

{3 is similar.

(b) The dual basis e


a is defined by <(;,a, ef3> aaf3' = If (;,a =

A r'.:Jf3
transposed inverse
and
fj Bf3Yey

' then the matrix
Bf3Y must be the

separate the t4 piece from the



of A To find B from A we can
f3'
(diagonal) r-O piece:
" 1 -
t 2 2
dt

(a) I gtt W
gcpcpl 0


1 1

cp (gCPCP)

dcp
W(gcpcp)2

484 PROBLEM BOOK IN RELATIVITY AND GRAVITATION

implies that

1 1

-Ct)2 cPl-2

wi gtt
gepep 1-
e....... 2
t gtt -

w e


o 2
(gepep)-
e.......
ep ecf:!.

Since the r-(} piece is diagonal, it is trivially inverted:

1 1
e' =
r ( /I)2

r eO

I- 2
e() .

(c) The 4-velocity has zero rotation if w =


0 (see Problem 5.18)
a {3
or
equivalently if u is hypersurface orthogonal:


U[a;,8uy ]

(see Problem 7.23). The quickest way to check this condition is to use
""'"

forms and the equivalence of


U[u ,8.......]
and d t.The condition ur....... .......u ....... =

,8 y]

....... .......'....... La ,

t -t -t 2

d CA)" \

is then equivalent to CA) =


O. But CA) =
a dt where a =
I gtt -

ep

= a
(r ,()) and thus
d t
= a
,r
d;"dt + a

(} M"dt

so dc':)t" i)t =
0 since dt" dt =
O.

(d) The 4-velocity of the ZAMO IS


2 -2
u = =

\gtt- W \
ep (et+weep)'
The vectors e and are Killing vectors. The vector field e +
t eep t weep
is not a Killing vector because w is not constant. However any particu-

woecp (w o
lar ZAMO is moving along the Killing vector =


+ a fixed

constant, which is chosen to equal w at the radius of the particular

ZAMO under consideration). Thus the results of Problem 10.14 apply

and we get

a = V log II; .1; I = V log I gtt + 2Ct)o +Ct)


cP I ・

cP
CHAPTER 17: SOLUTIONS 485

When we set and evaluate at this becomes


gtc:p
= -

CtJ cp CtJ CtJ


a =

} V log I -Ct)2
t cPl .

Solution 17.19. In Boyer-Lindquist coordinates, for t =


constant and


r+, the Kerr metric gives the metric of the horizon:

2 2 -I 2
(r
2 2 2
(2Mr+)2 sin20(r

ds =
+ a cos 0) d0 + +a cos 0) dcP .
(1)


(We have used the relation r + a =

2Mr+ in
gcPcP') The Gaussian

curvature K of a 2-surface is the same as the Riemannian curvature of

Problem 9.23, where the Riemann tensor is computed from the

2-dimensional metric. It is convenient to use an orthonormal basis dyad.

For a metric of the form

2a
)2 )2
2a
2 l l 2 2
ds =
e (dx + e (dx (2)
we let
a a

-1 1 -1 -
2 2
e.) =
e dx , e.) =
e dx (3)
and we have

K =
R =
R e
-2(al +a2) (4)
1212' 1212

The quickest way to find the Riemann tensor is with forms as in Problems

8.27 and 9.20. Since

-1


-1

1 -2 -1
-a


2 -1
de.) =
e da 1 A dx =
e a
1 2dx A dx = a
1 ,2e
e.) A e.)

and a similar expression with 1 and 2 interchanged, we find

-a
-1
-a
-1 -2
CA) e

a -e


,1CA) (5)
,2e.)

2 1 2

di} wS
(Recall that =

- i'SA and
waS -wj3a')

The only nontrivial

curvature form is
486 PROBLEM BOOK IN RELATIVITY AND GRAVITATION

'" '" '" '"

a>1 -1 -1 -a

J\2= 2+WaAW2
a -a
al- a 2-1

2 l 2

d(a 1 2e dx -a
2 le dx )+ 0
, ,

a -a a -a
1 2 -2 -1 2 1 -1-2

(a 1 e ) dx Adx -

(a 2 le ) dx Adx
" 2 2 "

-Ca 1 + a 2 ) a

-a



-a
2 -1 -


-e [(e ,1

),1 +(e ,2

),2]W A W ・

a> -1 -2 h ave
Since J\12 =
R W A we
121 2'

-Ca 1 +a 2 ) a -a a

-a

2 1
] (6)
K =

1212

-e [(e ,1

),1 +(e ,2 e ),2'

Using the explicit form of the metric in Equation (1), with () =


xl and

cp =
x , we find from Equation (6) that

222
3a cos ()

r+
K =

2Mr+ .
(7)
(r .+ a

cos

0)3

Note that if r -
3a < 0, then K is negative for a range of () around

the poles (()=0,1T). But when K is negative, the surface cannot be




globally embedded in a Euclidean 3-space. Since r+ =
M + (M -


)2,

2 a > 3 2 M/2.
the condition that
r -
3a be less than zero is equivalent to

The Gauss-Bonnet theorem says that



Kd S =
21TX

where X is the Euler characteristic of the surface ex =


2 for a sphere,
o for a torus, etc.). Since



d S =
(g) 2 d() dcp =

2Mr+ sin () d() dcp


we have
CHAPTER 17: SOLUTIONS 487

2TT TT

r2+ 2

1 1
-1

3a cos ()
X (217) dcp (2Mr+)2 () d()

sin
o 0
(r + a

Cos

())3
1 2 2 2

1 (r


3a x

(2Mr+) 2 dx

o + a



)3

(2Mr+)2 2 r II

L(r a2x2) +


so the surface is topologically a 2-sphere. (For further details see

L. Smarr, Phys. Rev. D. 7,289, (1973).)

Solution 17.20. The energy amplification described in Problems 17.15

and 17.16 corresponds to stimulated emission in quantum-mechanical


language. If we know the rate for stimulated emission, we can find the

rate for spontaneous emission as follows:

Let p be the probability per unit of phase space of the spontaneous


emission of a quantum. If we send N quanta in, the probability of getting
N + lout by stimulated emission is (N + 1) p, since the quanta (gravitons,
photons) are bosons. To simplify the argument, assume p? 1 so we do

not have to consider the emission of more than one extra quantum; dimen-

sionally the answer would not be changed if we were to include this re-

finement. The expectation value of the excess number of quanta emitted is

< N>(excess

number) x (probability of that number)

1 x (N + 1) p .

The amplification is

< : (1 )

A == =
+ p p (1)

in the classical limit (N 00). Fermi's Golden Rule says that the rate of

spontaneous emission is
488 PROBLEM BOOK IN RELATIVITY AND GRAVITATION


dN
dt p. (2)

phase space

In Equation (2) replace p by A and recall that the classical effect is


described in terms of modes with "quantum numbers" e and m and a

frequency w. Thus


e,m J A dw ・
(3)

N ow A goes rapidly to zero for large e and m, as in all barrier-

penetration problems. Also from Problems 17.15 and 17.16, A is non-

zero for w 11M over a range 11M, where M


w is the mass of the

black hole. Since A is a number of order unity for low e and m, re-

place A by 1 in Equation (3) and drop the summation. Then

dN
w!.
dt M

The hole loses energy at a rate

dE dN it
1rw
dt dt 2

This implies a loss of angular momentum at a rate

dJ mdE
--
1r
dt w dt M

Thus the time-scale for angular momentum loss is

10
r _
J/(dJ/dt)



(1r/M)


M /tr -
(M/I015 gm )3 10 years.

15 by
Thus for M S 10 gm, J will already have become essentially zero

33 54 in
the fraction lost will be 10-
this process. For M


10 gm,
10
10 years.

Solution 17.21. For all static spacetimes, the "surface of infinite red-

shift" (goo =
0) coincides with the horizon (Problem 17.12). Therefore,
CHAPTER 17: SOLUTIONS 489

if the above metric describes a black hole, its surface is at


goo

0 or

-1 1
fact, the surface p

h p / m.





were
p == We will now show that, in = -


IS not contained in the same manifold as the one which becomes asymp-

totically flat (p (0) and thus cannot be reached by objects in our universe.

A glance at the
gPP portion of the metric shows that p =
1.5 is an

infinite proper radial distance away from any p > 1.5. To see that p =
1.5

is also infinite proper time away and thus removed from the manifold it is

sufficient to consider radial null geodesics. (The reader can verify that
for nonradial or timelike geodesics, matters get even worse):

gp/pp)2 +
gOO(Poi = 0

or

1.
dp <P + )
d>'

( p

) Po
<P ; )2
_

Remembering that is constant of


where A is an affine parameter. Po

the motion (stationary metric), the above equations show that

-- A
P -2
(p- t)2 dp r


-1 _


dA '
Po

(p -

1 )(.0 + t) J>.

3 3 .

sur f ace IS

the left diverges at P th e p


Since the integral



on



an infinite affine distance away. There are no singularities for P

and p is a monotonically decreasing coordinate for radially infalling


well must be off the
photons and particles, so = as as p


p ,

phys ical manifold.


of
The above conclusions also indicate that the surface
a star


3. thus
vacuum metric can never reac


collapsing to form the given


p '

for
for realistic physical systems, the given metric is realizable only


P > .


CHAPTER 18: SOLUTIONS

Solution 18.1. To find the gravitational radiation we use the square of the

third time derivative of the quadrupole moment


Power =

(i') ""
GM2L 4 .

5 5 6
5c 5c t

The appropriate parameters for the mean Massachusetts motorist: M =


mass


of fist and forearm 2 x 10 gm, L
f'OW
length of forearm 50 cm, t = f'OW =

period of average shake


f'OW
.2 sec.The calculation in cgs units then
43
yields a radiative power on the order of 2 x 10- ergs/sec.
To calculate his total energy expended, use the fact that muscles are

almost totally nonconservative, so every shake uses up an energy roughly


equal to the arms maximum kinetic energy:


1 L 1 8
3 10 erg/sec
Power
M"2 x



51
The fractional "efficiency" is thus f'OW
10- .

Solution 18.2. The gravitational binding energy of the system is of order



/T yn'where
2 MR T char-
M /R and the kinetic energy is of order dyn
acterizes motions of the system. For a system in equilibrium the virial

theorem gives
2 3
T f'OW
R /M .

dyn

The rate at which energy is radiated gravitationally is governed by the

square of the third time derivative of the quadrupole


moment or roughly

GW 2 4
p f'OW
M R
/T yn .

490
CHAPTER 18: SOLUTIONS 491

The time for radiation reaction to affect the


system is the time it takes
to radiate a significant fraction of its energy as gravitational waves:

4 2
T RT
T =
K.E. ""J
dyn
""J
dyn
2 2
pGW MR M

The ratio of this gravitational wave characteristic time to the


dynamical
time is then of order

RT

C : Y-.
T dyn

""J -

T 2
dyn M

Evidently gravitational radiation reaction forces play an appreciable role


only in relativistically compact systems.

Solution 18.3. The three independent orientations of the dipole correspond


to the three components of a three dimensional vector. Since the quadru-
pole is a second rank, three dimensional tensor (9 component) which is

symmetric (3 constraints) and traceless (1 constraint) there are five in-

dependent" orientations." An example of five independent quadrupole

tensors is the five tensors with all components zero except:

1 1 1 1
I 1
=--1 1 Ixx
(i) Izz Ixx (ii)
Iyy
= - = = - = -- =

2 2 yy 2 2 zz

(iii) Ixy

Iyx (iv) I
xz


zx
(v) Iy z

Iz y ・

Solution 18.4. Assume the rod to be in the xy plane rotating about the

z axis. If fl is the mass per unit length of the rod the time dependent
parts of the reduced quadrupole tensor are

£/2


2 2
..f-
xx

Ii- cos (cut). 2 x dx

£3 2 Me


cos cut =
cos(2cut) + constant
12 24
492 PROBLEM BOOK IN RELATIVITY AND GRAVITATION


..r
Me
= -
cos 2wt + constant
yy 24


Me
-r

.-foxy sin 2wt


= =

yX 24

so that

6 2

( )
... ...


<-I- (2 ) Me 2 2

jk..r jk >
5 <2cos 2wt+2sin 2wt>
= =
GW 5 24

2 4



cu M e ・
(Note units with G =
c =
I!)

To calculate the charge centrifuged to the ends of the bar, balance

electrostatic and centripetal forces:


leV(I)\ =
rmw

where (I) is the electrostatic potential and e and m are the charge and

mass of an electron. Thus the charge density induced will be of order

2 2
p ,..., _V (1) ,...,
-(m/e)w .

This charge depletion in the main part of the rod will of course be

balanced by electrons squashed against the ends with charge (m/e)w eA
where A =
rod cross section. The rod thus becomes an electric quadru-
2 3
pole with quadrupole moment of order (m/e)w e A. The electric quadru-

pole radiation due to this induced moment will be roughly w times the
6 2
0(m/e)2e

square of the quadrupole moment, or roughly w A . The ratio

of electromagnetic damping to gravitational damping is then


L 6 2
0(m/e)2e
[(m/et ]
EM ,...,


A =
cu2 .

L 2 6
p2A

GW w e

21
In G =
c =
1 units (m/e) x 10- and p(10g/cm )

10- 27 and


cu(lkHz) x 10- so that

18
10-
LEM/L GW
,...,

For once gravitational radiation wins out.


CHAPTER 18: SOLUTIONS 493

Solution 18.5. The radiation reaction force on an element of the source is


dF.

= -
(I) .

pd x .

,1

The rate at which the source element loses energy is the rate at which the

reaction forces do work on the source element: vidF. 1


where v is the

velocity of the source element. For the whole source, then

J J
dE


l 3 l
(I) (v p
(I),i v p d X
),i d x
= - =

dt

Here we have used Gauss' divergence theorem and evaluated the surface

integral on a surface outside the source to obtain the second equality.

The equation of continuity tells us that V -


(vp)


-ap/at so

J J
dE = _
(I)
ap d

x = - 1. -I- 5) xjx kp d3x
dt at 5 Jk dt

--
1 (5) d
.f
5 jk dt
( 4k+ 3

Ujk


rpdx.


Since
4kOjk

0, we drop the final term to arrive at

1 (5)'
dE / dt = -

( ) -Ijk5

Now we time average over several oscillations. Since secular changes in

parameters of the source are assumed to be small over the several oscilla-

tions, we have
T. T...


・ ..

1 1
T --If )4kdt T
Jo
-I'
jk-l' jk dt

and
..

dE 1.
= -
<..f .f >.
dt 5 jk jk

To find the rate of loss of angular momentum we use the fact that for

an element of the source


494 PROBLEM BOOK IN RELATIVITY AND GRAVITATION

dJ 3
ra d latl0n reaction
. .


torque =
r x dF = -
r x V(I)p d x
dt

and for the whole source


dJ 0 O



- = -
c 1J xJ(I) kP
d x
dt '

J ( -Ifc xm) p d
ijk j 3
= -
E x x


2 2
= _

ijk
km
(5) xjx mp d3x = _

ijk
(5).f-.
km

5 Jm

Time averaging, and integrating (in time) by parts as above gives us


dJ 2
= _

ij k
< ".i-' km ..f Jm > 0

dt 5

Solution 18.6. Let m , m and r r be respectively the masses, and


1 2 1 , 2

distances from the center of mass, of the two stars rotating at angular

frequency cu. From Newtonian mechanics we know that r m




r m =
RJL,
1 2 2

where R == r

+ r

and JL

reduced mass =

ml m 2 /(m 1 + m
2 ).
The total

energy of the stars can now be written as a function of M, R, and JL:

1 /lM
m m
E 1 2 1 2 2 1 2 1 2 JLM












cu -



cu
R/l -


= -

2 1(

Here M =
m + m and Kepler's laws have been used.
1 2

The power dissipated as gravitational waves is easily calculated


once the reduced quadrupole moment,

f (XjXk-

8jk)d

4k ==
P r x ,

jkd3X

is evaluated. Note that the "reducing term" f pr 8 will be con-

stant in time and can be ignored. Suppose the z axis to be the axis of
CHAPTER 18: SOLUTIONS 495

rotation and cp the azimuthal angle from the x-axis to the line
joining the
stars, then (aside from time independent terms) we have

2 2 2 12
1 (r 1 m 1 ) cos cp R Jl. 2cp
_

+ r m =
cos + constant
xx 2 2

1 2
I = -
R Jl. cos 2cp + cons tant
yy 2

Ixy

IyX

R21L sin 2c/> .

Since cp =
cut, it is straightforward to differentiate three times, perform
. . .

the sum -I- and time average to find


jk jk

・ ・ ・ ・ ・ ・

1 1 1 2 2 2 2

<.f
jk...f jk >


(2w)6
( R2IL)

<sin 2wt+sin 2wt+2cos 2wt>

3 2
32 6 4 2 32 M Jl.
.\
R P
_

Jl.
_ _

UJ

5"
- -

5 S GW'

Since this power is radiated at the expense of orbital energy, it follows

that

dE =
1 Jl.M dR =
-P =
32 M Jl.2
R2 dt -5
dt 2 GW S

The resulting differential equation

3 dR 64 2
R = _
M Jl.
dt 5

is easily integrated to give

4 2
R = -
(256/5) M Jl.t + constant.

If we denote by to the time at which the separation goes to zero accord-

ing to this formula, then


4 2
R =

2 6 M 1L
(to

t) .

We can find the time to by noting that at t =


0, R must be the present

se p aration R
now
,thus
496 PROBLEM BOOK IN RELATIVITY AND GRAVITATION


5R
now
to


256 M 11

Solution 18.7. In terms of the total orbital energy E and angular momen-

tum L, the semimajor axis and eccentricity are

a = -




/2E (1)



1 +
2EL
(ml +m 2 )
e =

3 3 (2)
ml m 2

so that
m m
da 1 2 dE

dt

2 (It (3)
2E

m +m

(L2 )
de 1 2 dE dL

+ 2EL .
(4)
dt 3 3 dt dt
m m e.
1 2

If the separation of the two particles is r, where


a(l- e )
r =

1 + e cos () (5)

(see figure), then




2 m
r = r l
1 m + m
1 2



r =

2 m + m
1 2

so the components of the quadrupole


tensor are

2 2 ml m 2 2 2
I = + m x r cos ()
ml xl

xx 2 2 m + m
1 2

m m
1 2 2 2
I =
r sin ()
yy m + m
1 2
CHAPTER 18: SOLUTIONS 497

m m
1 2 2
slnt9 cost9

I =
r (6)
xy m
1 +m

m m
1 2 2
1==1 +1 =
r.
xx YY m.+m
1 2

According to the Newtonian equations of motion



2 2

[(ml + m 2 ) a(l- e )]
t9 =
(7)

so Equation (5) gives



ml

[ m)
r =
e sin t9 (8)
a(l -

Now we compute the successive time derivatives of I , using Equations


ij

(5), (7) and (8) to simplify the expressions. The results are

2m l m 2 r cos t9 sint9
Ix x



(9)
2 2
[(m 1 + m
2 ) (1
a -
e )]

..
-2m 1 m
2 3

xx


(cos 2t9 + e cos (9) (10)
a(l- e )

2mlm2 2'
t9 sin(9)t9 (11)
I =
(2 sin2t9 + 3e cos
xx

a(l- e )

2m 1 m 2

YY

r (sin t9 cos t9 + e sin (9) (12)
2 2
[(m 1 + m
2) a(l- e )]

2ml m 2 3 2

YY

(cos 2t9 + e cos t9 + e cos t9 + e ) (13)

a(l- e )

・ .. -

2ml m 2 2'
sin(9)t9 (14)

YY

(2 sin2t9 + e sint9 + 3e cos t9

a(l- e )
498 PROBLEM BOOK IN RELATIVITY AND GRAVITATION

2 2
m m r (cos 0 -
sin 0 + e cos 0)
I =
1 2
(15)
xy .1
2 2
[(m 1 +
m2) a(l -
e )]

2ml m 2
(sin 20

I =
+ e sin e + e sin 0 cos 0) (16)
xy 2
a(l- e )

, . ・ -

2ml m 2 3

1=
xy
(2cos20-ecosO+3ecos 0)0 (17)

a(l- e )

... ... -
2m 1 m 2 e sinOO
I = I
xx
+ I =
(18)
YY
a(l- e2)

The rate of loss of energy is then:

t (.
.. ...


... ... ...

dEl 1 2

i-jki-jk -5\ IjkIjk-
= - ==

(it 5 3


・ ・ . ・ ・ . ・ ・


...

1 2 2 2 l' 2
=--1 +21 +1 -

5 xx xy yy 3

2 2
_8m 1 m 2 2 2 '2
==
[12(1 + e cos 0)2 + e sin 0] 0
222
15a (1- e )

where we have used Equations (9) -

(18) and straightforward algebraic


simplifications. Now we must average over an orbit. The period of an

orbit is (Kepler's third law)

2" 3

1 !. 2" a
"2
T = dO =



o (m 1 + m

) 2"

so that

T 27T

I J
dE !. !.
( ) 1.
dE dE

dt =
dO
dt T dt T dt 1.
o 0 u

2 2
(m 1 2)
m m +m

( )
-32 1 2 73 2 37 4
= 1 + e + e (19)

24 96


(1_e

)7/2
CHAPTER 18: SOLUTIONS 499

Similarly, the rate of loss of angular momentum is

dL
.. ...

-2
.. ...

-2
Czij k kj 5 I
zij Iik kj
_



dt 5
..
.. ..

-2
[I ( I yy- I xx)+ I
Xy( Ixx Iyy)]


XY

2 2
m m
8 1 2 2 2

[4 + 10e cos e + e (9 cos e -

1)
5 2 2 2
a (1_ e ) .

3 3
+ e (3 cos e -

cos e)] e

which when time averaged gives us

2TT


dL 1. dL
< ) dt



dt
1. de
fJ

221.
-32
m m
(ml +m 2 )2 7 2
( )
1 2
_

- -
1 + e ・
(20)
5 7/2 8

(1_e

)2

From Equations (3) and (4) we have, finally:

da 2a2 IdE
( )
dt =m 1 m
2 \Cit )
m m
(ml +m 2 ) 73 2 4

37

-64 1 2
=-
l+-e +-e (21)
5 /2 24 96


(1_ e

)7

[ ]
a(l- e2)
+ m 2
ml ) dL
< t)
dE
( )
de
( ) P
2 -

= _

dt m e + m dt 1

ml 2 ml 2 a 2
(m 1 + m )

m m (m 1 + m )e
( )

304 1 2 2 121 2

1 + -e .
(22)
15 304


(1_ e

)5/2

From Equation (22) we see that de/dt is negative, so the eccentricity

decreases because of radiation reaction.


500 PROBLEM BOOK IN RELATIVITY AND GRAVITATION

If we put e =
0 in Equation (21), we recover the results of Problem

18.6. For the integration of Equations (21) and (22) when e 1= 0, see

P. C. Peters, Pnys. Rev. 136,1224 (1964).

Solution 18.8. The Ricci tensor for the wave must vanish. By contracting
the result in Problem 13.13, we have then


2RJLV haV,aJL haJL,av +
hJL;aa h
= = - -

,JLV

Since the perturbations functions of


are u t x, the third term 0
hJLV
= -

automatically vanishes and the components R


22 ,

23 ,
R33 automatically
vanish. For JL,v =
0,2 we find


d O 1
(h 2 h
2)

_ =


du

0 1
From this we conclude h

=h

_ [The integration constants are of no con-

sequence; they are not wavelike; they carry no energy, and since they

make no contribution to the Riemann tensor, they are gauge removable.]


Similarly we have
o 1
h =
h from JL,v

0,3
3 3

O 1 =!.
h o
_


h from Il,V

0,0

2 3


+ h


0 from Il,V

1, 0 -

Other choices of JLV give no further, independent relations.


These four constraints leave us with six independent components of

x) leaves the
ha{3' A gauge transformation of the form
'JL(t ha{3 as

functions only of t -
x and hence does not invalidate our four constraints

above. We choose then a gauge transformation of this type which annihi-

h h and which leaves with only the quantities


lates h
oo '
11 ,
02 '
h03 us

which invariant for such transformation.


h =
h and h
h33 are a

23 32 22
The explicit form of the gauge transformation is
CHAPTER 18: SOLUTIONS 501

I II
III
1 --

o(u) =

hoo(u)du gl (u) = -


h 11 (u)du

2(U) =

III h
02 (u)du 3(U) =

I\o3(ii)dii ・

Now we consider whether these are the same components as we would

get by projecting out the transverse traceless part of the perturbation.


O 1
Since we have h h transverse part is equiva-
0+ h 1 projecting out the

lent to projecting out the TT part. But projecting out the transverse

h did transformation.
part leaves h 22 ,
23 ,
h33 invariant, as our gauge

Thus we get the same components either way.

Solution 18.9. Let the z axis be the symmetry axis and let cp be the

At of time, t constant, in the


cyclic axial coordinate. an instant =

asymptotically flat region far outside the sources through which the waves

propagating, the angular


are momentum inside a closed 2-s urface r =

is (by symmetry only an axial component)



11 =-(16nT
/l;Vd2!./lV (1)
r= r

where e =
a/acp. (See Problem 16.23. The proof goes through even if

there is no time Killing vector and even in th presence of waves since

waves modify only the space-space parts of the metric



see MTW, Equa-
tion 19.5.) Similarly, the angular momentum inside a closed 2-surface

r =


(r 2 > r
1 ) is



12 =
-(1677)-1 /l;v

!.p.v ・
(2)

r=r

Thus the angular momentum of the waves between r



and r

is
502 PROBLEM BOOK IN RELATIVITY AND GRAVITATION

1 2 -1 1 =-(1617)-1

r=r2
1L;Vd2IILV+(1617)-1
r=r
f 1L;Vd2IILV

J TIL

d 3v

IL

r1SrSr2

where we have used Stokes' theorem as in Solution 16.23. Since TIL


vanishes for r
1 S r S r

, J2 -

J1 is zero and so the angular momentum

flux of the waves vanishes.

Solution 18.10. A plane wave propagating in the z-direction can be de-

scribed at the position of a detector by

icu t
Re{A + e-icutl h h J

h = -h = = =
Re { A e .

xx yy xy yx X

the plane monochromatic (and hence 100% polarized) and


If wave is
A+
will be constant. More generally if considering
Ax we are waves in a

bandwidth about cu, and will be slow functions of time


narrow
A+ Ax
(Le. slow compared to the wave oscillations). Since the intensity is pro-

portional to the absolute squares of the A's we can define a polarization


tensor

Aa A >
Pab

\A+12 +
1Axl2
in analogy with the electromagnetic approach. [See for example Landau
and Lifschitz, p. 122 ff.] Following this analogy further we notice that

this matrix is hermitean with a unit trace so we can write it as

1 +
3 -

1i 2

Pab



1i 2 1 -

As in electromagnetism will be called the Stokes parameters


l' 2' 3
of the wave. [Note: Some authors call these the normalized Stokes

para me ters.]
CHAPTER 18: SOLUTIONS 503

These parameters characterize the polarization state of the wave. To

get insight into the meaning of the parameters, let us consider a mixture

of (1) unpolarized waves, (2) circularly polarized waves, and (3) linearly
polarized waves with an orientation rotated about the x and y axis by

an angle t/J. Mathematically these three cases correspond to superposi-


tions of
2 2 2
(1) A+


1 (t) Ax

G (t)
2 with < IG 1 (t)1 > = < \G 2 (t)1 > == < IGI >

and with < G
1 (t)G;(t? =

(2) A+ =
H(t) Ax

:!: iH(t)
(3) A+ =
F(t) cos 2t/J Ax

F(t) sin 2t/J ・

We assume of course that G1 G F, and Hare uncorrelated. The



2 ,

polarization matrix is then

<\F\2>cos

2t/J + <IGI>2+ <IHI>2

<IFI >sin2t/J cos2t/J + i<IH\2>

p =-


2 2
+<1 H\2>

<IF\2>sin2t/J cos2t/J:!: i<\HI > <I F\2> sin 2t/J + <I G1 >

I ==
<\F\2> + 2<\GI

> + 2<IHI2>
so that



<IFI

> COS 4t,lr =

1 <\FI2> sin 4t,lr =

2:!: i <\HI > ・

radiation
finally that the polarization properties of the
can be
We see

similar to that of
found from the Stokes parameters in a manner very

electromagnetis m.

Fraction of radiation linearly polarized =

( ;g'f)2+

Orientation of linear polarization: tan 4t/J =


g 1 /g 3

Fraction of radiation circularly polarized \g2 1 =

(g 2 positive for right circular polarization)


i:2 i:2 .c 2 2
(s 1 + S 2 + S 3 )

Fraction of radiation polarized = ・


504 PROBLEM BOOK IN RELATIVITY AND GRAVITATION

Solution 18.11. The motion is mo easily


t analyzed in a nearly Newtonian
coordinate system. The motion of the particles (and the observer) then
has the form
x '-
Rx '-
hx .

Here R denotes the magnitude of the Riemann tensor for the waves and

h denotes the (dimensionless) metric perturbations for the waves. Since

the waves are weak and oscillatory, the positions are fair ly constant so

we can integrate the equation of motion to get


x(t) -

Xo

xoh(t) + at + (3 + <9(h )

initial position.
Xo

If we ignore the higher order corrections, we see that a and {3 must be

zero since the particles were at rest in their initial positions before the

wave burst arrived. After passage of the waves h =


0 and the particles

must again be at rest in their initial positions.

Solution 18.12. The normal modes of the rod (taken to be oriented along

the x-axis) are found from the equation

a2
-+



- -

2 a2
- =
0 . (1)
2 rat 2
at ax

Here t)
(x, is the displacement of an element of the rod, r is the damp-

ing time and a is the speed of sound. The modes are of the form

C -iwt -t/2r
s=e e ( )
ux (2)

where

d 1...
( )
2 u 2
a + w + u =
0 . (3)
2 2
dx 4r

Since the ends of the rod are free,


CHAPTER 18: SOLUTIONS 505

ag 0 (4)

ax x = :!:L/2

where L is the length of rod. The solutions of Equation (3) satisfying

Equation (4) are


sin n11x/L, odd


un (5)

cos n11x/L, n even

with frequencies
(Un

n11a/L if we assume
(Un? liT.
Consider a gravitational wave propagating in the z-direction. The
force field which it induces is

xk z) (6)
Fj RjOkO(t
= - -

If we take the rod to be at z =


0, we must add a driving term

F(t, x) = -
xR
x Ox oCt) (7)

to the right-hand side of Equation (1). We can express the resulting dis-

placement as a superposition of normal modes:

I Bn(t)un(x)


(8)

The un are orthogonal, so Equation (1) gives

L/2
r F dx
un

..

l' -L/2
B + -

B + UJ
.\
B =
(9)
n n n n
T L/2

r un
dx
-L/2

Since F is antisymmetric about x =


0, only the modes with n odd con-

tribute. For n odd the right hand side of Equation (9) is

2 2

Rxoxo(t) L I (211 n ) .
506 PROBLEM BOOK IN RELATIVITY AND GRAVITATION

If the rod is driven -iWn


on resonance, Le.
Rx Ox 0 ""-J
e t, Equation (9)
with Bn -iwn t
""-J
e gives

1Bn l
T 1
oc


oc _


(10)
W n n

The 2
energy flux F in the gravitational wave scales as
IRxoxolwnl2 oc
1/n ,

so 2
IB n 1 1
sensitiv ity oc oc -
(11)
2 4
1/n n

Solution 18.13. Inside the material we have

Ofl. Oi 0
TGW,fl. TGW,i F
= =


where F is the rate, per unit volume, at which external forces are doing
work on the gravitational waves. Isolate a chunk of the cement of cross

section A and put a Gaussian "pill box" around it. From Gauss' theorem

in three dimensions we have

J J
Oi Oi
TGWd i

T,i d Vol.

(T wl -T
au t in ) I.J FOd
A= Vol.

= rate at which wave energy increases inside

= -
rate at wave energy is absorbed by cement

and hence:

(rate at which momentum is absorbed



(rate at which energy is absorbed

per unit area per unit time) per unit area per unit time) .

Solution 18.14. Take the wave to be propagating in the z-direction and

the molecular displacement to be in the x-direct ton. If g is the displace-


ment of the molecule, the equation of geodesic deviation gives us a

dynamical equation for the molecule's motion. In a complex representa-


tion, the equation is the real part of
CHAPTER 18: SOLUTIONS 507

c i 2 x 1"
s + rs + W


-R
OxO


= -
h X
2 xx 0

or

2 2
_<:L A X e- 1wt

2 2

S (-w -iwr+w o ) = _<:L h X =

+ 0

2 xx 0

Here w is the frequency of the wave, r ==


(damping time)-l, and Xo
is the distance in the x-direction of the equilibrium position of the mole-

cule from the center of mass. We can easily solve for ,

1 2 iw t 2 2 A

[ r\W-W O ) -IW r]/Ll
- .

w A X
oe


2 +


_w )2
2 2
/). ==
(w + w r

and for

1 3 -iwt 2 2 A

_

_

w A X oe .r
[w r +l\W -w
O )]/Ll.
2 +

From this we can easily evaluate the rate at which the molecule of mass m

absorbs energy:

<xF

> =
Ret(g)(Fxtl

Re{(g)(- mw2A:Xoeiwt)}

m r
IA+12X .

To find the momentum absorption we must recall that there is a longitudi-

nal geodesic deviation, though it is smaller by a factor of order v /c

_mRz xxO xog hxx Xo



F = =

1 2 iw t l
Xo Re { A+ e } Re { s

= -
mw s .

The time averaged force is then

<F

> =
mw6X r<Re(A+e-iwt)'Re(A+e-iwt?
1 6 2 2


mw r I A+ \ Xo
508 PROBLEM BOOK IN RELATIVITY AND GRAVITATION

which demonstrates on a microscopic bas is the reason for the res ult in

Problem 18.13.

Solution 18.15. Consider two particles a distance d apart. Their rela-

tive velocity fluctuates under the influence of the wave by an amount

vhwd. If the two particles collide during this cycle of the wave,

their distance must have been of order d ,....,


v/w, so hv.
v""" On the

average then, particles collide with energies fractionally larger by (/1 v /v)2
than they would in the absence of the wave. Since the collision randomizes

their direction of motion, this energy is not subtracted out half a cycle
later when the wave is reversed. Another effect also adds of order ( v/v)2
fractional energy per collision: the fact that particles moving toward each

other at v + v have a greater chance of colliding than do particles with


v- We conclude that at temperature T every collision is,
v. on the

average, increased in energy by order (/1 v / v)2 kT ,....,
h kT . The number of

collisions per volume per time is no


v /f ,....,

(no/f)(kT /m) 2", where no is

the number density and m the mass of the particles. Thus the internal

energy absorbed per volume per time is approximately

dE.In
t h2 3



,....,
1 1 (nOkT)2",...., 1 1

in t

(1)
dt
n2"m2"f n2"m2"f
o o

This is integrated to give


.1..


.1 t


i nt(t 0) =

(2)
t (t)
,....,


to
In -



to,...., (f/h ) (m/kT) 2". Thus, E in in finite time of
where t
00 a
to;
course this does not happen: our Newtonian treatment breaks down at a

time t.,...., when the gas becomes relativistic.


to
The "front" of the loses energy at the rate given by Equation
wave

2 2 2
(1). Since its internal energy density is of order w h c /G, it is damped

in a distance
CHAPTER 18: SOLUTIONS 509

2 2 3
f cr
w h c /G
d amplng

,...., . ,....,
d amplng 3 1 1


(nokT)"2 /(ngm 2f)



3 2


( )
,...., w

\Gmno

thermal

(The first term in parenthesis approaches unity for a relativistic gas; the
second approaches unity for a wave at the "gravitational plasma frequency"
of the gas.)

Solution 18.16. The characteristic quadrupole gravitational-wave power

(G/c ) I QI
5 2 5 2 4
of a physical process is ,....,
(G/c )M v /T2 where M, v, T
are relevant masses, velocities and times for the process. This can be

conveniently written as



p2
Q.
( )
P ,....,
Mv ,....,
GW T
5 59
c 3 x 10 ergs/sec.


Here p,...., Mv IT represents the characteristic "power" of internal

energy flows.

For an explosion of energy E, characteristic time r the "internal

power flow" is E/r. If we assume that this internal power flow couples

efficiently to gravitational waves, then in c =


G =
1 units


GW
,....,
{E/r)2
and

E ,....,
P r ,....,
E /r
GW GW

The typical graviton from this explosion will have energy


emergin
1iw ,....,
"'h/r so the number of gravitons emerging will be of order

2 2
E2/r
( )
E E
N '" = '"

1i

h/T 16
10 ergs
510 PROBLEM BOOK IN RELATIVITY AND GRAVITATION

Solution 18.17. The mass motions relevant to gravitational wave genera-

tion are the collisions of electrons with lattice points, photon scattering,

etc. Since the thermal velocities of the electrons greatly exceed their

drift velocity, the energy involved in a collision is of order kT. Accord-

ing to the analysis of Problem 18.16, the gravitational wave power

generated by one electron should be (kT /T)2 where T is the mean time

between collisions of the electron with the lattice. The total energy, in

gravitational waves, emitted during the bulb lifetime must then be



GW
"""

(k;) T
L /(1059 ergs/sec.)

N =
number of conduction electrons in filament


T =
bulb life time '"
4 x 10 seconds.

The typical graviton generated should have a frequency of roughly



T- so that n, the number of gravitons is of order

EGW T -
N (kT)2 T L
n '" '" .

1r T1I 59
10 ergs/sec.

For the light bulb kT must be on the order of an electron volt, since the

thermal emission is primarily in the visible, and rough values of Nand


17 13
Tare 10 electrons and 10- seconds. For these values the number
19
of gravitons emitted during the bulb's lifetime is of order 10- .

To estimate the number of gravitons given out by the lightbulb when


it is dropped, we assume that the energy of the falling light bulb is con-

verted to kinetic energy of shattering; and we calculate the res ulting


to the formula in Problem 18.16
production of gravitational waves according
If the light bulb weighs 20 grams and is dropped from a height of 1 meter,
6 that the shattering is characterized
it gains '"
10 ergs. Let us assume


by a time of .1 sec, so that the total power in shattering is 10 ergs/sec.
If 10% of this is relevant to quadrupole oscillations, then
CHAPTER 18: SOLUTIONS 511



(10 ergs/sec)2 47
GW
""-J

59
10- ergs/sec
10 ergs/sec

48
E ""-J
P x .1 sec 10- ergs
GW GW

26
The number of -nw ""-J
if/.1 sec ""-J
10- erg gravitons is then on the order
22
of 10- .

Solution 18.18. To linearized order in the gravitational field, the inter-

action Hamiltonian with a system whose stress-energy tensor is T is


/lv

H =
(87TG) 2 T
/lvh/lV .
(1)


33
We shall use units with c =11"= 1, so G2 =
1.616 x 10- cm. For the

electron in the hydrogen atom,

P/lP V
T =

me uf.luv

(2)
f.lv m


we work in the transverse-traceless gauge for hf.l , so

0 h ij 0 (3)
hOf.l
= .


,j

and thus

1 p.p. h ij
1 j
H =
(87TG)2 (4)
me

Since a graviton has spin 2, the decay must be to an s state. The

transition matrix element from a 3d state to a Is state with the emission

of a graviton with wave-vector k and polarization A (for right-hand


circular polarization A = 1 and for left-hand circular polarization A =
-1)
IS
T = < Is; l, A I H I 3d; 0> ・
(5)
We now expand

d3k ij* t


1 -ik'r ij ik'r
h ij (e + e --e
k,A ak,A )
(6)
--ek,A ak,A
_

3 1

(217)2 (2W)2 A
512 PROBLEM BOOK IN RELATIVITY AND GRAVITATION

ij
Here w is the frequency of the wave and e the polarization tensor.

The creation and annihilation operators satisfy the


equal-time commuta-
tion relations

[ak,'\' at",\']


<!. .!{) 0.>v\'


and so


k, AIO> =



k,'\ I at,,'\' 10> = 0 (k .!{)o,\,\,


(7)

Using Equations (4), (6), and (7) in Equation (5), we get


..*
(8"G)2 1 1 ik'r IJ
T =

3 1
< 1s I e --PiP.j e
k,A I 3d> (8)
me (2") 2 (2w) 2
- -

8 4
Since kr,...., (leV) x 10- cm ,....,
10- , we can make the approximation of

putting ei .!.. =
1 (cf. the "dipole" approximation for electromagnetic

transitions). The transition rate is

dr = 2" I T\2 x (density of final states for graviton)


2 2

217 I T 1 w dO (9)

where dO is the solid angle into the


hich graviton is emitted. Hence


dr Gw ij 2


= -

1 <Is 1 p.p.e k 1 3d> 1


1 J
\ .
(10)
d 2 ,1\
17
me

If we use the explicit form of the wave functions, the matrix element in

this equation can be evaluated. Since we do not observe the spins of the

initial or final states of the atom, we will sum over the final spin states

and average over the initial spin states. This is most easily done by

introd ucing the pure s pin-2 operator



Q1J
0 .

p.p.
1 J


0..p2.
IJ
(11)

Since e\ =
0,replace PiPj by
we can
Qij in Equation (10). The
of
spherical components Qij are
CHAPTER 18: SOLUTIONS
513

Q2 '=
(Qxx -

Qyy + 2i Qxy)
Ql ==
Qzx +
iQZy
QO '=
n )t (Qzz- Qxx- Qyy) (12)

Q-l ==
Qzx

iQzy
Q-2 '=
(Qxx -

Qyy

2i
QXY) ・

ij
Since e
A has x-y components (for A =
:!:1):

+'

[1 ]
_1

2- 2
:!:i -1

we find


. .

1J

Qij e


22Q_2A. (13)

Since Qa has spin two, the Wigner-Eckart theorem says

<j.m.\Q
J J a U 11
m .> . =
<j.IIQI\

j.>
1 <2aj.m.U.m.>
11J J
(14)

where the "reduced" matrix is defined as


<j.IIQIU.
J 1
> =

2.J.+ 1
<2a J..m. l j.m.
1 1 J J
> <j.m.IQ
J J a \j.m.
1 1
> (15)

m.m.a
1 J

and where < jm ji miU j mj > denotes a Clebsch-Gordon coefficient. Thus

Equations (13) and (14) (with the n quantum number added) give

l<njjjmjIQijeij*lnijimi>12 2\<njjJQllnih>12\<2 -2A hmiljjmj>12



Now we sum over m. and avera g e over m., using


J 1


(2 j j +
1).. 2
I 1<2 I
1 2
-2A J.m.IJ.m.>1 \<J.-m.J.m .\2-2A>\
..

2'J .+ 1 1 1 1 J 2.J.+ 1 5 J J1 l
1 1
m.m. m.m.
1 J 1 J

2j j + 1


5 2j. + 1

514 PROBLEM BOOK IN RELATIVITY AND GRAVITATION

and thus

dr Gw 22j j +l 1< n.j.IIQlln.j.

1 1 >\ (16)

dO 2 5 2'J + 1 J J
17 m i

This is independent of the polarization of the graviton and of the angle of


emission. The total rate is obtained by multiplying by 2 (for the two

polarization states) and by 417 (solid angle), giving

2j + 1
Gw 16


r =

m2e
_

5 2.JJi + 1 1<n.j.IIQlln.j.>1
J J 1 1

(17)

The reduced matrix element can be evaluated from Equation (14) and any

convenient choice of m. and m.. For example,


1 J


lOOIQoI320>
< 1s 11Q11 3d> =

(18)
< 2020100>

The Clebsch-Gordon coefficient in Equation (18) is unity. From Equations

(11) and (12), 1

Qo

n f (PzPz- p2) ・

The p2 term does not contribute to the matrix element in Equation (18)
because it cannot couple j =
0 to j =
2. Thus

<lsIIQII3d> =

( f <lOO\pzpz\320>. (19)

In general one would introduce a complete set of states to evaluate the

matrix element in Equation (19) as a sum of terms of the form

<100\p In><n\p \320>.


z z
Since the 1100> wave function is so simple,

however, we can use

Pz -ia/az

so

1-.

2 2 1-. sin (J
( )(
sin (J
p2 z
_
a /az = _
cos (J
ar
_


a(J
cos (J
ar
_

r a(J
2 2
_/cos 2 (J

a sin (J



\ ar
r ar
CHAPTER 18: SOLUTIONS 515

when acting on the spherically-symmetric 1100> state. Thus

00 IT 2IT .1 .1
2 3

1 L i 2... 2
(3 cos 0-1) 2(30) 2 1.
( 1617)
2 2
<320 p 1 1 100> r dr S 1°nO dO d
()

'fJ



955 a

o 0 0


a2 2


3a
e- / 0 a 1 1.. e-r/a
r 2 sin
x cos 0 +
2 r ar 1 3
ar
(417)2 a

1 00
4r IT

L L
-6 2



-3a 2
dr
5 2 cos 0 sin 0
sinOdO(3cos 0-1)

r e

_

191 a

2 ra
0 0

1 00 4r

( )



62 8
--

3a
= -
dr e .!- + !:.-
191 a
6 15 2 a
0 a


_
- -
1 62 x 8 x 243 x 57

2 191 x 15 x 2 x 256



.19

where a =
(m e a)-l is the Bohr radius and a =
1/137 is the fine struc-

ture constant.

r =
Gw .
16 . 1. .
3 . (. 19 )2
5 5 2



a4

24

. 36 G m wa .

45
Now
Gm; =
1.75 x 10- and w =
12eV for 3p Is. Thus the life-
1 38
time is r- =
1.9 x 10 sec.

Solution 18.19. A star, and its thermal graviton flux are spherically sym-

metric only in the time averaged sense. Since a star's interior is com-

posed of subatomic particles it is clear that, at a given time the star may

be spherically symmetric to an astronomer, but to a mathematician, with


his much more stringent requirements for symmetry, the star has a fine

grain (atomic scale) graininess which makes all symmetry theorems in-

applicable. It is, of course, the time dependence of these fine scale


516 PROBLEM BOOK IN RELATIVITY AND GRAVITATION

particle inhomogenieties which generate gravitational (and electromagnetic!)

radiation.

The multipolarity of the flux is related to the scale of angular asym-

metries of the flux. (This should not be confused with the fact that the

generation of the waves is locally quadrupole, nor with the fact that the

time averaged flux is monopole -

Le. isotropic.) For a star like our Sun

most of the graviton flux will be generated in the high temperature


7o 3
(10 K), dense (50 gm/cm ) core. If we take as a characteristic radius
10
of this core one quarter of the sun's radius ,....,
2 x 10 cm and the inter-

particle distance of 10- cm implied by the density, we see that the
18
angular scale of asymmetry is 10- radians. The spherical harmonic
,....,

18
expansion of graininess of 10- radians requires e values of order
18
10 In this fine-grain sense then, the flux has characteristically a

18
10
2 -pole distribution.

Solution 18.20. In a nearly inertial coordinate system centered at the

center of mass, the effect of the Riemann tensor, according to the equation

of geodesic deviation, is to accelerate free particles in the following way


d xj k
Rj OkO x
= -


dt

We have here ignored the velocity dependent "magnetic" type terms which

are smaller by a factor v/c. (See solution to Problem 18.14.) This equa-

tion makes it clear that the Riemann tensor has a longitudinal effect if
and only if one of the spatial indices of R
jOkO
is z, the direction of

wave propagation. It follows that 'II 2' 'II


3'
'II

have longitudinal effects
but that 'II 4' 'II 4' (1)22 are purely transverse.

To investigate the spin of the waves, transform from x,y coordinates

to new coordinates x', y' rotated about the z-axis in the positive sense

by an angle cpo Under such a transformation the transformation for the

Riemann components is, for instance


CHAPTER 18: SOLUTIONS 517

Rx'Oz'O' =

xOzO
cos cp + R
yOzO
sincp


Rx'Ox'O'


xOxo
cos cP + R
yOYO sin2cP + 2 sincP coscP R
XOyO

From the form of the transformation for the Riemann components, the

transformation laws for the symbols defined in the problem are easily
calculated:
'I' 2' ;: -

Rz'Oz'o' = -

i R
zOzo

'1'2

'II =
e iCP 'II
3' 3

e- iCP
'113 =

'll3
2 i ,t,.

'1' ' =
e 'f'qt 'II =

-2i,t,.-
'f'qt
4 4 '
4' 4

(I)
2'2'

(1)22 .

Since 'II

and
(1)22 are unaffected by rotation about z, the waves are


scalar (spin 0) waves. For 'II and 'II a rotation of 180 returns
3 3'
the wave to the same state of polarization (e. g. pure real), so these sym-

bols must correspond to spin 1 waves. The symbols 'II and 'II need
4 4

only be rotated by 90 to be in the same polarization state so they repre-

sent spin 2 waves. These symbols in fact correspond to the circular

polarization states of the waves of general relativity.

Solution 18.21. In the figure are the six polarization modes of a weak,
plane, null gravitational wave permitted in the generic metric theory of
gravity. The figure shows the displacement that each mode induces on a

sphere of test particles. The wave propagates in the +z-direction (arrow


at upper right) and has time dependence cos (cut). Solid line, snapshot at

cut =
0, the broken line, one at cut =
'TT. There is no displacement perpen-

dicular to the plane of the figure. The above modes are obtained by using

the equation of geodesic deviation


518 PROBLEM BOOK IN RELATIVITY fl..ND GRAVITATION

y y
o o



x x
\\

( a ) Re'i'4 ( b) 1m 1'4

y y

-- --
o .
". "
" ....
II"
" "
/ ,

I \ ,
, \ ,

I \
x z
I I

, \
,\




, I
\ I , /
, , .II'
" ".
" ....

"- -"

(c ) 4>22 (d) '1t2

x y

z z

( e ) Re'll3 ( f) Im'l'3
CHAPTER 18: SOLUTIONS 519

2 k
d x


xJR
2 O J'o'
dt

Solution 18.22. The function H is determined by the vacuum field equa-


tions O. We first compute the Christoffel symbols from the
Ra{3

Lagrangian for geodesics (see Problem 7.25)



2 .2 ..
.2

x + y -uv + 2 Hu .

From the Euler-Lagrange equation

aL aL
ds ( ) ax
_

ax

we get

X + H 1i =

,x


ry

and therefore r = H .
Similarly y + H u =
0 implies = H
uu ,x uu
,Y ,Y

implies r a{3

and u =
0 =
O. From


v-2H u -4H xu-4H yu=O
,u ,x ,Y

we get
rv = -2H r

= _
2H r

= -
2H .

uu ,u xu ,x yu ,y

All other r's are zero. In the expression (MTW, Equation 8.51b)

1 1
/l r /l ra
Ra{3

/la{3,/l (log (- g) 2") ,a{3 + (log (- g) 2"),/l r

a{3 a{3
- -

all

only the first (note that (- g)


2"
term contributes =
1). We find the only
nonzero term to be
R uu =H +H .

,xx ,yy

Any harmonic function in x and y will therefore satisfy the field equa-

tions. For a plane wave, H is a quadratic function in x and y, so

that (exercise!) the Riemann tensor is a function of u alone and has no

singularities in the x-y plane.


CHAPTER 19: SOLUTIONS

Solution 19.1. The relevant Newtonian equations are

V 2<1> =
417 Gp (gravitational potential) (1)

ap/at + V .

(P =

) 0 (continuity equation) (2)

ay Vp

+ (v. V)v = - - -
V(I) (equation of motion) .
(3)
at


If the universe is static and homogeneous, then v




0, and p and pare

constant in position and time. The solution of Equation (1) is then


(I) = -
17
Gp (r. r) + C .
r + K
3 ----

where C

and K are arbitrary constants of integration and -
r is the posi-

tion vector from the (arbitrary) origin. Equation (2) is identically zero.

Equation (3), however is contradictory: the left side vanishes since .y =


0;
on the right, Vp vanishes by homogeneity, but V<I> is not identically

zero for any choice of C, K.



Thus, there is no solution to the equations.

Solution 19.2. Since the spacetime is isotropic everywhere, it is in par-

ticular spherically symmetric around some nonsingular observer. But by


Birkhoff's theorem (Problem 16.3) the only vacuum spherical solutions are

the Schwarzschild metrics, and the only one which is regular at the origin

where our observer is -
is the M =
0 case, Le. flat Minkowski space.

Solution 19.3. By Liouville's theorem the number density of photons in



phase space Iv/v is a constant along a light ray, and (see Problem 5.10)

520
CHAPTER 19: SOLUTIONS 521

it is also a Lorentz invariant. Usin g (V emltte d /V 0 b serve d)




1 + z' We

have then

Flux observed = n
fI bs
dv
obs
= n
f ( bS
)V v
obs
bS
dVobs


Iemit
f f
v dv
{} emit emit {} emit
= =
I dv .

3 4 4 v emlt

emlt

(1 + z ) (1 + z )

{} 1. 4

a T
17
(1+z)4

where a is the Stefan-Boltzmann constant. Note that since a redshifted

black body spectrum is also a black body spectrum, the flux and spectrum

observed is just that which would come from a nearby stationary object of
solid angle {} and temperature T/(l+z).
In all the above it doesn't matter whether the redshift is due to doppler
motion, gravitational redshift, or cosmological redshift, because Liouville's
theorem holds in general.

Solution 19.4. Homogeneity implies that the scalar curvature R of the

hypersurface must be constant. R can be found by specializing the


formulas of Problem 9.20, or from the formula

k .1 k .1 k m
R.. = r -(log 1 g12) ,1J + r (log I g 1 2) -
r r
k im
..

J.k
.. ..

lJ lJ, k 1J ,

where Iglt =
a3fr2sin and the r's are found from the Lagrangian for

geodesics or as in Problem 7.6. This gives

rrp rrp 2rO


ROO
= rr
OO,r

(log (sin 0)),00 + rr


00 (log fr2),r -

Orp Orp

Orrr 00
1 2 2 2

_(rf-2) ,r + cosec

0_r- f-3(r f) ,r -cot 0 + 2f-



[r (1- f -2)] ,r (2r)-1.
522 PROBLEM BOOK IN RELATIVITY AND GRAVITATION

Since
2 2 -1
Ree Ree (a r)

and

R;; =


Ree
by isotropy, we get

R =

R;; + Ree + R(j,(j,



3[r2(1- f- 2 )],r (2a 2 r 3 r 1 == A

tha t is

-2)

r (1 -

f =
Br + C .

Since f 1 as r 0, we have C =
O. Thus

2 2 -1
f =
(1 -

Br) .


If B 1= 0, we can scale r by letting r' =
I BI"2 r. The metric then takes

the prescribed form with radial coordinate r'.

Solution 19.5.

(a) Let

sinx k =
+1

r =
X k= 0

sinhX k =-1

then

cos
X dX
dr =
dX
cos h X dX
and

dX

dr 2


dX
1- kr 2
dX


dr 2 2 2 2

+ r d0 =
dX + l2(x)d0
1- kr
CHAPTER 19: SOLUTIONS 523

where


sin x ,
k =
+1

2Cx) X2 =
k= 0

sinh x, k =
-1 .

(b) Let dt =
R(17)d17 define a transformation from the variable t to
the variable 11. Then

2 2 2 2
ds =
-dt + R (t) (dX + 2d02)
2 2

R (11)(-d11 + dX2 + 2d02)

Solution 19.6. From the Robertson-Walker metric (in "trigonometric"

form), the metric for the spacelike 3-surfaces is

2 2 2 2 2
dcp2)]

da =

[dX + sin x (d0 + sin 0 .

By analogy with the metric for 2-spheres, it is easy to guess that this

metric can be represented as a 3-sphere embedded in a Euclidean 4-space

with cartesian coordinates W, X, Y, Z. In particular, if we define

W =
a cos

Z =
a sin X cos 0

X =

sinX sinO coscp
Y =

sinX sinO sincp

2 2
(which satisfy W + Z2 + X2 + y2 =
A ), the metric becomes

2 2
d =
dW + dZ + dX2 + dy2 .

In Euclidean 4-space one can find two planes (e.g. X- Y and W-Z)

which intersect only at the origin (a =


0). Thus, a rotation of the X- Y

plane changes all X and Y coordinates except the origin and leaves

all W-Z coordinates fixed; and vice versa for W-Z rotations. There-

fore, a combination of both rotations leaves no points fixed except the


524 PROBLEM BOOK IN RELATIVITY AND GRA VITA TION

origin and (for nonzero a) takes all points of the 3-sphere into different

points.

Solution 19.7. In some local proper frame suppose the bullet passes by
with a proper velocity V. When it has moved a proper distance dr farther,

it passes a cosmological observer whose velocity relative to the first

proper frame is
R dR
oV =
H dr =
HV dt = -
V dt =
V -

R R

This observer sees it moving


(according to the velocity addition formula)
at velocity
V -oV 2 2
V' = =
V -

(1- V )oV + f)foV )


1- V o V


Y _(1_y2)y d: + e(Oy)2 .


2 2
Thus, dV/dR =
-(1-V )V/R which is integrable, giving yV ==
(1_V )-2"V

constant/R =
constant x (1 + z). So V 1 and V

are related by
Y2 V 2/Y l V 1

(1+ z)-l. For a particle with nonzero rest mass, this result

says that the relativistic momentum redshifts away as one power of (1 + z)


as the universe expands. For a photon, in the limit V 1, yV h v and

we obtain the usual redshift factor. (Note that we have not taken into

account the slowing down of the bullet due to the gravitational attraction

of the cosmological matter at rest in the proper frame. The reader can

verify that this is a higher order effect, with dV oc:


(dr)2, so it does not

affect the differential equation for dV /dR.

Solution 19.8. Start with the 11, X form of the metric

2 2 2
ds =
R (11) [-d11 + dX2 + 2{x)dn2] (1)

(3rd form in Problem 19.5), where I{x) =


sinhX, X, sinX for k =
-1,
0, +1. Since conformal transformations preserve light cones, a natural

first step is to go to null coordinates, e. g.


CHAPTER 19: SOLUTIONS 525

u = 1. (11 + X)

(2)

v =

2 (11

X) ・

The metric now becomes

2 2 2 2
ds =
R (u+v)[-4dudv + I (u-v)d0 ] .
(3)

We now try a transformation which preserves the "nullness" of the coordi-

nates, and also preserves the symmetry between u and v:

a =
g(u) u =
f(a)
(4)
(3 =
g(v) v =
f(f3)

where g is the inverse function to f. With this transformation, Equation

(3) becomes

2 2 2 2
ds =
R (u+ v) [- 4f'(a)f'(,B)da d(3 + I (u -

v)d0 ]

[ ]
2 I (u-v) 2

R (u+v)f'(a)f' (f3 ) -4da d,Q
fJ + d0 .
(5)
f'(a) f'(f3)

If the metric is to be conformally flat, the term in brackets must have the

flat space form [- 4 da d(3 + (a -

(3)2 d0 ]. Using f'(a) =


du/da =
(da/du)-l

[g'(u)]-l, we get the condition

g'(u) g'(v) [I(u -

v)]2 =
[g(u) -

g(v)]2 .
(6)

For the case k= 0, we have I(u-v)= u-v, so g(x) =


x is a solution

by inspection. For the other cases Equation (6) can be solved by first

finding the solution for v close to u; that is, for v =


u + (. Expanding

Equation (6) in a Taylor series gives

(g')2 [1 + (g"/ g' + (2 g"'/(2g') ...] (2 (1- k (2 /6 ...)2


= (2 (g')2 [1 + (g"/(2g')+ (2 g'''/(6g') ...]2 ,

so that with p =
g', q =
p' and k =
+1, we must solve
526 PROBLEM BOOK IN RELATIVITY AND GRAVITATION

2q dq/dp -

4p =
3q2/p .

This Bernoulli differential equation gives



p(Ap-4)2 , A =
constant ,

and integrating twice we get


u + B =
tan- (Ap/4 -l)t

g(u) =
C tan(u+B) + D .

We can now verify by substitution that this is in fact the general solution
to Equation (6). Without loss of generality we can take g =
tan u. For

k =
-1, we can similarly find g =
tanh u. With these solutions, Equation

(6) turns into the trigonometric identities

secu secv sin(u-v) =


tanu -
tan v

sech u sech v sinh (u -

v) =
tanh u -
tanh v

and the metric of Equation (5) becomes

2 l l
2 R [tan- (a) + tan- ({3)] 1- 4 da 2
ds =

2 2
dtJ + (a _

tJ)2 d0 1
(1+a )(1+f3 )
or

2 l I
2 R [tanh- (a) + tanh- ({3)] 2
ds =
1-4dadtJ + (a _fJ)2 d0 1
(32)

(1- a ) (1 -

which is manifestly conformally flat. A conformally flat metric has zero

Weyl tensor, so the Riemann tensor is only composed of the Ricci tensor

and scalar curvature (see the introduction to Chapter 9).

RatJyo = 2°
[a[yRtJ 1] -

o[Ly0tJ1] R .
(8)

With this the Einstein equation


CHAPTER 19: SOLUTIONS
527


R 817
(TIf3 gaf3 T)

f3

and the stress energy tensor

a a
T (p+p)u u{3 +
pga{3

f3

give R
a {3.In terms 0f u /l , p, p.
yo g/lV'

Solution 19.9. If D is the physical size of an object, and a is its angle

subtended, d

== Dlo. From the figure, and the Robertson-Walker metric,
we see that D =
R(t) rl 0, so d



1 R(t 1 ). If the object is moving trans-

versely at a proper velocity V and with an apparent angular motion

r. Ro .

/ / now,R=R o

:i-I
'-I
I I I 1I

then, R=R(f.)
528 PROBLEM BOOK IN RELATIVITY AND GRA VITA TION

do /dt, then d == V /(do /dt). Let t' denote time measured at the emis-

sion of photons. Since V =
d(R(t 1 )r 1 o)/dt', and since dt'!dt =
R(t 1 )/R o
because of the cosmological redshift, and noting that R(t 1 ) can be con-

sidered constant since its change induces no transverse motion, we get


dM =

Rorl' If the object has an intrinsic luminosity L, and we receive


a flux f, then dL ==
(L/4 1Tf) 2". In a time dt' it emits an energy Ldt'.

This energy is redshifted to the present by a factor R(t 1 )/R o '


and is now

distributed over a sphere of proper area 41T(rl Ro)2 (see figure). Thus,
f =

(Ldt'R/Ro)(41TrlRo)-2/dt and dL =

R rl/R(tl)'Using R o /R(t 1 ) =

1 + z, we have now obtained (1+z)2 dA =


(l+z)d M =


Solution 19.10. From the solution to Problem 19.9 we have dL =

R rl/R(tl)
where r

is the radial coordinate of the object and R(t 1 ) is the
s ale
factor at the time t1 at which the light was emitted. Since Ho == R/R

and qo == -
R R/R 2 ,
the first terms in the power series expansion of R

are

R(t) =
Ro
C1 +
Ho(t to)
- -

qoH (tt o )2

+ ..

-) ・ (1)

d R o /R(t 1 ) 1 + z, but
We eliminate the factor R(t 1 ) from using

can L

we still need to find expressions for Ro and for r


Setting ds =

metric gives
to get the path of light rays in the Robertson-Walker

to
(1

dt _

dr (2)



R(t) Jo (1- kr

)!
to the lowest 2 orders in (to

t ) and r by
-Integrating Equation (2) 1 1

means of Equation (1) gives



[(tO-t1)+
0 Ho(to-td! +
...J (3)

o /R(t 1 )
in terms of 1+z R
and inverting Equation (1) t
to

to get -

io [z ..-J
gives
ROr1
= -

(1+qO)z2 +
CHAPTER 19: SOLUTIONS 529

so that

io [z (1-qo)z2 ..J

(l+z)Ror l



+ +

or
equivalently

LH
f L --..Q [1




(qo

1) z + ...] .

411 d

411 z2

Solution 19.11.

(a) Let n(t 1 ) be the number of sources per unit volume at time t1 .

The volume element is

1 1

I (3)gl2" drl
2"
kri) ri dr 1 sin e 1 del dcpl

del dcpl =
R (t )
1 (1- .

Thus the number of sources between r and r + dr at time t1 is


1 1 1



dN =
411R (t 1 )(1-kri)-2"ri n(t 1 )dr 1 .

The quantities r and t1 are related by the equation for null rays propa-

gating in the Robertson-Walker metric (Equation (2) in the solution of

Problem 19.10) i.e. r




r(t 1 ) where

2 2"
dr 1 =
(1- kr 1 ) dt 1 /R(t 1 ).

We have, therefore

2 2
dN =
411 R (t 1 ) r (t 1 ) n(t )
1 I d t1 1
and
to
N(z) =

f t

41TR2(tl)r2(tl)n(tl)dtl

where t ' the cosmological time corresponding to redshift z, is defined


implicitly by
R( t z ) 1

(1)
R(t o ) l+z'
530 PROBLEM BOOK IN RELATIVITY AND GRAVITATION

Since the number density of sources is conserved, we have


net) R (t) =
constant.

(This follows from 0) and hence


(null);{.l

to
N(z) 417n(t o )R\t o )
f R-l(tl)r2(tl)dtl'

If z is small, to R:: t ' and we can use the expansions


R(t 1 ) =
R(t o ) [1- Ho(t o -

1)
+ ...]

to t 1
[ 1+ 2 ]

r(t 1 ) =

HO(to-t 1 )+
...

(2)
R(t o )

(Solution 19.10, Equations (1) and (3)). Thus for small z

to
N(z) 417 n(t o )
f (to t )2 [1 + 2H o (t o t ) + ...] dt 1
- -

1 1

'7 n(to)(t o



)3 [1 +
Ho(t o



) +
...J ・

To evaluate to



to (9(z2) from Equation (1) we need the next term in

Equa tion (2):

R(t z ) =
R(t O
)[1 -

Ho(to-t z
) -

qoH (to-tz)2+..] ・

This gives

HO(t o



) =

[1 -
z +
) o..] +

and so, finally,


(to)
[ ]
417 3 3
1--
N(z) z
z(l+qo)+
...
= - - ・

3 2
H3o

Note that the field equations did not enter to this order in z.
CHAPTER 19: SOLUTIONS 531

(b) The flux received at the Earth is given by


LR (t 1 )
S =
(3)
411 r R4(to)

(cf. Problem 19.9, where S was called f). Thus, as in part (a),

to
N(S) =

J s
41TR2(tl)r2(tl)n(tl)dtl (4)

where, from Equation (3),



r (ts ) L
= (5)
2 4
R (t

) 411 SR (to)

When S is large, to -



is small and expanding the integral in Equation

(4) as in part (a) gives

N(S) =

4; n(t o Ht o -ts )3
[1+ HO(to-t s
)+
..] .

Inserting the expansions for r(t s ) and R(t s ) in Equation (5) g ives

1 1

) [


3 2


-t

( l
411 S
1 _



0 ( )411S
+ ...

and so
3 1

N(S)
4; n(t o )
(4 YS [1 3H
(4 rS ...] +
= -


Solution 19.12. Choose a new radial coordinate X, defined by dX2 =

2 2
dr /(1- kr ). Then for a radial photon dO =
dcp = 0 and the metric coeffi-

cients are independent of X along the ray. Therefore X is an ignorable


coordinate, and
pX

constant along the ray (see Problem 7.13). Rais ing

this index with the metric gives


pX =
C/R (t)
532 PROBLEM BOOK IN RELATIVITY AND GRAVITA TION

where C is some constant. But pX =


dx/dA, so


1 2 1 R
dA =
C- R (t)dX =
C- (t)d\2

(1- kr )2

Note that since R is a function of t it is therefore a function of r along


a pas t null ray.

Solution 19.13.
'"
'"
In the comoving orthonormal frame
'"
we have TO''''0 = _
p'
and
T;: =

TCPi> TOe= =
p.
-0
'"
The trace-reversed stress energy tensor

'"
_0

thus has components T 0 (p + 3p), and


Tli p). We equate

(p
= = -

2 2
this to 1/811 G times the Ricci tensor. The Ricci tensor can be calcu-

lated e.g. from the results of Problem 9.20 (general spherically symmetric
metric). The components are:


..




3R/R


.. O


Rli =

2 (RR
+ 2R + 2k) .

The equations in the problem now follow immediately. If, equivalently,


the Einstein tensor
Gf.LV (trace reverse Ricci tensor) had been equated
to 811T
f.Lv'
two linear combinations of the equations would have been
obtained.

Solution 19.14. Simply eliminating R from the simultaneous equations


in Problem 19.13 gives the first equation here. The second follows from

this, from the identity d[(R)2]fdR R, = and from Equation (1) of Prob-

lem 19.13:

PR2 =! (R)2
!..i.. 811G 4
2 dR ( 3 2 dR ) =
R = _


11G(p+3p)R


dR
(PR ) =
-(P+3p)R

d 3 2
dR
(pR ) =
-3pR 0
CHAPTER 19: SOLUTIONS 533

Solution 19.15. Equation (1) follows immediately from the definition


H == R/R and the first-order Friedmann equation (Problem 19.14)

R2 + k =



PR

If we now take
this equation, use the identity
d/dR on d[(R)2]/dR= R,
3 2
the other first order equation d(PR )/dR 3p R and = -
the definition

'2
..

q == -

R R/R , we
get Equation (2). If p? p then the left side of Equa-
tion (2) is negligible compared with the right side, and Equation (3) results

Equation (4) is immediate upon substituting Equation (3) into Equation (1).
If p =
1/3 p, then eliminating p from Equations (1) and (2) gives Equa-

tion (5), while eliminating the k/R term gives Equation (6).


Solution 19.16. Using Tg = -
p, and T

(no sum) =
p, and
P,j

(homogeneity) the 11

j component is

o -



v -




_
T vr



.I. ar vav
j; v j, v a jv j


pro.J o (p + p)ro.J o


p ,j +

pr .
+ pr .
= =

Jk JV

which is a tautology. The 11 =


0 component gives

a v
o =

cr;v =

cr,v -

T:r ov
+ T
<fr av

.1
dp °
= -

dt
+ pr 00

(p+p)(log\g 2),0


has been used (see Problem 7.7). Thus
where r
(log\g\2),{3

a {3

d P / dt = -
(p+p) (R )

3 dt

and
3 2
d(PR ) = -
3p R dR

which we recognize as one of the first-order Friedmann equations (see

Problem 19.14).
534 PROBLEM BOOK IN RELATIVITY AND GRAVITATION

Solution 19.17. With p =


p =
0, and k =
-1, the first-order Friedmann
equation
R2 + k =
817 G
PR

becomes R = 1 which implies R =


t and thus the metric given is correct.

Since the metric is spherically symmetric, the curvature radial coordinate

is
r =
t sinh X .

It is not hard to guess a time coordinate

T =
t coshX

and then to compute the transformed metric

2 2 2 2 2 2
ds =
_dT + dr + r (d0 +sin 0dcp2)
which is empty Minkowski space.

Solution 19.18.

(a) When matter dominates, we ignore press ure so that mass-energy

density decreases in proportion to the increase in volume of the universe,

and

p=PO( R )3 .
(1)

Define a new time coordinate ("development angle") by dl1 =


dt/R. Then
the Friedmann equation becomes

( t 8;G Po ( R Y

(dR::Y (2)
= = --


or


R-t dR .Q.. Rt 817 G 3

2 =

( P0 R0 kR

_

dl1 dl1 3

which integrates to give


CHAPTER 19: SOLUTIONS 535



-1 R2
sin k =
+1

( 1TG
POR r

l.
R2
R2 k 0 (3 )
dRt



1. G
11
(8; R )2

2 1 Po

(8; POR -kR)2 1

sin h

1 R2
k =-1


(8; Po R r
From Problem 19.15 we know

417 GPo
qo
= -
(4)
3 H2

and

R =
(k=:!:l) .
(5)
(2qo -

1) H

We see that since the left side is positive, k =


sign (2qo -

1). Thus in
Equation (3)
817 R
3 2qo (k :!:1)
'3 Po 0
= =

Ho I (2q o 1)1
- 2

Inverting Equation (3) gives

qo k 1

(1 -
c os 11) =


o (2q o -1)2

R= 1. H


3 n

k =

4 0 0.'

qo -1
(cosh 11- 1) k =

Ho(l- 2qOY2

Now integrating dt =
Rdl1 gives
536 PROBLEM BOOK IN RELATIVITY AND GRAVITATION

qo .


(11 -

sin
11 ) k =


o (2q o -

1)2


1 2 3 3
Ho Ro 11 k=O
=,

12

qo

(sinh 11-11) k = -1 .

Ho(1- 2q o)2

The fact that cannot be eliminated from the 0


Ro k =
case merely re-

flects the fact that the universe here has an arbitrary scaling and its

geometry looks the same at all times. The value of will not enter
Ro
the computation of any physically measurable quantity.

(b) When radiation dominates, the mass-energy in a given comoving


volume is not constant. There is an extra decrease in density from the
redshift of photons, so


Po ( f ・

The analog of Equation (2) is

( t (dR Y G( R)4

= =

8 Po
--


or

dR


dl1
( 17GPoR -kR2)2
which has solutions:

sin 11 k = 1


( 8; GPoR Y x k 0 (6)
= =
11

sinh 11 k = -1 .

Instead of Equation (4), we now get (Problem 19.15)

817 GPo
qo =

3 2


CHAPTER 19: SOLUTIONS 537

and instead of Equation (5), we have

2 k
Ro =
(k = :t 1) .

(qo -l)H

Equation (6) therefore becomes

qo
k = :t1

817 G (qo -

1)2 H
P 0 R4


2 4
H R k=O
o 0

and integrating for t gives

1..

1 qo

Ho qo [ -

] (1- cos 11) k=l

122
t k=O
2 Ho Ro 11

1..
1 qo2
Ho [ 1 -

qo ] (cosh 11-1) k =
-1 .

Solution 19.19. After the bullet has reached nonrelativistic velocity, we

have (from Problem 19.7)

dr dr
p p dr dR dr
constant = - -
RR (1)
R df dr dR dt dR

where r is the Robertson-Walker coordinate, related to proper separations

(r P ==
proper separation) by dr P =
R dr (hence the last equality above).
2 .

For a k = 0 Friedmann universe R ex f3 so R ex R-2 and Equation (1)


gives us dr/dRex R-2 and hence


r = A + B/R2 .

As t 00, the approaches r =


A. The proper distance between
rbullet

the r =
A observer and the bullet is R =
BR2.
r Thus the proper dis-

tance becomes infinite even though the velocities (according to Equation


(1)) approach each other.
538 PROBLEM BOOK IN RELATIVITY AND GRAVITATION

If k =
-1, then R becomes constant at large times and Equation (1)

gives dr/dR ex R- so that
r =
A + B/R .

As t 00 the bullets coordinate r approaches A, but the proper dis-

tance to the r =
A observer RL\r approaches B, a constant.

Solution 19.20. Measuring time in terms of development angle 11 defined

by dl1 =
dt/R(t), the Friedmann metric for a radial (de =
dcp =
0) photon
becomes
2 2 2 2
o =
ds =
R (11) (- d11 + dX )

dX2

where =
dr /(1- r ) is the "trigonometric" radial coordinate on the

3-sphere (see Problem 19.5). From Problem 19.18 we see that the universe

lives an interval L\ 11 =
21T (time between two zeros of R), so in this time

a photon can propagate L\X =


21T, Le. exactly once around the universe.

Solution 19.21. For a k =


0 Friedmann universe we have


R(t)/R o =

(2 Ho t ) ・

The energy emitted per source per unit proper time is L. This is red-

shifted by a factor R(t)/R o ' so the total energy per star which is now

present is to 2


3 3 3
L R(t) dt 5/3
& =

o Ro



( 2 HO ) to

or =

2L/5Ho since
to

2/3 Ho. The energy per volume, u, now is

thus H

Ln. Since the energy fills the universe isotropically, it

corres ponds to a flux


c c -

B =
U =
H L n.
41T 101T 0

[By comparison, a Newtonian universe where the stars "turned on" a

-1
time
to

( Ho) ago would have


CHAPTER 19: SOLUTIONS 539

I t

2-1
& =
L dt =

Ho L


and

B =

1TH O Ln
.J
Solution 19.22. From the solution of Problem 19.18, if > 0.5 we must
qo
have a k =
+1 cosmology which expands according to

qo
R(t)/R o =
(1- cos 11) ・
(1)

2Qo
_

Setting R(t) Ro' so that 11 gives a formula which relates to


110'
= =

110
at the present epoch. The epoch is arbitrary, however, so it also re-
qo
lates 11 and q at any epoch. Putting q =
0.5002 gives 1 -
cos 11

.0008

at z =
1500. Substituting this back into Equation (1) and putting Ro/R(t)

1+ z =
1501, we can solve for getting
qo'

qo

[2 -

(1 + z) (1- cos 11)]-1 =


1.25 .

If < 0.5, the k =
-1 analogy of Equation (1) is
qo

R(t)/R O =


(cosh 11

1) (2)
qo

so q

.4998 gives cosh 11- 1 =
.0008 at z =
1500 and

qo

[2 + (1 + z) (cosh 11- 1)]-1 = 0.312 .

Solution 19.23.

(a) At the present epoch t =

to' the spacelike 3-surface has the

metric

( )
2 dr2 2 2 2
da =

R 2
+ r (d0 + sin 0 dcp2) .

1_r

Noting that O:S r :S 1 covers half the 3-s phere, we have


540 PROBLEM BOOK IN RELATIVITY AND GRA VITA TION

I t=to (now)

I I I I I I I II
.-r-+


r=O Ir=r:0


t=O (singularity)

1 1 2rr
R30 r2
J I (3)gl t d
1 J I
3 2
V =
x =
2 dr d(cos 0) de/>

1.


217 R .

a -1 a (1 _
r )
3-sphere

For a matter dominated universe (see Problem 19.15)


RO


O (2Q O -l)-t (1)

so



=2rr2(2qO-1)-2 H


(2)

(b) Our past light cone sweeps outward a proper radial distance c dt

in a time dt (see figure). From the metric, the proper area of a 2-sphere
2 2
is 417 r R , where rand R are both functions of t along the light cone.

Thus, the visible volume is (c= 1 now)

ta


2 2
V =
dt 417 r (t) R (t) ・
(3)

The differential equation for r(t) follows from its definition as a null cone:

-dt R(t) dr


' (4)
2 "2
(1 _
r )

and r =
0 at t =

to' To solve Equation (4) and evaluate the integral in


CHAPTER 19: SOLUTIONS 541

Equation (3), we use the solution for R(t) parameterized by "development


angle" 11 (see the solution to Problem 19.18).

R =
A(l -

cos 11) , t =
A(l1- sin 11) (5)
where
qo
A== (6)

(2q o -1)2 Ho

and the current value of 11 is

C :o)


cos- ・
(7)


Equation (4) now becomes d(sin- r) =
dl1 so that

r =
sin
(110 -11) (8)

and Equation (3) becomes

T/O


2 2
v=
A(I-cos )d 417sin2( (l-cos
o- )A ) ・

This straightforward integral gives

3 61 5 5. 3 1.
4 'IT A
( 110 + 30 sin 3 110

V sin 2 2
110 + 4 110 sin cos

110

8 110

80 2

which can be reduced with Equations (6) and (7) to a function of


qo
and Ho.
(c) The farthest coordinate radius that we see is, according to Equa-
tion (8), 1

rO

sin
(110) =
(2q o -

1)2/qo

where Equation (7) has been used. The volume out to this coordinate

radius at the present epoch is (analogously to part (a))

ro R 3 2 d r



Or
ro(l- ro) ]
2 2
Ro [sin- ro
3 1
V =
4'IT = 2'IT -


o 2 2
(1- r ) 1

fcos- qo\
qo)(qo qo-1)21J
(1-


1-

217 1
_

E.
H (2qo
-1)2 L qo /
542 PROBLEM BOOK IN RELATIVITY AND GRAVITATION

Solution 19.24.

R=R

r+
r=O

R=R(f.)

From the figure and the Robertson-Walker line element, f =


[r 1 R(t 1 )]0. We

can express R(t 1 ) as R


o /(l + z); the hard part is now to compute r

Use the TJ,X form of the metric from Problem 19.5. Then r

I(X1)' The

path of a photon is 0 =
R2(dX2 dTJ2), -
so
X1

TJ(t o ) -

11(t 1 ), and using


the formula for sin (A -

B), sinh (A -

B) we have:

r1

(l1o)C(111) -

(111)C(l1o) ・
(1)

[Here C means cos, 1, or cosh for k =


1, 0, -1.] Now we need the

formula relating TJ of an epoch to its redshift and the current value of q

(Problem 19.18):


2qo -


C(l1) = -

1+z
qo

qoz qo + 1

C(17) =

qo(l + z)

(This is true for all three values of k.) With this result we have

1 1

\2qo 11 (2q o z+ 1)2

I(17) =

qo(l + z)
CHAPTER 19: SOLUTIONS 543

12qO 11 2 -

[1
1-


= -

qo + qoz

(1 -

qo) (1 + 2q o z) 2] ・

q (l+z)

So the apparent angle is

a =
E/r 1 R(t 1 ) =
E(1+z)/r 1 R o '

and when we use Equation (3) of Problem 9.15 to eliminate


Ro we get,

finally,

a =
EH O (1+z)2 q

[l- q o +
qoz -(1- q o)(1+2q o z)2]

Solution 19.25. We work with units which make c =


k =
1. The density

of matter (m p =
average baryon mass) is P ma tt
= m n. The density of


radiation is Prad

KaT , where a is the radiation constant =

5 3 4/3
(817 )/(15h ) and K is 1 (considering photons only) or 1 +
({I) =

1.454 (considering photons and neutrinos -


see, Weinberg, p. 537). Equat-
ing these give

1 Prad KaT
= =
(1)
Pmatt nmp

The Saha equation for the fractional ionization x of hydrogen is




(217mekT) exp(-i a2me/kT)
I- x
nh3

(m e

electron mass, a =
1/137). If recombination is defined by the left
side of this equation being about unity, Equation (1) gives us


15 (217 me T) 2
1 2 mp
exp( 2 a
me /kT ) =

817

KT

Obviously the exponential sets the scale of the solution, T


a2me;
guessing a coefficient, computing the right side, and iterating once gives
544 PROBLEM BOOK IN RELATIVITY AND GRAVITATION

more accurately: T io a2me (d. Problem 19.26). So, finally, when


matter dominance and recombination occur at the same temperature T, we

get

4 mp

4 aT
( )
KaT
a= =

3 11 3 TK m n

4 mp 320
= =

3K (137)2 (1836)
3 .1 a2m

(80 )


4 x 10 .

Solution 19.26. The Saha equation for the equilibrium fractional ioniza-
tion of hydrogen is (units with k=c= 1)


B /T

2 (217m e T)2 e-

(1)
I-x 3
nh

where B is the ionization energy =

a2me(a =
1/137), and n is the

number density of protons plus atoms. The value of n is related to a,

the entropy per baryon by



4 aT
a= --

3 n

5 3
so if we take x =
1/2 and use a =
817 115h , Equation (1) becomes


melT

2t 417 7 / 2 1 T
( me) exp

- - -

45 a
2(137)2

This is easily solved by iteration. For example, iterating twice:

(log 6.908 )
me 2 a 3 me

2(137) + log
If 2 T


2(1:;)2 [log 6.908

(10.53 + log log
)J-l
6. 08
0 8
4330 K for a =
10

140500K for a =
10 .
CHAPTER 19: SOLUTIONS 545

Solution 19.27. From Problem 19.18, near the singularity in the radiation-

dominated regime,
1 .1 2
817 4 2 817 4 2
11
R =

( 3 Po Ro ) 11, t =

( 3 Po Ro ) 2' (1)

(This equation holds for any value of k in the limit 11 0.) Thus we have


2 3R 3
t = =
(2)
3217 P
3217
Po R

where we have used P oc
R- for the radiation-dominated era. The photon

energy-density is 00


817 q dq
_ _

aT4
Py
- -

3 exp (q/kT) -


5 4 3
where a is the Stefan-Boltzmann constant 817 k /15h . For highly rela-
tivistic electrons and positrons (which obey Fermi-Dirac Statistics)

00

817 r q3 d q 7
aT

Pe-
= = =

Pe+

h3 J.. o
exp(q/kT)+l 8

Thus the total mass-energy density is

11
P aT4
Py+Pe++Pe-
= =

and so

4 .1
4 3
( )
_


T = .
t .

11 3217 a

If neutrinos are included and we ass ume they have zero chemical potential

(cf. Weinberg, Section 15.6), then since they have only one spin state,

1. 4

P- P P- aT
Pv e
= = =

v 16
e vf1. vf.l

11 4 7 4 9
P =
aT + aT =
aT4
4 4 2


2 3 4
T =

( 9

3217 a ) t- 2 .
546 PROBLEM BOOK IN RELATIVITY AND GRAVITATION

Solution 19.28. For relativistic particles, E p, the equilibrium

density at a temperature T is (see the solution to Problem 19.27):


aT for photons

f .2 4
peT) =
En(p)dp = aT for each neutrino species
16


aT4 for each electron or muon species.

5 4 3
Here a is Stefan's constant =
(81T k )/(15h ). For this problem, then,
we have
1- + 4 7 4
(1 + 4
) aT at
Rl
. ・

16 8

peT) = -
KaT ,

1- + 2 7 4
(1 + 4 ・

16

8 ) aT at R

K1 =
25/4 ,



18/4 .

We now use the fact that the expansion of the universe is isentropic. The

entropy per volume of such a relativistic gas is calculated from the first

law of thermodynamics to be



KaT .

Since V, the volume, varies as the expansion factor cubed, we have

constant =
S IX
Ka(RT)3 ,

giving for isentropic expansion

3 3 3 3
K R T =
K R T
1 1 1 2 2 2

and

( )(
R1 25 t




1 R

18 ) .

Physically, this result says that in annihilating, the muons dump their
energy (actually entropy) into the remaining particles, and increase their
CHAPTER 19: SOLUTIONS 547

temperature by a factor (25/18)3" over the expected first-power decrease

of T with R.

Solution 19.29. The standard model produces helium and deuterium by

(roughly speaking) (1) freezing out a nonequilibrium neutron/proton ratio

when the weak interactions for n p become slower then the timescale

for the expansion to drop the temperature, (2) making deuterium by n+ p

d+y while most free neutrons have not yet decayed, and (3) using up
3 3 3 3 4
almost all d by d+d He or H + etc., He or H +d He + etc.

We can thus answer the problem:

(i) There is no change in freezing out ratio, but higher baryon densi-
ties at any given temperature give more "cooking," therefore
4 4
more complete transformation to He : more He , less d.

(ii) The n p reactions is slower, so more frozen out neutrons;



other rates not changed, so: more He ,
more d.

The reaction right by the de-


(iii) n+ v p + e is driven to the

generacy (Fermi) energy of the neutrinos; fewer neutrons give



less He , less d.

(iv) This is similar to (iii) but for v+p n+e+, giving more He ,

more d.

(v) The expansion time is related to density by


2"
t =

( 32:GP ) + constant .

Increased G thus means smaller times spent in each density stage. This

gives a higher nip ratio. The small change in G does not substantially
affect the fact that the neutrons are almost all consumed to d, since the

decay time is still long, so: more d, more He .

Solution 19.30. The cosmological constant is equivalent to a perfect fluid


with effective density and pressure

P eff

(817)-1 A ,
Peff
= -
(817)-1 A ・
548 PROBLEM BOOK IN RELATIVITY AND GRAVITATION

The dynamical equations of Problem 19.14 become


R2 =


AR

with solution


t/To -

1. A

Ro e T -


The metric is then

2 2 2t/T 2 2 2
ds = -
dt +
R e 0(dr + r d( ) .

To transform this to a manifestly static form, introduce a "curvature

coordinate"

tiT 0

Ro e r

so that the metric becomes

2 2
ds = -
dt + (dr' -
r' dt/T
0)2 + {2 d0 2 .

Now eliminate by the transformation


gr't

,2

( )
1 !-
t =
t' + -

2 To log -

T2o
and find

(1 )dt'2 (1 rli

ds = - -
+ -
dr,2 + r,2 d0 2 .
(2)

The interpretation of Equation (1) as a nonstatic cosmological model


is based on the assumption that the coordinates are comoving with the
galaxies, the density of which is negligible in the dynamical equations.

Solution 19.31. The complete set of equations for isotropic, homogeneous


universes is (see Problems 19.13 or 19.14)

R 2
k 8rrp
( )
_

+ (la)
R 3
R2
CHAPTER 19: SOLUTIONS 549

R R 2



= _

( ) R
_

R2
_
877 P .
(lb)

Since the universe contains only dust stress-energy, and "vacuum


Po'

polarization" stress-energy A, we have

P =

Po + A/8rr (2a)

p =
-A/Srr .
(2b)

Since all time derivatives of R higher than the second may be expressed
as linear combinations of Rand R by Equations (1), the necessary and

sufficient condition for a static solution is

..

R=R=O. (3)

With the conditions of Equations (3) and (2), Equations (1) become


k/R =
(8rrP o+A)/3 (4a)


k/R = A (4b)

which imply

Po

A/4rr (Sa)

R = A-2 .
(5b)

Since Po > 0, Equation (4b) now implies k =


+1. To investigate

the stability of the "Einstein universe," first combine Equations (1) to

give ..

2R 8rr 8rr 2 A
= _
P 8rr p = _
P + (6)
3 o
_

R 3 3

and let

R=A-2+oR (7a)


+ a (7b)
Po

4rr Po
550 PROBLEM BOOK IN RELATIVITY AND GRAVITATION

Equation (6), to order a R, now becomes

2A t
817
(0 R) = _


aP
0 (8)

Now, since the total amount of dust is conserved, the dust stress-energy

obeys

R =
constant
Po


Po 8R
= _
3 (9)
Po R

Equation (8) then gives

(oR) -
AoR =
0 , (10)

Le. the perturbation oR grows exponentially in time. Hence, the universe

is unstable.

Solution 19.32. The line element of the "Einstein universe" (k=+l) IS

(see Problem 19.5)

2 2 2 2 2
ds =
R2[dX + sin x(d0 +sin 0d<f>2)] (1)
with

O X 0<0<17,
17, - -
o <f> 217 .

The volume V is therefore

17


3 2 3
v = 417 R sin2X dX =
217 R .
(2)

From Problem 19.31, we have the result, for this universe, that

R =

(417Po)-2 (3)

where Po is the matter (dust) energy density. Substitution of Equation

(3) into Equation (2) yields


CHAPTER 19: SOLUTIONS 551

C2
( )

V = 211
411
(4)
Gpo

(where we have put in the factors of c and G).

Solution 19.33. Let A be the Einstein value of the cosmological con-




A A 2"
stant so that the Einstein universe has
Po

E /417, and R =

E (see

Problem 9.31). All dust cosmologies, even with nonzero A, have pR =

constant. Write this as

1+ 1

pR =
(1)
417 A "2

This is equivalent to writing A =



E (1 + )2. The remaining dynamical

equation of Problem 19.14 becomes


R2 AR 1 + 2(1 H)

VCR) == _

(2)

3At R



For small R, R ,....,
l/R, Le. R ,....,
(3". Th expansion reaches a mini-

mum when R is a minimum. Setting dV /dR equal to zero, we find that

this occurs when 1 1

R=R m =(1+ )3A-"2. (3)

When R is close to
Rm'

R2 VCR m ) + !.(R-R )2V"{R


2 m m
) + ...

2 1 2

{1+ )3-1+[A2R-{1+ )3] .

From a table of integrals, we find

1 1
2 1
- -


R =

(1+ ?3 [1 + {1_(1+ )-'31 sinhA"2 (t-t )] m


(4)
A'i

where R at t t For small this becomes


Rm ,
= =

sinhA! (t-t )]

Rm [1+ m

552 PROBLEM BOOK IN RELATIVITY AND GRAVITATION

Thus R remains approximately equal to Rm until

fsinhA2(t-t)

Le. for a time

A-t
(t-tm ) log e).
Hence if f is made sufficiently small, the time can be made arbitrarily
long. Eventually R continues to expand, and asymptotically





=--')-A

2 3

and we get the de Sitter universe of Problem 19.30.


If quasar formation occurred at z =
2, then R =

Rm at z =
2 I.e.




=R

(1+z)=3R m =3A-2.

If this is so (with f? 1), then the density of matter now must be:

3 .1-1
Po

(417R OA2) =
(10817)-1A

R t (20A2 t
( )
1A ...1. 2
HO ( ) =
+ =
_

R 0 3 2 9
3A t
R 3
o R

-28 -1. -56 -2


10 A 4 x 10 1 2

T a k Ing
Ho I.e.
Po x


cm gives

cm, .

58 2 3
10- cm- =
1.6x 10-30gm/cm .

Solution 19.34. The effective Pc density corresponding to A is

-57 -2
A 10 cm 31 3
Pc
= _ =

817
4x 10- gm/cm .

817

Typical densities in the solar system are


CHAPTER 19: SOLUTIONS 553


Pbody
I"toJ
1 gm/ cm

33 15 3
Psol ar system
I"toJ
10 gm/(10 cm)3 '"

10-12gm/cm .

Thus, the effect of A in the solar system, in order of magnitude is

Pc 19
'"
10- .

Psolar system

Solution 19.35. The Friedmann equations (Problem 19.15) are

..


R = - -
17
G(p + 3p) R

RR + 2R + 2k =
417 G(p -

p) R

For a static solution, all time derivatives must vanish, so we get two

conditions, P + 3p =
0 and P =
3k/817 GR ,
which have no solution for a

fluid with positive energy density and pressure (i.e. any known fluid).

Solutions 19.36 and 19.37. "Easy" cases: Since the metric is time in-

variant and reversible, =


0 and it has the form
gOi

2 i 2 k i
dx j
gij (x )dx
ds
goo(x )dt +
= -

If it is homogeneous (Problem 19.37), clocks must tick at the same rate

everywhere, so
goo

constant; if it is inhomogeneous but pressureless
(Problem 19.36), it is possible to rescale the time coordinate locally to
make goo =
constant. (This was shown in Problem 16.25.) The standard

formula for 1 now shows by inspection that the only nonvanishing


1lva
components are r (Any 0 component is either a time derivative of a
ijk

derivative of Now be computed from


component gOj or a
goo.) Roo can

the standard formula involving the l's; since every term contains a zero

index on aI, it follows that


Roo = 0 and hence
554 PROBLEM BOOK IN RELATIVITY AND GRAVITA TION

1"
TO"° - -
TI"a =

and hence p + 3p =
O. But this is impossible for a pressureless fluid, or

for a physically reasonable perfect fluid (with positive p and p).


"Harder" cases: Let e be the time-symmetry Killing vector and u

be the 4-velocity of the fluid. Since u oc:


e (Problem 13.9) and IS

hypersurface-orthogonal, u is hypersurface-orthogonal and hence cu



(3

O.

In the Raychaudhuri equation (Problem 14.10)

dO ao
oc: =

dr at

where e =
a/at. Thus the Raychaudhuri equation becomes

2 2
j (Taf3 ffuf3T) f3

o =

aa;a 2a 0 811 u u (1)


- - - -

For p =
0 (Problem 19.36) we have a = 0 from the Euler equation

(Problem 14.3) and Equation (1) becomes

2 1 2
o = -
2a - -
0 -

41TP

which has no solution for positive p.

When there is pressure, then in general a is nonzero; in fact,


a = V log Ie. 12 .

If we assume homogeneity (Problem 19.37), then a =


0 and

2 1 2
o = -
2a -
0 -

41T(P + 3p)

which again has no solution for p + 3p > O.

Solution 19.38.

(a) A galaxy moves with xi =
constant implying that dx =
0 and
2 2
hence that ds = -
dr .
CHAPTER 19: SOLUTIONS 555

2 i j
(b) For dr =
0, ds =
g.. dx dx
1J

(c) If gOi and gij are independent of xi, then on constant r hyper-
surfaces, nothing depends upon xi; the space is homogeneous. On the
other hand, a bad choice of coordinates and hence a seemingly distorted
spacelike hypersurface may cause the metric functions of a homogeneous
space to depend upon position.

(d) In Problem 14.9 we showed that if the spatial connecting vector

between any two nearby geodesics is of constant length, then =


()= 0
aa(3
for the geodesic field. If gO1 '
and g..
lJ
are independent of r, then the

connecting vector will have constant length.


O i
(e) Since u =
+ 1, u =
0, we have

U =
-uyr =
-r = _! (g. 0 + g O 1 . ' .

g.. 0 )

..

iij Y1J 0 ij 1,j J, lJ,


(1)
1 1 .-

gij, 0 gij
= = .

2 2

1 1 3 3
f -2 (f

() == u



\g\-Z (\g\z u ) , a =

,0 )
,a

(2)

3 f

2 f

Now, from Equations (1), (2), and the formula (see Problem 5.18)

a = !. ( u pJL (.). + u (.). pJL a ) - !. 0p (.). (3)


(.).
a;fJ 2 a;JL fJ fJ;JL 3 afJ

we arrive at
・ .

1 1 1 1
fg f 0 (4)
2(uiij+Ujii)- 3 Ogij

a gi j

= = .

ij 2 ij 2

aa

uf3 ra 00
(f) u
ua;o
= = =

;f3


(!!.aD, 0 + !!.aD, 0 goo,a) goa,O

aa

= = -

aDO
556 PROBLEM BOOK IN RELATIVITY AND GRAVITA TION

If
aa

0, then gOa,o =
0 and galaxies fall on geodesics if and only if

O.
gOi,O
(g) From Problem 7.23 we showed that the necessary condition for a

family of hypers urfaces to be orthogonal to u is (U =


O. Since u =
a/aT
a{3
and the hypersurface!" of constant T are spanned by a/ax\ then w
a {3f 0

implies that (a/at). (a/ax ) f 0 and hence that gOi f O. If w a {3.o f 0,
then 1= 0 and from part (f), this means that the galaxies do not fall
(3,

on geodesics.

Solution 19.39. From Solution 14.9 we know that the spatial connecting

vector, between
, two nearby world lines obeys

Vu V +
u(e.a)u (1)

Putting e == Rn in Equation (1) gives

Rna u{3 + naR =


Rn{3 u

+ Ru

nYa (2)
;{3 ;{3 y


where R == dR/dt =
uaR
, a.
If we now dot n into Equation (2) and use

;{3 (see
the decomposition equation for u Problem 5.18)


()
u (U + a +
(ga{3 + u
u(3) aa u{3

a{3

a ;{3 a{3 3 a

we obtain

R n{3
[ ]


Rn w + u +
( {3 u(3) + u
aa u{3
= -

a {3 a {3 a

+ RuanYna a .
(3)


N ow the antisymmetry of (U and the fact that n is a spatial
a{3
unit vector (naua =
0) used in Equation (3) give us

R =
a n

n{3 + 1. ()
R a{3 3'
CHAPTER 19: SOLUTIONS 557

Averaging u

n{3 over directions produces proportional to the
a {3n a term

trace of Since is traceless, left with


aa(3. aa(3
we are

<:> =
e .

Solution 19.40. Since u we have


eo

r Ya


;{3


a ,{3

{3uy

0 + r
°a{3
so that
1 1

O=u .ag

(3 =r

ag

(3 =_(_g)-z[g
-Oa-
(_g)z] ,a (1)
a ,p afJ

(see part (f) of Problem 7.7). Hence for a Robertson-Walker cosmology

1 .1 ・

o =
(- g)-Z (- g),Zo =
3R/R .

The rotation is

CU
P(3YU[a;y] 0
= =

a (3

since r
°a{3 is symmetric. For the shear we have

a a
= P YU 1. 0P
{3 (3

afJ caiY ) 3 a

a =

Oa


a..
lJ

U '

C 1,J )


Ogoo
1J

1 R
(g o 1,J + g g.. 0 ) g..
= - .
- -
. ' .

2 O J,l lJ, R lJ


goo R/R g.. =

lJ 0
= -

2 , 1J

For the anisotropic metric, Equation (1) still holds, so

1 .1
o =
(_g)-Z (-g)Zo =
a + b + c

CU =

a (3
a =

Oa

1 1 1
a.. goo 0 g.. A
gij
= =

lJ 0

lJ 2 , 3 1.1 3
558 PROBLEM BOOK IN RELATIVITY AND GRAVITA TION

where

28. -
b -
6 for i=j=x

A =
2b -
a -
6 for i=j=y

26 -
a -
b for i=j=z.

Solution 19.41. The quickest way to compute the Ricci tensor for this

metric is to use the formulas of Problem 9.33. We have

1 ・

K. .
= -

gij (1)
lJ 2


0 =



K.. Kij -K (2)
lJ

t i
0 =

.J


K .

J,1


K .

,J
(3)

i i i
0 =






. K _
K .

.J
(4)

where a dot denotes d/dt. We have set (3)R ij to zero in Equation (4)
because the 3-geometry is flat, and we have replaced covariant deriva-

tives by partial derivatives in Equation (3).


Contracting Equation (4) gives R =


and hence

K = -
lit (5)

with a suitable choice of the constant of integration. Substituting Equa-

tion (5) back into Equation (4) gives

R i .J .
= _
Ki./t

and hence




Ai./t

(6)

where Ai. is a constant matrix. We now choose the coordinates so that


Kij is diagonal at some time t =

to. Then by Equation (6)


Kij remains

diagonal and can be written in the form


K .

_oi. P./t (7)
J J.J
CHAPTER 19: SOLUTIONS 559

where
Pj constant. For this equation to be consistent with Equation (5)

we must have

I Pi i

1 ・
(8)

Equation (3) is automatically satisfied, Equation (2) implies

Ii P = 1 (9)

and Equation (1) becomes


goo
1J

-2g.1m K


2g..
lJ
P./t

and implies that


2P.

goo = 0.. t
1J 1J

Thus the final form of the metric ("Kasner metric") is

2P 1 2P 2P
2 2 2 2 3 2
ds = _
dt + t dx + t dy2 + t dz (10)

subject to the constraints of Equations (8) a (9).



The volume of this universe is proportional to


1 "2
2lp.
[(3)goo]"2
lJ

(t 1) =


which can be contrasted with the t 2" behavior of the radiation-dominated

Friedmann model.

A convenient way of representing the constraints of Equations (8) and


(9) is to draw a circle of radius 2/3 centered at y =
1/3 and to inscribe

an equilateral triangle. The y-coordinates of the vertices of the triangle


will satisfy Equations (8) and (9). From this we see that, except for the
special case (1,0, 0) which can easily be shown to represent flat space,

two of the P's lie between 0 and 1 and one P lies between 0 and

-1/3. Thus space must contract along one axis while expanding along

the other two.


CHAPTER 20: SOLUTIONS

Solution 20.1. Note that there are no "real" (Le. tidal) gravitational

forces; the light goes in a straight line in any inertial frame, but in the

accelerated frame of the tube it seems to bend.

Consider two inertial frames -


one comoving with the tube at the in-

stant the light enters it -


the other comoving with the tube at the instant

the light reaches the end. Since the length of the tube is seen in both

frames as f and since () is always small, observers in either frame see

it take f/c for the light to cross the tube. At this time their relative

velocity is {3 =
gf/c .
(An observer in the second frame sees the first

frame moving upward with velocity (3.) The direction angles of the
photons path are related by (see Problem 1.8 and figure on p. 561):

() ()
cos tfr' + {3
tfr

cos =
sin = =
{3 .

1+ {3 cos tfr'
Theref ore
2 2 -16 -1
() =
gf/c , g /c

10 m,

15
so, if the tube is 10 meters long () =
10- radians, and the total dis-
2 14
placement (g/c )f2 10- meters. This is very small compared to

the wave length of light; it is difficult to imagine interferometric (or any

other) techniques capable of detecting this. When lasers are used for

alignment (as in the Stanford linear accelerator) they are meant to correct

for the curvature of the surface of the earth -


a much larger effect, ob-

viously, since a light ray at the earth's surface is not bent so strongly as

to be pulled into a circular orbit.

560
CHAPTER 20: SOLUTIONS 561

1 gl
/J =
;::!

Initial frame S' final frame S

Solution 20.2.
YI

r::
.,,=B+dB+;
Sun local inertial frame Sft local inertial frame 5ft+1

Consider a sequence of freely falling observers, each occupying a

local inertial frame of length de. Let the nth freely falling observer Sn
be momentarily at rest relative to the sun when the photon hits the left

side of his frame and let him see the photon move at an angle 0 relative
to its inertial direction (see figure). When the photon leaves his frame
and enters
Sn+1' which is now momentarily at rest, he is falling with
speed {3 =
gd f =



in the cf> direction. This photon enters frame

Sn+1 moving in the direction 0 + dO: the transformation of photon angles

(see Problem 1.8) is

cos i/i {3
e(f32)


cos t/J = =
cos I/J -

(3 s in 1/J +
1-{3 cos t/J
so that
2-

cos t/J -
cos t/J = -
{3 sin t/J
562 PROBLEM BOOK IN RELATIVITY AND GRAVITATION

and

dO =
(3 sintP (3 sincp

(1)

(R G:e2) (R


de ・


2 2"
+ e )

Thus the total deflection, a i.s

oo

J J
de 2GM
a =
dO =

GMRo 3

(2)
Ro
-00
(R +e2)2
which is one half the observed value!

In performing the calculation we assumed that a sequence of rulers

was placed along the line y =

Ro parallel to the line y 0, with the =

(n+ l)th ruler locally parallel to the nth ruler. It was the deflection

with respect to these rulers that we calculated. But we did not correct

for the fact that spacetime around the sun is curved (tidal forces !); in

fact, a locally parallel sequence of rulers forms a curved line with re-

spect to the y = 0 line and it is with respect to this curved line that we

actually calculated the photon deflection. Thus the real deflection, as

measured by a distant observer, must include both effects: the deflection

of the photon with respect to a set of locally parallel rulers, and the bend-

ing of the line (with respect to y =


0) of the sequence of rulers.

Solution 20.3. We may consider purely equatorial motion with no loss of

generality. In the gravitational field of the sun, the metric may then be

approximated by


ds
_(1_ 2)dt2+(1+ 2)(dr2+r2d 2). (1)

Null geodesics for this metric have the form

b 2M

sincp +

(1- cos cp) (2)
CHAPTER 20: SOLUTIONS 563

+=0

"'-+E + 8.

The angle a between the sun and the star (see figure), as measured

by an astronomer on earth, satisfies

oa
tan a =

tan(1T-4>E+ oa ) -tan4>E +

cos
c/>E

(l+ )r d4>/dA

uc/> dc/>

---



(1 )dr/dA

[ J

dr

(3)


where uc/> and u



are the orthonormal components of the photon's
4-velocity, and we have taken advantage of the isotropic form of the

metric in Equation (1) to define angles. If Equation (2) is now used,

Equation (3) becomes


564 PROBLEM BOOK IN RELATIVITY AND GRAVITATION

tan'PE
A... -
Oa =
sin4>E

2: (l-cos4>E)
2 2M.
CPE
cos A... A...

cos'PE b sln'PE

2M (1- cos CPE) (4)


tanCPE - -


cos2CPE
and hence

2M 2M 1+ 2

( )
c osa
oa (l+cosa) (5)

R 1 -

cos a

It is oa which is the deflection angle measured by an astronomer at

earth. [See also MTW, Section 40.3, and references given therein.]

Solution 20.4. Since we wish to compute only the lowest order deflection

due to J, we can look for terms linear in J and set M =


0 in the line
-- -

element. The total first order deflection will then be the sum of the usual

term linear in M and a term linear in J. Since the metric (for our pur-

poses) is (see Problem 17.1),

2 2 4J 2 2 2 2
ds = _
dt -

sin e dt dcp + dr + r d0 (1)


(J points along z-axis) the equations of motion may be deduced from

the variational principle (Problem 7.25)


J (_12 -


sin2ei +r2+r2(P+r2sin2e 2) dA =
0 (2)

where t == dt/dA etc., and A is affine parameter. The Euler equations

for Equation (2), to first order in J, are

4J
(2t

E-
dA


sin e
')
\0

(3a)

(2 r2sin2e-
4; ei)

sin =
0 , (3b)

which imply that


CHAPTER 20: SOLUTIONS 565

2J


Po 0 cp

t +
== sin
(3c)


2 .
2 2J .

0 0 P (3d)
_

'P r sin sin
cp
= -

r 0

are constants, and

'2 4J
・ ・

r= rO
+rsin20cp2+-sin20cpPo r

(3e)

d 2 2 2 3
(r 0 ) OJ r 0

(3f)
Pcp

cos sin
dA

From Equation (3f), one may deduce a third constant of the motion (in

addition to and
Po Pcp):




Pcp L
2 2

0 ). (4)
o 2.
- =
constant
(PO == r

sin 0

We will now decompose the bending into three contributions, each de-

fined by the relationship (see figure) of the ray trajectory (a straight line
to lowest order) to

J. (Since we are working only to linear order, the
total bending must be the sum oJ such contributions.)

In both cases (a) and (b), the angular momentum of the light beam
about the sun's center has zero projection onto J, in case (c) it has unit

projection.

J J

(0) ( b) (c)
566 PROBLEM BOOK IN RELATIVITY AND GRAVITATION

First consider (a) and (b): In both (a) and (b) 0


Pcp
we cases cases =

(since
Pcp is a constant of the motion). We wish to calculate reO). From

Equation (3d) we have


2JP o
cp =



(9(J) (Sa)

and from Equation (4)



0 L (Sb)
Po r constant.
= = =

Equations (3e) and (Sa), to (90), now give us


..


r =
r O , (Sc)

but this result and Equation (Sb) guarantee straight-line motion! Thus in

cases (a) and (b) J makes no contribution to the bending of light.

Now we consider case (c): In case (c), we may set 0 =


'TT/2, and
. ..

o =
0 (Equation (3f) guarantees that 0 will always then be 0) and solve

our equations accordingly. Using

Pcp 0\ d

d dcp d 2J P

_ _

- -

ax -;2 ) dcf>

dA dcf> 3

and defining u == l/r, e ==


Po/Pcp, we may put Equation (3e) in the form


u" + u = -
6u J e -

2J e u,2 -

4J e uu" (6)

where u' ==
du/dcp. We will solve Equation (6) by perturbation techniques;
we define
u = U


ul (7)

with ul =
(90). To lowest order, the equation is U
o + U


0 which

implies




cos cp (8)

where b is the impact parameter and where we have chosen our coordinate

system such that the ray moves parallel to the y-axis. For the second
order solution, u

must satisfy the equation
CHAPTER 20: SOLUTIONS 567

" 2J e 2 2 2J e
(sin cjJ cjJ)

u + u = - -
+ cos = - -
(9a)
1 1
2 2
b b

or

2J e


= - ----


(9b)

Thus, from Equations (7), (8), (9b), the solution for u IS

1 2J e


cos cjJ -



(10)

The bending at each asymptote, a, is found by setting u =


0 (r =
00). At
one asymptote cjJ TT /2 + a so that cos cjJ -
a and

2J e
a (11)
-T'

There is an equal contribution at the other asymptote, so the total bending


IS
4J
ocjJ =
2a =


(12)


have used the fact that e b- ).
(where we
Po/Pcp

From this result, and the results from cases (a) and (b), we may write

the deflecti on of light by J as

4J. n

ocjJ (13)


where n is a unit vector in the direction of the angular momentum of the

ray about the Sun's center.

Note that the dimensionless small parameter in which we have been

expanding is

J 1./ 1048 g m-cm 2 /sec 7

(_)
x 12
_

_ -

10- 2 x 10- arc sec .

2 10 2

sun (7x 10 cm)
568 PROBLEM BOOK IN RELATIVITY AND GRAVITATION



)"2

Solution 20.5. The index of refraction of a plasma is n =
(1- v p Iv
2 2
where v P frequency p
is the lasma V == n e 117m The well-known
pee

equation describing the path of a ray through an inhomogeneous medium is

[see, e.g. B. Rossi, Optics (Addison-Wesley, 1957), p. 54]

d(nm)/de


Vn
- (1)

where is a unit vector tangent to the ray and e is the distance along
the ray. Since the deflection angle we are computing is small, we can

correctly find the first-order change in ]E by integrating Equation (1)


along the unperturbed trajectory y

b, -
00 < x < + 00. From the formula

given for ne we have


v -6 -2


( ) Q ( 10 )
n 1 -
R: 1- .0101
2 2
v HZ
or

-7.5 -2
0657

(R ) (1O HZ
"-


) (2)

R: r.

RQ


Thus, using n = 1 at :!: 00 and r =
(x + y2)"2 ,

oo
-2 -7'S
.0657
[X ( ) ( ) ] dx.(3)

( ) J

( )
L x
ml + 00 -m\
-- -- -
00
R 9 R r
+ Y r
Q 10 Hz Q
-00

The term in x vanishes by symmetry in :!: x, while the y term can be


00

written as 2
I_oo. Since !E is a unit vector, for small angles

(}El oo

!EI-o) y .
is the scattering angle we seek. Defining

11 == b/R Q , z ==
rib

and using

y =
b, x

= b 2 (z2_1), dx =
bz(z2_1)-t dz ,
CHAPTER 20: SOLUTIONS 569

we get

f OOZ-7.S
-2
-6.S -t dz
( Z2_1 ) (4)

( )
() .131 TJ .

coronal 9
10 Hz 1

On putting t =
1/ z2, we recognize the integr il in Equation (4) as a beta

function, B( 3. 7 5, .
5) =
.158. Thus our final expression for the coronal

deflection is


-6 5

( )

() .021 (5)

coronal TJ .


10 Hz

The general relativistic deflection, ()GR' is


8.5x 10-
()GR (6)
TJ

(see e. g. Problem 15.6). The two deflections are equal when

8.5 x 10- 6 -2
TJ-6.S
( )

.021
TJ 9
10 Hz
or

-.36

( )

TJ == 4 1 ・

RQ 10 9 Hz

For smaller impact parameters b, the coronal deflection dominates.


570 PROBLEM BOOK IN RELATIVITY AND GRAVITATION

Solution 20.6.

8"
-,-'
b "

band h are measured

in units of 4mm.

The relation of cp and () (see figure) is given by Snell's law

n =
cp /() .
(1)

The refraction required to simulate the sun's gravitational bending is

a ==
cp -
() =
1. 75"/b .
(2)

From Equations (1) and (2) () must be


() =
1. 75"/b(n -1) =
1.6 x 10- /b .
(3)

From the geometry of the figure () = -


dh/db, so Equation (3) gives


dh =
1.6 x 10-
db b


h 1.6 10-

ho x log (b)


CHAPTER 20: SOLUTIONS 571

Solution 20.7. In the Schwarzschild metric, in curvature


coordinates, the
equation for noncircular orbits may be put into the form (see e.g. solution
to Problem 15. 9)
" M 2 2
u +U=-+ - 3M u ==a+ bu (1)


where u == l/r, u" == d u/d<;b2, and L ==
angular momentum per unit mass

of the orbiting particle (= constant). The solution to the lowest order,


the linear portion of Equation (1), is




a + A cos <;b (2)

where A is a constant. The second order solution to Equation (1),


u1'
then satisfies the equation


bu
2 2
ui + u


b(a + 2aA cos <;b + A cos <;b)

2 2 2

ba + bA + 2abA cos cp + bA cos 2cp .
(3)

The inhomogeneous solution to Equation (3) is

b(a )+ abAcp sincp


2 2 2
Ul

+ A -

(bA j6) cos2cp (4)

and

U = U

+ u


(5)

The second term in


u1
causes the nonperiodicity; we may put it into
more useful form by use of the identity

f <;b sin cP =
cos (cp -

cp) -

c os <;b + <9( 2)
f .
(6)

Since ab is a small quantity according to Equation (1), Equation (6) may

be used to put Equation (4) into the form

2 2
Ul
b(a2+ A
)+ A cos(cp-abcp)-A coscp-(bA j6) cos2cp (7)
572 PROBLEM BOOK IN RELATIVITY AND GRAVITATION

and the perihelion shift, per revolution, is


ocp =
217 ab =
617 M /L 2 .
(8)

We may express L in terms of the semimajor axis a and eccentricity e

of the orbit using standard classical mechanics

2 2
L =
Ma(l -

e ) .
(9)

Equation (8) then becomes

617M
ocp =
, (10)

a(l -

e )

which in the limit of small e agrees with the solution of Problem 15.7.

Solution 20.8.

(a) Yes. First note that particle mass is constant:


dm dp'p dp 0
dr

-(iT
= _

2p .

dr

Since p

mu and m is constant, a factor of m can be taken out of

each term in the force equation leaving


d u/l / dr = -
(l1/l (I)
,v
+ ua(l)
,a
u/l)

and showing that particle motion is independent of mass.

(b) Yes. Near the earth the earth is flat. Take z to be in the verti-

cal direction, then we have (I) =


(I)(z) and the "source equation" for (I)
2 2
is a (1) /az =
O. Above the earth's surface then, the potential is

(I) =
az + b .

The constant a is determined by dropping a massive particle from rest


and setting its initial acceleration to g. With uO =
1, u = 0 in the force

equation, we have
CHAPTER 20: SOLUTIONS 573

mg =
dpz /dr = -
m(l)
,z

-ma

and hence a =
g.

Now consider a photon moving vertically upward. It loses energy


dpO = _
pU(I) ,U dx = _
pZ(I) ,z dt

in time dt =
dz. Since pZ =
pO for a photon, the energy loss is then

o °
dp = -
p g dz .

For an earth-based experiment g z? 1 and

pO
/po =
v/v = _
g z

in agreement with the Pound-Rebka experiment.

(c) No. For a photon PfJ dxfJ = 0 so the path of a photon is governed

by
d plL = -
pU(I) ,

dxlL = -
dxu(I)

U plL .

Since dp is proportional to p, the photon must move in a straight line;


there is no bending.
[Note: Also in disagreement with observations, this theory predicts a

regression of the perihelion of a planet's motion, of magnitude

a =
217
M /J2
where a =
shift per orbit, J =
angular momentum of planet per unit mass.

For nearly circular orbits, this becomes

a =

27TME)/r

which is 13"''/century for Mercury's orbitJ

Solution 20.9. Any standardized clock can be calibrated against a clock

at infinity (which meas ures time t). Then the ratio of the ticking rate of
574 PROBLEM BOOK IN RELATIVITY AND GRAVITATION

a clock at (r,O), 4-velocity u, to the rate of a clock at infinity is

dr 1
(1)
_

dt
uO(r,O)

In Solution 16.19, we showed that uO =


constant on the surface of a

rigidly rotating, equilibrium configuration of a perfect fluid. Thus

dr/dt =
constant

and all clocks on the surface of the earth have the same ticking rate!
(The doppler shift effect exactly cancels the redshift effect.)
CHAPTER 21: SOLUTIONS

Solution 21.1.

(i) One way of finding the formula for the derivative of a detenninant

is from the identity

det A =

Tr { 10 g A I

valid for any nonsingular matrix A. (Here log A is the inverse



function of e , where he exponential is defined as a power

series.) Thus we have

o(det A) =

Tr {log A J o(Tr {log AI)


(detA)Tr(A- 0A)

and for det


g ==
('6a{3)
8g =
g gaf3 8f!u.f3

where we have used the symmetry of The desired result


'6a{3'
follows immediately:

.1 1 _.1
0(_g)2 =
-_(_g)

20g


(- g) t gaf3 8f!u.f3 .

(ii) From the definition of ga{3,

ga{3 oa
g{3y

Y'

we have

8gaf3 gf3y + gaf3 8gf3y = 0

575
576 PROBLEM BOOK IN RELATIVITY AND GRAVITATION

Multiplication by gYo gives

8g

{38,l + gy8 ga{3 8 {3 0


and hence the answer:

8 ga8 = _
gy8 ga{38g{3y .

Solution 21.2. The variation in L due to changes in (l)A is given by

(- g) taL ((I)
( )
A A aL ocI>A aL

(I)
,11
, g v)
Il

+ O(l)A (_ g) t .
(1)
a<f)A a(l)A ,11
,11

The operation of varying cI>A commutes with partial differentiation

a ((I) A /I) =
(o(l)A)
, t'" , 11

so Equation (1) becomes

JL(-g)t 8e1>A aLC-g)t


(-g) taL (_g)t
( ) ( )

aL o(l)A + _
L 0(1)

a(l)A a<f)A axil a<J>A


,11,11 ,11

The second term is a perfect divergence, which gives a surface term in

the integral aS, and hence is zero since o(l)A is taken to vanish on the

surface. Thus
(?L(-g)t

0 =
8L =
JL _
L 1

\ acI>

8e1> acI>A axil ( g) t
_

Il



Solution 21.3. Since L (- g)2 d x is a scalar, it is unchanged under the
infinitesimal coordinate transformation

xll =
xll + 11


By renaming the dummy variable of integration, d x can be changed back


to d x. The net change in L(- g)2 is therefore the Lie derivative (see
Problem 8.17), so that
CHAPTER 21: SOLUTIONS 577

f .£1; (L(-g)!)d

0= oS =

(1)
o(L(-g)!) .£
J[ ]
OL

(.£t; elJA)(_g)! 0

t; g/lV
d4x .

oeIJA g/lV

The first term in the integrand vanishes by the equation of motion (Problem

21.2), thus

f T/lV'£1; g/lV (_g)! d



o = x ・

Since

f gl1 v

fl;v V;11

we get

f T/lV /l;V(_g)! d

o = x

(2)

f (TI1Vfl )(_g)t d
f (_g)t d 4

= x -

Tfl x .

;v ,v 11

The first term is

J )(_g)t d
J ((-g)t TflVl1,v)d 4

(TflV x = x
l1;v

and gives a surface integral which vanishes. Since is arbitrary,


11

Equation (2) then gives


TI1V = 0 .

;v

Solution 21.4.

(a) Under the independent variations

+ a
gflV gl1v gl1 V





+ ora
flv I1 V I1v

we have

1 1 .L.L
(1)
O?_g)2 R) =
(_g)2 g/lVORp.v + Ro(-g)2 + (_g)2 R/lVogllV .
578 PROBLEM BOOK IN RELATIVITY AND GRAVITATION

Since

R/l r v,a
-r a,vrC:Za r


-r ar a (2)


the first term on the right-hand side of Equation (1) becomes

1 1

(_g)2 g/lVoR =
((_g)"2 g/lVofYl ) _

((_g)"2 g/lVora )
/lV /lV ,a p.a ,v
1 1
_

((_g)"2 g/lV) ,a a['a/lV + ((_g)"2 gIlV) ,vora (3)


Jla



+(_g)2" g/lV(oraaa r +:ra ora _ora ra _r a ora)
/lv aa
/lV va
/la va p.a.

The first two terms on the right-hand side of Equation (3) are divergences,
which therefore do not contribute to aS. (The integral of a divergence
can be converted to a surface integral and or = 0 on the surface of the

region of integration.) By appropriate relabeling of dummy indices, Equa-

tion (3) can be written

1 1

(_g)"2 g/lVoRI1 V =
(_g)2" (Al1v

+ oVBI1)or
a /lv
(4)

where
1 1

A/lV ==

gf.1J/fYlaa -

(-g)- 2" [(_g) 2" gl1 ] ,a _

gavr/laa _

gal1rVaa (5)

BIL ==
(- g)-t [(_g)t glLt\p +
gaPrlLaP .
(6)

By Problem 21.1 we have

.1
1 "V
U [ (_g)2 ] = - -
(_g)2 g U


(7)
2 I1 V

so

J (_g)t

0= aS =
(1617)-1 d x(A/la

+ oVBI1)or

/lv

(1617)-1
J (_g)! d 4 x(R 1..
)OgI1

+ g VR
V I1V

2 I1

o(Lmatter(-g)2")

I1v d 4 x
+ ug .
(8)

a gl1
CHAPTER 21: SOLUTIONS 579

Since Ola is symmetric, the fIrst term in Equation (8) gives


lLv

AILV 1. 0/'/ Bil L 0 11 B




+ + (9)

-.

2 a
2 a

Contracting on a and v in Equation (9) yields

A +
a2BIL + BIL =
0 .
(10)

But contracting Equation (5) and comparing with Equation (6) shows

A =

a _
BIl .
(11)

Equations (10) and (11) imply BIl =


0, so by Equation (9)

A =

V0 .
(12)

If the explicit expression in Equation (5) is used, Equation (12) can be

wr i tten
av V
gfV C a =
gllV,a + g III + gall i (13)
aa aa

where
_L .1
C ==
(- g ) 2
(- g ) ,a
2 -
Ia .
(14)
a aa

When we multiply Equation (13) by gllv we find that

V a
4C


glL g + 2i
aa
,a flV

= -
2(- g)-t (_ g)t ,a
+ 2i

aa


-2C

(where Problem 21.1 has been used in the second line) and therefore that
Ca =
O. If we define
i =
g ill (15)
Aaa Il aa

and if we now multiply Equation (13) by gAll ryv' we get


580 PROBLEM BOOK IN RELATIVITY AND GRAVITATION

o r

rAya + (16)

ryA,a

yAa
and thus

(ryA,a+ rya,A -gaA,y) =
r (17)
2 yAa

which is one of the required results.

We now return to Equation (8), and use Problem 21.3:

1 1

(Lmatter(-g)2) 0 o(Lmatte/- g )2) 0 1L1/ !. ( ! TI1vo
g g)

gl1v
_
_

gl1 v


og/lv og/lV
(18)
1 .1 s:- "V


(-g) T V u gr

2 I1

We conclude that

(1617)-1 (RW -

glL1/ R) -


1L1/

or


I1V

811 T
I1 V

(19)

(b) Although r (now taken symbols formed



to be the Christoffel
I1 V

from is not tensor, or is (Problem 8.26). Thus
ga(3) a
l1v
a tensor we

can simplify the calculation of OR I1V by working in a local inertial frame,

where the r's vanish. In this frame, Equation (2) gives

oR = ora _ora =
ora _ora. (20)
/lV /lv,a /la,v I1 v ;a p.a;v

Since this is a tensor equation, it is valid in any coordinate system. The


r's are Christoffel symbols, so 0 and
g/lv;a

1 1
V v
(_g)2 gl1 (_g)2 [(gl1
oR
ora/lv);a (orapag/lv);v]
= -

l1v
1 1

((_g) 2" gl1V ora )
_

(oraIla g11v (_g)2") ,v


I1v,a

(where we have used Problem 7.7 (g) in the second line). Since this is a

perfect divergence, it does not contribute to aS. The remaining terms in

Equations (1) and (18) lead to Equation (19) as in part (a).


CHAPTER 21: SOLUTIONS 581

Solution 21.5. Taking the variation of the action

(&r )-1
f (lI>;a 11>2)(-g) t d
2 4
s = -
lI>;a + m x

gives us

oL
[2m (1) _2(_g)-t[(_g)t(l),a] ,a] (_g)t

o = =

0(1)

or


0(1) -
m (1) =
0 .
(1)

From Problem 21.3 the stress-energy is

o [(-g)2L]

MV
==
-2(-g)-t
ogMV
...L (_g)- t rII> II> (_g) t 1.(11) II> ,a + m 11>2)(-g) t g


= _

411' ,M L
,v 2,a MV

4 [11>'11lI>,v II> ,a + 11>2


)]

gl1v (lI>,a (2)


= -

and hence its divergence is

TM = 1. ((I)' (I)'v +(I),M 0(1) -<1> ;M(I),a -




(1) (I),M) .
(3)
,v 417 ,v ,a

For a scalar field (I), we also have the relation

(I) ,M =
(I) ;11 (4)
;v ,v'

thus Equation (3) becomes

(I),M 2
T MV = -
(o(l)-m (I)) (5)
;v 411'

which, by Equation (1), vanishes.

Solution 21.6. Varying the action integral gives us


582 PROBLEM BOOK IN RELATIVITY AND
GRAVITATION

(A[/l;v] A4t;v] [_g]t) 8(A[/l,v] A[/l,v] [_g]t)




o(L[-g]2") - -
1 _


477

a aA 477
All aA
11

= -
-1.2.2.
477
axil
(A[/l,V][_g]t)=_l-L(FV/l[_g]t) 277
axil

[ g] 2"

F VI1 ' (1)
277 ;11

where we have used Problem 7.7 (i).

The prescription for the stress-energy gives

oL 1
o(F I1v F arg/lagvr)
2 1 v

TafJ +
L fJ F
/lvF/l
== -

fJ

8gafJ
811
8g fJ
a 1611
(2)
1 v 1
F F Fllv
Fa
_

{3v {3'
- -

477 1677 I1V

Consider now the Lagrangian density

-1.. 1-

( F F I1V F flll A
)( g) t (3)


- -
_

I1 V

1677 477 ;v

Variation of L with respect to yields


F/l V ,
= 0 .
(4)


Variation of L with respect to F/l yields

F/l V =
2A [v ;11] .
(5)

Solution 21.7. Variation of the Lagrangian with respect to (I) yields

(I),a(_g)t
and thus
(_g)t (R


(l)2,a
(I) <I> ,a
) =
-2w
ax

( (I) )
-1 2 a
2w (I) (I)

0 (I) -
w <I>


<I>' + R = 0 .
(1)
CHAPTER 21: SOLUTIONS 583

(We have used here

((I) ,a(_g) { ) ,a


o <I> == (I) ;a = .

,a 1-
(_g) 2

Variation of L with respect to involves three terms:


'6a{3
Q(I)-1 (-g) t)
o( --<u(I) ,atl>,(3 gu fJ
ga{3(_g)2J
1 1

(i)

(3
= _
(I) rct>,a(l),{3(_g)2 _!(I),1lq>
2,/1

(2a)
and
1 1 Q 1 Q

(ii) 0((I)R(-g)2) (_g)2 (I) GafJ 0 {3(I)(_g)2 gafJ oRa{3


+ (2b)

(_g)t ga{3 oRafJ Q =


[(_g)t (ga{3 orYafJ Q
_

gay arTaT )] (2c)


axY

(see Problem 21.4). Also, we have from the expression for the r's as

derivatives of the g's

or V-lpogpuru/lV
= + gAp [(0 gP/) ,v + (ogpv),/l (og/lv),p] -

(2d)

lp[(ogp/l);v


(ogpv);/l (og/lv);p]

Using Equations (2c) and (2d), we can then write

<1>(-g) t ga{3oR oga{3(_ <I>,a;(3 + 0 <I>)(-g) t (2e)


ffu.{3
->

{3

where denotes the operation of dropping all exact divergences (which

vanish when integrated over). Combining Equations (2b) and (2e) we have

1 1 Q

0((I)R(-g)2) =

(_g)20gafJ[-(I),a;{3+ {3 (I) o + (I) G


a {3] ・
(2f)

The remaining term needed is

1677
0(Lmatter<-g)2) 877 (_g)t -rz{3 (2g)
(iii) = _

0 {3
584 PROBLEM BOOK IN RELATIVITY AND GRAVITATION

From Equations (2a) and (2f), and (2g), we get the field equations ob-
tained by varying ga(3 :



{3
+ 11>-1
( 0 II>
{3 -1I>,a ;(3)

(iJ 11>-2
(lI>,a1I>,{3 -

(3I1>'/lIl>'/l) (3)
81711>-1 0
Ta{3
- =

Solution 21.8. For simplicity choose coordinates (Gaussian normal



coordinates; see Problem 8.25) in which the metric takes the form ds

2 i
dx dx j xi(i=
dn +
(3)gij The coordinates .
1,2,3) span the surface layer
which is taken to be at n =
O. From the initial value equations we have





(3)G iJ ,


(K J _oi.J,n

TrK) -

(TrK)K J

, + 1. oi.(TrK)2
J 2 J

1 i


ai, Tr(K 2 ) =
877 T


We now integrate over n from -


f to + f, using the fact that n repre-

sents proper distance perpendicular to the 3-surface, and we let f -+ O.

Since the intrinsic geometry of the 3-surface is well defined, the only

term that survives this limiting integration is the term involving the

derivatives of the extrinsic curvature, so for [K] the discontinuity in K,

we find

[K

.] -
ai,

Tr [K] =
877 si.J

or

i'
477 (3) g S J

Tr [K] = -
477 S,1 = -

lJ
..

and




i 1 i k
[K j] =
877 S




S k ・

Solution 21.9. From the initial value equations, we have

-{K,m-o,mTr(K)J I

G _{K,m -(TrK).J = =
877 Tn,1 .


, =

11m ,1 11m

The discontinuity of this equation across the 3-surface is


CHAPTER 21: SOLUTIONS 585

] _o.m Tr [K]l

{[K.11m = -
877 [Tn.]

By combining this with our result from Problem 21.8, we have then

S.m = _
[Tn.]


1 m 1

Solution 21.10. The surface stress energy of the thin shell of dust is

sa{3 =
au

u{3. From the solution to Problem 21.8, we have

{ }
l'
. .


[Kj] =
877 au u

2 oj (- a)
or

{ }

[KOo]
lJ

877a u. u. +
1 J 2
(3)g..IJ .
(1)

Since there is no stress-energy outside the shell [T i ]




0 and the equa-

tion of motion (see Problem 21.9) gives us

m m m m
O = S1 =a u u.+au u .+au' u
Im Im l

,m 1 11m

da
= U +
auiu 1m


aUilmu

(2)
dT i

By contracting this with u we verify the second relation

da m

dT
+ au
1m


(3)

Comparison with Equation (2) shows that uilmu



= O. Thus

a = V

u =
uiV.(uje.)= ui(uj , .e.+ujK..n)= uiujK..n.
1 J 1 J lJ lJ
(4)

(We have used Equation (2) of Solution 9.32 to get the second-last

equality.) The third relation now follows from Equations (1) and (4):

a+ -
a- =
uiu j [Koo]
IJ
n =
477an .
586 PROBLEM BOOK IN RELATIVITY AND GRAVITA TION

To verify the fourth relation we need to show

+ -

i j
(K ..+ K ..

)u u =
0 .

13 lJ

by using the initial value equation [G n ]



We do this =
817 [T

] =
O. Since

(3)R is continuous, this gives us


0= [Tr(K )-(TrK)2]

K K+

+ K3

K-


-(K1 +)2 (K _)2



(KJ ++K3 {K1
-) -

+K3 -
-(3)gi j (K a a+

Ka-)J


(K33
++K {[KJ ]o1 Tr [K] J
-) 1

where we have used the easily verified fact that K3 and K


+ -
commute.

Since

[K\] =

81TU(U jU +
i (3)81) .

it follows that

[K 1.] =
417U

and that

[Kj.]


oj.1 [K i1.] =
817U (uju.) 1

and therefore finally

(K3 ++KJ uju.1


-) =
0 .

Solution 21.11. If T is proper time measured by an observer at rest in

the dust, then the motion of the shell is specified by r(T), where r is

the radial coordinate of the shell in either the interior or exterior metric.

The metric of the shell can be specified by

2 2
ds 2 = -
dT + R

(TXd0

+ sin 0 dcp2) .


Here 417R =
the surface area of the shell at T, and thus R(T) =
r(T).
CHAPTER 21: SOLUTIONS 587

From the second relation in Problem 21.10 we have that

0=
da
dr
+ aui



(aui) I 1


(a((3)g)t u

) , 1./((3)g)t .

4 2
(3)g

But u = 1 and =
R (r) so that (aR ) =
O. This implies that



477 R a =
/1, which we identify with the rest mass of the shell, is a

constant.

To find the equation of motion we use the junction condition from

Problem 21.10

[ (}(}] =

81TU(U(}U(} +
(3)g(}(}) =

41TU(3) (} /l =

Evaluating KOO we have

a 1 r r

KOO na r 00 r n


nO;O
= -

gOO,r
= = - =

and
r+ r
[KOO] = -
r(n -
n -) =
/l

where nr+ and nr- are the radial components of the normal evaluated

in the exterior and the interior geometry.

By using o.n = 0 and n.n =


-0'0 =
1, we can evaluate the compo-

nents of 0 and n. Exterior to the shell we have

1 =

(1- 2) (u )2

_

(u

)2/ (1- 2)

ntut


nru


1 = -

(1_ 2 t
(ni +
(1_ 2) (n )2 r


and by eliminating u
,nt we find

1-

1+(ur)2/ (1 2)

_

n+ =

(1_ 2 )

588 PROBLEM BOOK IN RELATIVITY AND GRAVITATION


On the shell r =
R(T) and u =
dR/dT == R so the contravariant component

of n, exterior to the shell is


2:

(1- R
r+
n = + .

The calculation for nr- is identical, and we get the same answer as

above with M set to zero. Thus we have

1 1

{[ }
"2

"2

2M

r+ r- 2 2
/l
= -
R(n -
n ) = -
R 1 -

+ R -

[1+ R ]

which can be solved for M to give the equation of motion

. 1

2 2 2
M =
1L(1 + R ) -

IL /2R .

If R = 0 at R = 00
(i.e. infall from rest at 00), then M =
IL and

R = -

{R (1 4:Y + .

This can easily be integrated to give

3 1

: T = _

(1 +
)2:R (1 + +
YR

where we have taken T =


0 to correspond to R =
O.

Solution 21.12. To find an instantaneous metric, we need only to con-

sider the metric and field equations on a spacelike Cauchy surface S,


the surface of time symmetry. Six of the ten Einstein field equations

determine the metric off S; these we do not need to worry about. The

remaining four equations are initial-value equations and can be written

in the form (see Equations (6) and (7) of Problem 9.33)

!. (3)R _!. ( K i )2 + 1.. K




e =
0 (la)

2 2 1
2 m

Kiml Kmml

0 (lb)

m i ,
CHAPTER 21: SOLUTIONS 589


where (3)R is the curvature scalar on S, K

is the extrinsic curvature,

a slash represents covariant differentiation with respect to the three

geometry (3)gij on S, and where we have used the vacuum initial value

equations.

At a moment of time symmetry the (timelike) normal to S, from which

K is computed, goes into minus itself under time reversal. Thus K =

ij ij

K and hence the extrinsic curvature of S must vanish. For K =

ij ij
on S, Equation (lb) is trivially satisfied and Equation (la) takes the form

(3)R = 0 .
(2)

As a solution to Equation (2), we begin with the ansatz

g.. = (1)411.. (3)


lJ "1)

and see if there is a (I) which satisfies Equation (2):

(3) R ij m m m e f m
_

g (r ..

r . . + r o. r .

r 0 r . .
) .
(4)
1m,) 1) , m Ll) m mL 1)

With the ansatz of Equation (3) we have, for the Christoffel symbols



o .

Ll

2 -1
((I) . am. +(1) oom.
1 1

,m1Jo.)
Ll
, (Sa)
, 1 , L

gij r

. . = _

6(1)-6(V(I))2 + 6 V 2(1)
-5 (5b)
1m,)

gij r

..

lJ,m

2(1)-6 (V(I))2 _
2(1)-5 V 2(1) (Sc)

r e.
ij
g rm
el

)m
= _
4(1)-6 (V(I) )2 (5d)


ij


me

lJ
..
= _
12(1)-6 (V ) 2 (5e)

where we have used the symbols


590 PROBLEM BOOK IN RELATIVITY AND GRAVITATION

ij
(V (I) )2 == (I) (I)
,1

,J l1

(Sf)

2 ij
V (I) (1)..11 (5g)
_

= .

,1,J

Combining Equations (4) (5), -


we see that Equation (2) reduces to

(3)R = 0 =
8(1)-5 V 2(1) , (6)

which is satisfied by any solution of Laplace's equation. One such solu-

tion is
M.
(I) =
1+1-2. (7)
i 2r,

leading to a metric 4
Mi
g.,
IJ

( 1 + 1
i 2 r.1
) 1/..
1J
(8)

The lowest order term in M/r for the metric in Equation (8) is

2M.1
glJ
' ' 1 + l



) 11..
1J

which is the "Post-Newtonian" approximation to the spatial part of a

metric generated by point masses M located at positions r . Thus the


i "'"

metric in Equation (8) is, in fact, the instantaneous spatial metric for

point masses at arbitrary positions at a moment of time symmetry.

Solution 21.13. Begin by noticing that the square of if, which is a



AA
Lorentz invariant, is just the determinant of U . The analog of the

Minkowski metric will then be a set of matrices which serve to raise and
AA
lower indices in constructing the determinant of U ':

AB FC EG
oC

det (U )


U U c
FE

DG

where

CFE=C ) (

aD =

Evidently, a pair of c matrices plays the role of the Minkowski metric.


CHAPTER 21: SOLUTIONS 591

To find the analog of a Lorentz transformation, denote such a trans-

formation by
F'C' F'C' Fe
U =
L U
FC

Then, invariance of the dot product of two vectors U, V, requires

AB CD E'F' EF G'H' GH
U V L U L V
cACc BD cE'G,cF'H"

EF GH

Thus we must have

E'F' G'A'
L L
cE'G,cF'H"

CEG cFH EF GH

This implies that

GM FK
aMOK L
E'F'

G' A"

EE'G'CF'H' C

E H EF GH

and hence that

E'F'
det (L 1
EF )

Thus the Lorentz transformation matrices must be unimodular (of unit

determinant).

Solution 21.14.

(a) The result follows from the fact that the indices B, C and D

range over only two values and hence cannot all be different.

(b) Writing out the result of part (a) and using the antisymmetry of

cAB gives C c 0
CAB cCD +
cAe +
cAD

DB BC

CD
Contracting now with gives (watch out for the signs!)

-CAB + - BAO.
AB =

Hence we have

AB

(AB) [AB] (AB)


+ =

cAB ・


592 PROBLEM BOOK IN RELATIVITY AND GRAVITA TION

cd. C D C' D' C D C' D'
(0 A 0B 0B' 0 A'

0B 0 A 0 A' 0B')

SolutIon 21.15. Since C


ab
1 = -

(see e. g.

F. A. E. Pirani, Lectures on General Relativity, p. 315) and *T =

ab

E T
cd'
the result follows immediately.

Solution 21.16. Since primed and unprimed indices commute, we have



Cab )

2 (TABA'B' +
TBAB'A')

(TABA'B,-TBAA'B'+ TBAB'A,-TABB'A')
1 1



Cab )

4 (TBAA'B' +
TABB'A' ) .

With the result of Problem 21.14, this can be written as



Cab )

2T[AB][A'B'] +
2 (TBAA'B' +
TABB'A')
1 c c' 1

2 cABcA'B,T c c'


(TBAA'B'+ T ABB'A' ) ・

Finally, using the result of Problem 21.15, we get

1 c C'
TBAA'B'


Cab )

2 cABcA'B,T c c'
+ *T
ab

T(ab)

gab T + i *T
ab

INDEX

References are to problem numbers, except that integers (1.0, 2.0, etc.) refer to
chapter introductions (Chapter 1, Chapter 2, etc.).

aberration of light gravity, 12.4


or Birkhoff's theorem, 16.3
accelerated observer, 1.17 bivector, 3.29
accelerated system, thermal equili- black holes, 15.10, 15.11, 17.0 ff.
brium, 5.29 black-body radiation, 5.13; redshift
acceleration, 1.0, 1.14, 1.15, 1.16, of flux, 19.3
2.13, 10.14, 14.12 Bohr radius of gravitational atom,
accretion of perfect gas, 15.18 12.19
adiabatic index, 5.23; of Maxwell- boosts, 1.0; effect on Riemann
Boltzmann gas, 5.32 tensor, 15.14
affine connection, see connection Bose-Einstein gas, 5.34
coefficient Boyer-Lindquist coordinates, 17.0
affine parameter, 7.0, 7.11 Brans-Dicke theory of gravity, 13.3,
angular momentum, 11.1, 11.3, 11.4, 15.20,16.5,21.7
11.6, 20.4; in spherical metric, buoyant force, 16.2
15.1; tensor, 11.2. See also spin
4-vector

Cauchy hypersurface, 13.10


angular velocity of lab frame, 14.12 causality, 12.13
antisymmetric tensor, totally, 3.20- center of mass frame, 2.11
3.24, 3.28
Chandrasekhar limit, 16.9
antisymmetrized direct product,
charged particle motion, 4.10 4.14;

3.24
5.9; from Hamiltonian, 14.19;
antisymmetry, 3.14 3.18

integral of, 14.20


astronaut, inebriated, 7.22 Christoffel symbol, 7.0, 8.0. See
automobiles, junked, 17.2 a Iso connection coefficient
axisymmetric system, radiation from, collapse, see star, collapsing
18.9
collapse of shell, 21.11
colliding beam, 2.6
basis vectors or one-forms, 8.1;
comma goes to semicolon rule, 14.0
orthonormal, 8.0 commutator of vectors, 8.0
Bernoulli equation, relativistic,
Compton scattering, 2.1, 5.14; in-
14.7, 14.8
verse, 2.2
Bianchi identities, 9.15, 9.16
conformal invariance of Maxwell
big bang model, 19.0 equations, 14.21

593
594 INDEX

conformal tensor, 9.0 decay, particle, 2.5, 2.10


conformal transformation of metric, deceleration parameter, 19.0,19.15,
6.7 19.22

conformally flat metric, 9.18,13.2; derivative, covariant, see connec-

if isotropic, 9.27; Riemann tion coefficient


tensor of, 9.19 derivative, directional, 7.0, 8.0
connection coefficients, 7.0, 7.4; derivative, exterior, 8.3, 8.5, 8.6,
of a diagonal metric, 7.6; identi- 8.24
ties for, 7.7; not tensors, 7.1, derivative, Lie, 8.13-8.17, 8.21,
8.11; for polar coordinates, 7.2; 9.30; of metric tensor, 10.2
with same geodesics, 8.26; of
derivatives, commutation of, 9.8-9.10
spherical metric, 8.27; symmetries DeSitter universe, 19.30
of, 8.12
conserved momentum of cyclic determinant; covariant derivative of,
7.8; related to antisymmetric
coordinate, 7.13
tensor, 3.23; of metric tensor, 3.9
conserved vector, 3.35
distance measures, cosmological,
contraction, 3.0 19.9, 19.24
convective stability, 16.21 doppler shift, 1.18, 1.19, 1.21,1.22,
coordinate basis vector or one-form, 15.8, 20.9
8.0 dot product, 1.0
coordinate transformations, 3.8,3.10 dragging of inertial frames, 13.18,
cosmic microwave background, 19.0 20.4
cosmological constant, 13.4, 19.0, dual basis, 8.0
19.30 -19.34 dual of a tensor, 3.25, 3.26, 4.9
cosmology, 19.0 ff.; anisotropic, dynamics, particle, 2.3, 2.4, 2.6-
19.41; empty and isotropic, 19.2; 2.12, 2.14
Newtonian, 19.1; static, 19.35-
19.37
Eddington-Finkelstein coordinates,
covariant components, 1.0
16.11
covariant derivative, see connec-
Eddington luminosity limit, 12.5
tion coefficient
Einstein field equations, 13.0
covariant differentiation, 7.0 ff.
Einstein tensor, 9.16,13.0
covariant vector, 8.0
Einstein universe, 19.31, 19.32
curvature, 9.0 ff. See also Riemann
electromagnetic current density, 4.0,
tensor, Weyl tensor
4.7
curvature, extrinsic, see extrinsic
curvature
electromagnetic energy density, 4.0
electromagnetic field; duality rota-
curvature, mean, 9.31
tion, 4.20; Green's function for,
curvature, principal, 9.34
4.22; invariants, 4.2 4.4; Killing

curvature scalar, sign of, 13.5 vector, 10.16; measurability, 9.22;


curvature of two-sphere, 9.1 tensor, 4.0, 4.8, 4.9; two-form, 8.7
curvilinear coordinates, 3.0 electromagnetic stress-energy tensor,
4.0
cyclic coordinate, 7 .13
electron does not fall in closed con-
cyclotron, 2.14
ductor, 12.6
INDEX 595

embedding, 15.15 Frobenius' theorem, 7.23


energy conditions, 13.0; weak, 5.6, frogs, giant, 1.24
13.6; dominant, 13.7
energy conservation in Friedmann gas, distribution function for a rela-
model, 19.16 tivistic, 5.34
energy density,S. 0 gauge transformation, 13.12, 13.14,
energy flux, 5.16 18.8

energy of particle, minimum value, Gauss-Bonnet theorem, 17.19


10.15 Gauss-Codazzi equations, 9.32,
entropy; flux,S. 27; generation of, 9.33, 9.35
5.28; per baryon, 19.0, 19.25, Gaussian normal coordinates, 8.25,
19.26 9.32, 9.33
Eotvos experiment, 20.8 geodesic, 7.0, 7.4, 7.9; null, 1.13
equations of motion, 5.0, 13.0, 14.0 geodesic deviation, 9.13, 9.14
equivalence principle, 12.14, 14.14 geodesic equation, 7.0, 7.10 7.12;

ergosphere, 17.0 for Kerr black hole, 17.0


ergos urface, 17.12 geodesics, of flat 2-space, 7.3;
Euler equation for perfect fluid, particle motion on, 7.12, 14.1; of
14.3 velocity-space metric, 7.15
expansion of fluid flow, 5.18, 14.10 geodetic precession, 11.11
expansion, cosmological, 19.38- geometrized units, 12.17
19.40 gradient, 8.0
experimental tests, 20.0 ff. gravitational potential, 12.0
extrinsic curvature, 9.0, 9.28-9.34 gravitational waves, 13.16, 18.0 ff.;
strong, 18.22
Fermat's principle, 7.14 gravitons, 18.16 18.19-

Fermi-Dirac gas, 5.24, 5.34 gravity, theories of other than general


relativity, 13.1- 13.3. See also
Fermi-Walker transport, 7.0, 7.17- Brans-Dicke theory of gravity
7.19, 8.18, 8.20, 11.4, 11.7
gyroscope, 11.4, 11.10, 11.11
force field, constraints on, 2.15
form, differential, 8.0
Hawking's area theorem, 17.14, 17.15
freely falling frame, 8.0 heat conduction, stress-energy
Friedmann equations, 19.13, 19.14, tensor of, 5.28
19.16
heat flux, stress-energy tensor of,
Friedmann universe, 19.0; brightness 5.26
of night sky, 19.21; bullet in,
19.19; deuterium production, homogeneity, 19.15; and isotropy,
9.26, 19.4, 19.35
19.29; distances, 19.24; helium
horizon of black hole, 17.0
production, 19.29; matter domi-
nated, 19.18; photon circling, Hubble constant, 19.0
19.20; radiation dominated, 19.18; hydrogen atom, gravitational decay,
matches to Schwarzschild, 16.30; 18.18
temperature, 19.27, 19.28; volume hydrostatic equilibrium, 12.9, 12.10,
of, 19.23 14.4, 14.5
596 INDEX

hypersurface-orthogonal vector Lie derivative, see derivative, Lie

field, 7.23 f., 10.8; null, 7.24 Lie transport, 8.18 8.20

light, area of beam, 3.5


ignorable coordinate, 7.13 light deflection, 13.2, 20.1, 20.3-
index-free notation, 7.20 20.6
index of refraction of static space- light, gravitational attraction be-
time, 7.21 tween beams, 13.17
initial-value equations, 21.0 Lightman-Lee theory of gravity,
17.21
initial-value problems, 13.10,21.12
linearized theory, 13.0, 13.13,13.14,
injection energy, 16.22
isentropic flow, 5.20; of perfect 13.17,13.18,13.20,18.0; field
equations, 13.15; gravitational
fluid, 14.3
waves, 13.16; inconsistency of,
isotropic coordinates, 15.13 13.19
isotropy, see homogeneity little group, 1.23
Lorentz boost, 1.12, 1.27
Kasner universe, 19.41 Lorentz force equation, 4.0, 4.10-
Kepler's law, 12.1, 17.4 4.12, 4.19
Kerr metric, 17.0 ff. Lorentz gauge, 13.14, 13.15, 18.0
Kerr-Newman black hole, 17.0 Lorentz group, 1.25, 1.26, 1.28
Killing horizon, 17.12 Lorentz transformation, 1.2, 1.23;
Killing vector, 10.0 ff., 14.11; com- appearance of, 1.5; of solid angle,
mutator of, 10.3, 10.5; and con- 1.9; of electromagnetic field, 4.5,
served quantity, 10.11- 10.13; 4.6; of angles, 1.7, 1.8; of light
and constant of motion, 10.10; in beam, 3.5; of radiation flux, 5.11
Euclidean space, 10.4; and four- luminosity-redshift relation, 19.10
acceleration, 10.14; identity for,
10.6, 10.7; linear combinations Mach's principle, 13.18
of, 10.3; in Minkowski space,
10.9; of two-sphere, 10.1; and magnetic field, of wire, 4.1; frozen
into fluid, 4.23
staticity, 10.8; variational
principle, 10.6; as vector poten- magnetic monopole, 4.21
tial, 10.16 map of earth, cylindrical or stereo-

Killing's equation, 10.0, 10.2 graphic, 6.3; Mercator, 6.4


kinematics, particle, 2.5 mass; inertial, 5.17; negative, 13.20

kinetic energy, 2.0 Massachusetts motorist, 18.1


Kronecker delta tensor, 3.19,3.27, Maxwell-Boltzmann gas, 5.32 5.35 -

3.28 Maxwell equations, 4.0, 4.8, 4.9,


Kruskal coordinates, 15.15 14.15, 14.18, 21.6; conformal in-
variance, 14.21; curvature coupling
in, 14.16, 14.17
LeMaitre coordinates, 15.16
Maxwell tensor, 4.0
Lemaitre universe, see Friedmann
metric tensor, 3.0, 3.8, 3.9, 6.0 ff.,
universe
8.1; ambiguity in dimensionality,
lens to simulate light deflection of
6.5; analytic extension, 6.9; con-
sun, 20.6
formally flat, 9.0; covariantly con-
Lense-Thirring effect, 11.10, 11.11 stant, 7.5; of Euclidean hypersphere,
INDEX 597

6.2; of flat 2-space, 6.1; for map Poynting-vector, 4.0, 4.4


of Earth, 6.3, 6.4; spherically
Poynting-Robertson effect, 5.12
symmetric, 6.10; static, 10.8;
static, generated by perf.ect precession of Earth, 11.9
fluid, 13.9; stationary, 10.8; syn- principal directions, 9.34, 9.35
chronous, 8.25; of velocity space, product, direct, 3.0, 3.11, 3.12; of
6.8 forms, 8.4
mirror,1.18, 1.19 product, outer, antisymmetrized,
momentum, four-, 1.0, 1.20, 2.0;
3.24
conservation of, 2.0; of photon, product, scalar, 8.0
1.0 product, wedge, 8.0
momentum density, 5.0 projection tensor, 6.6
Mossbauer effect, 2.10 proper time, 1.0

N avier-Stokes equations,S. 31 quadrupole tensor, reduced, 18.0


negative mass, 13.20 quantum effects on black holes, 17.20
neutron star mass limit, 16.9 quasars, 19.33
Newtonian gravitation, 12.0, 12.8,
12.11-12.13 radiation, 5.11-5.14; intensity of,
non-coordinate basis, 8.2 5.10. See a Iso gravitational waves
Nordstr theory
m of gravitation, rapidity parameter, 1.0
13.2 Raychaudhuri equation, 14.10
null curve, 1.13,6.7 recombination of hydrogen, cosmo-
null surface, 8.8 logical, 19.22, 19.25, 19.26
null vector, 3.2 3.4

redshift, 8.28, 15.10, 19.0, 20.9


number counts, 19.11 Reissner-Nordstrom black hole,
17.0, 17.5, 17.17
Ohm's law, 4.18 Ricci tensor or scalar, 9.0
Olbers' paradox, 19.21 Riemann tensor, 9.0; in 1,2,3 dimen-
one-form, 8.0
sions, 9.7; in N dimensions, 9.2;
effect of boost on, 15.14; for com-
Oppenheimer- V olkoff equation, 16.0 9.3; of conformally flat
puter,
metric, 9.19; of doughnut, 9.6; of
Palatini variational method, 21.4 flat spacetime, 9.17; identities
parallel transport, 7.0, 7.19, 8.18- for, 9.15; of isotropic space, 9.25;
8.20; around a circuit, 9.11; on a measurability of, 9.22; of spheri-
2-sphere,7.16 cal metric, 9.20; symmetries of,
9.0; of two-Minkowski space, 9.5;
perfect fluid, 5.20, 5.21, 14.3; in
of a two-sphere, 9.4; in linearized
static metric, 13.9
theory, 13.13; of a plane gravita-
perihelion shift, 15.7, 20.7 tional wave, 9.21; zero, 9.17
phase space density, 3.34, 5.10 Riemannian curvature of a two-
Planck units, 12.18 surface, 9.23, 9.24
polytropic stars, onset of instability, rigid-body motion, 14.9
16.8 Robertson-Walker metric, 19.0, 19.5,
Pound-Rebka-Snider experiment, 19.6; conformally flat form, 19.8;
13.2, 20.8 null rays in, 19.12; redshift in, 19.7
598 INDEX

rocket, 2.13,7.15 spinor, 21.0, 21.14


21.16; identified
with 4-vector, 21.13; representa-
rope, breaking strength, 5.6
tion of Lorentz group, 1.25, 1.26,
rotating stars, 10..5, 16.0 1.28
rotation group, 1.25, 10.1, 10.4
star, binary, 18.2, 18.6, 18.7; collap-
rotation, cosmological, 19.38, 19.40 sing, 16.25-16.30; of Fermi gas,
rotation of fluid flow, 5.18 16.13; rotating, 16.17 -16.19;
1-6.22, 16.23; of uniform density,
16.12
scalar field, 21.5; curvature coup-
static metric, 10.8
ling of, 14.14; equation of
motion, 14.13; in Kerr geometry, stationary limit of black hole, 17.0,
17.16; in Schwarzschild geometry, 17.12
15.19 stellar structure, relativistic, 16.0 ff.

scattering, 2.8, 2.9,2.11 Stokes parameters for gravitational


scattering angle, 2.7, 2.12 wave, 18.10

Schur's theorem, 9.26 Stokes theorem, 8.10


Schwarzschild coordinates, 15.13 stressed medium, 5.17
Schwarzschild geometry, 15.0 ff., stress-energy pseudotensor, Landau-
21.11; accretion of gas into, Lifschitz, 13.11
15.18; light rays in, 15.6; orbits stress-energy tensor, 5.0; of capaci-
in, 15.2 15.5, 15.7, 15.11

tor, 5.8; eigenvectors of, 5.16; of


Schwarzschild interior solution, electromagnetic field, 4.15 4.17, -

16.12 5.9, 5.30; of heat conduction,


5.28; of heat flux, 5.26; of
shear, cosmological, 19.38 -19.40
isotropic gas, 5.2; of magnetic
shear of fluid flow, 5.18; of light
field, 5.4; of Newtonian gravity,
rays, 9.18
12.8; of scalar field, 14.13; of
shell,self-gravitating, spherical, single particle, 14.1; of groups of
16.14 16.16

particles, 5.1; suddenly nonzero,


simultaneity, 1.24, 3.1 13.8; of perfect fluid, 5.3; of rod
under tension, 5.5; of rotating rod,
soap film, 9.31
5.7; of vacuum, 13.4; of viscous
sound, velocity of, 5.22, 5.23; in
Fermi gas, 5.24 fluid, 5.30
summation convention, 1.0, 3.6,3.7
spacelike interval, 3.1
sun; corona modifies light deflection,
spacelike vector, 1.0, 3.4
20.5; gravitational position in sky,
spacelike surface, 8.8 12.4; oblateness of, 15.7
sphere, hollow, 16.4 surface, integration over, 8.9
spherical symmetry, metric, 6.10, surface layer, 21.8-21.11
16.0 ff.
symmetrization, 3.14 3.18-

spherical star, 16.0


synchronization of clocks, 1.11,1.17,
spin 4-vector, 11.3, 11.4, 11.6,11.7; 1.24
coupling to Riemann tensor, 11.8,
11.9
tachyons, 1.6, 12.13
spin of zero-rest-mass field, 12.15,
12.16, 18.20, 18.21 temperature, 5.13, 5.14; in acceler-
ated system, 5.29; in static gravi-
tational field, 14.2
INDEX 599

tensors, 3.0 vector analysis (3-dimensions),


thermodynamics, first law, 5.19, 8.22, 8.23
14.3 vector; null, see null vector;

thin shells, 21.0 tangent, 7.0; transformation of,


Thomas precession, 11.7 1.0, 3.0; unit, 1.10
tidal forces, 12.0 velocity addition, 1.3, 1.4
tides; solid Earth, 12.3; spring and velocity, four-, 1.0, 1.1
neap, 12.2 velocity space, metric of, 6.8, 7.15
time symmetry, instant of, 21.12 virial theorem, tensor, 5.15
timelike interval, 3.1 viscosity, 5.30, 5.31
timelike vector, 1.0, 3.4 volume element, 3.30 -

3.32; of
momentum space, 3.33
trace-reverse, 13.14
transverse-traceless (TT) gauge,
18.0 weight; of bottle when heated, 12.7;
twin paradox, 1.11, 15.12 pressure contribution, 14.6
Weyl tensor, 9.0, 9.18
units, geometrized, 12.17 white-dwarf mass limit, see
Chandrasekhar limit
universe, matter or radiation domi-
nated, 19.0 wind tunnel, relativistic, 5.25
wor ldline of an observer, 1.0

Vaidya metric, 16.11


variational methods, 7.25, 21.0- ZAMO's, 17.18
21.7
LIBRARY OF CONGRESS CATALOGING IN PUBLICATION DATA

Main entry under title:

Problem book in relativity and gravitation.

Includes bibliographical references and index.


1. Relativity (Physics) -Problems, exercises, etc.
2. Gravitation-Problems, exercises, etc. 3. Astrophysics-Problems, exercises,
etc. I. Lightman, Alan P., 1948-
QCI73.55.P76 530.1'1 74-25630
ISBN 0-691-08160-3
ISBN 0-681-08162-X pbk.

You might also like